Sei sulla pagina 1di 453

H. A.

MAPOH

Ill1<1><l>EPEHUl1AJlbHOE
l1 l1HTEfPAJlbHOE
l1C4l1CJ1E Hl1E
B OPHMEPAX l1 3AilA4AX

<l>YHKUliJliJ OJJ,HOfl IlEPEMEHHOfl


113J],ATEJlbCTBO HAYKA
MOCK BA
I. A. MARON

Problems
in Calculus
of One Variable

(With elements of theory)

TRANSLATED

FROM THE RUSSIAN


by
LEONID LEVANT

MIR PUBLISHERS
MOSCOW
UDC 517=20

First published 1973

Revised from the 1970 Russian edition

Ha artcAut'icKoM fl3b1Ke

0223-200
M
041(01)-73
Contents

From the Author . . . . . 9


Chapter I. Introduction to Mathematical Analysis 11
I.I. Real Numbers. The Absolute Value of a Real Number 11
1.2. Function. Domain of Definition 15
1.3. Investigation of Functions . . . . . 22
1.4. Inverse Functions . . . . . . . . . 28
1.5. Graphical Representation of Functions 30
1.6. Number Sequences. Limit of a Sequence 41
1.7. Evaluation of Limits of Sequences 48
1.8. Testing Sequences for Convergence . 50
1.9. The Limit of a Function . . . . . 55
1.10. Calculation of Limits of Functions 60
I.I I. Infinitesimal and Infinite Functions. Their Defini-
tion and Comparison . . . . . . . . . . . . 68
1.12. Equivalent Infinitesimals. Application to Finding
Limits . . . . . . . . . . . . . . . . . 71
1.13. One-Sided Limits . . . . . . . . . 75
1.14. Continuity of a Function. Points of Discontinuity
and Their Classification . . . . . . . . . . . . . . . . 77
1.15. Arithmetical Operations on Continuous Functions.
Continuity of a Composite Function . . . . . . . . . . 84
1.16. The Properties of a Function Continuous on a Clo-
sed Interval. Continuity of an Inverse Function 87
1.17. Additional Problems 91

Chapter II. Differentiation of Functions 93


2.1. Definition of the Derivative 98
2.2. Differentiation of Explicit Functions JOO
2.3. Successive Differentiation of Explicit Functions.
Leibniz Formula . . . . . , . . . . 107
6 Contents

2.4. Differentiation of Inverse, Implicit and Parametri-


cally Represented Functions . . . . . . . . . . . . . . 111
2.5. Applications of the Derivative . . . . . . . . . . 115
2.6. The Differential of a Function. Application to App-
roximate Computations 122
2.7. Additional Problems . . 126

Chapter III. Application of Differential Calculus to Investigation of


functions . . . . . . . . . . . . . 131

3.1. Basic Theorems on Differentiable Functions . . . . 131


3.2. Evaluation of Indeterminate Forms. L'Hospital's Rule 138
3.3. Taylor's Formula. Application to Approximate Cal-
culations . . . . . . . . . . . . . . . . . . . . . . . 143
3.4. Application of Taylor's Formula to Evaluation of
Limits . . . . . . . . . . . . . . . . . 147
~ 3.5. Testing a Function for Monotonicity . . . 148
3.6. Maxima and Minima of a Function . . . 152
3.7. Finding the Greatest and the Least Values of a
Function . . . . . . . . . . . . . . . . . . . . 159
3.8. Solving Problems in Geometry and Physics . . . 162
~ 3.9. Convexity and Concavity of a Curve. Points of
Inflection . . . . . . . . . . . . . . . . . . . 166
3.10. Asymptotes . . . . . . . . . . . . . . . . 170
3.11. General Plan for Investigating Functions and Sket-
ching Graphs . . . . . . . . . . . . . . . . . . . . 174
3.12. Approximate Solution of Algebraic and Transcen
dental Equations . . . . . 183
3.13. Additional Problems . . . . 190

Chapter IV. Indefinite Integrals. Basic Methods of Integration 190


4.1. Direct Integration and the Method of Expansion 195
4.2. Integration by Substitution 199
~ 4.3. Integration by Parts 202
4.4. Reduction Formulas 211

Chapter V. Basic Classes of Integrable functions 214

5.1. Integration of Rational Functions . . . . . 214


5.2. Integration of Certain Irrational Expressions 219
5.3. Euler's Substitutiom . . . . . . . . 222
5.4. Other Methods of Integrating Irrational Expressions 224
5.5. Integration of a Binomial Differential . . . 228
5.6. Integration of Trigonometric and Hyperbolic Functions 230
5.7. Integration of Certain Irrational Function~ with the
Aid of Trigonometric or Hyperbolic Substitutwns . . . 237
Contents 7

5.8. Integration of Other Transcendental Functions 240


5.9. Methods of Integration (List of Basic Forms of In-
tegrals) . . . . . . 242

Chapter VI. The Definite Integral 247


6.1. Statement of the Problem. The Lower and Upper
Integral Sums . . . . . . . . . . . . . . . 247
6.2. Evaluating Definite Integrals by the Newton-Leib-
niz Formula . . . . . . . . . . . . . . . . . 256
6.3. Estimating an Integral. The Definite Integral as a
Function of Its Limits . . . . . . . . . . . . . . . . . 262
6.4. Changing the Variable in a Definite Integral . . . 275
6.5. Simplification of Integrals Based on the Properties
of Symmetry of Integrands . . . . . . . . . . . 288
6.6. Integration by Parts. Reduction Formulas 294
6.7. Approximating Definite Integrals 301
6.8. Additional Problems . . . . . . . . . . 307
Chapter VII. Applications of the Definite Integral . . . . . . 310
7.1. Computing the Limits of Sums with the Aid of De-
finite Integrals . . . . . . . . . . . . . . . . . 310
7.2. Finding Average Values of a Function . . . . 312
7.3. Computing Areas in Rectangular Coordinates . 317
7.4. Computing Areas with Parametrically Represented
Boundaries . . . . . . . . . . . . . . . . . . . . . . 327
7.5. The Area of a Curvilinear Sector in Polar Coordinates 331
7.6. Computing the Volume of a Solid . . . . . . . . 336
7.7. The Arc Length of a Plane Curve in Rectangular
Coordinates . . . . . . . . . . . . . . . . . . . . . . 345
7.8. The Arc Length of a Curve Represented Parametri-
cally . . . . . . . . . . . . . . . . . . . . . . 348
7.9. The Arc Length of a Curve in Polar Coordinates 351
7.10. Area of Surface of Revolution . . . . . . . . . 354
7.11. Geometrical Applications cf the Definite Integral 360
7 .12. Computing Pressure, Work and Other Physical
Quantities by the Definite Integrals . . . . . . . . . . 367
7.13. Computing Static Moments and Moments of Inertia.
Determining Coordinates of the Centre of Gravity 372
7.14. Additional Problems . . . . . . . . 383
Chapter VIII. Improper Integrals . . . . . . . . . . . . . . . 387
8.1. Improper Integrals with Infinite Limits . . 387
8.2. Improper Integrals of Unbounded Functions 397
8.3. Geometric and Physical Applications of Improper
Integrals . . . . . . . . 409
8.4. Additional Problems 415
Answers and Hints . . . . . . . . . . . 418
In fond memory
of my parents

From the Author

This textbook on mathematical analysis is based on many years'


experience of lecturing at a higher technical college. Its aim is to
train the students in active approach to mathematical exercises,
as is done at a seminar.
Much attention is given to problems improving the theoretical
background. Therefore standard computational exercises are supple-
mented by examples and problems explaining the theory, promo-
ting its deeper understanding and stimulating precise mathema-
tical thinking. Some counter-examples explaining the need for cer-
tain conditions in the formulation of basic theorems are also in-
cluded.
The book is designed along the following lines. Each section
opens with a concise theoretical introduction containing the prin-
cipal definitions, theorems and formulas. Then follows a detailed
solution of one or more typical problems. Finally, problems with-
out solution are given, which are similar to those solved but
contain certain peculiarities. Some of them are provided with hints.
Each chapter (except Chap. IV and V) closes with a separate
~ection of supplementary problems and questions aimed at reviewing
and extending the material of the chapter. These sections should
prove of interest to the inquiring student, and possibly also to
lecturers in selecting material for class work or seminars.
The full solutions developed in the text pursue two aims: (1)
to provide lecturers with a time-saver, since they can refer the
students to the textbook for most of the standard exercises of a
computational character and concentrate mainly on the solution
of more sophisticated problems, thus gaining time for more rewar-
10 From the Author

ding work; and (2) to meet the needs of those who are working
on their own or following correspondence courses, providing a sub-
stitute for the oral explanations given to full-time students.
The student will find the book most useful if he uses it acti-
vely, that is to say, if he studies the relevant theoretical material
carefully before going on to the worked-out solutions, and finally
reinforces the newly-acquired knowledge by solving the problems
given for independent work. The best results will be obtained
when the student, having mastered the theoretical part, immedia
tely attacks the unsolved problems without referring to the text
solutions unless in difficulty.
Isaac Maron
Chapter }

INTRODUCTION
TO MATHEMATICAL ANALYSIS

I.I. Real Numbers.


The Absolute Value of a Real Number
Any decimal fraction, terminating or nonterminating, is called
a real number.
Periodic decimal fractions are called rational numbers. Every
rational number may be written in the form of a ratio, .!!... , of two
q
integers p and q, and vice versa.
Nonperiodic decimal fractions are called irrational numbers.
If X is a certain set of real numbers, then the notation x E X
means that the number x belongs to X, and the notation x ~ X
means that the number x does not belong to X.
A set of real numbers x satisfying the inequalities a< x < b,
where a and b are fixed numbers, is called an open interval (a, b).
A set of real numbers x satisfying the inequalities a~ x ~ b is
called a closed interval [a, b]. A set of real numbers x, satisfying
the inequalities a~ x < b or a< x ~ b, is called a half-open in-
terval [a, b) or (a, b]. Open, closed, and half-open intervals are
covered by a single term interval.
Any real number may be depicted as a certain point on the
coordinate axis which is called a proper point. We may also intro-
duce two more, so-called improper points, + oo and -oo infinitely
removed from the origin of coordinates in the positive and nega-
tive dirtctions, respectively. By definition, the inequalities -oo <
+
< x < oo hold true for any real number x.
The interval (a-e, a+ e) is called the e-neighbourhood of the
number a.
The set of real numbers x > M is called the M-neighbourhood
of the improper point +oo.
The set of rral numbers x < M is called the M-neighbourhood
of t .e improper point -oo.
The absolute value of a number x (denoted Ix I) is a number
that satisfies the conditions
Ix I= -x if x < O;
Ix I = x if x ~ o.
12 Ch. I. Introduction to Mathematical Analysis

The properties of absolute values are:


(I) the inequality Ix I::s;;;; a means that -a ::s;;;; x ::s;;;; a;
(2) the inequality Ix I~ a means that x ~a or x ::s;;;; -a;
(3) Ix YI~ Ix I+ IYI;
(4) lxyl~llxl-IYll;
(5) lxyl=lxllYI;
(5) I; I= \: l <Y =F O).
t.1. t. Prove that the number
0.1010010001. .. 1000 ... 01. ..
~
n
is irrational.
Solution. To prove this, it is necessary to ascertain that the
given decimal fraction is not a periodic one. Indeed, there are n
+
zeros between the nth and (n 1)th unities, which cannot occur
in a periodic fraction.
1.1.2. Prove that any number, with zeros standing in all deci-
mal places numbered ton and only in these places, is irrational.
t. 1.3. Prove that the sum of, or the difference between, a ra-
tional number a and an irrational number ~ is an irrational
number.
Solution. Consider the sum of a and ~ Suppose a+~= y is a
rational number, then ~ = y-a is also a rational number, since
it is the difference between two rational numbers, which contra-
dicts the condition. Hence, the supposition is wrong and the number
a+~ is irrational.

1.1.4. Prove that the product a~ and the quotient a/[~ of a


rational number a =F 0 and an irrational number ~ is an irrational
number.
1.1.5. (a) Find all rational values of x at which y = V- x 2 + x + 3
is a rational number.
Solution. (a) Suppose x and y = V x2 + x + 3 are rational 11um-
bers. Then the difference y-x = q is also a rational number. Let
us now express x through q
y-x= V x 2 +x+ 3-x= q,
v x +x+ 3= q+x,
2

x2 +x+3 = q2 --f-2qx+x 2 ,
q~-3
X= J-2q.

By a direct check it is easy to ascertain that q =F 1/ 2


I.I. Real Numbers. The Absolute Value of Real Number 13

Prove the reverse, namely, y=Vx 2 +x+3 is a rational number


if x=r-=-2~, where q is any rational number not equal to 1/ 2

Indeed,

Y= Vx2+x+3= ..V/(q2-3J2 q2-3


(1-2q)2+I=21i +3=
= ,,/q 4 -2q 3 +7q~-6q+9 = ,,/(q 2 -q+3) 2 =q 2 -q+3
v (l-2q) 2 v
(l-2q)"J J 1-2qJ
(
q=I= 2
I) .
The latter expression is rational at any rational q not equal to 1/ 2

(b) Prove that V2 is an irrational number.


l. l.6. Prove that the sum V3 + V2 is an irrational number.
Solution. Assume the contrary, i.e. that the number V3 + V2
is rational. Then the number
V-3- v-2= V3 + v
3
I
2
is also rational, since it is the quotient of two rational num')crs.
Whence the number
V2 = f [(V3+ V2)-(V3-V2) I
is rational, which contradicts the irrational nature of the number
V2 (see Problem 1.1.5). Hence, the supposition is wrong, and the
+
number V3 J/2 is irrational.
l.1.7. Prove that for every positive rational number r satisfying
the condition r 2 < 2 one can always find a larger rational number
+ +
r h (h > 0) for which (r h) 2 < 2.
Solution. We may assume h < 1. Then h2 < h and (r + h) 2 <
+
< r 2 + 2rh h. That is why it is sufficient to put r 2 + 2rh + h = 2,
i.e. h=(2-r 2 )/(2r+l).
l. l .8. Prove that for every positive rational number s satisfying
the condition s 2 > 2 one can always find a smaller rational number
s-k (k > 0) for which (s-k) 2 > 2.
l. l .9. Solve the following inequalities:
(a) I 2x-3 I< 1;
(b) (x-2) 2 ;:?4;
(c) x2 +2x-8~0;
(d) I x 2 - 7x+ 12 I> x2 - 7x+ 12.
Solution. (a) The inequality I2x-3 I< 1 is eqivalent to the
inequalities
-1<2x-3 < 1,
whence
2 < 2x < 4 and 1 < x < 2.
14 Ch. I. Introduction to Mathematical Analysis

(d) The given inequality is valid for those values of x at which


x 2 - 1x+ 12 < 0, whence 3 < x < 4.
1.1.10. Find out whether the following equations have any
solutions:
(a) jxJ=x+5; (b) lxl=x-5?
Solution. (a) At x ~ 0 we have x = x + 5. Hence, there are no
solutions. At x<O we have -X==x+5, whence x=-5/2. This
value satisfies the initial equation.
(b) At x~O we have x=x-5. Hence, there are no solutions.
At x < 0 we have -x=x-5, whence x=5/2, which contradicts
our supposition (x < 0). Thus, the equation has no solution.
1.1.11. Determine the values of x satisfying the following equa-
lities:

(a) I:~: I=:+: ;


(b) jx2 -5x+61=-(x2 -5x+6).
1.1.12. Determine the values of x satisfying the following equ-
alities:
(a) I (x 2 + 4x+ 9) + (2x-3) I= Jx 2 + 4x+9 I+ I 2x-3 J;
(b) I (x 4 -4)-(x 2 + 2) J=I x 4 -4 l-1x 2 +2 J.
Solution. (a) The equality ja+bl=lal+lbl is valid if and
only if both summands have the same sign. Since
x 2 +4x+9=(x+2) 2 +5 > 0
at any values of x, the equality is satisfied at those values of x
at which 2x-3 ~ 0, i.e. at x ~ 3/2.
(b) The equality la-bl=laJ-lbl holds true if and only if a
and b have the same sign and Ia I~ I b I
In our case the equality will hold true for the values of x at
which

Whence
X2 - 2 ~ I; Ix I ~ va:
1.1.13. Solve the inequalities:
(a) \3x-5J-j2x-t-31>0;
(b) x 2 -5xJ > Jx 2 J-J5xl.
1.l.14. Find the roots of the following equations.
(a) JsinxJ=sinx+ I;
(b) x 2 -2Jxl-3=0.
Solution. (a) This equation will hold true only for those values
of x at which sin x < 0, that is why we may rewrite it in the
1.2. Function. Domain of Definition 15

following way:
-sinx=sinx+ 1, or sinx=-1 / 2 ;
whence X=:rtk-(-I)k:rt/6 (k=O, 1, +2, ... ).
(b) This equation can be solved in a regular way by considering
the cases x~O and x~O. We may also solve this equation re-
writing it in the form
lxl 2 -21xl-3=0.
Substituting y for Ix I. we obtain
y2 -2y-3=0,
whence y1 = 3, y2 = -1. Since y =Ix I~ 0, the value y2 = -1 does
not fit in. Hence
y=jxj=3,
i.e. X 1 = -3, X 2 = 3.

1.2. Function. Domain of Definition


The independent variable x is defined by a set X of its values.
If to each value of the independent variable xE X there corres-
ponds one definite value of another variable y, then l/ is called
the function of x with a domain of definition (or domain) X or,
in functional notation, y = y (x), or y = f (x), or y =qi (x), and so
forth. The set of values of the function y (x} is called the range
of the given function.
In particular, the functions defined by the set of natural num-
bers 1, 2, 3, ... , are called numerical sequences. They are written
in the following way: X 11 x2 , , Xn or {xn}
1.2. t. Given the function f (x) = (x+ l)/(x-1). Find f (2x), 2f (x),
f (x 2 ), [f (x)]2.
Solution.
2x+ I x+I
f (2x) = 2x- I ; 2f (x) = 2 x-l;
x2 + I
f(x2)=x2-1; [f (x)] 2 = (:~: )2.
t.2.2. (a) Given the function
f (x) = log 1-x
1+ x

Show that at Xi, x2 E(-l, 1) the following identity holds true:

f (Xi)+ f (X2) = f ( t+~;;2 )


16 Ch. I. Introduction to Mathematical Analysis

Solution. At xE(-1, 1) we have (1-x)/(l+x)>O and hence


f( ) f( ) l I-xi I 1-x2 l (l-x1)(1-x 2 )
Xi+ x2 = og 1+x1 + og 1+x2 = og(l+x1)(1+x2J. (I)
On the other hand,
I- x1+x2
x1+x
f (- - 2 ) l
=Og l
l+x 1 x 2 =og l+x 1 x 2 -x 1 -x2 =
I +x 1 x 2 +x
1 1 +x 2 I +x1 x 2 +x 1 +x2
I +x1x2
- I (I -xi) (l-x2)
- og (I +x 1) (I +x 2 ) '

which coincides with the right-hand member of expression ( 1).


(b) Given the function f (x) =(ax+ a-x)/2 (a> 0). Show that
f (x + y) + f (x-y) = 2f (x) f (y).
1.2.3. Given the function f(x)=(x+l)/(x' 1 -J). Find f(-1);
f(a+l); f(a)+l.
1.2.4. Given the function f (x) = x3 - 1. Find
f (bt:~; (a) (b =I= a) and f ( a~ h).
1.2.5. Given the function
3-x- J, -I< x < 0,
f (x) = { tan (x/2), 0 x < < n,
x/(x 2 -2), n < x < 6.
Find f (-1), f (n/2), f (2n/3), f (4), f (6).
Solution. The point x = - I 1ies within the interval [-1, 0).
Hence
f(-1)=3-l-])_1=2.
The points x = n/2, x = 2n/3 belong to the interval [O, n).
Hence
f (n/2) =tan (n/4) =I; f (2:rt/3) =tan (n/3) = V3.
The points x = 4, x = 6 belong to the interval [n, 6]. Hence
4 2 6 3
f( 4 )=15-2=7; f( 6 )=35_z=17
1.2.6. The function f (x) is defined over the whole number scale
by the following law:
2x 3 + 1. ifx< 2,
f (x) __ { l/(x- 2), if 2 < x ~ 3,
2x-5, if x > J.
1.2. Function. Domain of Definition 17

Find: f (Jl2), f (VB), f (Vlog 2 1024).


1.2.7. In the square ABCD with side AB= 2 a straight line
MN is drawn perpendicularly to AC. Denoting the distance from
the vertex A to the line MN as x, express through x the area S
of the triangle AMN cut off from the square by the straight
line MN. Find this area at X= V2/2
and at x = 2 (Fig. 1). 8
Solution. Note that AC= 2 V2;
hence 0 < x < 2 v-2.
then
S(x)=Sl::. AMN=X2

If x > V2, then


S(x) = 4-(2 V2 -x) 2 =-x2 +
+4xV2 -4. ]}

Thus, Fig. I

S(x)={ x2 , O<x~J/2,
-x2 +4xV2 -4, V2 < x~2Jl2.
Since V2/2<V2, S(V2/2)=(V2/2) 2 = 1/ 2 Since 2>V2,

1.2.8. Bring the number an, which is equal to the nth decimal
place in the expansion of V2 into a decimal fraction, into cor-
respondence with each natural number n. This gives us a certain
function an=qi(n). Calculate qi(l), qi(2), qi(3), qi(4).
Solution. Extracting a square root, we find V2 = 1.4142 ...
Hence
cp(l)=4; qi(2)-=l; qi(3)=4; qi(4)=2.
1.2.9. Calculate f (x)= 49/x2 + x2 at the points for which 7/x + x=3.
Solution. f(x)=49/x 2 +x 2 =(7/x+x) 2 -l4, but 7/x+x=3,hence
f(x)=9-14=-5.
1.2. IO. Find a function of the form f (x) = ax 2 +bx+ c, if it is
known that f(0)=5; f(-1)=10; f(l)=6.
Solution.
f (0) = 5 =a 02 + bO+c,
f (-1)= lO=a-b+c,
f (1) = 6 =a+b+c.
18 Ch. I. Introduction to Mathematical Analysis

Determine the coefficients a, b, c from the above system. We have:


a=3; b=-2; c=5; hence f (x)=3x 2 -2x+5.
1.2.11. Find a function of the form
f (x) =a+ be" (c > 0),
if f (0) = 15; f (2) = 30; f (4) = 90.
1.2.12. Find cp['(x)] and 'ljl[cp(x)] if
cp (x) = x2 and ' (x) =2x.
Solution.
cp [' (x)] = ['(x)]2 = (2x}2 =22",
' [cp (x)] = 2" <xi= 2x2.
1.2.13. Given the function
f (x} = 5x2+ I
2-x
Find f (3x); f (x3 ); 3f (x); [f (x) ]3.
1.2.14. Let
3x at -1 < X < 0,
{
f (x) = 4 at 0 ~ x < 1,
3x-1 at 1 ~x~3.
Find f (2), f (0), f (0.5), f (-0.5), f (3).
1.2.15. Prove that if for an exponential function y =ax (a > O;
a=I= 1) the values of the argument X=Xn (n= 1, 2, ... ) form an
arithmetic progression, then the corresponding values of the func-
tion Yn = axn (n = 1, 2, ... ) form a geometric progression.
1.2.16. f (x) = x 2 + 6, cp (x) = 5x. Solve the equation f (x} =I cp (x) I
1.2.17. Find f(x) if
f(x+ l}=x2-3x+2.
1.2.18. Evaluate the functions
f (x) = x 2 + l/x2 and cp (x) = x4 + l/x4
for the points at which l/x + x = 5.
1.2.19. f (x) = x+ l; cp (x) = x-2; solve the equation
If <x> + cp (x) I= If (x) I+ Icp (x) 1.
1.2.20. A rectangle with altitude x is inscribed in a triangle
ABC with the base b and altitude h. Express the perimeter P and
area S of the rectangle as functions of x.
1.2. Function. Domain of Definition 19

1.2.21. Find the domains of definition of the following functions:


(a) f (x) = Vx=T + V6-x;
(b)f(x)=Vx 2 -x-2+V 1 ;
3+2x-x2
(c) f (x) = V x ;
x 2 -x-2
(d) f (x) = V sin x- 1 ;
(e) f (x) = ..V/ log -5x-x~
4 -;
(f) f (x) = logx 5;
x2-5x+6
(g) f (x) = log xLHx + f:i ;
x-3
(h) f (x) =arc sin- 2 -Iog (4-x);
I , __
(i) f(x)= 1og(l-x)+V x+2;
(j) f (x) = log cos x;
3
(k) f (x) = arc cos 4 + 2 sin x ;
(I) - I
Y-YIXFX
Solution. (a) The domain of definition of the given function
consists of those values of x at which both items take on real
values. To ensure this the following two conditions must be satis-
fied:
x-1~0,
{
6-x~O.

By solving the inequalities we obtain x ~ l; x ~ 6. Hence,


the domain of definition of the function will be the segment [ 1,6].
(e) The function is defined for the values of x for which
5x-x:i
Iog-4-~0.

This inequality will be satisfied if


5x-x:i 2
-4-~ 1, or x -5x+4<0.
Solving the latter inequality, we find 1 ~ x ~ 4. Thus, the seg-
ment [ 1,4] is the domain of definition of the function.
(f) The function is defined for all positive x different from unity,
which means that the domain of definition of the function consists
of the intervals (0, l) and (1, oo). +
20 Ch. I. Introduction to Mathematical Analysis

(k) The function is defined for the values of x for which


3
- l ~4+2sinx~ I.
Since 4+2 sin x >0 at any x, the problem is reduced to solving
the inequality
3
4+2sinx ~I.
Whence
3 ~ 4 + 2 sin x, i.e. sin x ~ - 1/
2

By solving the latter inequality we obtain


:rt 7:rt
- 6 +2kn~x ~ 6 +2kn (k=O, +I, 2, ... ).
(I) The function is defined for the values of x for which
Ix I - x > 0, whence Ix I > x. This inequality is satisfied at x < 0.
Hence, the function is defined in the interval (-oo, 0).
1.2.22. Find the domains of definition of the following functions:
(a) f (x) = V arc sin (log2 x) ;
(b) f (x) = log 2 log 3 log4 x;
(c) f (x) =...!...+2arc in x +-1-;
x Yx-2
(d) f (x) =log I4 - x2 I;
(e) f (x) = V cos (sin x) +arc sin 1txx 2

Find the ranges of the following functions:


(f) Y -- 2-cos
I .
3x '
x
(g) Y = l-f-x2

Solution. (a) For the function f (x) to be defined the following


inequality must be satisfied
arc sin (log 2 x);;;;:: 0,
whence 0 ~ log 2 x~ l and I~ x~ 2.
(b) The function log 2 log 3 log 4 x is defined for log 3 log4 x > 0,
whence log 4 x > I and x > 4. Hence, the domain of definition is
the interval 4 < x< oo. +
(c) The given function is defined if the following inequalities are
satisfied simultaneously:
x=;t=O; -1 ~x~ I and x> 2,
but the inequalities -1 ~ x~ I and x > 2 are incompatible, that
is why the function is not defined for any value of x.
1.2. Function. Domain of Definition 21

(e) The following inequalities must be satisfied simultaneously:


cos (sin x) ;;;;::: 0 and j 1 tx 2
j ~ l.
The first inequality is satisfied for all values of x, the second,
for Ix I= I. Hence, the domain of definition of the given function
consists only of two points x = +I.
(f) We have
2y-I
cos3x=--.
y
Since
2y-l
-I~ cos3x~ 1, we have -1 ~--~I,
y
whence, taking into account that y > 0, we obtain
1
-y~2y-1~y or 3 ~y~ I.

(g) Solving with respect to x, we obtain

x = I VI
--~-----''-
-4y 2
2y
The range of the function y will be determined from the relation
l-4y 2 ;;;;::: 0.
Whence
I I
-2~Y~2
1.2.23. Solve the equation
arc tan V x (x + I)+ arc sin V x 2 + x + I = n/2.
Solution. Let us investigate the domain of definition of the func-
tion on the left side of the equation. This function will be defined
for

whence x 2 + x = 0.
Thus, the left member of the equation attains real values only
at x1 = 0 and x 2 = - I. By a direct check we ascertain that they
are the roots of the given equation.
This problem shows that a study of domains of definition of a
function facilitates the solution of equations, inequalities, etc.
1.2.24. Find the domains of definition of the following functions:
2x-3
(a) !J = V x + 2x + 3
2 ;
(b) y=log sin(x-3)+Vl6-x 2 ;
22 Ch. I. Introduction to MafhPmatical Analy~is

--
(c) y= V 3-x+arc cos - 3- ;
x-2

x
(d) Y=1og(l+x)'
1.2.25. The function f (x) is defined on the interval [O, 1]. What
are the domains of definition of the following functions:
(a) f (3x 2 ); (b) f (x-5); (c) f (tan x)?
Solution. The given functions are functions of functions, or su-
perpositions of functions, i. e. composite functions.
a) Let us introduce an intermediate argument u = 3x2 Then the
function f(3x 2)=f(u) is defined if O~u~ 1, i.e. 0~3x2 ~ 1,
whence -1/V3~x~ 1/V3.
(c) Similarly: 0 ~tan x ~ 1, whence
kn~ x~ n/4+kn (k =0, 1, + 2, ... ).
1.2.26. The function f (x) is defined on the interval [O, 1]. What
are the domains of definition of the functions
(a) f (sin x); (b) f (2x+ 3)?
1.3. /n'lJestigation of Functions
A function f (x) defined on the set X is said to be non-decreasing
on this set (respectively, increasing, non-increasing, decreasing), if
for any numbers x1 , x 2 EX, x 1 < x 2 , the inequality f (x1 ) ~ f (x2 )
(respectively, f (x1 ) < f (x2 ), f (x1 ) ~ f (x 2 ), f (x1 ) > f (x2 )) is satisfied.
The function f (x) is said to be monotonic on the set X if it pos-
sesses one of the four indicated properties. The function f (x) is
said to be bounded above (or below) on the set X if there exists a
number M (or m) such that f (x) ~ M for all x EX (or m ~ f (x)
for all x EX). The function f (x) is said to be bounded on the set X
if it is bounded above and below.
The function f (x) is called periodic if there exists a number
+
T > 0 such that f (x T) -= f (x) for all x belonging to the domain
of definition of the function (together with any point x the point
+
x T must belong to the domain of definition). The least number T
possessing this property (if such a number exists) is called the
period of the function f (x). The function f (x) takes on the maxi-
mum value at the point x 0 EX if f (x 0 ) ~ f (x) for all x EX, and
the minimum value if f (x 0) ~ f (x) for all x E X. A function f (x)
defined on a set X which is symmetric with respect to the origin
of coordinates is called even if f (- x) = f (x), and odd if f (- x) =
=-f(x).
In analysing the behaviour of a function it is advisable to de-
termine the following:
1.3. Investigation of Functions 23

1. The domain of definition of the function.


2. Is the function even, odd, periodic?
3. The zeros of the function.
4. The sign of the function in the intervals between the zeros.
5. Is the function bounded and what are its minimum and ma-
ximum values?
The above items do not exhaust the analysis of a function, and
later on their scope will be increased.
1.3.1. Find the intervals of increase and decrease of the f unc-
tion f (x) = ax 2 +bx+ c, and its minimum and maximum values.
Solution. Isolating a perfect square from the square trinomial,
we have
f(x)=a ( x+-
b )2 +--.
4ac-b2
2a 4a
If a> 0, then the function f (x) will increase at those values of x
satisfying the inequality x+b/(2a) > 0, i.e. at x>-b/(2a), and
decrease when x+b/(2a) < 0, i.e. at x<-b/(2a). Thus, if a> 0,
the function f (x) decreases in the interval ( -oo, -:a)
and inc-
reases in the interval (-b/(2a), +oo). Obviously, at x=-b/(2a)
the function f (x) assumes the minimum value
b) 4ac-b2
fm1n=f ( -2Q =~
At a> 0 the function has no maximum value.
Similarly, at a< 0 the function f (x) will increase in the inter-
val ( - oo, -:a)and decrease in the interval (- b/(2a), oo); at
x= - b/(2a) the function f (x) takes on the maximum value
fmax= f ( - :a)= 4ac4-;;b2

whereas it has no minimum value.


1.3.2. (a) Find the minimum value of the function
Y= 3x2 +5x-l.
(b) Find the rectangle with the maximum area from among all
rectangles of a given perimeter.
Solution. (a) Apply the results of Problem 1.3.1: a= 3 > 0, b = 5,
c = - l. The minimum value is attained by the function at the
point x=-5/6
4ac-b 2 37
Ym111=~=-12

(b) We denote by 2p the length of the perimeter of the required


rectangle, and by x the length of one of its sides; then the area S
24 Ch. I. Introduction to Mathematical Analysis

of the rectangle will be expressed as


S=x(p-x) or S=px-x 2
Thus, the problem is reduced to the determination of the maximum
value of the function S (x) = - x 2 + px. Apply the results of Prob-
lem 1.3. l: a= - 1 < 0, b = p, c = 0. The maximum value is attai-
ned by the function at the point x = - b/(2a) = p/2. Hence, one
of the sides of the desired rectangle is p/2, the other side being
equal to p-x = p/2, i. e. the required rectangle is a square.
1.3.3. Show that
(a) the function f (x) = x 3 + 3x + 5 increa<ses in the entire domain
of its definition;
(b) the functiong(x)=x/(l+x 2 ) decreases in the interval (1, +oo).
Solution. The function is defined for all points of the number
scale. Let us take arbitrary points Xi and x2, Xi < x 2 on the number
scale and write the following difference:
f (x2)-f (x1) = (x~ + 3x2+ 5)-(xf + 3xi + 5) =
= (X2-X1) (x: + X1X2 + x~ + 3) =

= (x2 - X1) l(
X1 + ~ X2) 2 + ! x~ + 3] .
Since x2-Xi > 0 and the expression in the brackets is positive
at all x1 and x 2, then f (x 2)-f (xi)> 0, i.e. f (x2) > f (x1), which
means that the function f (x) increases for all values of x.
1.3.4. Find the intervals of increase and decrease for the follo-
wing functions:
(a) f (x) =sin x+cosx;
(b) tan(x+n/3).
Solution. (a) Using the familiar trigonometric formulas, we find
f (x) = V2 cos (x-n/4).
It is known that the function cos x decreases in the intervals
2nn ~ x~ (2n+ 1) n
and increases in the intervals
(2n-l)n~x~2rm (n=O, +l, 2, ... ).
Hence, the intervals of decrease of the function f (x) are:
n/4+2nn~x~n/4+(2n+l)n (n=O, 1, ... ),
and the intervals of increase of the same function are:
n/4+(2n-l)n~x~n/4+2nn (n=O, + 1, ... ).
l.3.5. Find the minimum and maximum values of the function
f (x) = a cos x + b sin x (u 2 +b > 0).
2
1.3. Investigation of Functions 25

Solution. The given function can be represented as:


f (x) =Va+ b2 cos (x-a),
where cos a= a/ Va2 + b2 , sin a= b/ Va 2 + b2 Since I cos (x-a) I~ 1,
the maximum value of f (x) equals + Va 2 + b" (at cos (x-a)-= 1),
the minimum value of f (x) being equal to -Va 2 + b2 (at
cos (x-a) =-1).
1.3.6. Find the minimum value of the function
f (x) = 3<x'-2>3+ s.

Solution. We denote by qi (x) the exponent, i. e.


qi (x) = (x 2 -2) 3 + 8.
The function f (x) = 3!/l <x> takes on the minimum value at the
same point as the function qi (x).
Hence
qi (x) = x 6 -6x 4 + 12x2 = x 2 [(x2 -3) 2 + 3].

Whence it is clear that the function qi (x) attains the minimum


value (equal to zero) at x = 0. That is why the minimum value of
the function f (x) is equal to 3 = 1.
1.3.7. Test the function
f (x) =tan x+ cot x, where 0 < x < n/2,
for increase and decrease.
1.3.8. Given: n numbers a1 , a 2 , , an- Determine the value
of x at which the function
f (x) = (x-a1 ) 2 + (x-a + ... + (x-an)
2) 2 2

takes on the minimum value.


Solution. Rewrite the function f (x) in the following way:
f(x)=nx 2 -2(a 1 +a 2 + ... +an)x-t-(a~+ai+ ... +a~).
wherefrom it is clear that f (x) is a quadratic trinomial ax2 +bx+ c,
where a= n > 0. Using the results of Problem 1.3.1, we find that
the function assumes the minimum value at x = -b/(2a), i. e. at
x= (a 1 +a2 + ... +an)/n.
Thus, the sum of the squares of deviations of the value of x
from n given numbers attains the minimum value when x is the
mean arithmetic value for these numbers.
1.3.9. Which of the given functions is (are) even, odd; and
which of them is (are) neither even, nor odd?
26 Ch. I. Introduction to Mathematical Analysis

(a) f (x) log (x + V 1 + x2);


=
1-x
(b) f (x)=log 1+x;
(c) f (x) = 2x3 -x+ l;
ax+1
(d ) f (x) = x ax-1
Solution. (a) It can be seen that f(+x)+f(-x)=O. Indeed,
f (+ x) + f (-- x) = log (x + V 1 + x2) +log (- x + V 1 + x2) =
=log (1+x2 -x2 )=0,
hence, f (x) = - f (- x) for all x, which means that the function is odd.
l+x (l-x)-1 1-x
(b) f(-x)=log 1_x=log l+x =-log 1+x
Thus, f (- x) = - f (x) for all x from the domain of definition
(-1, 1). Hence, the function is odd.
1.3.10. Which of the following functions is (are) even and which
is (are) odd?
(a) f (x) = 4-2x4 +sin 2 x;
(b) f (x) = V 1 + x + x 2 -V1-x + x 2 ;
1+akx
(c) f (x) = 1-akx;
(d) f(x)=sinx+cosx;
(e) f (x) = const.
1.3.11. Prove that if f (x) is a periodic function with period T,
then the function f (ax+b), where a> 0, is periodic with period T/a.
Solution. Firstly,
f [a (x+ T/a) +b] = f [(ax+ b)+ T] = f (ax+b),
since T is the period of the function f (x). Secondly, let T1 be a
positive number such that
f [a(x+ T 1 )+b] =f (ax+b).
Let us take an arbitrary point x from the domain of definition
of the function f (x) and put x' = (x-b)/a. Then
f (ax'+ b) = f (a x:b + b) = f (x) = f [a (x' + T 1) + b] =
= f (ax' +b+aT 1) = f (x+aT 1).
Whence it follows that the period T:::;;;. aT 1 , i.e. T 1 ~ T/a and
T /a is the period of the function f (ax+ b).
Note. The periodic function f(x)=Asin(wx+cp), where A, w, cp
are constants, is called a harmonic with amplitude I A I. frequency w
J ..'3. Investigation of Functions 27

and initial phase qi. Since the function sin x has a period 2n, the
function A sin (mx +qi) has a period T = 2n/m.
1.3.12. Indicate the amplitude I A I. frequency m, initial phase qi
and period T of the following harmonics:
(a) f (x) = 5 sin 4x;
(b) f (x) = 4 sin (3x + n/4);
(c) f (x) = 3 sin (x/2) + 4 cos (x/2).
1.3.13. Find the period for each of the following functions:
(a) f (x) =tan 2x;
(b) f (x) =cot (x/2);
(c) f (x) =sin 2nx.
Solution. (a) Since the function tan x has a period n, the function
tan 2x has a period n/2.
1.3.14. Find the period for each of the following functions:
(a) f (x) =sin' x+ cos 4 x;
(b) f(x)=lcosxl.
Solution. (a) sin' x+cos4 x= (sin 2 x+ cos 2 x) 2 -2 sin 2 xcos2 x=
= l-..!...sin 2 2x-1-_!_ (I-cos 4x)-~ +...!..sin (4x+ ~)
2 - 4 -4 4 2 '

whence T = 2n/m = 2n/4 = n/2.


(b) f (x) =I cos x I= V cos2 x ~= V (I+ cos 2x)/2; but the function
cos 2x has a period T = n; hence, the given function has the same
period.
1.3.15. Prove that the function f (x) =cos x2 is not a periodic one.
Solution. Let us prove the contrary. Suppose the function has a
period T; then the identity cos (x + T) 2 =cos x 2 is valid.
By the conditions of equality of cosines for a certain integer k
we have
x2 + 2Tx+ T2 x2 = 2nk.
But this identity is impossible, since k may attain only integral
values, and the left member contains a linear or quadratic function
of the continuous argument x.
1.3.16. Find the greatest value of the function
2
f (x) = V 2x2 -4x+3
1.3.17. Which of the following functions are even, and which are
odd:
(a) f (x) = V (l -x) 2 + V (I+ x) 2 ;
28 Ch. I. Introduction to Mathematical Analysis

(b) f (x)=x 2 -lxl;


(c) f(x)=xsin 2 x-x 3 ;
(d) f (x) = (1+2X) 2/2x?
1.3.18. Find the period for each of the following functions:
(a) f (x) =arc tan (tan x);
(b) f (x) = 2 cos x 3 n.

1.3.19. Prove that the functions


(a) f (x) = x+ sin x; (b) f (x) =cos Vx
are non-periodic.

1.4. /n'Verse Functions


Let the function y = f (x) be defined on the set X and have a
range Y. If for each y E Y there exists a single value of x such that
f (x) = y, then this correspondence defines a certain function x = g (y)
called inverse with respect to the given function y = f (x). The suf-
ficient condition for the existence of an inverse function is a strict
monotony of the original function y = f (x). If the function increa.,es
(de~reases), then the inverse function also increases (decreases).
The graph of the inverse function x = g (y) coincides with that of
the function y = f (x) if the independent variable is marked off along
the y-axis. If the independent variable is laid off along the x-axis,
i. e. if the inverse function is written in the form y = g (x), then
the graph of the inverse function will be symmetric to that of the
function y = f (x) with respect to the bisector of the first and third
quadrants.
1.4.1. Find the inverse to the function y = 3x + 5.
Solution. The function y = 3x + 5 is defined and increases through-
out the number scale. Hence, an inverse function exists and in-
creases. Solving the equation y = 3x+ 5 with respect to x we obtain
x = (y-5)/3.
1.4.2. Show that the function y = k/x (k *
0) is inverse to itself.
Solution. The function is defined and monotonic throughout the
entire number scale except x = 0. Hence, an inverse function exists.
The range of the function is the entire number scale, except y = 0.
Solving the equation y=k/x with respect to x, we get x=k/y.
1.4.3. Find the inverse of the function
y=loga(x+Vx2 +l), (a>O, a* 1).
Solution. The function y = loga (x + V x 2 + 1) is defined for all x,
+
since Vx 2 1 > lxl, and is odd [see Problem 1.3.9 (a)J. It increases
1.4. Inverse Functions 29

for positive values of x, hence, it increases everywhere and has an


inverse function. Solving the equation
Y = loga (x + VX 2 + 1)
with respect to x, we find
aY =x+ Vx 2 + l; a-Y= - x+ V x2 + l,
whence
x= ~ (aY-a-Y)=sinh(ylna).

l .4.4. Show that the functions


f(x)=x 2 -x+l, x;;;::=l/2 and cp(x)=l/2+Vx-3/4
are mutually inverse, and solve the equation
x2 -x+ 1=1/2 + V" x-3/4.
Solution. The function y=x 2 -x+l=(x-l/2) 2 +3/4 increases
in the interval 1/2 ~ x < oo, and with x varying in the indicated
interval we have 3/4 ~ y < oo. Hence, defined in the interval
3/4 ~ y < oo is the inverse function x = g (y), x ;;;::= 1/2, which is
found from the equation
x2 -x+(l-y)=O.
Solving the equation with respect to x, we obtain
X=g(y)= 1/2+ Vy-3/4=cp (y).
Let us now solve the equation
x 2 -x+ I= 1/2+ V x-3/4.
Since the graphs of the original and inverse functions can intersect
only on the straight line y=x, solving the equation x 2 -x+ 1 ~~x
we find x= I.
1.4.5. Find the inverse of y =sin x.
Solution. The domain of definition of the function y =sin x is
the entire number scale, the range of the function is the interval
[- 1, l]. But the condition of existence of an inverse function is
not fulfilled.
Divide the x-axis into intervals n:rt-:rt/2~x~n:rt+:rt/2. If n is
even, then the function increases on the intervals n:rt-:rt/2~ x~
~n:rt+:rt/2; if n is odd, the function decreases on the intervals
n:rt-:rt/2 ~ x ~ n:rt + :rt/2. Hence, on each of the indicated intervals
there exists an inverse function defined on the interval [-1. l].
30 Ch. !. Introduction to Mathematical Analysis

In particular, for an interyal - n/2 < x < n/2 there exists an in-
verse function x =arc sin y.
The inverse of the function y =sin x on the interval nn-n/2 <
< x < nn + n/2 is expressed through arc sin y in the following way:
X=(- l)narcsiny+nn (n=O, + 1, +2, ... ).
1.4.6. Find the inverse of the given functions:
(a) y= sin (3x- l) at-(n/6+ 1/3) < x< (n/6+ 1/3);
(b) y=arcsin(x/3) at-3<x<3;
(c) y = 5tog x;
(d) y= 2x(x-ll.
1.4.7. Prove that the function y=(l-x)/(l+x) is inverse to
itself.

1.5. Graphical Representation of Functions


1.5.1. Sketch the graph of each of the following functions:
(a) f (x) = X 4 -2x 2 3; +
2x
(b) f (x) =I + x2 ;
(c) f(x)=sin 2 x-2sinx;
(d) f (x) =arc cos (cos x);
(e) f (x) = V sin x;
(f) f (x) =xi/log x.
Solution. (a) The domain of definition of the function f (x) is the
entire number scale. The function f (x) is even, hence its graph is
symmetrical about the ordinate axis and it is
sufficient to investigate the function at x ~ 0.
Let us single out a perfect square f (x) =
= (x 2 -1 )2 + 2. Since the first summand
(x2 - l) 2 ~ 0, the minimum value of the func-
tion, equal to 2, is attained at the points
-2 -1 o 1 z .x x = 1 (see Fig. 2).
The function f (x) decreases from 3 to 2 on
the closed interval 0 < x < 1 and increases
unboundedly on the open interval 1 < x < oo.
Fig. 2 (b) The domain of definition of the func-
tion f (x) is the entire number scale. The fun-
ction f (x) is odd, therefore its graph is symmetrical about the origin
of coordinates and it is sufficient to investigate the function at
x >-0.
Since f (0) = 0, the graph passes through the origin. It is obvious
that there are no other points of intersection with the coordinate
1.5. Graphical Representation of Functions 31

axes. Note that lf(x)l~l. Indeed, (l-Jxj) 2 ~0or l+x 2 ~2lx!,


whence
2\x\
I~ 1+ x2 =If (x) J.
Since f(x)~O at x~O and f(l)=l, in the interval [O, oo) the
maximum value of the function f (x) equals 1, the minimum value
being zero (see Fig. 3).
y

~~~-'-~-'-~--1-~~.__~...._~-'-___,_x

-2 -1 1 2 J

~--=1+------------------
Fig. 3

and f (x 2 ) > f (x 1 ).
Similarly, we can show that on the interval (1, oo) the function
decreases. Final! y,
f (x) = 2x/( 1 + x2 ) < 2x/x 2 = 2/x,
whence it is clear that f (x) tends to zero with an increase in x.
(c) The domain of definition of the function f (x) is the entire
number scale. The function has a period 2n, that is why it is quite
sufficient to investigate it on the interval (0, 2n], where it beco-
mes zero at the points x = O; x = n; x = 2n.
Writing the given function in the form
f (x) =(I -sin x) 2 - l,
we note that it increases with a decrease in the function sin x and
decreases as sin x increases. Hence, the function f (x) decreases on
the intervals [O, n/2] and [3n/2, 2n], and increases on the interval
[n/2, 3n/2]. Since f (n/2) = - 1, and f (3n/2) = 3, the range of the
function is - 1~f(x)~3 (Fig. 4).
32 Ch. I. Introduction to Mathematical Analysis

(d) The domain of definition of the function is the entire number


scale. Indeed, I cosx I::::;; 1 at any x, hence, arc cos (cosx) has a
meaning. The function f (x) is a periodic one with the period 2n,
hence, it is sufficient to sketch its graph on the interval [O, 2n].
But on this interval the following
!J
equality is true:
3 f(x)=~ x, o::s;;x::s;;n,
z \ 2n - x, n ::::;; x ::::;; 2n.
Indeed, the first assertion follows
--i.--'--;----,,__---:!.___~--a: from the definition of the function
o arc cosx, while the second one can be
-I proved in the following way. Let us
put x'=2n-x, n::s;;x::s;;2n; then
Fig. 4 O::::;; x' ::::;; n and
f(x)=arc cos [cos(2n-x')] =arc cc1s(cosx') =X'=2n-x.
Taking all this into consideration, we draw the graph (see Fig. 5).
(e) The function y = V-sin x is a periodic one with period 2n;
that is why we may confine ourselves to the interval [O, 2n). But
g

Fig. 5

the function is not defined in the whole interval [O, 2n), it is


defined only in the interval [O, n], as in the interval (n, 2n) the
radicand is negative. The graph is symmetrical about the straight
!J

Fig. 6

line x = n/2, as well as the graph y =sin x (see Fig. 6 ). Here we


have an example of a periodic function which does not exist in
the infinite set of intervals.
(f) The domain of definition of the function is
0 < x < 1 and 1 < x < oo.
1.5. Graphical Representation of Functions 33

Reduce the formula to the form


f (x) =xi/log x = xlog.~ 10 = 10.
Hence, the graph of the given function is the half-line y = 10
in the right-hand half plane with the point x = 1 removed (see Fig. 7).
1.5.2. Sketch the graphs of funGtions defi-
ned by different formulas in different inter- !I
vals (and in those reducible to them):
sinx at -n~x~O, 10 . - . . - " - - - - - -
(a)y= { 2 at O<x~l,
l/(x-1) at l<x~4;
-2 at x > 0,
(b) y= { 1/2 at X= 0,
"":'.o+--1---- :i:
-x 3 at x < O; Fig. 7
(c) y=x + V x 2 ;
(d) y=2/(x+ Vx2 ).
Solution. (a) The domain of definition of the function is the
interval [-n, 4]. The graph of the function consists of a portion
of the sinusoid y =sin x on the interval -n ~ x ~ 0, straight line

!f
y

1/2
I
-1
)o :c
0 1
x -f
-7t
-2
Fig. 8 Fig. 9

y = 2 on the interval (0, l] and a part of the branch of the hyper-


bola y= l/(x-1) on the interval (1,4] (see Fig. 8).
(b) The graph of the function consists of a portion of a cubic
parabola. an isolated point and a half-line (see Fig. 9).
(c) The function may be given by two formulas:
2x, if x ~o.
y= {
0, if x < 0.
34 Ch. I. Introduction to Mathematical Analysis

Thus, the graph of our function is a polygonal line (see Fig. 10).
(d) From (c) it follows that the function is defined only in the
interval (0, + oo ), y being equal to 1/x (x > 0). Thus, the graph
of our function is the right-hand part of an equilateral hyperbola
(see Fig. 11 ).
9

Fig. 10 Fig. 11

Fig. 12

the points where cosx < 0, we can sketch the desired graph (the
solid line in the same figure).
(b) The function lx+2lx may be given by two formulas:
{ (x+ 2)x at x ~-2,
Y= -(x+2)x at x~-2.
Plotting separately both parabolas: y=(x+2)x=(x+ 1)2 -1,
and y=-[(x+ 1)2 - I], retain only the parts corresponding to
the above indicated intervals. Drawn in a solid line in Fig. 13 is
the graph of the given function, the broken line showing the de-
leted parts of the constructed parabolas.
1.5. Graphical Represmtation of Functions 35

1.5.4. Sketch the graph of the function


y=2 Ix-21-1x+1 l+x.
Solution. At x ~ 2
Y= 2 (x-2)-(x+ l)+x= 2x-5.
At -1 ~x~2
y= - 2(x-2)-(x+ l)+x= - 2x+3.
y !J

I
I
I
I
I
I
I
I
I f
I
_ __.:..I-l--_,_,~+--1_,...---;.,._ :C
-1 0
-f

Fig. 13 Fig. 14

Finally, at x ~-1

Y=-2(x-2)+(x+ l)+x=5.
Hence, the given function can be rewritten in the following way:
5, x~-1,
y= -2x+ 3, -1 ~x~2,
{ !J
2x-5, x~2.

Therefore the graph is a polygonal line


(see Fig. 14).
1.5.5. Sketch the graph of the function
Y= 2x-2-x.
Solution. Draw graphs for the functions
y 1 = 2x and y 2 = - 2-x (broken lines in
Fig. 15), and add graphically their ordina- Fig. 15
tes. In doing so bear in mind that y 2 y < <
< y1 , and that y 2 tends to zero with an increase in x, whereas y 1
tends to zero with a decrease in x (the solid line in Fig. 15).
1.5.6. Sketch the graph of the function
y=x sin x.
36 Ch. I. Introduction to Mathematical Analysis

Solution. Being the product of two odd functions y1 = x and


y 2 =sin x, the function y is an even one, that is why we shall
analyse it for x;;;;:::: 0.
We draw graphs for y 1 = x and y 2 =sin x (the broken lines in
Fig. 16).
At the points where y 2 =sinx=O, Y=Y 1 y 2 =0, and at the
points where y 2 =sin x = 1, y = y 1 = x. The latter equality
y /
/
/
/
/

Fig. 16 '
indicates the expedience of graphing the auxiliary function y 3 = - x.
Marking the indicated points and joining them into a smooth
curve, we obtain the required graph (the solid line in Fig. 16).
l.5.7. Sketch the graph of the function y=x(x 2 -l) by multi-
plying the ordinates of the graphs y 1 = x and y2 = x 2 -1.
1.5.8. Graph the following functions:
(a) y=x/(x 2 -4), (b) y= l/arc cosx.
Solution. (a) Since the function is odd, it is sufficient to inves-
tigate it for x;;;;:::: 0.
Let us consider it as the quotient of the two functions:
Yi= x and y 2 = x2 -4.
Since at x = 2 the denominator y 2 = 0, the function is not de-
fined at the point 2. In the interval [O, 2) the function Yi increases
from 0 to 2, the function y 2 is negative and I y 2 j = 4-x2 decreases
from 4 to O; hence, the quotient f (x) = Yif Y2 is negative and in-
creases in absolute value, i.e. f (x) decreases in the interval [0,2)
from 0 to -oo.
In the interval (2, oo) both functions are positive and increasing.
Their quotient decreases since from 2 ~Xi < x 2 it follows that
- (X1-X2) (x1x2+4) < 0
Y Y -
2- i--2---2--
X2 Xi
2 4) 2 4)
X2-4 Xi -4 (X2- (Xi -
1.5. Graphical Representation of Functions 37

The indicated quotient tends to zero asx-+ oo, since y = 1 __}__~~x~---+ 0.


The general outline of the graph is presented in Fig. 17 (three
solid lines).
(b) Denote y 1 =arc cos x. The domain of definition of this func-
tion [x[::;:;;;l. At X=l we have y 1 =0, hence, y=l/y 1 -..oo at

!I

!J
',,
I\

I \
1!

lI ' ' 2
I ',
I
I
/ I I
/
/ I I
/ I I
/
/ I
/ I
/ I
/ I -1 0 1
Fig. 17 Fig. 18

x-+ 1, i. e. x = 1 is a vertical asymptote. The function y 1 decreases


on the entire interval of definition [ -1, 1), hence y = l/y 1 incre-
ases. The maximum value y 1 = n is attained at x = - 1. Accor-
dingly, the minimum value of the function is l/n. The solid line
in Fig. 18 represents the general outline of the graph.

Simple Transformations of Graphs


I. The graph of the function y = f (x +a) is obtained from the
graph of the function y = f (x) by translating the latter graph along
the x-axis by [a [ scale units in the direction opposite to the sign
of a (see Fig. 19).
II. The graph y=f(x)+b is obtained from the graph of the
function y = f (x) by translating the latter graph along the y-axis
by [ b I scale units in the direction opposite to the sign of b (see
Fig. 20).
III. The graph of the function y = f (kx) (k > 0) is obtained from
the graph of the function y = f (x) by "compressing" the latter graph
against the y-axis in the horizontal direction k times at k > 1 and
by "stretching" it in the horizontal direction from the y-axis 1/k
times at k 1 (see Fig. 21 ). <
38 Ch. I. Introduction to Mathematical Analysis

IV. The graph of the function y = kf (x} (k > 0) is obtained from


the graph of function y = f (x) by "stretching" it in the horizontal
direction k times at k > 1 and "compressing" it against the x-axis
(i. e. vertically) l/k times at k < 1 (see Fig. 21).
!I
!I
!Jf(x)+b, b>O
,o y=f(x)
/ y=f(x)+b,b<O

Fig. 19 Fig. 20

V. The graph of the function Y= - f (x) is symmetrical to that


of the function y = f (x} about the x-axis, while the graph of the
function y = f (-x) is symmetrical to that of the function y = f (x)
about the y-axis.

!f

---!-,--+------ x
y=f(x) I o
Fig. 21 Fig. 22

VI. The graph of the function y = f ( Ix I) is obtained from the


graph of the function y = f (x) in the following way: for x ~ 0 the
graph of the function y = f (x) is retained, then this retained part
of the graph is reflected symmetrically about the y-axis, thus de-
termining the graph of the function for x ~ 0 (see Fig. 22).
VII. The graph of the function y =If (x) I is constructed from
the graph y = f (x) in the following way: the portion of the graph
1.5. Graphical Representation of Functions 39

of the function y = f (x) lying above the x-axis remains unchanged,


its other portion located below the x-axis being transformed sym-
metrically about the x-axis (see Fig. 23).
VIII. The graphs of the more complicated functions
y=/...f (kx+a)+b
are drawn from the graph of y = f (x) ap- !J
plying consecutively transformations I to V.
1.5.9. Graph the function
Y= 3V--2 (x+ 2.5)-0.8
by transforming the graph y = VX.
Solution. Sketch the graph of the function
I
y= Vx (which is the upper branch of the I
parabola y 2 = x) (Fig. 24, a), and transform I
I !f=f(:c)
it in the following sequence.
Sketch the graph of the function y=3Vr2x Fig. 23
by enlarging 3 V2 times the ordinates of
x
the points on the graph of the function y = JI and leaving their
abscissas unchanged (see Fig. 24, b).
Then sketch the graph of the function y = 3V - 2x which will
be the mirror image of the preceding graph about the y-axis (see
Fig. 24, c).

!J

!J

x -::+-:~--l-3:
(a) 0 f/2 (b)

Fig. 24

By shifting the obtained graph 2.5 scale units leftward and then
0.8 unit downward draw the desired graph of the function
y=3V-2(x+2.5)-0.8 (see Fig. 24, d).
1.5.10. Graph the function y=3cosx-V3sinx by transforming
the cosine curve.
40 Ch. I. Introduction to Mathematical Analysis

Solution. Transform the given function

y=3cosx-V"3sinx=2V~3(v;- cosx- ~ sinx )=


= 2 V3 cos ( x + ~) .
Thus, we have to sketch the graph of the function
Y= 2V3cos (x+ n/6),
which is the graph of the function y= 2 V3 cosx
translated by n/6
leftward. The function has a period of 2n, hence it is sufficient to
draw its graph for -n :::;;;;x:::;;;; n
!J (see Fig. 25).
2'/J

!J

-t
Fig. 25 Fig. 26

The graph of any function of the form y =a cos x + b sin x, where


a and b are constants, is sketched in a similar way.
1.5.11. Graph the following functions:
x+3
(a) Y== x+ Ii
I
(b) y==~gi
x-
(c) { x2 +x+ 1, if -1:::;;;;x<0,

y= Stn X, 2
if 0:::;;;; x:::;;;; n,
(x-1)/(x+ 1), if n < x :::;;;;5;
(d) y=x+ 1/x;
(e) y=x2 -x3 ;
(f} y=x+sinx;
(g} y= 1/cosx;
(h} y= 3 sin (2x--4);
O> u=2V -3<x+ 1.5)- t.2;
1.6. Number Sequences. Limit of a Sequence 41

(j) y= lx 2 -2x-l I;
(k) y= 11x1-11:
(I) y =cos (sin x);
(m) y =I sin x I+ sin x on the interval [0,3 n];
1 at x > 0,
(n) y = x 2 sign x, where sign x = { 0 at x = 0,
-1 at x < 0.
1.5.12. The function y = f (x) is given graphically (Fig. 26).
Sketch the graphs of the following functions:
(a) y=f (x+ l);
(b) y = f (x/2);
(c) Y =If (x) I;
(d) y =(If (x) I f (x))/2;
(e) Y =If (x) l!f (x).

1.6. Number Sequences. Limit of a Sequence


The number a is called the limit of a sequence x1 , x2 , , Xm
... as n-+oo, a= lim xn if for any e > 0 there exists a number
n-+"'
N (e) > 0 such that the inequality lxn-a I< e holds true for all
>
n N (e).
A sequence which has a finite limit is said to be convergent.
A sequence {xn} is called infinitely small if lim Xn = 0, and infi
nitely large if lim Xn =co.
1.6.1. Given the general term of the sequence {xn}:
sin (nn/2)
Xn= n .

Write the first five terms of this sequence.


Solution. Putting consecutively n = 1, 2, 3, 4, 5 in the general
term Xm we obtain
sin (n/2) l
X1= I = ;

sin (2n/2) O,
X2= 2
sin (3n/2) 1
Xa= 3 =-3;
sin (4n/2) O
X4= 4 = ;

sin (5n/2) I
x.= 5 =5
42 Ch. I. Introduction to Mathernatical Analysis

1.6.2. Knowing the first several terms of the sequence, write one
of the possible expressions for the general term:
2 5 10 17 26
(a) 3' 8' T3' 18' 23;
1 1 I 1
(b) 1, 2' 2, 3' 3, 4' 4, 5
Note. A knowledge of the first several terms of a sequence is not
sufficient to define this sequence. That is why this problem should
be understood as one of finding a certain simple inductive regula-
rity compatible with the given terms.
Solution. (a) Note that the numerator of each of the given terms
of the sequence equals the square of the number of this term plus
+
unity, i.e. n 2 1, while the denominators form the arithmetic prog-
ression 3, 8, 13, 18, ... with the first term a 1 = 3 and the com-
mon difference d = 5. Hence,
an=a1 +d(n-1)=3+5 (n-1) =5n-2,
thus we have

(b) Here the general term of the sequence can be written with
the aid of two formulas: one for the terms standing in odd places,
the other for those in even places:
k atn=2k-l,
{
Xn = l/(k+ 1) at n = 2k.
It is also possible to express the general term by one formula,
which will be more complicated, for instance,

Xn = nt I [1-(-l)n] + n!2 [l + (-l)n].


1.6.3. Find the first several terms of the sequence if the general
term is given by one of the following formulas:
(a) Xn =sin (n:rt/3);
(b) Xn = 2-n COS n:rt;
(c) Xn =(l +
l/n)n.

1.6.4. Using the definition of the limit of a sequence, prove that


(a) Jim Xn = 1 if Xn = (2n- l)/(2n + 1),
(b) Jim Xn = 3/5 if Xn = (3n 2 + l)/(5n 2 - 1). Beginning with which
n is the inequality I Xn-3/5 I< 0.01 fulfilled?
Solution. (a) For any e > 0 let us try to find a natural number
N (e) such that for any natural number n > N (e) the inequality
IXn- l I < B is fulfilled.
1.6. Number Sequences. Limit of a Sequence 43

For this purpose let us find the absolute value of the difference

I2n+l-
2n-1 11 = I2n+I
-2 I 2
=2n+1
Thus, the inequality IXn-1 I < 8 is satisfied if 2n ~ 1 < 8, whence
n > l/e- 1/ 2 Hence the integral part of the number l/8- 1 / 2 may
be taken as N(e), i.e. N=E(l/e- 1/ 1 ).
So, for each e > 0 we can find a number N such that from the
inequality n > N it will follow that I Xn-1 I< 8, which means that
. 2n-1
hm 2n +1=1.
n-+rn

(b) Let us find the absolute value of the difference lxn-3/51:


3n2+ I 31 8
15n2-I -5 = 5 (5n2- I)

Let 8 >0 be given. Choose n so that the inequality


8
5 (5n2 - I) <8'
is fulfilled.
Solving this inequality, we find
2 8 +I. n > ..!... ... /8+5e
n > 25e 5' 5 V e
Putting
N=E (; y-s~5g)'
we conclude that at n >N
lxn-3/51<8,
which completes the proof.
If e = 0.01, then

N =E (; y 8 ~ 511 ) =E(; Vso5) =5,


and all terms of the sequence, beginning with the 6th, are contai-
ned in the interval (3/5-0.01; 3/5+0.01).
l.6.5. Given a sequence with the general term Xn=;~+!. It is
known that lim Xn = 1/3. Find the number of points Xn lying out-
n -+ "'
side the open interval

L= (-}-1 o~o; ~ + 1~oo)


44 Ch. I. Introduction to Mathematical Analysis

Solution. The distance from the point x 11 to the point 1/3 is


equal to
Ix,,-3 I II
=
19
---3 (9n+4)
I=a 19
(911+4)"

Outside the interval l there will appear those terms of the se-
quence for which this distance exceeds 0.001, i.e.
19 I
3(9n+4) > 1000'
whence
18 988 7
1~ n <2 7= 703 v.
Hence, 703 points (x 1 , x2 , , X703 ) are found outside the inter-
val l.
1.6.6. Prove that the number l = 0 is not the limit of a sequen-
ce with the general term x11 =(n2 -2)/(2n 2 -9).
Solution. Estimate from below the absolute value of the diffe-
rence
In2-2
2n 2 - 9 - 0 =
I J n 2 -2 \
\ 2n 2 -9 J

At n ~ 3 the absolute value of the difference remains greater


than the constant number 1/ 2 ; hence, there exists such e > 0, say,
e = 1 / 2 , that the inequality

l2~2 ~ - 0 1>+
holds true for any n ~ 3.
The obtained inequality proves that l = 0 is not the limit of the
given sequence.
1.6. 7. Prove that the sequence
I I 2 I 3 I 4
l, 2 3 3 5 4 7 5
with the general term
l/n, if n=2k- 1,
{
Xn = n/(n + 2), if n = 2k,
has no limit.
Solution. It is easy to show that the points x 11 with odd num-
bers concentrate about the point 0, and the points x,, with even
numbers, about the point 1. Hence, any neighbourhood of the point 0,
as well as any neighbourhood of the point 1, contains an infinite
set of points x11 Let a be an arbitrary real number. We can always
choose such a small e > 0 that the e-neighbourhood of the point a wi II
1.6. Number Sequences. Limit of a Sequenc't' 45

not contain at least a certain neighbourhood of either point 0 or


point l. Then an infinite set of numbers xn will be found outside
this neighbourhood, and that is why one cannot assert that all the
numbers xn, beginning with a certain one, will enter the e-neigh-
bourhood of the number a. This means, by definition, that the
number a is not the limit of the given sequence. But a is an arbit-
rary number, hence no number is the limit of this sequence.
1.6.8. Prove that limxn=l if Xn=(3n+l)/3n.
Prove that Jim xn = 2 if xn = (2n + 3)/(n + 1). Find the
1.6.9.
number of the term beginning with which the inequality
1(2n+3)/(n+l)-2l<e, where i-:=0.l; 0.01; 0.001, is fulfilled,
1.6.10. Prove that the sequence
3 7
2 2 4 4 8 a
with the general term

If 1 - I
2Cn+l)/2
if n is oddJ
Xn= { I
l 2n/2
if n is even,
has no limit.
1.6.1 I. Prove that at any arbitrarily large a> 0 Jim xn = 0 if
Xn =an/n!
Solution. Let a natural number k > 2a. Then at n > k

~~ = T' ~ . ~ ~.~ T) (k~ I . k~2. ~) <


+r- r.
= ( ..

< ak ( k = (2a)k ( ~

have:
lim (an/n!)
({ r
Since Jim (l/2)n=0 (prove it!), then at a sufficiently large n we

= 0.
< ,2:)k and, hence, an/n! < E, which means that

1.6.12. Test the following sequences for limits:


(a) Xn = l/(2n);
1 for an even n,
(b) x -- {
n - 1/n for an odd n;
I n:n
(c) x11 =-ncos T;
(d) x,,=n[l-(-1)"].
46 Ch. I. Introduction to Mathematical Analysis

1.6. 13. Prove that the sequence with the general term
Xn = l/nk (k > 0)
is an infinite-!y small sequence.
Solution. To prove that the sequence xn is infinitely small is to
pro\e that lim Xn = 0.
n - ""
Take an arbitrary 8 > 0. Since I xn I= l/nk, we have to solve the
inequality
l/nk < e,
whence n > V
l/8. Hence N may be expressed as the integral part
of V -1/8, 1.
. e. N = E (k/-)
v l/8 .
1.6.14. Prove that the sequences with the general terms

(a) Xn= 1-(-1~


n , I
(b) Xn=-nsin [ (2n-1)2
n]

are infinitely small as n - oo.


1.6.15. Show that the sequence with the general term
xn = (-l)n 2/(5 Vn+ 1) is infinitely small as n - oo. Find a num-
ber N beginning with which the points xn belong to the interval
(-1/10, 1/10).
Solution. Take an arbitrary 8 >0 and estimate Ixn I:

lxnl=
5
v2

'' +- I
< 5 v-<
n 2
2
av-= v-
2

n
I

n
Ixn I< 8 a; soon as n > l/8
That is why
the sequence is infinitely small.
We take now 8= 1/10. Since I xn I<
" ""
3

I!Vn.
xn will necessarily
Hence Jim Xn
- = 0, i.e.

be smaller than 1/10 if l/Vn


< 1/10 or n > 1 000. Hence N may
be taken equal to 1 000. But we can obtain a more accurate result
by solving the inequality
2 I
lxnl=
5
V n+l <w
It holds true at n > (19/5) 3 = 3.8 3 = 54.872. Hence N may be
taken equal to 54 ~ 1 000.
1.6 .16. It is known that if Xn =a+ an, where an is an infinite-
si mal as n--+- oo, then Jim xn =a. Taking advantage of this rule,
find the limits:
3n+ 1 +sin (nn/4)
(a) Xn = 3n ;
1.6. Number Sequences. limit of a Sequence 47

Solution. (a ) Xn = 3n+ t +sin


3n
(n:rc/4)
=
+
3 an, W here an = sin (n:rc/4) .
3n IS
an infinitesimal as n- oo, hence lim Xn = 3.

t.6.17. Prove that lim Vn=l.


n-+ "'
Solution. Let us prove that the variable }:In
can be represented
as the sum I +an, where an is an infinitesimal as n - oo.
Let us put }:In= 1 +an. Raising to the nth power we obtain
- ( l + an)n-
n- - 1 + nan + n (n-1)
21 an + . . . + ant
n2

wherefrom we arrive at the conclusion that for any n > l the fol-
lowing inequality holds true:
n>
l +n(n-1) 2
21 an
(since all the terms on the right are non-negative). Transposing the
unity to the left and reducing the inequality by n-1 we obtain
n 2
l >2a",

whence it follows that 2/n > a~ or V 2/n > an > 0. Since


lim V 2/n = 0, lim an also equals zero, i. e. an is an infinitesimal.
n-+<: n-+oo
Hence it follows that
lirn
11- 00
V/n= 1.
1.6.18. Prove that the sequence with the general term

is infinitely large as n--+ oo.


Solution. Let us take an arbitrary positive number M and solve
the inequality
3V; >M.
Taking the logarithm, we obtain
Vn> log,, M, n >(log. M) 3
If we now take N = E (log 3 M)3, then for all n > N the inequa-
lity IXn I > M will be fulfilled, which means that the sequence is
infinite! y large.
1.6.19. Prove that
lim Va- 1 (a > 0).
n-+ "'
48 Ch. I. Introduction to Mathematical Analysis

1.7. Eivaluation of Limits of Sequences


If the sequences {xn} and {yn} are convergent, then
(I) lim (xn Yn) = lim Xn lim Yn;
(2) Jim (XnYn) =Jim Xn lim Yn;
(3) lim Xn = 11 ~m Xn (lim Yn =I= 0).
Yn 1myn
If Xn ~ Yn then lim Xn ~ lim Yn
1. 7 .1. Find lim Xn if
n-+""

( d) Recall that
l2+ 22 + 32 + ... + n2 = n (n+ 1~(2n+ I).
Hence

1.7.2. Find Iim xm if


n-+""
I .7. Emluation of limits of Sequences 49

(c) In solving this example, and also the rest of the examples
of Problem 1.7.2, take advantage of the following equalities (see
Problems 1.6.17 and l.6.19):
Jim
n-~
V' n = 1 and lim
n-+oc
V' a - 1. ( 1)
We have
lim xn = lim V' 5n = JY5 lim V' n,
lim
n-+ "'

but from (1) it follows that Jim V's 1 and lim V'"n l; hence
Jim Xn = 1 1 = 1.
1.7.3. Find
. ' 2n3 1-Sn2 )
}~tn,., \2n2+3 +Sn+ I

Solution. Summing the fractions, we obtain


2n 3 -13n 2 +3
Xn = 10ns+2n2+ 1Sn+3
Whence

Note. If we put
2n 3 1-Sn2
Yn = 2n2 + 3 ; Zn= Sn+ I '
then the limit of their sum Jim (Yn +Zn)= 1/5, though each of the
summands is an infinitely large quantity. Thus, from the conver-
gence of a sum of sequences it does not, generally speaking, follow
that the summands converge too.
l. 7 .4. Find lim Xn if
50 Ch. I. Introduction to Mathematical Analysis

Solution. (a) Xn=Vn(V2+3/n-Vl-l/n)-+ +oo as tt-+ oo,


since the second multiplier has a positive limit.
n 2 (n-Yn 2 +1) -n 2
(c) xn-- I
- ----
-n+Y1i2+T
I
=-n __ -+-oo as n-+ oo.
I+ y1+~2

I ) I ) a
( -;i--1 2/ 3
-
(
n-1 1/
+1
It means, Xn--+ 1/3.
(e) Factoring out the terms of the highest power in the numera-
tor and denominator, we have:

1. 7 .5. Find lim Xn if


n-oo

1.8. Testing Sequences for Convergence


Bolzano-Weierstrass' theorem. A monotonic bounded sequence has
a finite limit.
Theorem on passing to the limit in inequalities. If xn ~ Yn ~ zn
and Jim xn = lim Zn = c, then lim Yn = c too (c is a number,
oc
+ oo
fl -

or - oo but not oo ).
1.8. Testing Sequences for Convergence 51

1.8.1. Prove that the sequence with the general term xn =


= +
(2n- l)/(3n 1) is an increasing one.
Solution. We have to prove that Xn+l > Xn for any n, i.e. to
prove that
2n+ I 2n- I
3n+4 > 3n+ I
The latter inequality is equivalent to the obvious inequality
6n2 +5n+ l > 6n 2 +5n-4.
Hence, xn+t > Xn-
1.8.2. Given a sequence with the general term
10n
Xn=nr.

Prove that this sequence decreases at n ~ 10.


Solution.
IQn+ I !On JO 10
Xn+t = (n+ 1)1 = nf n+ I =Xn n+ I '

Since n~ 1 < 1 at n ~ 10, then Xn+t < xn beginning with this


number, which means that the sequence decreases at n ~ 10.
1.8.3. Test the following sequences for boundedness:
5n 2
(a) Xn=n2+3;

(b) Yn = (- l)n n~l sin n;


(c) Zn= n cos nn.
Solution. (a) The sequence {xn} is bounded, since it is obvious that
5n 2
0 < n 2 + 0 < 5 for all n.
(b) The sequence {Yn} is bounded:

IYn I= J(-l)n J n~ 1 I sin n I< n~ I< 2.


(c) The sequence {zn} is not bounded, since
I Zn J = In C<JS nn J = n.
1.8.4. Prove that the sequence
x-~ t
X = Xn-1
1-a+xo' n a+ Xn-1 '
(a> l, x0 > 0) converges.
Solution. Let us prove that this sequence is monotonic and
bounded. Firstly, xn < xn_ 1 as

Xn
Xn-1
= a+ < Xn-1'
Xn-1
52 Ch. I. Introduction to Mathematical Analysis

Hence, the given sequence is a decreasing one. Secondly, all its


terms are positive (by condition a> 0 and x 0 > 0), which means
that the sequence is bounded below. Thus, the given sequence is
monotonic and bounded, hence it has a limit.
1.8.5. Prove that the sequence with the general term
I l I I
= 5 + I + 52 + I + 5a + I + ' ' ' + 5n + I
Xn
.
( 1.e. I I I I I I )
X1=5+1; X2=5+1+5 2 +1; Xa=5+1+5TTf+5a+1;
converges.
Solution. The sequence {xn} increases, since xn+i = xn + l/(5n"" 1 -:- 1)
and, hence, Xn+i > Xn Besides, it is bounded above, since l/(5n+ l) <
< 1/5" at any n and
I I I I
Xn = 5 + I + 52 + I + 5a + I + ' ' ' + 5n + I <
I I I I 1/5- l/5n+1 I ( I) I
<s+52+5a++5n= J-1/5 =4 l-5,, <4
Hence, the sequence converges.
1.8.6. Taking advantage of the theorem on the existence of a
limit of a monotonic bounded sequence, prove that the following
sequences are convergent:
n2-I
(a) Xn=~;
I
(b) Xn = 2 + 21 + 3JI + ... + lllI .
1.8. 7. Prove that the following sequences converge and find their
limits:
(a) x1 =V2; x 2 =V2+V2;

X3 = ( 2+ V 2 + V 2; . , Xn = }/ 2 + V 2 + ... + V2; ... ,


'--------
n radicals
2"
(b) x,, = (n+2)1 ;
E (ny).
(c) Xn=-n-'
(d) the sequence of successive decimal approximations l; 1.4;
l.41; 1.414; ... of the irrational number J.F2;
(e) Xn = n!/nn.
Solution. (a) It is obvious that x1 < x 2 < X3 < ... < Xn <
< x 11 + 1 < ... , i.e. the sequence is increasing. It now remains to
prove that it is bounded.
1.8. Testin{!, Sequences for Convergence 53

We have Xn=Jl2-txn-u n=2, 3, ... Since x 1 =V2< 2


x2 =V-2-\-x1 <V2-f-2=2, X 3 =V2-tx 2 <V'2+2=2, .... Let it
be proved that x,,_ 1 <2. Then x 11 =V2-tx11 _ 1 < V2+2=2. Thus,
with the aid of mathematical induction we have proved that x,, < 2,
i.e. the sequence is bounded. Hence, it has a finite limit. Let us
find it. Denote
Jim Xn=Y

Then, Xn= V2+xn_ 1 ; raising to the second power, we obtain


X~=2+xn-l'
Passing to the limit, we can rewrite this equality as follows
Iimx~= lim (2+xn_ 1 ), or y 2 =2+y.

The roots of the obtained quadratic equation are:

Y1 = 2; Y2 = -1.
The negative root does not suit here, sincexn>O. Hence, lim Xn=Y 1 =2.
<I-<

(c) We have ny- 1 < E (ny) ~ ny or y-J_ < E n(ny) ~ y. But the
n
sequences {u-*} and {Y} converge, their limit being y, that is
why Jim Xn = y.
n- oo
(d) This sequence is non-decreasing, since each following term
Xn+i is obtained from the preceding one Xn by adding one more
significant digit to the decimal fraction. The sequence is bounded
above, say, by the number 1.5. Hence, the sequence converges, its
limit being V2.
(e) The sequence decreases monotonically. Indeed,
(n+ 1)1 n! n! nn nn
Xn+l = (n+ J)n+1 (n+ J)n = nn (n+ J)n = (n+ l)n Xn

nn
Since (n+ l)n < 1, Xn+l < Xn.
Then, since Xn > 0, the sequence is bounded below, hence lim xn
exists. Let us denote it l. Obviously, l = Jim xn ~ 0. Now let us
n- a;,
show that l = 0. Indeed,

(n~})n =(n~I r =(l+ ~r~ 1-tn ~ =2.


54 Ch. I. Introduction to Mathl'matical Analysis

nn
Hence, (n+ l)n < 2I and Xn+i <2
I
xn. Passing over to the limit,
we obtain
I
[~ 2l

which, together with l ~ 0, brings us to the conclusion:


[=0.
1.8.8. Find the limits of the sequences with the following gene-
rat terms:
n n
x = . z = ;
n vn 2 +n' n n 2 +1
I I I
Yn= ,.r::o-;--;-
r n-+1 + .rr n 2 +2 + + .r
r n +n 2
Solution. Let us prove that Jim xn = 1. Indeed,

1 I= I n- y;zq:-;i
Y n2 +n
I=
n <_!__
Y n2+n (n+ Y n2+n) 2n
We can prove similarly that
Jim Zn= 1.
Then,
I I I n
Yn< Yn 2+1 + Yn2+1 +" '+
~~~~~-,--~~~~--"'
Yfi2+1 = Yn 2 +1 =Zn
n
On the other hand,
I I n
Yn > .r
r n2+n + .rr n +n + + .rr n2+n
2

n
Thus,

and according to the theorem on passing to the limit in inequalities


Jim Yn = 1.

1.8.9. Using the theorem on passing to the limit in inequalities


prove
Jim fla= 1 (a> O).
n .... "'
1.9. The Limit of a Function 55

1.8.10. Prove the existence of the limit of the sequence Yn = a 1 12 n


(a> 1) and calculate it.
1.8.11. Taking advantage of the theorem on the limit of a
monotonic sequence, prove the existence of a finite limit of the
sequence
I I I
Xn= l +p+32+. . +fi2.
1.8.12. Taking advantage of the theorem on passing to the limit
in inequalities, prove that
Jim Xn=l if Xn=2n(Vn 2 +l-n).
n - oo

1.8.13. Prove that the sequence


Xi= Va; X2 o:c -v a+ v a;
x3 = (-a-+-:v::--;r=a=+=V=a; ... , xn = {-a-+--=Vr=a=+=.=.=.=+=V=a
'--~~--,-~~----
n radicals
(a> 0)
has the limit b=(V4a+l+l)/2.
1.8.14. Prove that the sequence with the general term
I I I
Xn = 3 + I + 32 + 2 + + 3n + n
has a finite 1imi t.
1.8.15. Prove that a sequence of lengths of perimeters of regular
2n-gorn inscribed in a circle tends to a limit (called the length of
circumference).

1.9. The Limit of a Function


A point a on the real axis is called the limit point of a set X
if any neighbourhood of the point a contains points belonging to X
which are different from a (a may be either a proper or an impro-
per point).
Let the point a be the limit point of the domain of definition
X of the function f (x). The number A is called the limit of the
function f (x) as x ___,.a, A = Jim f (x), if for any neighbourhood V
x-a
of the number A there exists a neighbourhood u of the number a
such that for all x EX lying in u, f (x) EV (the definition of the
limit of a function after Cauchy). The number A may be either
finite or infinite. In particular, if the numbers A and a are finite
we obtain the following definition.
56 Ch. I. Introduction to Mathematical Analysis

A number A is called the limit of a function f (x) as x - a,


A= Jim f (x), if for any e > 0 there exists a number cS (e) > 0 such
"-+a
that for all x satisfying the inequality 0 < /x-a / < cS and belon-
ging to the domain of definition of the function f (x) the inequality
If (x)-A j < e holds true (the "e-cS definition").
If a=+ oo, the definition is as follows. A number A is called
the limit of a function f (x) as x -+
oo, A= lim f (x), if for any
x-+ + 00
e > 0 there exists a number M (e) > 0 such that for all x satisfying
the inequality x > M (e) and belonging to the domain of definition
of the function f (x) the inequality If (x)-A I< e holds true (the
"e-M definition").
The notation lim f (x) = oo means that lim If (x) / =
x ... a
oo. The
x-+ a
+
rest of the cases are considered similarly.
The definition of the limit of a function after Heine. The nota-
tion lim f (x) =A means that for any sequence of values of x con-
" -+a
verging to the number a

(belonging to the domain of definition of the function and differing


from a) the corresponding sequence of values of y
Y1 = f (X1); Y2 = f (X2); ; Yn = f (xn),
has a limit, which is the number A.
1.9.1. Taking advantage of the definition of the limit after
Heine (i.e. in terms of sequences) and of the theorems on the limits
of sequences, prove that
1. 3x+I I
}~\ 5x+4 = 2'
Solution. Let us consider any sequence x1 , x2 , satisfying
the following two conditions: ( 1) the numbers x1 , x2 , belong
to the domain of definition of the function f(x)=(3x+ l)/(5x+4)
(i.e. Xn =F-4/5); (2) the sequence {xn} converges to the number 2,
i.e. lim Xn = 2.
n -+""
To the sequence {xn} there corresponds the sequence of values
of the function

proceeding from the theorem on the limits ( 1. 7),


1. f 3xn+ I lim (3xn+ I) 6+ I I
Im hm 5 +4=r (5 +4)=10+4=2
(xn}= n-+
n-+ "" "" Xn 1m Xn
I .9. The Limit of a Function 57

Thus, independently of the choice of a sequence {xn} which


converges to the number 2 (xn ~ -4/5), the corresponding sequences
of values of the function f (xn) converge to the number 1/2, which,
according to the definition of the limit of a function, means that
I" 3x+l I
x:~ 5x+4=2
Note. The definition of the limit after Heine is conveniently
applied when we have to prove that a function f (x) has no limit.
For this it is sufficient to show that there exist two sequences {x~}
and {x~} such that lim x~ = Jim x~ =a, but the corresponding
n-oo n-+oo
sequences {f (x~)} and {f (x~)} do not have identical limits.
t.9.2. Prove that the following limits do not exist:
(a) lim sin - 1- 1 ; (b) Jim 2*; (c) lim sin x .
.t - I X- x-+ 0 .t-+ oo

Solution. (a) Choose two sequences


Xn=l+ n~ and X~=l+(4 n~l)n (n=l, 2, ... ),
for which
lim Xn = lim x~ = 1.

The corresponding sequences of values of the function are:

f (xn) =sin 1 +l/(~n)_ 1 =sin nn = 0


and
f (Xn.) =Stn
. I . 4n I . ( n ) l
I+ 2/[( 4n+l)n]-l =sm-2- n=sm 2nn+-~i = .
+
Hence,
lim f (xn) = 0 and Jim f (x~) = l,
Kn -+ I x~ -+ J

i.e. the sequences {f (xn)} and {f (x~)} have different limits, whence
it follows that Jim sin x~ 1 does not exist.
x- I
(c) Choose two sequences, xn = nn and x~ = 2nn + n/2 (n = 1,
2, ... ), for which lim xn = Jim x~ = oo. Since
n-+oo n-+oo

lim sin Xn = Jim sin nn = 0,


fl-+OO n-+ao
and
Jim sin x~ = Jim sin (2nn + n/2) = 1,
n-+<:JJ n-~

Jim sin x does not exist.


58 Ch. I. Introduction to Mathematical Analysis

Note. The above examples show that one cannot draw the con-
clusion about the existence of the limit of a function proceeding
from the sequence of values of x of a particular form (for example,
proceeding from Xn = 1+2/((4n + 1) n) in the item (a) of this prob-
lem), but it is necessary to consider an arbitrary sequence x1'
x 2 , , Xn, having a given limit.
I .9.3. Proceeding from the definition of the limit of a function
after Cauchy (i.e. in the terms of "e-cY'; "e-M", etc.), prove that
(a) lim (3x-8) = -5;
JC-> I

1. 5x+I 5.
(b) Im 3 ...L.9=3'
x-+oo X'

(c) li~I (I _!x)2 = + oo;


(d) Jim loga x = oo (a> l);
(e) lim arc tan x = n/2;
(f) lim sin x = 1/2.
JC ... :t/6

Solution. (a) According to the "e-cS" definition we are to prove


that for any e > 0 there exists cS > 0 such that from the inequality
lx-ll<cS it follows that Jf(x)-(-5)J=lf(x)+5J<e.
In other words, it is necessary to solve the inequality
J3x-8+51 =3 lx-1 I< e.
The latter inequality shows that the required inequality If (x)+5 I <e
is fulfilled as soon as Ix-1 I< e/3 =IS. Hence, lim (3x-8) = -5.
JC-+ I
(b) According to the "e-M" definition of the limit one has to
show that for any 8 > 0 it is possible to find a number M > 0
such that for all x > M the inequality

1 3x+9
5x+l_~I< 8
3
(*)
will be fulfilled.
Transforming this inequality, we obtain

'
5x+I_~, 14
3x+9 3 j 3x+9 I < 8
Since x > 0, it remains to solve the inequality
14
3x+9 < 8
J.9. The Limit of a Function 59

whence

x
> ~
14-98.

14-98
hence M=~
14-98
Thus, for e > 0 we have found M = ~ such that for all
values of x >M the inequality ( *) is fulfilled, and this means that
I' 5x+ I 5
tm 3X +9=3
x -+oo

14-0.09 2
Let, for example, e=0.01; then M= o.o3 =463 3 .
(c) We have to prove that for any K > 0 there exists fl> 0
such that from the inequality
/x-1/<fl
there always follows the inequality

/ (l~x)2 I= > K.
(I I x)2

Let us choose an arbitrary number K > 0 and solve the inequality


I '-, K (**)
(l-x) 2 "' '
whence
l
/ 1-x I< VR (K > 0).
Thus, if we put fl= ;K , then the inequality (**) holds true as

soon as Ix- 1 I < fl, which means that !i~n 1 (I ~x) 2 = + oo.
(d) We have to prove that for any K > 0 there exists M > 0
such that from the inequality x > M there always follows the ine-
quality loga x > K. Let us choose an arbitrary number K > 0 and
consider the inequality 1ogax>K. If we put aK=M, then at
x > M the inequality loga x > K holds true. Hence,
Jim logax=
x-+CD
+ oo.
1.9.4. Prove that Jim cos x does not exist.
X-+ oo
1.9.5. Using the sequences of the roots of the equations sin (l/x)= 1
and sin ( l/x) = - 1, show that the function f (x) =sin (1/x) has no
limit as x - 0.
60 Ch. I. Introduction to Mathematical Analysis

1.9.6. Proceeding from Cauchy's definition of the limit of a


function prove that:
. x-1
(a) Jim (3x-2) = l; (b} I1m .r- = 2;
-+ I x ... 1 r x-1
(c) Jim sin x = 0; (d) Jim cosx= I;
" .... 0 x ... 0

(e)
I.
im
2x-I 2
3 +2=3;
x -++oo X
(f) Jim ax=+ oo (a> l);
x- + 00

. m
(g ) II sinx O
-=.
X-+ CD X

1.10. Calculation of Limits of Functions


I. If the limits lirn u (x) and lim v (x) exist, then the following
r .... a X-+Q
theorems hold true:
( 1) Jim [u (x) + v (x)J =Jim u (x) +Jim v (x);
X-+a x-a x-a
(2) Jim [u (x) V (x)] =Jim u (x) -Jim v(x);
lim u (x)
. u (x)) = 1rh a ( ) (I'1m v (x) =/= O) .
(3) 11m - (
" ... a v x im v x x ... a
X-+Q

II. For all main elementary functions at any point of their do-
main of definition the equality Jim f (x) = f (lim x) = f (a) holds true.
X-+Q X-+a
Ill. If for all values of x in a certain neighbourhood of a point a
(except for, perhaps, x =a) the functions f (x) and qi (x) are equal
and one of them has a limit as x approaches a, then the other one
has the same limit.
IV. The following limits are frequently used:
. m
(1) I1 sinx l
-=;
X-+ 0 X

(2) lim(l + l/x)x=Jim(l +a) 1'a=e=2.71828 ... ;


x-+oo a-o
(3) I .tm loga (I +x) = Iogae (a> O; a=/= l);
..... 0 x
(4 ) Jim In (I +x) = l;
x-+0 x
aX-1
(5) Jim --=Ina (a>O).
X-+0 X
I.JO. Calculation of Limits of Functions 61

1.10.1. Find the limits:


. 4x 0 +9x+7 (b) lim x 3+3x2-9x -2.
(a) !~13x 6 +xa+1; x- 2 xa-x-6 '
(c) Jim x+ I ; I. xq
(d) ..1!!1i
xP-1
_ 1 (p an
d
q integers);
x _,. - 1 V 6x~ + 3 +3x
. V9+5x+4x2-3
(e ) 1tm (f) liill
V-
10 - ; - 2 ;
;
X-+ 0 X X-+ 2 X-

()I . JIX+7-3 V2X=3 ~ J;


g tm2 V x+6-2 V3x-5
x-+
3 ; (h) liill [Ioga
x-3 x+6-3
. . x3-x 2-x+ I Vx+S- .r
V8x+1 .
hmI xa - 3x +2
(1) X-+ ; (J') 1till.~
x- 1 r 5-x- r 7x-3

Solution. (a) Since there exist limits of the numerator and deno-
minator and the limit of the denominator is different from zero,
we can use the theorem on the limit of a quotient:

(b) The above theorem cannot be directly used here, since the
limit of the denominator equals zero as X-+ 2. Here the limit of
the numerator also equals zero as x-+ 2. Hence, we have the
indeterminate form ~ . For x =I= 2 we have
xa+3x 2 -9x-2 (x-2) (x 2 +5x+ I) x2+5x+ l
xs-x-6 = (x-2) (x 2 +2x+3) = x2 +2x+ 3

Thus, in any domain which does not contain the point x = 2 the
functions

are equal; hence, their limits are also equal. The limit of the
function qi (x) is found direct! y:
. . x2+5x+ I 15.
hill
x-+2
qi(x)=hill
x_,.zX
2 + + =n
2x 3

hence,
. f ( ) 1. xa+3x2-9x-2 15
Itill
X-+2
X=till
X-+2 X
3__ 6
X
=n
62 Ch. I. Introduction to Mathematical Analysis

(c) Just as in (b), we remove the indeterminate form %by


transforming
lim x+I _ lim (x+l)(Y~-3x)
x .... -1 Y6x~+3+3x x--1 3-3x2
= lim V6X2+3-3x -1
x--1 3 (1-x) - '
1.10.2. Find the limits:

(a)!~~
. (xa x2)
3xi=4-3x+2 ;
(b) lim (V 9x2 + l-3x);

( ) I. 2 Vx+3 V-;+5 V-;.


cim,~v'
x-+"' r 3x-2+ 2x-3
(d) lim (V2x 2 -3-5x);
(e) lim x(V x 2 +I - x);

(g) lim 52x/(x+3l.

Solution. (a)!~~ xt
(3 4- 3xx: 2 )
Here we have the indeterminate form oo - oo; let us subtract the
fractions
I. ( x3 x2 ) 1. 2x3 +4x2
Im 3x2-4-3x+2 = Im 9x3 +6x2-12x-8=
%-+al X-+al

= lim 2+4/x =~.


x .... "' 9+6fx-12/x 2 -8/x3 9

Note. We see that in such examples the limit is equal to the


ratio of the coefficients at the superior power of x (provided the
polynomials are of the same degree).
. (Y9x2+ l-3x) _
(b) IIm .
1Im I _ O
- - .
.... +"' I x .... +"' Y 9x 2 + I + 3x
(c) In handling such examples bear in mind that the function
f (x) = 'V
Pn (x), where Pn (x) is a polynomial of degree n, tending
to infinity in the same way as the function xn. This allows us
to single out the superior power of x and divide both the nume-
v
rator and denominator by this power of x. In the given example
I. JO. Calculation of Limits of Functions 63

the divisor is Vx; then we obtain:


. 2Vx+3 vx-+s vx- . 2+31v-x-+s1\;Y'xa
hm = hm .:--;:-::==----'<;-;-======-
..... u, V3x-2+ v2x-3 X-++CJ> V3-2fx+ v 4/x-l2/x 2 +9fx 3
2
=y-3.
(d) Since the sum of two positive infinitely large quantities is
also an infinitely large quantity, then
!im (V2x 2 --3-5x) = Jim jJl2x2 -3+(-5x)] = + oo.
x--oo X-+-oo

(f) At x >0 we have VX2 = x, therefore


. Vx2 (2+3/x 2 )
Iun .
= 11m
x l'r2+3/x2 V2
=-- .
.<-++ex> x(4+2/x) X-++o:> x(4+2/x) 4
At x < 0 we have VX2 = -x and, hence,
. V x2 (2+3/x2 )
I1m =
. -x V 2+3fx2 = -V2
11m --
.... _,,,
x(4+2/x) x(4+2/x)
x ... -oo 4

Note. From this it follows, incidentally, that Jim Jl-2X2'+3 does


..... 00 4x+2
not exist.
lim 2x/(x+3)
(g) lim 52x/(x+3) = 5x ... "' = 52 = 25.
K-+ <Xl

1.10.3. Fincl the limits:


(a) Jim 2x-2 . (b) Jim 4 x+I ;
x ... I v26+x-3 ' x ... -1 Vx+I7-2

(c)
.
I1m 1+vx-
VX ; (d) Jim
Vi+x- (k positive in-
x ... -1 I+ x ... o x
teger);
(e) Jim sin (x-n/6) . (f) I .1m - - - -
cos x
X-+:JT/6 V 3-2cosx' x-+n/2 V (I- sin x) 2 '

() I" 2sin2x+sinx-I
g .. 2~ 6 2 sin 2 x-3 sin 1 x+
Solution (method of substitution). (a) Let us put 26+x=z 3
Then x = z3 - 26 and z __,. 3 as x __,. 1; hence
. 2x-2 . 2z 3 - 54 1. 2 (z-3) (z2+3z+ 9)
I1m = 11m = 1m =
X-+ I V26+x-3 z.,.3 z-3 Z-+ 3 z-3
=Jim 2 (z 2 +3z+ 9) = 54.
Z-+ 3
64 Ch. I. Introduction to Mathematical Analysis

(d) Let us put 1 +x= zk; then x= zk-1 and z - l as x- 0.


Hence,
{'./T+x-1 . z-1 I
Jim = hm k- I = 7i (see Problem l.10.1 (d)).
X-+0 X Z-+lz

(e) Let us put x-n/6=z; then x=z+n/6 and Z--+0 as


x - n/6. On substituting we obtain
. sin (x-:rc/6) 1.
IIm sin z
= Im --=-------
x ... '!f/6 V3-2cosx V3-2cos(z+:rc/6)
2 ... 0
. sin z .
1Im 2 sin (z/2) cos (z/2)
= IIm = -
z ... oY 3- Y 3 cos z+ sin z z .. o 2 Y 3 sin 2 (z/2) +2 sin (z/2) cos (z/2)

= lim .r cos (z/2) l.


z .. o " 3 sin (z/2) +cos (z/2)
t.10.4. Find the limits:
(a) lim 1- ~os x; (b) Jim tan x-;sinx;
X-+0 X X-+0 X
. cos (:rcx/2)
(c ) IIm I .
I
X-+ -x
. ( . I - cos x -_ 1.Im 2 sin 2 (x/2) -_..!_I"Im (sin (x/2))2 -_..!_.,
Sol ut ton. a ) 1Im
X-+0 X
2
X-+0 X
2 2 X-+0 X
12 2
. tanx-sinx 1. sinx(l-cosx)
(b) IIm = Im -----=---'-
x ... o x3 X-+0 COSXx3
. I sinx 1-cosx
=II m - - - - - - -
x ... 0 cos x x x2
(c) Let us put l-x=z. Then X= 1-z and Z--+0 as X--+ I.
Hence,
cos ~ x cos ( .:!_ _ ~ z) sin .:!. z
II. m -2- = 1Im 2 2 - =1I I T I -2 -=- :re
.
X-+I 1-X Z-+0 z l-+0 z 2

Note. For a simpler method of solving similar problems see l.12.


1.10.5. Find the limits:
(a) Jim(l+l/x)7:x; (b) lim(l+x)''<3x>;
.t-+ al x _.. 0

<c> !~~ (i ~xY: (d) lim(l+k/xrx;

() I . ln(l+x).
(f} hm -1 - ;
e4X-I
e Im 3x_1 '
x ... 0 x .. 0 an x
(g) Jim ln(a+x)-lna; (h) Jim ex -:-e-:x ;
.< ... 0 x x .. o smx
(I")I"I m lnx-1
--.
Je-+B x-e
/./(). Calrulation of Limits of Functions 65

Solution. (a) Jim ( 1+ ~) 7 x = lim [(I + +)x] 7 =

rr
x-::n X X-+oo

= u~~ (I++ = e1;


(e) lim In (I +x) _ lim [In (I +x). _x- ] __I_
x -+ 0 3x - I - x-+ 0 ~ 3x - I - In 3 '

(i) Put x/e-l=z; then Xo=e(z+l); z~O as x-e. On substi-


tuting we obtain
. In x- I 1. In (x/e) l . In (I +z) I
IIm - - - = 1111 = - 1ltll = - .
x-c x-e x-+ee(x/e-1) e z-o z e
1.10.6. Find
Jim
K-+CC
(I +~)x. X

Solution. lim ( I+~ I ) x


=Jim l
' (
I+ I ) x'
xz
JI/x =e 0 =1.
r-~oo x-,.oo

1.10.7. Find the limits:


()I . (l+x)(J-l'>)/(1-x).
a 1m __ .
X-+I 2+ X
b I. ( xz+2x-I )(2x+l)/(x-1)
( ) ltll 2x 2 -3x-~
X-+ "
Solution. (a) Denote:
f (x) = (1 + x)/(2 + x);
qi (x) 1-vx.
1-x '

Jim f (x) =Jim 2'++x = 32 ;


X-+ I X-+ 1 .x

I1mqi(x ) 1.
= tm
1-vx 1
l-x = 2 .
x .... J x-1

But at finite limits lim f (x) =A> 0, Jim qi (x) = B the following
relation holds true:
lim Ql (x) 111 / (x)
Jim [f (x) j!Jl (x) =ex-a = eB In A =AB.
X-+G
Hence,
I . (I +x)(1-1'%L'n-xl = (~)112 = ~ / ~
}~~ 2+x 3 V 3
Note. If in handling examples of the form lim [f (x)]!Jl ex> it turns
X-+a
out that Jim f (x) = l and Jim qi (x) = oo, then the following

3-3148
66 Ch. I. lnfroduclion to Mathematica/ Analysis

transformation may be recommended:


Jim [f (x) ]'I- (X) = li.n {I + [f (x)- I] }'f (X) c=
x-a .r ~a

lim rp (x) [/ (x)- I]


=Jim {[I+ (f (x)- l)jll<I (xl-1>}~ (Al LI (x)-1] =e1' .. a (*)

t. I0.8. Find the limits:


. (2x:l + 3 )8x'+3 . (l+tanx)l/>mx
(a) !:n;, 2x2+ 5 ; (b)
Iltn
.,_, 0 1+.
sin x
;
(c) lim (1 +sin nx)cot nx;
x-+ 1
- (sinx)l/(x-a) .
(d) llin -.- (a =I= kn, with k an integer).
x~a sma

Solution. (a) Let us denote:

f (x) = 22xx2 +
+ 3.
2
5, cp (x) = 8x 2 + 3;
rtm f()x = rtm 22x2+3
xLt 5 = J,
X-+00 X-+oo

Jim cp (x)
X-+00
= Jim (8x 2
X-+00
+ 3) = oo.
Use the formula (*):
. (2x2+ ~\Bx'+3 _ l~m00 rp(x)[/(x)-!] 0
ltm
X->00
2X 2+ D"/ -e1' '

2x 2 +3 2
f (x)- I= 2x2+ 5 - I= - 2xi+ 5;
. . 2 (8x2+3)
lun cp (x) [f (x)-1] = - 1tm +
2x2 5 = - 8.
f-+CCI X-+ClO

Therefore
)" ( 2x2 + 3)
Bx'+ 3 _ _~
tm 2x2-f- 5 -- e .
x-oo
1. I0.9. The function f (x) is given with the aid of the limit
. x 2" - I
f (x) = Jun
n-+ocx
""+ 1
Investigate this function and graph it.
Solution. Consider three cases:
(I) / x / > l. Since in this case Jim x 2 " = oo, then
. l-l/x 2n
f (x) = 11111 1
n-rs:
+
1;x:in = l.
(2) Ix I < 1. In this case Jim x 2 " = 0; thPrefore
fl-+ iS)
f (x) =-1.
I .JO. Calculation of Limits of Functions 67

(3) x-= + 1. In this case x211 = l at any n, and therefore f (x) = 0.


Thus, the function under consideration can be written in the
fol lowing way:
liflxl>l
f (x) = { - I if Ix I < I
0 if X= +1
or, briefly, f(x)=sign(\xl-1) (see Problem 1.5.ll(n)).
The graph of this function is shown in Fig. 27.
1.10.10. The population of a cou- Y
ntry increases by 2/o per year. --_., -1 r-
By how many times does it increase I 1
I
in a century? 1 I
Solution. If we denote the ini- l I
I
tial number of inhabitants of a -11 o T1 - - x
I
given country as A, then after a I I
I
year the total population will amo- 1 I
unt to 1 -f

A+1~02 =(l+~o)A. Fig. 27

After two years the population will amount to A ( l + 50I . After )2


I 00 years it will reach the total of A ( l + 510 )1 0
, i. e. it wil 1

+fa rr
*r
0
have increased [ ( l times. Taking into account that
!~~ (I + = e, we can approximately consider that (I + 510 ~ e.
Hence, after 100 years the population of the country will have
r0

increased e2 ~ 7.39 times.


Of course, this estimation is very approximate, but it gives an
idea as to the order of the increase in the population; (the quan-
tity f\ l t I ')100
= 7 .245 to within three dt'cimal places ) .
. 50
/
1.10.11. Find the limits:
() r cosx+4tanx.
a ImoX->
2-x-2x4 '
. 2x 2 +5x- 7
(b) x~~\ 3x2-x-2
. Y5-x-2
(c) 11m ;
x ... 1 Y2-x-I

. 2x 2 -5x+ 4
(d) hm 5x2 -2x-3 ;
X- 00

3
68 Ch. I. Introduction to Mathematical Analysis

(e) Jim (V x 2 + 1-J,/ x2 - I);

V-
X-+IXl

(f) Iim ( 2x 3 + 2-x').


-cc I+ Bx

l. I0.12. Find the limits:


vx+4-2 (b) limsin(l-xl.
(a) lim . ;
x-O sin 5x x-1 Vx-1 '
. si11 a
(c)hm 1 2 , 2 ; (d) Jim tan2x lan(;cc/4-x);
a-Jt - a 1n :c-Jt/4
(e ) Itm. tan3 x-3 tan x
x-rr/ 3 cos (x n/ 6)
. +
l. I0.13. Find the limits:
e-X-1
(b) I1
(a) Jim (I+ 4/x)x+a; m--;
x-ao x-o x
I

(c) l t m - - ;
x
a2X_
( d) lim (I + 3 tan 2 x)cot' ";
:c-o x-o
. f2x-I\".
(e) "~~~4 (sin 2x)tan' 2x; (f) !~1! \2x+ I) '
(g) Jim (tan x)tan 2 "; (h) Jim (sin x) 18 " ";
x-n/2 x-11/2
(I') 1. (3x2+2x+l )(6x+IJ/(3x+2>,
Im
x-ao X 2+ X +2 '
(J') J' (1+3x)(I-Vx)/(l-x).
tm 2+3X
x-rs:: '
eX _ ePX
(k) ltm .
'.o x
1.10.14. Find the limits:
(a) limarccV(l-x); (b) Jim lntanx;
-0 x x-Jt/ 4 I -cot x
(c) Jim - 1- In (I+ a sin x).
-O Sill X

I.II. Infinitesimal and Infinite Functions.


Their Definition and Comparison
The function a (x) is called infinitesimal as x __,a or as x __, oo
if Jim a (x) = 0 or Jim a (x) = 0.
x-.,.a X-+ a:i

The function f (x) is called infinite as x-> a or as x ~ oo 1f


Jim f (x) = oo or Iirri f (x) = oo.
-------=-_J_.11. Infinitesimal and Infinite Functions 69

A quantity inverse to an infinite quantity is called an infinite-


simal.
Infinitesimal functions possess the following properties:
(1) The sum and the product of any definite number of infinite-
simal functions as x ___...a are also infinitesimals as x---+ a.
(2) The product of an infinitesimal function by a bounded function
is an infinitesimal.
Comparison of Infinitesimals. Let the functions a (x) and ~ (x) be
infinitesimal as x ---+ a. If
I . a (x)
x'_?;! ~ (x) = C,
where c is a certain finite number different from zero, then the
functions a (x) and ~ (x) are cal led infinitesimals of the same order.
If c = 1, then the functions a (x) and ~ (x) are called equivalent;
notation: a (x),...., ~ (x).
If c = 0, then the function a (x) is called an infinitesimal of a
higher order relative to ~ (x), which is written thus: a (x) = o (~ (x)),
and ~ (x) is called an infinitesimal of a lower order with respect
to a (x).
If !~n; c;<~~n = C, where 0 < Ic I < + 00, then the function a(x)
is called an infinitesimal of the nth order as compared with the
function ~ (x). The concept of infinite functions of various orders
is introduced similarly.
l.11.1. Prove that the functions
2x-4
(a) f (x) = x2 +s as x---+ 2,

(b) f (x) = (x-1) 2 sin 3 x 1


1 as x---+ 1 are infinitesimals.
Solution. (a) It is sufficient to find the limit
. f (x ) = l'1m~
11m 2x-4 = 0 .
t2
5
X->2X-r

(b) Firstly, the function cp(x)=(x-1) 2 is infinitesimal as X-+ 1;


indeed, lim (x-1) 2 =0. Secondly, the function
x~I

' (x) = sin 3 x-


-
1 ; x =I= I,
1
is bounded:
I'.
stn x-1 ~
3 I I -1 .

Hence, the given function f (x) represents the product of the


bounded function ' (x) by the infinitesimal cp (x), which means that
f (x) is an infinitesimal function as x---+ 1.
70 Ch. I. Introduction to Mathematical Analysis

1.11.2. Prove that the functions


f (x) = 3x-12
(a) 2x2 7 + as x--+ 4;
(b) f (x)= sinx
x
as X--+OO
are infinite~imal.

l. l l.3. Find
Jim x sin ( l/x).
x .... 0

Solution. Since x is an infinitesimal as X--+ 0 and the function


sin (l/x) is bounded, the product xsin (l/x) is an infinitesimal, which
means that Jim x sin (l/x) = 0.
x .... 0

1.11.4. Compare the following infinitesimal functions (as x--+ 0)


with the infinitesimal qi (x) = x:
(a) f 1 (x)=tanx 3 ; (b) f 2 (x)=Vsin 2 x;
(c) f 8 (X)=V9+x-3.
Solution. (a) We have
. -
Itm tanxs . rtanx-3 x2 ] = 11m
- = 1tm - -
. -tanx
-
3 . 2 = O.
11mx
3 3 -
o x
x .... x .... o x x .... o x x .... o
Hence, tan x is an infinitesimal of a higher order relative to x.
3

(b) We have
.
ltm a ,_
vsin2x 1 ,-~ si112x I ]
x .... o x
= 1m
x .... o
- - - - =oo
x2 v-x
Hence, V
sin 2 x is an infinitesimal of a lower order as compared
with x.
(c) We have
.
Itm V9+x-3 = 11111
. I
= -I .
x-o x x-o Y9+x+3 6
Hence, the infinitesimals V9 + x-3 and x are of the same order.
1.11.5. Determine the order of smallness of the quantity ~ with
respect to the infinitesimal a.
(a) ~=cos a-cos 2a; (b) ~=tan a-sin a.
Solution. (a) ~=cosa-cos2a=2sin{asin ~.
Whence
r1111 1.._ _ r1m 2 sin (3a/2) sin (ct/2) - ~

cx-o a cx .... o a
2 - 2 2 -

Hence, ~ is an infinitesimal of the same order as a 2 , i. e. of the


second one with respect to a.
1.12. Equivalent ln(inilc1imals 71

1.11.6. Assuming x ~ oo, compare the following infinitely large


quantities:
(a) f (x)=3x 2 +2x+5 and <p (x) = 2x 3 + 2x- l;
(b) f (x) = 2x 2 + 3x and <p (x) = (x + 2) 2 ;
(c) f(x)=Vx+a and <p (x) = V-
x.
Solution. (a) The infinite function 3x 2 + 2x + 5 is of a lower order
as compared with the infinite function 2x 3 + 2x- I, since
r 3x2+2x+5 - r 3/x+2/x2 +5/x 3 - 0
Im 2x3-j-2x-I- x-oo
X-+00
im 2t2/x2 -l/x3 -

1.11.7. Prove that the infinitesimalsa=x and ~=xcos(l/x)(as x---0)


are not comparable, i.e. their ratio has no limit.
Solution. Indeed, lirn x cos (l/x) = lim cos (l/x) does not exist (prove
X-+0 X X-1>0

it!), which means that these infinitesimal functions are not com-
parable.
1.11.8. If X-+ 0, then which of the following infinitesimals is
(are) of a higher order than x; of a lower order than x; of the
same order as x?
(a) lOOx; (b) x 2 ; (c) 6 sin x; (d) sin 3 x; (e) V tan 3 x.
1.11.9. Let x ~ 0. Determine the orders of the following infini-
tesimal functions with respect to x:
(a) 2sin 4 x-x; (b) Vsin 2 x+x 4 ;
(c) VI+x3 -I; (d) sin2x-2sinx;
(e) 1- 2 cos ( x + ~) ; (f) 2 V sin x;
x
(g) x- I ; (h) tanx+x 2 ;
(i) cosx- V cosx;
-
1.11.10. Assuming the side of a cube to be an infinitesimal, de-
termine the order of smallness of the diagonal of the cube (d); of
the area of its surf ace (S); of its volume (V).

1.12. Equivalent Infinitesimals.


Application to Finding Limits
If the functions a (x) and ~ (x) are infinitesimal as x ~a and if
a (x),..., )' (x), ~ (x),..., 6 (x), then
hm ~((x)) = lim I((x)) (replacing an infinitesimal by an equivalent one).
x-aPX x-..aux
72 Ch. I. Introduction to Mathematical Analyst5

If
Jim f(x)=k, 0 <!kl< oo,
x~a

then
f (x) a (x) - ka (x).
If
a (x) - y (x),
~ (x) - y (x),
then
a (x) - ~ (x).
For two infinitesimal functiom to be equivalent it is ne~essary
and sufficient that their difference be an infinitesimal of a higher
order as compared with each of the two.
Listed below are infinitesimal functions:
(a (x) is an infinitesimal as x _,. 0)
(I) sin a (x) - a (x); (2) tan a (x) - a (x);
(3) I -cos a (x) - [a (x)]2;2;
(4) arc sin a (x) - a (x); (5) arc tan a (x) - a (x);
(6) ln[l+a(x)]-a(x); (7) alx>-1.--a(x)lna
(a> 0), in particular, elx> - I.-- a (x);
(8) [l+a(x)]P-1.--Pa(x), in particular, v l+a(x)-1.--a.(x).
n
t.12.t. Prove that as X-+0
I I I
(a) l - y l+x .-- -2 x; (b) 1 - -1+ - x;
x

(c) sinJ;/xVx - Y x2 +Vx 3


Solution. (a) By formula (8) at P = 1/2 we have
l- I = l (JI 1 + x- l) ,_, l. _!_ x.
Yl+x YI +x 2

(c) By formula (1) we have

sin}/xVx ---Y~ =x3f4,


Vxz + V x3 =x3/4Vl + xrl~ ,_ xaf4,
whence sin y' x i1 x .-- l/ x + Jfxa 2

1.12.2. Replace each of the following infinitesimals with an equi-


valent one:
(a) 3sina-5a 3 ; (b) (l-cosa) 2 + 16a:1 +5a 4 -t 6a 5
1.12. Equivalent /nfinite~imals 73

Solution. (a) Note that the sum of two infinitesimals ex. and ~
of different orders is equivalent to the summand of the lower order,
since the replacement of an infinitesimal with one equivalent to it
is tantamount to the rejection of an infinitesimal of a higher order.
In our example the quantity 3 sin ex has the order of smallness 1,
(-5a :1) - the order of smallness 3, l'.ence
3 sin a+ (-5cx. 3 ) _. 3 sin ex. _. 3a.

(b) (l-cosa) 2 + 16a.3+5a 4 +6ab=4sin 4 1"+ 16cx. 3 +5cx.4 +6cx..


The summand 16cx.3 is of the lower order, therefore
(I -cos cx.) 2 + 16cx.3 + 5cx. 4 + 6cx. _. 16a3
l.12.3. With the aid of the principle of substitution of equiva-
lent quantities find the limits:
() r sin 5x (b) Jim
I-cos x
a x~m0 ln(l+4x); x-0 I-cos x '
2
(c) lim Vin cos x ; (d) Jim YI+~ +x2 - I
x-o 1+x2_ 1 ,_ 0 sm4x
(e) I . sin 2x+ arc sin 2 x- arc tan 2 x
tm 3 ;
x-0 x
(f) r 3sinx-x2 +x 3
un tan x+2 sin 2 x+5x4
x-0
;

( ) r1 (sin x-tan x)2 +(I - cos 2x)' + x0


g x _?10 7 tan' x+ sino x+2 sin x '

(h) (' sin v; In (I +3x) .


x !!Tl ( s v - )'
- o
(arc tan .r--
r x ).2 e " - I

(1') rtm l-cosx+2sinx-sin3x-x2+3x4

- 0 tana x-6 sin~ x + x-5x:i

Solution. (a) We have sin 5x _. 5x; In (I+ 4x) _. 4x (see the list
of equivalent infinitesimals on page 72). Therefore
1. sin 5x 1. 5x 5
!III I (I -j- 4 ') = IITJ -4 = -4 '
x-o n x x-o x
(c) ll'rn
x-0
v'n cos x
I+x2-1
=
,.
Jill
x-0
In II+ (cos x-1))
A/4
X
=
_ . cosx-1 _ _ 4 1. x2;2 _ _ 2
- 4 11m 2 - 1m 2 -
x-o x x-o x
(d) From the list of equivalent infinitesimals we find:
Ii 1 +x +x 2- 1 ,._ (x + x )/2 ,..... x/2,
2 sin 4x _. 4x.
7! Ch. I. Introduction to Mathematical Analysis

Therefore
Jim }' I+~ +x2 - I= Jim x/2 = _!_.
x~ 0 sm 4x x ~ 0 4x 8

(e) Using the list of equivalent infinitesimal functions given on


page 72 we obtain
sin 2x +arc sin 2 x-arc tan 2 x - sin 2x - 2x.
Hence,
. sin2x+arcsin 2 x-arctan 2 x 1. 2x
Iirn 2
x~O
3
X
= x~OX
Hn 3=3

(h) sin Vx- Vx; In (1+3x) - 3x;


V-; -
i'
arc tan Vx,...., Vx; e5
sin V -xln(l+3x) -I" v-s Vx;
t ,...,
X3x_3
x ~r~ (arc t an 1 ,r-::)2 (
r x e
s V x- 1) - .'~110 x-5 V x -5
1. 12.4. Find the approximate values of the roots V l.02 and
V0.994. Estimate the absolute error.
Solution. Use the approximate formula
l+x/2Vt+x,..., (*)
{for x sufficiently close to zero). In our case
Jlt + 0.02,...., I+ 0 ~ 2 = 1.01;
Vt -0.006 ~ t- 0 ~06 = o.997.
To estimate the error we note that
;-(VI +x- I)=~ (x-2 VI +x+ 2) =
= + +(
(x + I - 2 V x+ I + 1) = V x+ I - I) f (; ) = i 2
,....,
1
.
Hence, the absolute error of the approximate formula (*) is esti-
x2
mated by the quantity 8 .
Using this estimate we find that the absolute error of the root
- (0 02)2
v1.02~1.01 is ~--8-=0.00005, and the absolute error of
;-- (0 006) 2
Jt 0.994 ~ 0.997 amounts to ~ --8- ~ 0.000005.

1.12.5. Prove that, as x---.0,


J,-- I
(a) V 1 + x-- I,...., 3 x;
1.13. One-Sided Limits 75

(b) arc tan mx ,._, mx;


(c) l-cos3 x,....,{sin 2 x.

l.12.6. For x----. 0 determine the order of smallness, relative


to the infinitesimal ~ (x) = x, of the following infinitesimals:

(a) v-sin 2 x+x 4 ; (b) x 2 (1+x).


1+Vx

1.12.7. For X-+2 determine the order of smallness, relative to


the infinitesimal ~(x)=x-2, of the following infinitesimals:
(a) 3(x-2) 2 +2(x 2 -4); (b) Vsinnx.
1.12.8. Making use of the method of replacing an infinitesimal
with an equivalent one, find the following limits:
. sin 3x (b) lim In (I+ sin 4x) .
(a) !~1~ In (I -t-5x); x~ o esin 5x - I '
. ein 3x_ 1
(d) lim arc tan 3x.
(c) hm
x~ 0 In (I+ tan 2x)' x~o.arc s1112x'
. In (2-cos 2x) (f) lim YI.+ sin 3x- I.
(e) xl1tn I 2
~ 0 n
( . 3
sin x +
I) ; x~o ln(l+tan2x) '

( ) Jim In (I +2x-3x2-j-4x3). . V1+x 2 -1


(h) Iltfl
g x ~ 0 In (I - x + 2x 2- 7x") ' x~ 0 I --cos x

1.12.9. Find an approximate value of the root V 1042.


1.13. One-Sided Limits
A number A is called the limit to the right of the function f (x)
X---+- Xo +0
+
as x----. x 0 (A= lim f (x) = f (x 0 0)) if for any c > 0 there exists
6 (i::) > 0 such that for all x satisfying the inequ-ality 0 < x-xQ 6 (e) <
and belonging to the domain of definition of the function f (x) the
inequality If (x)-A [ < e holds true. The limit to the left of the
function f (x 0 -0) as X-+X 0 -0 is defined in a similar way. If x 0 =0,
then we write simply x----++0 or X-+-0 and, respectively, f(+O)
and f (-0).
1.13.1. Find the one-sided limits of the functions:
I -2x+3 if x~ 1,
(a)f(x)=\3x-5 ifx>l asx-+1;
x2 - I
(b) f(x)= Ix-II as x-+ I;
76 Ch. I. Introduction to Mathematical Anulym

(c) f (x ) = V I - x
cos 2x
as x--.. ;
O

I
(d) f (x)=3+ l+ 711 , 1_x> as x--.. l;
(e) f (x) =cos (n/x) as x--.. O;
(f) f (x) = 5/(x- 2) 3 as x---.. 2.
Solution. (a) Let x ~ 1. Thenf(x)~-=-2x+3. Hence, f(l-0)=
Jim f (x) =I is the limit to the left.
!I x- 1-0
If x > I, then f (x) = 3x-5; hence_,
f (l + 0) = Jim f (x) = - 2 is the limit
x-1+0
to the right (see Fig. 28).

) - V1-cos2x -_ V2sin 2 x -_
C f( X--
()
x x
= V2jsinxf
x
but
sin x, if 0 < x < n/2,
\sin x I= {
-sinx, if -n/2 < x < 0.
Fig. 28 Hence,

f (--0) = Jim f (x) = Jim ( - V-2 sin


-x-
x) = -
1-
~ 2,
x- -0 .r--.-0

f (+0) = Jim f (x) = Jim ( V2 si~ x) = V2.


x-+o x-+o

(d) The expression l/(1-x) tends to + oo, when x tends to 1,


remaining less than 1, therefore
Jim 711< 1 -x>
x~1-o
= + oo Jim 1
'x .... 1-01+7 11 <1-xl
=0
'
f (l-0)~~3.
Further, as x-l+O we have 1/(1-x)--..-oo. Therefore
lim 7 1!< 1 -x>=O,
:c- I +0

f(l+O)= Jim (3+ 1/(i-xi)=3+1=4.


1
x-1+0 1+7

(e) Let us choose two sequences, {xn} and {x~}. with the general
terms
I , 2
xn == 21i and Xn = 2n + 1 (n = I, 2, ... )
respectively.
1.14. Continuity of a Function. Points of Discontinuity 77

Then lim xn = lim x.~ = 0 and


lim f (xn) = lim cos 2:rtn = l;

lim f (x~) = Jim cos (2n + l)


n-+CTJ n- ~
i = 0.
Hence, the function f (x) has no limit to the right at the point 0;
taking into account that f (x) is an even function, we cone! ude that
it has no limit to the left either (see Fig. 29).
!I
1

-1
Fig. 29

1.13.2. Prove that, as X----+ 1, the function


f (x) = J x+ l at 0 ~ x < I,
\ 3x +2 at I < x <3
has a limit to the left equal to 2 and a limit to the right equal to 5.
1.13.3. Find the one-sided limits of the following functions as
x----+0:
I
(a) f (x) = 2-21/x;
(b) f (x) = e11x;
(c) f(x)= jsinxl.
x

J.14. Continuity of a Function.


Points of Discontinuity and Their Classification
Let the function y = f (x) be defined on the set X and let the
point x0 EX be the limit point of this set. The function f (x) is said
to be continuous at the point x0 if lim f (x) = f (x0 ). The latter con-
x - x0
dition ~s equivalent to the condition Jim liy (x0 ) = lim [f (x 0 +/ix) -
Ax-o Ax-o
-f (xo>J =0.
78 Ch. I. Introduction to Mathematical Analysis

The function f (x) is continuous at the point x 0 if and only if


f (x0 - 0) = f (X 0 + 0) = f (x 0 ).
The function f (x) is continuous on the set X if it is continuous
at every point of this set.
Points of Discontinuity of the First Kind. Let the point x 11 be the
limit point of the domain of definition X of the function f (x).
The point x11 is called a discontinuity of the first kind of the fun-
ction f (x) if there exist the limits to the right and to the ldt and
they are finite. If f(x 11 -0)=f(x 0 +0)=l=f(x 0 ), then x., is called
a removal;[e discontinuity. Further, if f(x 0 --0)=l=f(x0 +0), then x 11
is a non-removable discontinuity of the first kind, and the d.ifference
f (x 0 +0)-f (x 11 --0) is called a jump discontinuity of the function
f (x) at the point x""
Points of Discontinuity of the Second Kind. If at least 01w of
the limits of f (x0 -0) and f (x 0 +0) is non-existent and infinite,
then point x0 is called a discontinuity of the second kind of the fun-
ction f (x).
1.14.1. Using only the definition prove discontinuity of the fun-
ction f (x) = 3x 4 + 5x'1 + 2x 2 + 3x + 4 at any x.
Solution. Let x0 ee an arbitrary point on the number scale. Fir~t
find lim f (x):

X-+ Xo X-+ Xo

Then compute the value of the function at the point x 0 :


f (x 0) = 3-' t + 5x~ + 2x~ + 3x11 + 4.
Comparing the results thus obtained, we see that
Ii n f (x 0 )-= f (x..).
X -+ Xo

Hence, the function f (x) is continuous at the point x0 by definition.


Since x 0 is an arbitrary point on the number scale, we have proved
continuity of the function for all values of x.
1.14.2. Given the functions:

(a) f (x) =
(~ + (2x 2 + 3)
6- 5x
for - oo
for
< x ~ l,
l < x < 3,
l x-3 for 3 ~ x < oo;
x~3.
(b) f (x) = J - 2.\ 2 for
for x > 3;
\ 3x
() f( ) _ !2x-3J
c x - 2x-3
1.14. Continuity of a Function. Points of Discontinuity 79

Find the points of discontinuity (if any). Determine the 1ump


discontinuities of the functions at the points of discontinuity of the
first kind.
Solution. (a) The domain of definition of the function is the entire
number scale (-oo, oo). In the open intervals (-oo, I), (1, 3),
(3, oo) the function is continuous. Therefore discontinuities are pos-
sible only at the points x= I, x=3, at which analytic representa-
tion of the function is changed.
Let us find the one-sided limits of the function at the point x ~c I:

f ( I - 0) = Ii m ~ (2x 2 + 3) ~ I;
x-1-0
f (I+ 0) = lim (6-Sx) =I.
x-1+0

The value of the function at the point x = 1 is determined by the


first analytic representation, i. e. f (I)= (2 + 3)/5 =I. Since
f (1-0) = f (1+0) = f (I),
the function is continuous at the point x = 1.
Consider the point x = 3:
f (3-0) = li:n (6-Sx) = -9;
x- 3-0
f(3+0)= lim (x-3)=0.
x- 3 + 0

We see that the right-hand and the left-hand limits, though finite,
are not equal to each other, therefore the function has a disconti-
nuity of the first kind at the point x = 3.
The 1ump of the function at the point of discontinuity is
f (3+ 0)-f (3-0) = 0-(-9) = 9.
(c) The function is defined and continuous throughout the entire
number scale, except at the point x = 3/2. Since 2x-3 > 0 for
x > 3/2 and 2x - 3 < 0 for x < 3/2,
f (x) = { 1 at x > 3/2,
- I at x<3/2.
Hence,
f (3/2 + 0) = 1, f (3/2-0) = -1.
Therefore, at the point x = 3/2 the function has a finite discon-
tinuity of the first kind. The jump of the function at this point
f(3/2+0)-f(3/2-0) is equal to 1-(-1)=2.
1.14.3. Test the following functions for continuity:
( sin x
(a) /. (x) = < -"- for x =/= 0,
l l fur x = O;
80 Ch. I. Introduction to Mathematical Analysis

(b) f (x)=sin (l/x);


(c) f(x)={xsin (1/x) for x=,60,
0
for X= 0;
f 4 3x for x < 0,
(d) f(x)= \ 2a+x for x;;?:=O;
(e) f (x) =arc tan ( l/x); (f) f (x) = (x 3 +
I )/(x 1). +
Solution. (a) The !unction is continuous at all points x =F 0. At
the point x = 0 we have
.
f (0) = 1; Iun sin t . sin x
- = Jun - = I.
.t:--o X X-++O X

Hence, at this point the function is continuous as well, which


means that it is continuous for all values of x.

Fig. 36

(b) The function is defined and continuous for all x =I= 0. There
a,re no one-sieled limits at the point x=O (cf. Problem 1.13.l
(e)). Therefore, at the point x=O the function suffers a disconti-
nuity of the second kind (see Fig. 30).
(d) f (-0) = 4, and f (+O) = 2a; the equality f (-0) = f (+0) = f (0)
will be fulfilled, i. e. the function f (x) will be continuous at the
point x=O if we put 2a=4, a=2.
(f) f (-1-0) = f (-1 +0) = lirn (x 2 -x+ I)= 3, i.e. both one-
x--1
sided limits are finite and coincide. But at the point x = -1 the
.~ 1.14. Continuity of a Function. Points of D1swntinuity 81

function is not defined and, therefore, is not continuous. The graph


+
of the function is the parabola y = x 2 -x 1 with the point M (-1, 3)
removed. If WP redefine the function putting f(-1)=3, then it
will become continuous. Thus, at x = -1 the function has a n:mo-
vable discontinuity.
l.14.4. Test the following functions for continuity:
(a) f (x) = E (x). It should be borne in mind that the function
E (x) is defined as the maximum integer n contained in the nutn
ber x, i. e. as a number satisfying the inequality n ~ x.
(b)
J 1 if x is rational,
A.(x)= \ 0 if xis irrational.

A(x) is cal led the Dirichlet function. For instance, A (0) = 1; A.(- I /2) = 1;
1' (V..2) = O; 1' (n) = 0, de.
Solution. (a) The function E (x) is defined throughout the entire
number scale and takes on only integral values. This function is
discontinuous at every integral value n of the independent va-
riable, since E (n-0) = n-1;
E(n+O)=n (see Fig. 31). !J
(b) Let us choose an arbitrary 3 --------r--'-"!
point x0 on the x-axis; two cases I :
2 ----_,-.J I
are possible: ( 1) the number x 0 I I I
is rational; (2) the number Xo is 1 -~ l II
irrational. -3 -2 -1 I I I
~--.~.....--+-.....~::--=---:-----x
In the first case A (x 0 ) = 1. In I 0 f 2 J 4
any vicinity of a rational point -f
there are irrational points, where I I

A.(x)=O. Hence, in any vicinity I


I
+--1
I
-z
of x0 there are points x for which !-I -3
I 11y I= J A(x0 ) - A(x) I= I. Fig. 31
In the second case A (x 0 ) = 0.
In any vicinity of an irrational point there are rational points
at which /.., (x) = 1. Hence, it is possible to find the values of x for
which
Ji1yJ=IA(x 0 )-A.(x)J =I.
Thus, in both cases the difference i1y does not tend to zero as
i1x __,. 0. Therefore, x0 is a discontinuity. Since x0 is an arbitrary
point, the Dirichlet function A (x) is discontinuous at each point.
The graph of this func.tion consists of a set of points with irratio-
nal abscissas on the x-axis and of a set of points with rational
abscissas on the straight line y = 1, that is why it is impos-
sible to sketch it.
82 Ch. I. Introduction to Mathematical Analsis

1.14.5. Using the definition of continuity of a function in terms


of "e-6", test the following functions for continuity:
(a) f (x) =ax+ b (a =I= O);
/ x2 if x is rational,
(b) f (x) = \ -x2 if x is irrational.
Solution. (a) Choose an arbitrary point x 0 According to the "e-- &"
-definition it is necessary to show that for any preassigned, arbitra-
rily small number e > 0 it is possible to find a number &> 0 such
that at I x-x0 I < & the inequality If (x)-f (x 0 ) I< c holds true.
Consider the absolute value of the difference
.1f (x)-f (x 0 ) I= I(ax+b)-(axd-b) I= Iax+b-ax 0-b/ =I a 11 x-xo I.
Let us require that lf(x)-f(x0 )/<e. This requirement will be
fulfilled for all x satisfying the inequality
Jal/x-x 0 /<e or lx-x 0 /<e/lal (a=j=O).
Hence, if we take & ~ e/I a/, then at Ix-x0 I < & the inequality
If (x)-f (xo) I< eis fulfilled. Continuity is thus proved for any
point x =X0
(b} Choose an arbitrary point x0 If {xn} is a sequence of rational
numbers tending to x0 , then Jim f (xn) = x~. If {x~} is a sequence
x,.-xo
of irrational numbers tending to x0 , then ,Jim f (x~) = -x~. At x 0 =I= 0
Xn-+Xo

the indicated limits are different and hence the function is discon-
tinuous at all points x =I= 0.
On the other hand, let now x = 0. Find the absolute value of
the difference lf(x)-f(O)/:
If (x)-f (0) I= I+ x2 -0 I= x 2
It is obvious that x2 < e at Ix I < Ve. If e > 0
is given, then,
putting &~Ve and /x-0/=/xj<&, weobtain/d{(O)/=x 2 <e.
Hence, ?t the point x = 0 the function is continuous. And so, the
point x = 0 is the only point at which the function is continuous. Note
that the function under consideration can be expressed through the
Dirichlet function (see Problem 1.14.4 (b)): f(x)=x 2 [2A.(x)-l].
1.14.6. Determine which kind of discontinuity the following
functions have at the point x = x 0 :
(a) f (x) = J x+ 2
for x < 2,
\ x2 - 1 for x ~ 2; X 0 = 2;
I
(b) f (x) =arc tan x-S; X 0 = 5; (c) f (x) = 1+I211x; Xe= O;
(d) f (x) =tan x; x 0 = n/2;
1.14. Continuity of a Function. Points of Discontinuity 83

(e) f (x) = Vx-E (Vx); x0 ="' n2 , where n is a natural number.


Solution. (a) Find the one-sided limits at the point x0 = 2;
f(2-0)= Jim (x+2)=4;
X--!>2-0

f(2+0)= Jim (x 2 -1)=3.


X-+ 2 +0
Here the limits to the right and to the left exist, are finite but
do not coincide, therefore the function has a discontinuity of the
first kind at the point x 0 = 2.
(e) The function E (l/.x) has discontinuities of the first kind at
every point x = n 2 , where n is a natural number (see Problem
1.14.4 (a)), whereas the function Vx is continuous at all x ~ 0.
Therefore the function f (x) = v x-E
(Vx) has discontinuities of
the first kind at the points I, 4, 9, ... , n 2 ,
l.14.7. Test the following functions for continuity
eX-J
(a) f (x) = -x- ;

(b) f(x)=rx::-1 for x=FO,


l 3 for x = 0;
(c) f(x)= f e f'' 1
for x=FO,
\ 0 for x ~= O;
(d) f (x) = Jim (sm x) 2"; (e) f (x) = I sin xI
sin x '
n-+ oc:

(f) f (x) = E (x) + E (- x).


1.14.8. For each of the following functions find the points of
discontinuity and determine the jumps of the function at these
points:
4
(a) f (x) =
x2-2x+ I;
x+2
(b) f(x)=x+lx+21;
(C) f (x ) c~ 2x2-x3
IX-- J I .
'
f - x for x~ l,
(d)f(x)={ 2
l x- 1 for x > I.
l.14.9. Redefine the following functions at the point x= 0 so as
to make them continuous:
(a) f (x) =ta: x;
84 Ch. I. Introduction to Mathematical Analysis

(b) f (x) = 5x2:;;3x;


f (X)= V1_;_x-1
(c) x ;

(d) f (x) = sin1 x


I -cos x

1.15. Arithmetical Operations on Continuous


Functions. Continuity of a Composite Function
If the functions f (x) and g (x) are continuous at the point x = x0 ,
then the functions
(1) f(x)g(x); (2) f(x)g(x); (3) ~~;) (g(x 0 )+0)
are also continuous at this point.
If the function u =qi (x) is continuous at the point x = x 0 and
the function y = f (u) is continuous at the point u0 =qi (x0 ), then
the composite function y=f [qi(x)] is continuous at the point X=X 0
l .15.1. Test the following functions for continuity:
2x0 -8x2+ 11
(a) f (x) = x4+4x3 +Bx2 +8x+ 4;
( b) f (x) = 3 sin3 x+ cos2 x+ I
4 cos x-2 '
(c) f (x) =- xa cos x+ ~2
sin x.
cos (l/sm x)

Solution. (a) A function representing a ratio of two continuous


functions (polynomials in this case) is discontinuous only at points
for which the denominator becomes zero. But in our case
x~ + 4x 3 +8x 2 +Bx+ 4 =(x2 + 2x +2) 2 ,
and since x2 + 2x + 2 = (x+ 1)2 + 1 > 0 at any x, the denominator
never becomes zero. Hence, the function f (x) is continuous through-
out the entire number scale.
(b) The function f (x) suffers discontinuities only at points for
which the denominator equals zero, i.e. at points which are the
roots of the equation
4 cos x- 2 = 0 or cos x = 1/2,
whence
X=Xn= + n/3+2nn (n=O, + 1, + 2, ... ).
Thus, the function f (x) is continuous everywhere, e:xcept at the
point xn.
/. 15. Operations on Continuous Functions 85

(c) Just as in the preceding example, the numerator is continu-


ous throughout the entire number scale. As far as the denominator
is concerned, according to the theorem on continuity of a composite
function, it is continuous at points where the function u = l/stn x
is continuous, since the function cos u is continuous everywhere.
Hence, the denominator is continuous everywhere, except at the
points x =kn (k an integer). Besides, we must exclude the points
at which cos ( l/sin x) = 0, i.e. the points at which I/sin x =
=(2p+l)n/2 (pan integer), or sinx=2/[(2p+l)n]. Thus, the
function f (x) is continuous everywhere except at the points x =kn
and x = (- l)" arc sin ( 2P: I) 31 +nn (k, p, n =0, 1, + 2, ... ).

1.15.2. Test the following composite functions for continuity:


(a) y =cos xn, where n is a natural number;
(b) y =cos log x;
(c) y= l/l/2-cos2 x.
Solution. (a) We have a composite function y =cos u, where
u = x". The function y =cos u is continuous at any point u, and
the function u = x" is continuous at any value of x. Therefore, the
function y =cos xn is continuous throughout the entire number scale.
(c) Here y= V1;2-u 2 , where U=cosx. The function Vl/2 -u 2
is defined and continuous on the interval I-
V 2/2, V2;2 I, the
.function u =cos x is continuous throughout the entire number scale.
Therefore, the function y= Vt/2-cos 2 xis continuous at all values
of x for which
,_ f n/4+2nn~x~3n/4+2nn,
Icos x I~ l1 2/2, i.e. \ 5n/4 + 2nn ~ x ~ 7n/4 + 2nn.

1.15.3. For each of the following functions find the points of


discontinuity and determine their character:
I I
(a) y= u-.,+ u- 2 , where u =x--1 ;
i x-1 for x ~ 0,
(b) y=u2 , where u = \ x+ 1 for x < O;
l-u 2
(c) y = 1 _._ ui , where u =tan x.
I

SoluUon. (a) The function


I
U=<p(X)=-
x-1
86 Ch. I. Introduction to Mathematical Analysis

suffers a discontinuity at the point X= 1. The function


I
Y = f (u) = u2+u-2

suffers a discontinuity at points where u2 + u-2 = 0, i.e. u 1 = - 2


and u2 = l. Using these values of u, find the corresponding values
of x by solving the equations:

whence x = 1/2 and x = 2.


Hence, the composite function is discontinuous at three points:
x 1 = 1/2, x 2 = 1, x3 = 2. Let us find out the character of disconti-
nuities at these points.
hm y= Jim y=O,
X'-+ J LI - J

therefore x 2 = 1 is a removable discontinuity.


lim y = lim y = oo; limy= limy=oo;
x - 1/2 u - - 2 X-+2 u-l

hence, the points x 1 = 1/2, X:i = 2 are discontinuities of the second


kind.
l.15.4. Given the function f(x)= 1/(1-x). FiHd the points of
discontinuity of the composite function
y = f {f [f (x)]}.
Solution. The po-int x = 1 is a discontinuity of the function

V=f(x)= 1 ~x
If x -=t= 1, then
I x-1
u= f [f (x)] = 1-1/(1-xJ = -..-- .

Hence, the point x = 0 is a discontinuity of the function


U=f[f(x)j.
If x -=t= 0, x =F I, then
I
y=f {f [f (x)]~ = l-(x-l)/x=X

is continuous everywhere.
Thus, the points of discontinuity of this composite function are
x = 0, x = 1, both of them being removable.
1.16. Funct. Cont. on Closed Interval: Properties 87

1.16. The Properties of a Function Continuous on a


Closed Interval. Continuity of an Inverse Function
I. The function f (x), continuous on the int<'rval [a, b], possess-
es the following properties:
(1) f (x) is boundl:'d on [a, b];
(2) f (x) has the minimum and maximum values on [a, b];
(3) If m = min f (x}, M = max f (x), then for any A satisfy-
a< x< b a<;x.<;b
ing the inequalities m ~A~ M there exists a point x 0 E [a, b] for
which f (x 0 ) =A.
In particular, if f (a) f (b} < 0, then we can find a point
c (a< c < b) such that f (c) = 0.
I I. Continuity of an Inverse Function. If the function y = f (x) is
defined, continuous and strictly monotonic on the interval X, then
there exists a single-valued inverse function x = cp (y) defined, con-
tinuous and also strictly monotonic in the range of the function
y -= f (x).
1.16.1. Does the equation sinx-x+l=O have a root?
Solution. The function
f (x} =sin x-x+ 1
is continuous over the entire number scale. Besides, this function
changes sign, since f (0) = 1, and f (3:rr/2) = - 3:rr./2. Hence, by pro-
perty (3) within the interval [O, 3:rr./2] there is at least one root
of the given equation.
1.16.2. Has the equation x&-18x + 2 = 0 roots belonging to the
interval [-I, l)?
1.16.3. Prove that any algebraic equation of an odd power v. ith
r<'al coefficients
a 0x 211 +i + a 1x 2n + ... + a 2nX + a 2n-t 1 = 0
has at Least one real root.
Solution. Consider the function
f (x) = aox2" + a1x2n + ... + a2nx + a2n+ t
t-J

v.hich is continuous throughout the number scale.


Let, for determinacy sake, a0 > 0. Then
Jim f (x) = + oo, and Jim f (x) = - oo.
x-+oo x~-oo

Hence, we can find numbers a, b, a < b such that f (a) < O;


f (b) > 0.By property (3), between a and b there exists a number
c such that f (c) = 0, which proves that the equation (*) l:ias at
least one real root.
88 Ch. I. Introduction to Mathemathil Analysis

t.16.4. Let the function f (x) be continuous on [a, bj and let


the equation f (x) = 0 have a finite number of roots on the inter-
val [a, b). Arrange them in the ascending order:
a < x1 < x < x < ... < x,, < b.
2 3

Prove that in each of the intervals


(a, x 1 ), (x 1 , x 2 ), (x 2 , Xa), , (x,,, b)
the function f (x) retains the same sign.
Solution. If the function changed its sign on a certain interval,
then we could find one more root of the function, which contradicts
the condition. To determine the sign of the function on any of the
indicated intervals it is sufficient to compute the value of the func-
tion at an arbitrary point of the appropriate interval.
1.16.5. Given a function on the interval [- 2, + 2):
f x2 + 2 if -2 ~ x < 0,
f (x) = \ -(x2 + 2) if 0 ~ x ~ 2.
Is there a point oR this closed interval at which f (x) = O?
Solution. At the end-points of the interval [- 2, +2] the given
function has different signs:
f(-2)=+6; f(+2)=-6.
But it is easy to notice that it does not become zero at any point
of the interval [-2, +2). Inrleed, x2 +2 > 0 and -(x2 +2) ;:: 0
at any x; this is due to the fact that f (x) has a discontinuity at
the point x = 0.
1.16.6. Does the function
f (x)=x3 /4-sin nx+3
take on the value 2 ! within the interval [-2, 2)?
Solution. The function f(x)=x 3 /4-sinnx+3 is continuous
within the interval [-2, 2). Furthermore, at the end-points of this
interval it attains the values
f (-2) =
l; f (2) = 5.
Since l < 2+ < 5, then, by property (3), within the interval
(-2, 2) there exists at least one p~lint x such that f (x) = 2+.
1.16.7. Show that the function
2x + I for - l ~ x < 0,
f (x) = { 2x for x = 0,
2x - l fnr 0 < X ~ l,
I.Iii. Funrt. Cont. on Closed Interval: Properties 89

define<l and bounded on the interval[-!, I], has neither maximum,


nor minimum values.
Solution. In the interval (-1, 0) the function increases from 3/2
to 2 and in (0, l] it increflses from 0 to 1, it does not attain either
the value 2 or 0. Therefore the function is bounded but never reaches
its upper and lower bounds. This is because there is a discontinuity
at the point x = 0.

t.16.8. Show that on any interval [a, b] of length greater than


unity the function f (x) =x-E (x) attains its minimum value but
never reaches its maximum.
Solution. In flny interval [n, n +I), where n is an integer, the
given func1.ion f (x) increases from 0 to 1, never attaining the maxi-
mum. Hence, 0 ~ f (x) < I for any
x-. Since on the interval [a, b] we !J
can find at least one internal in-
tegral point n, then f (n) = 0 and
Jim f (x) = 1, but f (x)=F I for any
x~n-0 x
x. It means that the function reaches -2 -f a 1 2 3
its minimum value but never
reaches its maximum. This is be- Fig. 32
cause there is a discontinuity at the point x = n (see Fig. 32).
L 16.9. Prove that the function y = 2 n+ Vx (n a naturai number)
is continuous throughout the number scale, consirlering it as a function
inverse to y =x 2 "+ 1
Solution. The function y = x 211 + 1 is continuous and increases from
- oo to oo over the entire number scale. Hence, the inverse function
V!/
x = 211 + is defined for all y, continuous and increasing. Denoting
the independent variable again as x, we find that the function
y = 211 + ~/x possesses the required properties.
1.16.10. Prove that for any function of the form
y=aox2n+1+a1x2n-1+a2x2n-3+ ... +a,,x+an+t> (*)

where a11 , al> a 2 , , a,,, a,,+ 1 are positive numbers, there exists
an inverse function increasing and continuous throughout the num-
ber scale.
Solution. As is known, the functions x, x 3, x", ... , x 211 + 1 incrrase
throughout the entire number scale. Then, since the coefficients
+
a; (i = 0, 1, ... , n 1) are positive, the function f (x) =a 0 x 211 + 1 +
+a 1x2"- 1+ ... +a,,x+a,,+ 1 also increases. Furthermore, it is con-
tinuous. Therefore, for a function of the form (*) there exists an
inverse function increasing and continuous over the entire number
scale.
90 Ch. I. Introduction to Mathematical Analysis

Note. This example establishes only the existence of an inverse


function x = g (y), but gives no analytic expression for it. It is not
always possible to express it in radicals. The problems of the exis-
tence of an inverse function and of expressing it analytically should
not be confused.
1.16.11. Prove that there exists only one continuous function
x = x (y) (- oo < y < oo) which satisfies the Kepler equation:
x-esinx=y (O<e<l).
Solution. Let us show that y (x) is an increasing function. Let
x 1 < x 2 be arbitrary points on the number scale. Then
y (x 2 )--y (x 1 ) = (x 2 -e sin x 2 )-(x 1 -e sin x 1 ) =
= (x 2 -x1 )-e (sin x 2 -sin x 1).
Estimate the absolute value of the difference I sin x 2 -sin x1 j:
Ism x 2 -sin x 1 I=21 sin x2 -;-x1 II cosx2 tx 1 j ~
---21 X2-X1,__....2/X2-X1/
::::::::: sm - 2 - ::::::::: 2 = IX2-X1 I = (X2-X1}
Since 0 < e < I,
ejsinx2 -sinx 1 I < (x 2 -x1 ),
whence
(x2 -x 1)-e (sin x 2 -sin x 1 ) = y (x 2 }-y (x1 ) > 0.
Since y (x) is a continuous function in the interval (-- oo, oo), the
inverse function x is a single-valued and
!! continuous function of y.
1.16.12. Show that the equation
x 3 -3x+ I =0
has one root on the interval [I, 2]. Cal-
--<--------x culate this root approximately to within
1 two decimal places.
1.16.13. The function f (x} is defined
-1 on the interval [a, b] and has values of
Fig. 33 the same sign on its end-points. Can one
assert that there is no point on [a, b]
at which the function becomes zero?
1.16.14. Prove that the function
f x+I at -I~x~O,
f (x) = \ - x at 0 < x~ I
is discontinuous at the point x = 0 and still has the maximum and
the minimum value on [- 1, I] (see Fig. 33).
1.17. Additional Problems 91

1.17. Additional Problems

(a) n! < (nt 1


I 3 5
r
1.17.1. Prove the inequalities:
for a natural
2n- l I
n> 1;
(b) -2 ' -4 ' -6 - 2 -
n
< .r
r 2n 1- I
1.17.2. Prove the inequalities:
(a) 202 303 > 303 202 ;
(b) 200! < 100200
1.17.3. Solve the inequalities:
(a) \lxl-21~1;
(b) II 2-3xl- l I> 2;
(c) (x-2)Vx2 +l >x2 +2.
1.17.4. Can a sum, difference, product or quotient of irrational
numbers be a rational number?
1.17.5. Do the equations
(a) lsinxl=sinx+3, (b) jtanxl-=tanx+3
have any roots?
1.17.6. Prove the identity ( x+Jx' r +( x-Jxl r
=X2

1.17.7. Prove the Bernoulli inequality


(1 + X1 ) (I+ X2 ) (1+Xn)~1 + XJ + X 2 + ... + Xn,
where x 1 , x 2 , , xn are numbers of like sign, and 1 + x; >O
(i = 1, 2, ... , n).
1.17.8. Find the domains of definition of the following functions:
(a) f (x} = VXCX2;
(b) f (x) =}/sin V x;
(c) f (x) = V - sin 2 nx;
(d) f (x) = .~ and g(x) = 1 ;
r lxl-x Yx-Jxl
(e) f (x) =arc sin (Ix l-3);
I
(f) f (x) =arc cos sin x .

1.17.9. Are the following functions identical?


(a) f (x) =!...x and cp (x) == 1;
~b) f (x) =log x 2 and cp (x} = 2 log x;
92 Ch. I. Introduction to Mathematical Analysis

(c) f (x) =x and qi (x) = (Vx) 2 ;


(d) f (x) =--= I and qi (x) = sin 2 x+ cos 2 x;
(e) f(x)=log(x-l)+log(x-2) and qi(x)=log(x-l)(x-2).
t.17 .10. In what interval are the following functions identical?
(a) f(x)=X and qi(x)=lOlogx;
(b) f(x)=VxJlx-1 and fp(x)=Vx(x-1).
1. 17 .11. An isosceles triangle of a given perimeter 2p = 12 revol-
ves about its base. Write the function V (x), where V is the volume
of the solid of revolution thus obtained and x is the length of the
lateral side of the triangle.
t.17.12. lnves.tigating the domain of definition of functions.
(a) solve the inequality
Vx-t2+Vx-5~V5-x;
(b) prove that the inequality
log 2 _x (x-3) ~-5
has no solutions.
1.17.13. The function y=signx was defined in Problem 1.5.11 (n).
Show that
(a) Ix I = x sign x;
(b) x = Ix I sign x;
(c) sign (sign x) =sign x.
1.17.14. Prove that if for a linear function
f (x) =ax+b
the values of the argument x = xn (n = 1, 2, ... ) form an arithmetic
progression, then the corresponding values of the function
y,,=f(x,,) (n=l,2, ... )
also form an arithmetic progression.
1.17.15. Prove that the product of two even or two odd functions
is an even function, whereas the product of an even and an odd
function is an odd function.
1.17.16. Prove that if the domain of definition of the function
f (x) is symmetrical with respect to x = 0, then f (x) + f (- x) is an
even function and f (x)-f (- x) is an odd one.
1.17 .17. Prove that any function f (x) defined in a symmetrical
interval (- l, l) can be presented as a sum of an even and an odd
1.17. Additional Problems 93

function. Rewrite the following functions in the form of a sum of


an even and an odd function:
(a) f (x) = -x+2
-,-., ;
1,x (b) y =a".

1.17.18. Extend the function f(x)=x 2 +x defined on the inter-


val [O, 3] onto the interval [- 3, 3] in an even and an odd way.
1.17 .19. The function {x} = x- E (x) is a fractional part of a
number x. Prove that it is a periodic function with period I.
1.17 .20. Sketch the graph of a periodic function with period
T=I defined on the half-open interval (0, l] by the formula y-=X2
1.17 .21. Let us have two periodic functions f (x) and cp (x) defined
on a common set. Prove that if the periods of these functions are
commensurate, then their sum and product are also periodic functions.
1.17.22. Prove that the Dirichlet function A. (x) (see Problem
1.14.4(b)) is a periodic one but has no period.
1.17 .23. Prove that if the function
f (x) =sin x +cos ax
is periodic, then a is a rational number.
1.17 .24. Test the following functions for monotony:
(a) f(x)=lx/; (b) f(x)=jx/-x.
1.17 .25. Prove that the sum of two functions increasing on a
certain open interval is a function monotonically increasing on this
interval. Will the difference of increasing functions be a monotonic
function?
1.17.26. Give an example of a non-monotonic function that has
an inverse.
1.17 .27. Determine the inverse function and its domain of de-
finition if
x if - oo < x <I,
(a) y -- tanh x; (b) y = { x~ if I~ x~ 4,
2" if 4 < x < 00.
1.17.28. Show that the equation x2 +2x+l=-l+Vx has no
real roots.
1.17 .29. Construct the graph of the function
y= f (x-1) + f (x + /),
94 Ch. I. Introduction to Mathematical Analysis

where
/ k(l-lxl/l) at lxl~l
f(x) = \ 0 at Ix I> l.
1.17.30. Knowing the graph of the function y=f(x), sketch the
graphs of the following functions:
(a) y=f 2 (x); (b) y=V-f(x); (c) y=f[f(x)].

1.17.31. Prove that the graphs of the functions y=logax and


y = logan x can be derived from each other by changing all ordinates
in the ratio I: l/n.
1.17.32. Prove that if the graph of the function y=f (x), defined
throughout the number scale, is symmetrical about two vertical
axes x =a and x = b (a< b), then this function is a periodic one.
1.17 .33. Let the sequence xn converge and the sequence Yn diverge.
What can be said about convergence of the sequences
(a) xn+ Yn; (b) XnYn?
1.17 .34. Let the sequences xn and Yn diverge. Can one assert that
the sequences xn + y,,, xny,. diverge too?
1.17 .35. Let an be an interior angle of a regular n-gon (n = 3,
4, ... ). Write the first several terms of the sequence a,.. Prove
that Jim a,.= n.
1.17 .36. Prove that from Jim xn =a it follows that Jim jx,,j = lal.
n-oo n~oo

Is the converse true?


1.17.37. If a sequence has an infinite limit, does it mean that this
sequence is unbounded? And if a sequence is unbounded, does it
mean that it has an infinite limit? Prove that x,, = n<- nn is an
unbounded but not an infinite function.
1.17 .38. Prove that the sequence {a,,}, where a,, is the nth digit
of an arbitrarily chosen irrational number, cannot be monotonic.
1.17 .39. Prove that if the sequence {a,,/b,,} (bn > 0) is monotonic,
then the sequence

will also be monotonic.


1.17.40. Prove the existence of limits of the following sequences
and find them.
(a) v~r. V 2 ~T. v 2 V 2 V2 ... ;
1.17. Additional Problrms 95

(b) Xn=C 11 !1/nf (c > 0, k > O);


(c) x11 = a,,/n, where a,, is the nth digit of the number :n:.
1.17.41. Prove that at an arbitrarily chosen x the sequence
{E ~nx) { is bounded.
t.17.42. Prove that the sequence

{ (x)+E (2x) 4- ... +E (nx)}


n~

has the limit x/2.


1.17.43. Prove that
Jim ah= 1 (a > 0).
h - 0

l.17.44. Given the function

f (x) = { 1 +x for x =l= 0,


0 for x=O.
Prove that
Jimf(x)=l.
<- 0
l.17.45. Let
p ( ) = a 0 x11 f- a 1 x11 - 1 f- ... + a 11

X b0 x"'+b 1 x"'-'--J- ... +bm


(a 0 =I= O; b0 =I= 0).
Prove that
oo, if n > m,
lim P (x) = { a11 /b 0 , if n = m,
x- 00 0, if n < m.
1.17.46. Find the constants a and b from the condition:

(a) }~1! ( ffi-ax-b) ~ 0;


(b) lim (J;-x 2 -x+ 1 -ax-b)=O.
x --t- - 00

1.17.47. Sketch the graphs of the followin6 functions:


(a) f (x) = lim j'/ I + xn (x;;;?: O);
ll -)> 00

(b) f (x) = lim sin 2 " x.


fl-+ 00

1.17.48. Prove that


Jim [(l--j-x)(l--j-x 2 )(1--j-x 4 ) (l \-x 211 )}= l~x(lx!<l).
11 -+ 00
96 Ch. I. lntroductinn to Mathematical Analysis

1.17.49. Can one replace infinitesimal summands by equivalent


infinitesimals in computing a limit?
1.17 .50. Determine the order of smallness of the chord of an
infinitely small circular arc relative to the sagitta of the same arc.
1.17.51. Determine the order of smallness of the difference of the
perimeters of an inscribed and circumscribed regular n-gons rela-
tive to an infinitely small side of the inscribed n-gon.
1.17.52. The volumetric expansion coefficient of a body is con-
sidered to be approximately equal to the triple coefficient of linear
expansion. On equivalence of what infinitesimals is it based?
1.17.!13. Does the relation log(l +x)- x hold true as x-0?
1.17.54. Will the sum of two functions f(x)+g(x) be mcessarily
discontinuous at a given point x0 if:
(a) the function f (x) is continuous and the function g (x) is dis-
continuous at x = x0 ,
(b) both functions are discontinuous at x= x0 ? Give some examples.
1.17 .55. Is the product of two functions f (x) g (x) necessarily
discontinuous at a given point x 0 if:
(a) the function f (x) is continuous and the function g (x) is dis-
continuous at this point;
(b) both functions f (x) and g(x) are discontinuous at x= x1?
Give some examples.
l .17 .56. Can one assert that the square of a discontinuous func-
tion is also a discontinuous function? Give an example of a func-
tion discontinuous everywhere whose square is a continuous function.
1.17 .57. Determine the points of discontinuity of the following
f undions and investigate the character of these points if:
I
(a) f(x) = l-ex/(1-x);
(b) f(x)=2-21/0-x>;
(c) cp(x)=x[l-2A.(x)], where A.(x) is the Dirichlet function (see
Problem 1.14.4 (b)).
1.17.58. Test the following functions for continuity and sketch
their graphs:
(a) y=x-E(x);
(b) y=x2 +E(x2 );
(c) y = (- l)E<x'>;

(d)y=lim +( 2 x. )2n
n-+"" 1 sm x
1.17. Additional Problems 97

1.17.59. Investigate the functions f[g(x)] and g[f(x)] for conti-


nuity if f (x) =sign x and g (x) = x (l -x2 ).
1.17.60. Prove that the function
2x at -1 ~x~O,
{
f(x)= x+ 1 / 2 at O<x~l
is discontinuous at the point x = 0 and nonetheless has both maxi-
mum and minimum values on [-1, l].
1.17.61. Given the function
-{ (x+l)2-< 111x1+i/x) if x=;i=O,
f (x) - O - 0.
1'f x -

Ascertain that on the interval [-2, 2) the function takes on


all intermediate values from f (-2) to f (2) although it is discon-
tinuous (at what point?).
1.17.62. Prove that if the function f (x): (l) is defined and mo-
notonic on the interval [a, b]; (2) traverses all intermediate values
between f (a) and f (b), then it is continuous on the interval [a, b].
1.17.63. Let the function y=f (x) be continuous on the interval
[a, b], its range being the same interval a~y~b. Prove that on
this closed interval there exists at least one point x such that
f (x) =X. Explain this geometrically.
1.17 .64. Prove that if the function f (x) is continuous on the
interval (a, b) and x1 , x 2, ... , xn are any values from this open
interval, then we can find among them a number s
such that

f (s) =-nI [f (x1) + f (x2)+ ... + f (xn)].


1.17 .65. Prove that the equation x 2x =l has at least one posi-
tive root which is less than unity.
1.17 .66. Prove that if a polynomial of an even degree attains at
least one value the sign of which is opposite to that of the coeffi-
cient at the superior power of x of the polynomial, then the latter
has at least two real roots.
1.17 .67. Prove that the inverse of the discontinuous function
y=(l +x2 ) sign xis a continuous function.

4--3118
Chapter 2
DIFFERENTIATION
OF FUNCTIONS

2.1. Definition of the Derivattve


The derivative f' (x) of the function y = f (x) at a given point x
is defined by the equality
f'()
x = 11m - tJ.y
= 11m
. f(x+tJ.x)-f(x)
l'J.X
-+ o tJ.x t;.x o
-+ 11x
If this limit is finite, then the function f (x) is called differen-
tiable at the point x; and it is infallibly continuous at this point.
Geometrically, the value of the derivatiye f' (x) represents the
slope of the line tangent to the graph of the function y = f (x) at
the point x.
The number
f , (x) = Jim f (x+ tJ.x)- f (x)
+ t;.x-+ + O tJ.x

is called the right-side derivative at the point x


The number
f ,_ (X)-
-
.
1Im f (x+tJ.x)
l1
- f (x)
6.x-+-0 X

is called the left-side derivative at the point x.


The necessary and sufficient condition for the existence of the
derivative f' (x) is the existence of the finite right- and left-side
derivatives, and also of the equality f'_ (x) = f ~ (x).
If f' (x) = oo, the function f (x) is said to have an infinite deri-
vative at the point x. In this case the line tangent to the graph
of the function y = f (x) at the point x is perpendicular to the
x-axis.
2.1.1. Find the increment ~y and the ratio ~~ for the following
functions:
(a) y = vx at X= 0 and ~X=0.0001;
I
(b) y= x2 + x- 6 at X= I and ~x=0.2.
2.1. Definition of the Derivative 99

Solution. (a) !iy = ~; x+ lix-Vx = Vo.0001 =0.01;


Ay 0.01
IOO. tJ..x = 0.0001 =
2.1.2. Using the definition of the derivative, find the derivatives
of the following functions:
(a) y=cosax; (b) y=5x2 -2x.
Solution. (a) liy=cosa(x+lix)-cosax=
=- 2sin (ax+; !ix) sin ; x;

Ay -2sin ( ax+-f Ax) sin-f Ax


tJ..x tJ..x '
. a A
. 8 . . . Sin 2 X
y' = ltm ~ Y = -2 l11n sin (ax+!(:-- !ix) !rm Ax = - a sin ax.
11.x - 0 X 11.x -+ 0 2 11.x ... 0
In particular, if a= l, then y =cos x and y' =-sin x.
2.1.3. Show that the following functions have no finite derivati-
ves at the indicated points:
(a) y = V
x3 at the point x =0; !I
(b) Y=Vx-1 at the point x=l;
(c) y=31xl+ 1 at the point x=O.
Solution. V (x + lix) -Vx
(a) !iy = 3 3

At x=O we have liy= V l\xa, 8y = VAx = 3

Ax tJ..x

=vi . ; hence, y' (0) = lim vi


J Ax 2 11.x ... o Ax2 = oo, __.__ __ . . _ _ . _ . :C '
-1 0 1
i.e. there is no finite derivative. .
(c) At l\x > 0 the increment of the func- Fig. 34
tion !J.y atx=O will be: liy=3(O+lix)+1-1 =31ix. Therefore
I"un ;:r-
Ay = 3.
11.x-+ +O X

At !ix< 0 the increment of the function !iy will be


liy = - 3 (0 + !ix)+ 1-1 = - 3/ix,
hence,
Jim tJ..y _ _ 3
11.x-+ -0 Ax- '

Since the one-sided limits are different, there is no derivative at


the point x=O (see Fig. 34).
4
100 Ch. If. Difjerentiation of Functions

2.1.4. Investigate the function y =I In x I for differentiability at


the point x = 1.
Solution. At x = 1
11y =I In (1+11x) 1-1 ln l j =I ln(l + 11x) I.
i. e.
11 = j ln (1 + /1x) I= { Jn ( 1+ 11x) at 11x:;;;::: 0,
Y - In (1 + 11x) at 11x < 0.
Therefore
fly
In (I+ 11x)
!1x a
t A
uX
> O,
{
L1x = - In ( 111: 11x) at 11x < 0,
whence
lim ~Y = + 1 and lim ~Y = - 1.
/'>.x - +0 X il.x - - 0 X

Since the one-sided limits are different, there is no derivative.


Hence, the function y = j ln x I is not differentiable at the point
x=l (see Fig. 35).
2.1.5. Find the average velocity of
!f motion specified by the formula
S=(t 2 -5t+2) m
from t 1 = 5 sec to t 2 = 15 sec.
2. 1.6. Using the definition of the
0 1 x derivative, find the derivatives of
the following functions:
Fig. 35 (a) y=x 3 ; (b) y= l/x2
2.1.7. Investigate the function Y=lcosxl for differentiability at
the points x = n/2 + nn (n an integer).

2.2. Differentiation of Explicit Functions


I. Basic Rules of Differentiation
(l)c'=O;
(2) (u+v)'=u'v';
(3) (cu)' =cu';
+
(4) (uv)' = u' u uv', the product rule;
v =
(5) ( u )' u'v-uv'
v2 (v =I=- 0), the quotient rule.
Here c =canst, and u and v are functions of x which have deri-
vatives at a corresponding point.
(6) If the function u =qi (x) is differentiable at the point x0 , and
the function y=f(u) is differentiable at the point u0 =qi(x 0 ), then
2.2. Differentiation of Explicit Functions IOI

the composite function y = f (qi (x)) is differentiable at the point x0


and y~ (x0 } = y~ (u 0 ) u~ (x0 }, the function of a function, or chain, rule.
II. Differentiation of Basic Elementary Functions
(1) (u")' = nu 11 - 1 u'; (2) (sin u)' =cos U u';
(3) (cosu)'=-sinu-u';
u' u'
(4) (tan u)' = cos 2 u; (5) (cot u)' = - sin2 u;
u'
(6) (In u)' = u;
(7) (au)'= au In a u'; 8) (e")' = euu';
(9) (sinh u)' =cosh U u';
(10) (coshu)'=sinhu-u';
u'
(11) (arcsinu)'=y . =-(arccosu)';
l-u 2
u'
( 12) (arc tan u)' = 1+u 2 = - (arc cot u)'.
2.2.1. Find y', if:
(a) y = 5x 2 13 -3xl 2 + 2x- 3;

(b) y =
a
V x. -
2 x
v-
x
b t
(a, b constan s).

Solution. (a) y'=5: x 213 -i_3. ~ xl 2 - 1 -2.3x- 3 - 1 = }~----


3V x
-~xVx-:4
2.2.2. Find y', if:
. sinx+cosx
(a) y=3cosx+2smx; (b} y=sinx-cosx;
(c) y = (x 2 + 1) arc tan x; (d) y = x 3 arc sin x.
Solution. (a) y'=3(cosx)'+2(sinx)'=-3sinx+2cosx;
(b) , =(sin x+ cos x)' (sin x-c~s x)- (sin x-cos x)' (sin x+ cos x) =
Y (sm x- cos x) 2
(cos x-sin x) (sin x-cos x)-(cos x+ sin x) (sin x+cos x)
(sin x- cos x)2
2
(sin x-cosx) 2 '

(d) y' = (x 3 )' arc sin x+ (arc sin x)' x 3 = 3x2 arc sin x+ y 1 x~ x2 .
2.2.3. Find the derivative of the given function and then com-
pute the particular value of the derivative at the indicated value
of the argument:
(a) f (x} = 1-V
x2 + 16/x at x = - 8;
102 Ch. II. Differentiation of Functions

(b) f(x)=(l-Vx) 2 /x at x=0.01;


(c) f(t)=(cost)/(1-sint) at f=n/6.

v3 ,--
Solution. (a) f'(x)=- 2 x- 113 -l6x- 2 = - 2 ~.1
3
3 x x
Putting x = - 8, we obtain
f' (-8) =
3 v
2
-8
16
(-8)2=12;
l

(c) f'(t)=-sint(l-s.inl)+cos 2 t = 1. .
(1-smt) 2 1-smt
Whence f' (n/6) = 2.
2.2.4. Taking advantage of the differentiation formulas, find the
derivatives of the following functions:
(a) y=2x 3 +3x-5; (b) y=Vx + /~-+O.lx 10 ;
2x 2 +x+ I
(c) y = x2 -x-t- I ' (d) y =
x+ Vx
V ,
x-2 x
e y = cosqi+sinqi
() ---o-~-~ (f) y = 2ex + In x;
1-cosqi
(g) y=ex(cosx+sinx); (h) y=ex~e~inx.
2.2.5. Taking advantage of the rule for differentiation of a com-
posite function find the derivatives of the following functions:
(a) y=sinax; (b) y=lntanx; (c) y=Scosx;
+
(d) y =In sin (x 3 l); (e) y =arc sin Vf=Xi;
(f) y =In" (tan 3x); (g) y = sin 2 V1/( 1-x).
Solution. (a) Here the role of the external function is played by
the power function: sin x is raised to the third power. Differentiating
this power function with respect to the intermediate argument
(sin x), we obtain
3
( Slll )' 3 . 2
X sin x = Sl!J X,

but the intermediate argument sin x is a function of an independent


variable x; therefore we have to multiply the obtained result by
the derivative of sin x with respect to the independent variable x.
Thus, we obtain
y: = (sin 3 x):in x (sin x)~ = 3 sin 2 x cos x;

(b) , = (I n t an X )tan x ( t an x )'x =tanl x cosl2 x =sin2 2x;


Yx
(c) y; =(Seas x)~os x (cos x)~ = 5cos x In 5 (-sin x) = - 5cos x sin x In 5;
2.2. Diflerentiation of Explicit Functions 103

(d) y~= [In sin (x 3 + l)fsin <x'+1> [sin (x 3 + 1)]~+ 1 [x 3 + l]~=


I
= sm . (x.3 + I) cos (x3 + 1) 3x2 = 3x2 cot (x 3 + 1)

(e) y~ =(arc sin Vl-x2 )'vi-::-;:2 (Vl - x2)1-x (l-x 2 )~=


I I x
Vt-(l-x2 ) 2 Vt-x 2 (- 2x) =- lxJ J!1-x2 (x=#=O).
2.2.6. Find the derivatives of the following functions:
(a) y = (1+3x + 5x2 ) 4 ; (b) y = (3-sin x)3;
(c) y= VSTr12X+ l/cos 2 x;
(d) Y=V2ex+2x+l+ln&x;
(e) y =sin 3x +cos (x/5) +tan Vx;
(f) y =sin (x 2 - 5x + 1) + tan (a/x);
(g) y =arc cos Vx;
(h) y =arc tan (In x) +In (arc tan x);
(i) y = In 2 arc tan (x/3);

(j) y = v + -{ + vx.
x x
Solution. (a) y' = 4 ( l + 3x+ 5x2) 3 (1+3x+ 5x2) ' =
=4(1 +3x+5x2 ) 3 (3+ lOx);
(g)y'=- I (Vx)' =- 1 ~=- t ;
Vt-(Vx) 2 Yl-x 2 Yx 2 fx(l-x)

(j) y' =
2
I i
x+Vx+Vx
l + V x+Yx
1
2
i ( 1 + 2.1r-x-)
Y
J.
2.2.7. Find the derivative of the function
. 2x
y=arcsm l+x 2

We have

i. e.
, { 1 ~x 2 at Ix I < l,
y = 2
- I+ x2 at Ix I > 1.
At Ix I = l the derivative is non-existent.
2.2.8. Find the derivatives of the following functions:
(a) y = sinh 5x cosh (x/3);
104 Ch. II. Differentiation of Functions

(b) y = coth (tan x)-tanh (cot x);


(c) y =arc cos (tanh x) + sinh (sin 6x);
(d) y = sinh 2 x3 + cosh 3 x2 ;
esinh ax
(e) Y = sinh bx- cosh bx
Solution.
(a) y' = (sinh 5x)' cosh ; + sinh 5x ( cosh ; ) '=

= 5 cosh 5x cosh ; +-} sinh 5x sinh ; ;


(c) y' = jtanh x)' +cosh (sin 6x) (sin 6x)' =
l-tanh2 x
= _ l/cosh 2 x +
y (cosh x- sinh 2 x) /cosh2 x
2

+ 6 cos 6x cosh (sin 6x) = - -cos1h-x + 6 cos 6x cosh (sin 6x).


2.2.9. Find the derivatives of the following functions:
(a) y= il V xa (x2+ I)
5 -x; (b) y=[u(x)]v x> (u(x)>O);
1

(c) y= v~ l-x sin 3 xcos 2 x


I +x2 '

(d) y= JI ( r--)x+l
tanx .
Solution. (a) Apply the method of logarithmic differentiation.
Consider, instead 6f y, the function
\x3\(x2+l) I I
z=ln!y)=ln V V 15-xl
0
=lnlxl+-
3 ln(x2 +1)--
15 lnl5-x[.

Taking into account that (In I u) )' = u' /u, we have


, I 2x I -24x3+ 125x2 -14x+ 75
z =x+3(x 2+1)+15(5-x) 15x(x2 +1)(5-x)
But z' =(In I y I)'= y' /y, whence
,_ z ' - V x 3 (x 2+I). -24x3+125x2-J4x+75
y -Y - Vs-x 15x(x2 +1 )(5-x)
(b) Suppose the functions u (x) and v(x) have derivatives in the
given domain of definiti.on. Then the function
z= lny=vlnu
also has a derivative in this domain, and
u'
z' = (v In u)' = v' In u + v -u
. 2.2. Differentiation of Explicit Functions 105

Hence, the function


y =elny== ez
also has a derivative in the indicated domain, and
y' = ezz' = yz'.
Thus,
y' = uv ( v' In u+ v~') =vuv- 1 u' +uv In Uv'.

2.2. IO. Show that the function y = xe-x'f 2 satisfies the equation
xy' = (l-x2 } y.
Solution.
y' = e-x12-x2e-x12 = e-x 2 12 (l-x2);
xy' = xe-x'f 2( l -x2 }.
Hence,
xy' = y (l-x2 ).

2.2.11. Show that the function y = xe-x satisfies the equation


xy' = (1-x} y.
2.2.12. Investigate the following functions for differentiability:
(a) y=arcsin(cosx}; (b) y=y1-V1-x2

Solution. (a) y' = {cos x)' sin x - ~ .


Y I - cos2 x V sin2 x Ism x I
Hence, y' = -1 at points where sin x > O; y' = I at points where
sin x < 0. At points where sin x = 0, i.e. at the points x =kn
(k = 0, +I, 2, ... ) the function, though continuous, is not dif-
ferentiable.
(b) The domain of definition of this function is the interval
-l~x~l.

y' -- 1 .r-I (-2x) at x =;b 0 and x =;b 1.


-- 2V1-v 1-x2 2 r l--x2

As X-+ 1-0 or x-.- l + 0 we have y' -++ oo. Let us find


out whether the derivative y' exists at the point x = 0, i.e. whe-
ther
.
I 1m
JI I- YI -
A
fJ.x 2
exists.
I\~- 0 u.X

Since V. l-i1x2 - l,.., _ I


2 dx2 , then

.
J.im
VI - .r
r I - fJ.x 2
= Jim
V1-
r-
-
2
fJ.x 2
__
{ ,}.,,
r 2 I
as L1x-+O,

o
i\x ... fJ.x i\x... 0 fJ.x - VT as dX-+-0.
106 Ch. 11. Differentiation of Functions

Thus, y'_ (0) =I= y: (0), which means that the function under consi-
deration has no derivative at the point x = 0, though it is cont i-
nuous at this point.
Note. There are cases of failure of existence of f' (x) and even of
t: (x) and f'_ (x) at a given point, i.e. when the graph of the f unc-
tion has neither a right-, nor a left-side tangent at the given point.
For instance, the function
f(x)= f xsin(l/x) at x=1=0,
\ 0 at x=O
is continuous at the point x = 0, but does not have even one-sided
. t
denva . tl{(x) . I
1ves, smce -XX= sm !1x.
2.2.13. Find the derivatives of the following functions:
(a) f (x) = sinh (x/2) + cosh (x/2);
(b) f(x)=ln[coshx]; (c) f(x)=2Vcoshx-l;
(d) f(x)=arcsin[tanhx];
(e) f (x) =Vt+ sinh 2 4x;
(f) f (x) = eax (cosh bx+ sinh bx).
2.2.14. Applying logarithmic differentiation find the derivatives
of the following functions:
(a) y=(cosx)slnx; (b) y= V sin3x
l-sin3x;
Vx-1
(c) y= V (x+2)2 JI (x+ 3) 3

2.2.15.
cos 2 x
f (x) = I -1- sin2 x;
show that
f (n/4)-3f' (n/4) = 3.
2.2.16. Show that the function
x-e-x
y= 2x2

satisfies the differential equation


+
xy' 2y = e-x.
2.2. 17. Find the derivatives of the following functions:
(a) y =In cosVarc sin 3- 2 " (x > O);
(b) y= Varc tan V co~ ln 3 x.
2.3. Successive Di(Jerentiation. Leibniz Formula 107

2.3. Successi'Ve Differentiation of Explicit Functions.


Leibniz Formula
If the derivative of the (n- l)th order of a function y= f (x) is
already found, then the derivative of the nth order is determined
by the equality
y<n> (x) = [y<n-ll (x)]'.
In particular, y"(x)=[y'(x)]', y"'(x)=[y"(x)]', and so on.
If u and v are functions differentiable n times, then for their
linear combination c1 u + c2v (c 1 , c2 constants) we have the following
formula:
(c1 u + c2v)<n> -
-1c u<n + c2
v<n>,
and for their product uv the Leibniz formula (or rule)
(uv)<n> = u<mv+nu<"-uv' + n(n-1)
1. 2 u<n- 2
,,
v +
n
+ ... + uv'n> = ~ C~u<n-k>v<k>,
k=O
<o>_ <o>- d Ck_n(n-1) ... (n-k+I)_ n!
where u - u, v -v an n- 1. 2 . 3 . .. k - kl (n-k)! are
binomial coefficients. Here are the basic formulas:
(1) (xmr>=m(m-1) .. . (m-n+ l)xm-".
(2) (ax)<ll) =ax Inn a (a > 0). In particular, (ex)<n> =ex.
(3) (lnxr> =(-l)n-1 (n nl) I.
x
(4) (sinx)<n>=sin(x+nn/2).
(5) (cosx)<n> =cos(x+nn/2).
2.3.1. Find the derivatives of the nth order of the following
functions:
(a) y=lnx; (b) y=ekx; (c) y=sinx; (d) y=sin5xcos2x;
(e) y=sinxcosx; (f) y=sin3xcos2 x; (g) y=ln(x2 +x-2).
Solution.
(a) y'=+=x- 1 ; y"=(-l)x- 2 ; y"'= 12x- 3 ;
y<4l '---=- l 23X-4; ... ; y<n> =(-l)n-1 (n-1) ! x-"= (-1)11-:"(n-I) I.
(c) y' =cosx= sin (x+n/2);
y" =cos (x + n/2) =sin (x + 2n/2).
In general, if we assume that for a given n = k

y<k> =sin ( x + k ; ) ,
108 Ch. II. Differentiation of Functions

then it will turn out that


y<k+o=cos (x+k ~)=sin [ck+ 1) ~ +x].
Whence, by virtue of mathematical induction we conclude that for
afly natural n
y<n>=sin(x+n ~)
(d) y =sin 5xcos 2x= f [sin 7x+ sin 3x]
Therefore
yim=f [1nsin(7x+n~)+3"sin(3x+n~)J.
I 2x+1
(g) Y = x2+x-2
To simplify the computations let us transform the obtained
function:
,= 2x+ I = (x+2)+(x- l) =-'-+-'-= (x- l)-1+-(x+ 2)-1.
Y x2 +x-2 (x-l)(x+2) x-1 x+2
Whence
y" =-1 (x-1)- 2 - l (x+2)- 2 ;
y"' = l 2 (x-1)- 3 + l 2 (x + 2)- 3 ;

y<n> = (-l)n-l (n- 1) ! [(x-1)-n + (x+ 2)-"] =


-- (- l)n-I (n- 1) I [ - 1- -t - 1 - ]
(x-1)" (x+2)"
- fin d y<n>.
ax+b
2 ' 3 2 ' y = cx+d'
-
Solution. Transform the given expression in the following way:

Y
= ax+ b
ex+ d
= ~
c
+ cbe- ad = ~ + be - ad (ex
(ex+ d) c c
_L
1
d)- 1
Whence
y' = (-1) bc-:-ad e (ex+ d)-2,
e

y" = (-1) ( -2) be-; ad e2 (ex+ d)-a,

y"' = (-1) (-2) (-3) be-ad e3 (ex+ d)- 4 ,


c

y<n) = (-l)"n! be-ad en (ex +d)-(n-t-1) =


e
n' cn-l
= (-1)" (ex +d)n+1 (be-ad).
2.3. Successive Differentiation. Leibniz Formula 109

2.3.3. y=x/(x2 - l); find y(m.


Solution. Transform the given expression

therefore (see Problem 2.3.2):


(nJ_(-l)nn! [ I --j I ]
Y - 2 (x+ l)n+l (x-l)n+l

2.3.4. Using the Leibniz formula, find the derivatives of the


indicated orders for the following functions:
(a) y = x2 sin x; find y( 25 l;
(b) y=ex (x2 - l); find y( 24 >;
(c) y = ex sin ~x; find y<n>.

Solution. (a) y( 25 >=(sin x. x2 )( 25 i = (sin x)< 25 i x 2 + 25 (sin x)< 241 (x2 )' +
+ 25 ; 24 (sin x)( 23 >(x 2 )'', since the subsequent summands equal zero.
Therefore
y( 25 i = x 2 sin ( x + 25 ; ) + 50x sin ( x + 24 ; ) + 600 sin ( x + 23 ; ) =

= (x2 -600) cosx+ 50x sin x.

2.3.5. Compute the value of the nth derivative of the function


3x+2 .
y = x2 _ 2x+S at the po mt x = 0.

Solution. By hypothesis we have y (x) (x 2 -2x+ 5) = 3x+ 2. Let


us differentiate this identity n times using the Leibniz formula;
then (for n ~ 2) we obtain
n (n-1)
yn(x)(x 2 -2x+5)+nyu- 1l(x) (2x-2)+ 2 y<n- 2 >(x)2=0.

Putting x = 0, we have
5y<n> (0)- 2ny(n-u (O)+ n (n- 1) g1n-z) (0) = 0.
Whence
y(n1 (0) = ~ ny<n-1) (0)- n (n;: I) y<n-2> (0).

We have obtained a recurrence relation for determining the nth


derivative at the point x = 0 (n ~ 2). The values y (0) and y'(O)
are found immediately: y (0) = 2/5;
110 Ch. fl. D1flerent tal ton a) Fuw twn.~

, -3x2 -4x+ 19 f (0 19
Y (x) = (x2-2x+5J' ; y ) = 25'

Then, successively putting n = 2, 3, 4, ... , find the values of


the derivatives of higher orders with the aid of the recurrence
relation.
For example,
" 2 19 2.1 2 56
y (0)=5 2 25-55=125

y'" (0) = ~ . 3. 152~- 3~2. ~~ = -~;:.


2.3.6. Find the derivatives of the second order of the following
functions:
_ arc sin x . ( ) _ _ x
(b) Y- .r ' c y-e .
r l-x
2

2.3.7. Given the function


y = C1e2x +c2xe2x + eX.
Show that this function satisfies the equation
y"-4y' + 4y=ex.
2.3.8. Using the Leibniz formula give the derivatives of the in-
dicated orders for the following functions:
(a) y=x 3 sinx; find y' 20 >;
(b) y = e-x sin x; find y'";
(c) y=eX (3x 2 -4); find y<n;;
(d) y= (l-x2 ) cosx; find y< 2 n>.

2.3.9. Using the expansion into a linear combination of simpler


functions find the derivatives of the lOOth order of the functions:
I
(b) y= +x
I
(a) Y = x2-3x+ 2 ;
vr=x
2.3. IO. Show that the function
y = xn [c 1 cos (In x) +c2 sin (In x)]
(c1 , c2 , n constants) satisfies the equation
x2 y" + (l -2n) xy' + (1 +n 2) y =0.
2.3.11. Prove that if f (x) has a derivative of the nth order, then
[f (ax+ b)] ''" = a"f'n> (ax+ b).
2.4. Inverse, Implicit and Parametrically Represented Functions 111

2.4. Differentiation of ln'Verse, Implicit and Parametri-


cally Represented Functions
I. The Derivative of an Inverse Function. If a differentiable
function y = f (x), a < x < b has a single-valued continuous inverse
function x = g (y) and y~ =I= 0 then there exists also
, I
Xy=-,
Yx
For the derivative of the second order we have
..
x" __ Yxx
yy- (l/~)3.

2. The Derivative of an Implicit Function. If a differentiable


function y = y (x) sat is fies the equation F (x, y) = 0, then we have
to differentiate it with respect to x, considering y as a function
of x, and solve the obtained equation d~ F (x, y) = 0 with respect
to y~. To find y;, the equation should be twice differentiated with
respect to x, and so on.
3. The Derivative of a Function Represented Parametrically. If
the system of equations
x=qi(t), y='IJ(t), a< t < ~.
where qi (t) and 'iJ (t) are differentiable functions and qi' (t) =I= 0,
defines y as a single-valued continuous function of x, then there
exists a derivative y~ and
Yx=
, ipi (t) Yi
-,-=--;
\Pl (t) Xt

The derivatives of higher orders are computed successively:


,. - (y~)i
YX y"' = (y;x)'i and so on
X- I ' XXX I '
Xt Xt

In particular, for the second derivative the following formula is


true:
" Xt Ytt -Xtt Yt
Yxx=
(x1)a
2.4. l. For the function
(a) y=2x 3 +3x&+x; find x~;
(b) y = 3x-(cos x)/2; find 11 ; x;
(c) y= x+ex; find x;11
Solution. (a) We have y~=6x 2 + 15x 4 + 1, hence,
' l l
X11= y~ = -6x_2 _+_1-5x-,+-I
112 Ch. II. Differentiation of Functions

(c) y~= 1+ex, !f'xx=ex, hence,


, I ex
Xy= I +ex' Xyy= (I +ex)3.

2.4.2. Using the rule for differentiation of an inverse function,


find the derivative y~ for the following functions:
(a) ii= f/ x; (b) y =arc sin V x; (c) y =In V~l+-x2- .
Solution. (a) The inverse function X= y 3 has the derivative
x~ = 3y2 Hence,
, I I I
Yx = x~ = 3 y2 = 3 V x2 .

(c) At x >0
the inverse function x = Ve 2Y - 1 has the derivative
x~= e Y/Ve Y-1. Hence,
2 2

. I Jf~.v=i Jf x2 x
Yx = x~ = e2Y = x2 + I = x2 + I .
2.4.3. For each of the following functions represented paramet-
rically find the derivative of the first order of y with respect to x:
(a) x=a(t-sint), y=a(l-cost);
(b) X=ksint--sinkt, y=kcost+coskt;
(c) X= 2 ln cott, y= tan t +cott;
(d) X=ect, y=e-ct.
Solution. (a) Find the derivatives of x and y with respect to the
parameter t:
x;=a(l-cost); y/=asint.
Whence
dy a sin t t t
dx = a (I-cost) =co 2 (t =I= 2kn).
dx - 2 cosec2 t 4
(c) dt = cot t =- sin 2t ;
dy 2 t 2 t 4. cos 2t
dt =sec -cosec = - sin 2 2t ,
dy _ 4 cos 2t sin 2t =
dx - 4 sin 2 2t co
t 21 (t =I= kn)
2

2.4.4. The functions are defined parametrically:


(a) {x=acos3 t, (b) { X= t 3 +3t+ 1,
y=bsin 3 t; y= t 3 -3t + l;
(c) { x =a (cos t + t sin t}, (d) {x=e1 c~~t.
y= a (sin t-t ms t); y= e1 sin t.
Find for them the second derivative of y with respect to x.
2.4. Inverse, Implicit and Parametrically Represented Functions 113

Solution. (a) First find y;.


y/ = 3b sin 2 t cos t; xi = - 3a cos 2 t sin t;
, = _ 3b sin~2 t c~s t = _ !!_ tan t ( t =f: (2k + 1) ~)
Yx 3a cos t sm t a 2
Then we shall find y"xx using the formula
" (y;);
Yxx=-,
Xt
where
(Yx')'1 = b
- a cos 2 t
Whence
b f)
Yxx = - a cos 2 t ( - 3a cos 2 t sin t) = 3a2 cos 4 t sin t'
(d) xl=e1 cost-e1 sint=e'(cost-sint);
y; = e1 sin t +et cost = e1 (cost+ sin t);
, cos t+sint
Yx= cost-sin t;

,. (y~)I
( cost+sin
cos t- sin t 1
t)' 2
Yn =--:; = et (cost-sin t) et (cos t-sin t)3

2.4.5. Find y;~x:

(a) x =e- 1; y = t 3 ; (b) x =Sect; y= tan t.


Solution. (a) First find
x; =-e- 1; y; = 3t 2 ,
whence
y: = - 3t2/e-t = - 3ett2.
Then find the second derivative

Yxx-
- (y;); - -(3ett2+6tet) = 3te2t (t
, - -e-t
+ 2).
Xt

And finally, find the third derivative


y~;x = (y~~)I = 3e21 [2 (t2+2t)+ 2t + 2] _ 6eat (t 2 + 3t + 1).
Xt -e-1

2.4.6. Find the derivative y; of the following implicit functions:


(a) x 3 +x2 y+y2 =0; (b) lnx+e-ulx=c;
(c) x 2 +y2 -4x-10y+4=0;
+
( d) xi. y'la =a''
Solution. (a) Differentiate with respect to x, considering y as
a function of x; we get:
+
3x2 2xy x2y' 2yy' = 0. + +
114 Ch. 11. Diflerentiation of Functions

Solving this equation with respect to y' find


, 3x 2 +2xy
Y = - x 2 +2y
2.4.7. Find y;x if:
(a) arctan y-y+x=O; (b) ex-eY=y-x;
(c) x+ y =ex-y.
Solution. (a) Differentiate with respect to x, considering y as a
function of x and determine y':
' 1+
l~y 2 -y'+l=O, whence y'=---l-=y- 2 +1.
2

Differentiate once again with respect to x:


y" = - 2y-3y'.
Substituting the value of y' thus found, we finally get
" 2 (I+ y2)
Yxx =- y

2.4.8. Find the value of y" at the point x = l if


x 3 -2x2 y 2 + 5x + y-5 = 0 and Y lx=t = l.
Solution. Differentiating with respect to x, we find that
3x 2 -4xy 2 -4x2 yy' + 5 + y' = 0.
Putting x = 1 and y = 1, obtain the value of y' at x = 1:
3-4-4y' +s+ y' =O; y' =4/3.
Differentiate once again with respect to x:
6x-4y 2 -8xyy' -8xyy' -4x2 y' 2 -4x 2 yy" + y" = 0.
Putting x= 1; y== 1 and y' =4/3, find the value y" at x~~ 1:
6 - 4 - 64
3
- 64 -
9
3y" = 0 y" = - 8 27.
22

2.4.9. Find y~ for the following implicit functions:


(a) x+Vxy+y=a; (b) arctan(y/x)=lnVx 2 +y 2 ;
(c) exsiny-e-Ycosx=O;
(d) eY+xy=e; find y~ at the point (0, 1).
2.4.10. Find y;x ot the following implicit functions:
(a) y=x+arctany;
(b) x 2 +Sxy+y 2 -2x+y-6=0; find y" at the point (1, 1).
2.5. Applications of the Derivative 115

2 .4.11. For each of the following functions represented parame-


trically find the indicated derivatives:
a sin t c cos t
(a) X=1+b.cost' Y=1+bcost; find y;;
(b) x =In (1 + t 2 ), y = t-arctan t; find y;;
(c) X=t 2 +2, y=t 3 /3-t; find y;x;
(d) x=e-t 2 , y=-arctan(2t+ l); find y;;
(e) x=4tan 2 (t/2), y=asint+bcost; find y~;
(f) x=arcsin(t 2 -l), y=arccos2t; find y;;
(g) x=arcsint, y=Vl-t 2 ; find y;x.
2.4.12. Show that the function y = f (x), defined by the parametric
equations x=e1 sint, y=e1 cost, satisfies the relation y"(x+y) 2 =
= 2(xy'-y).

2.5. Applications of the Deri'Vati<ve


The equation of a line tangent to the curve of a differentiable
function y = y (x) at a point M (x 0 , y 0), where Yo= y (x 0 ), has the
form
Y-Yo = y' (x0 ) (x-x 0 ).
A straight line passing through the point of contact perpendicu-
larly to the tangent line is called the normal to the curve. The
equation of the normal at the
point M will be !I
I
y-yo= -y' (xo) (X-Xo),
y' (x0) =I= 0.
The segments AT, AN are
called the subtangent and the
subnormal, respectively; and the _D.,.......T-~---A~--~N~ :c
lengths MT and MN are the Fig. 36
so-called segment of the tangent
and the segment of the normal,
respectively (see Fig. 36). The lengths of the four indicated segments
are expressed by the following formulas:

AT=I:,/; AN=lyy'I; MT=l:,/v1+(y') 2;

MN= I y I Vl + (y') 2

2.5.1. Write the equations of the tangent Iine and the normal:
(a) to the curve y = x 3 -3x + 2 at the point (2, 4);
116 Ch. II. Differentiation of Functions

(b) to the parabola y=2x 2 -x+5 at X=-0.5;


(c) to the curve y = x 4 + 3x2 - l6 at the points of intersection
with the parabola y = 3x 2
Solution. (a) Find the derivative at the point x = 2:
y' = 3x~-3, y' (2) = 9.
The equation of the tangent line has the following form:
y-4= 9(x-2) or 9x-y-14=0.
The equation of the normal is of the form:
l
y-4=--g(x-2) or x+9y-38=0.

(c) Solving the system of equations


y =X4 + 3x2 -16,
{
y=3x2 ,
we shall find the points of intersection of the curves
Xi=-2, X2=2, Y1=Y2=12.
Now we find the derivatives at the points x = - 2 and x = 2:
y' =4X3 + 6x, y' (-2) =- 44, y' (2) =44.
Therefore, the equations of the tangent lines have the form
y- 12 =- 44 (x+ 2), y- 12 -= 44 (x-2).
The equations of the normals have the form
I I
y-12 = 44 (x+ 2), y-12 = - 44 (x-2).

2.5.2. Find the points on the curve y=x 3 -3x+5 at which the
tangent line:
(a) is parallel to the straight line y = - 2x;
(b) is perpendicular to the straight 1ine y = - x/9;
(c) forms an angle of 45 with the positive direction of the x-axis.
Solution. To find the required points we take into consideration
that at the point of tangency the slope of the tangent is equal to
the derivative y' = 3x2-3 computed at this point.
(a) By the condition of parallelism
3x2 -3=-2,
whence x1 =-1/V3, x 2= 1/V3. The required points are:
M.(-1/V3, 5+8V3/9), M2(1/V3, 5-8V3/9).
2.5. Applications of the Derivative 117

(b) By the condition of perpendicularity


3x2 -3 =9,
whence x1 = - 2, x 2 = 2. The required points: M 1 ( - 2, 3), M 2 (2, 7).
2.5.3. Find the angles at which the following lines intersect:
(a) the straight line y=4-x and the parabola y=4-x 2 /2;
(b) the sinusoid y =sin x and the cosine curve y =cos x.
Solution. (a) Recall that the angle between two curves at the point
of their intersection is defined as the angle formed by the lines tan-
gent to these curves and drawn at this point. Find the points of
intersection of the curves by solving the system of equations
y=4-x
{
y=4-x:/2.
Whence
M 1 (0, 4); M 2 (2, 2).
Determine then the slopes of the lines tangent to the parabola at
the points M 1 and M 2 :
y' (0) = 0, y' (2) = - 2.
The slope of a straight line is constant for all its points; in our
case it equals -1. Finally, determine
the angle between the two straight ff
lines:
tan qi 1 = 1; qi 1 = 45; K
-1+2 1
tan qi 2 = l+:2 = 3;
q> 2 = arc tan ~ ~ 18.5.
0 Xo
2.5.4. Prove that the segment of Fig. 37
the tangent to the hyperbola y = c/x
which is contained between the coordinate axes is bisected at the
point of tangency.
Solution. We have y' = - c/x 2 ; hence, the value of the sub tangent
for the tangent at the point M (x0 , y 0 ) will be

I:. l=lxoj,
i.e. Ox 0 =x0 T (Fig. 37), which completes the proof.
Whence follows a simple method of constructing a tangent to the
hyperbola y = c/x: lay off the x-intercept OT= 2x0 Then MT will
be the desired tangent.
118 Ch. II. Differentiation of Functions

2.5.5. Prove that the ordinate of the catenary y =a cosh (x/a) is


the geometric mean of the length of the normal and the quantity a.
Solution. Compute the length of the normal. Since
y' = sinh (x/a),
the length of the normal will be
MN= Iy I Vl + (y') 2 = YV 1 + sinh 2 (x/a) =ycosh (x/a) =y 2/a,
whence y2 =a-MN, and y=Va-MN, which completes the proof.
2.5.6. Find the slope of the tangent to the curve
f X=f +3t-8,
2

'l y = 2t 2t - 5
2 -

at the point M (2, -1).


Solution. First determine the value of t corresponding to the gi-
ven values of x and y. This value must simultaneously satisfy the
two equations
I t 2 +3t-8=2
l 2t 2 -2t-5= - 1.
The roots of the first equation are t 1 = 2; t 2 = - 5, the roots of the
second equation t 1 = 2; t 2 = - 1. Hence, to the given point there
corresponds the value t = 2. Now determine the value of the deri-
vative at the point M:

y
'I X=2 = (y;)
Xf 1=2 =
(4t-2)
2/ +3 1=2
6
=7
And so, the slope of the tangent at the point M (2, -1) is equa I
to 6/7.
2.5.7. Prove that the tangent to the lemniscate p=aVcos20at
the point corresponding to the value 00 = n/6 is parallel to the x-axis.
Solution. Write in the parametric form the equation of the Iem-
niscate:
x= p cos0 =a V cos 20 cose,
y= p sin 0 =a V cos 20 sin 0.
Whence
,
Xe
a cos 8 sin 28
=-----=--=--a
v--
cos 20
. 0, sin
f cos 28
a sin 8 sin 28
Yo = - y +a
v--
cos20 0,co~
cos 28
x~ (n/6) = - a V2, ye (n/6) = o.
2.5. Applications of the Derivative 119

y~ (n/6)
Thus, the slope k= . =0. Consequently, the line tangent to
x0 (n/6)
the lemniscate at the point with 00 = n/6 and Po= aVr cos 200 =
= a/V-2 is parallel to the x-axis.

2.5.8. Find the equations of the tangent and the normal to the
following curves:
(a) 4x 3 -3xy2 +6x2 -5xy-8y2 +9x+ 14=0 at the point (-2, 3);
(b) x+y-2xy=0 at the point (1, 1).
Solution. (a) Differentiate the implicit function:
12x2 -3y2 -6xyy' + 12x-5y-5xy' -16yy' + 9 = 0.
Substitute the coordinates of the point M (-2, 3):
48-27 + 36y'-24-15 + IOy' -48y' + 9 =0;
whence
y'=-9/2.
Thus the equation of the tangent line is
9
y-3 = - 2 (x + 2)
and the equation of the normal
2
y-3 =g-(x+2).

2.5.9. Through the point (2, 0), which does not belong to the
curve y = x4, draw tangents to the latter.
Solution. Let (x 0 , xt,) be the point of tangency; then the equation
of the tangent will be of the form:
y-xg = y' (x0 ) (x-x 0 )

y-xt, = 4xi (x-x 0 ).


By hypothesis the desired tangent line passes through the point
(2, 0), hence, the coordinates of this point satisfy the equation of
the tangent line:
-x3=4xH2-x0 ); 3xg-8xi=o,
whence x0 = O; x0 = 8/3. Thus, there are two points of tangency:
M 1 (0, 0), M 2 (8/3, 4096/81).
Accordingly, the equations of the tangent lines will be
4096 2048 ( 8 )
y=O, y-81=21 X-3 .
120 Ch. JI. Differentiation of Functions

2.5.10. f (x) = 3x&- 15x3 + 5x- 7. Find out at which of the points x
the rate of change o.f the function is minimal.
Solution. The rate of change of a function at a certain point is
equal to the derivative of the function at this point
f' (x) = 15x4 -45x2 + 5 = 15 [(x2 - l/2) 2 + 1/12].
The minimum value of f' (x) is attained at x = + l/V2. Hence the
minimum rate of change of the function f (x) is at the point
x = t;V2 and equals 5/4.
2.5.11. A point is in motion along a cubic parabola 12y=x3
Which of its coordinates changes faster?
Solution. Differentiating both members of the given equation with
respect to t we get the relation between the rates of change of the
coordinates:
12yi = 3x2 xi
or

Hence,
(1) at -2 < x < 2 the ratio y;:xi is less than unity, i.e. the rate
of change of the ordinate is less than that of the abscissa;
(2) at x = 2 the ratio y;: x; is equal to unity, i.e. at these
points the rates of change of the coordinates are equal;
(3) at x < -:-2 or x > 2 the ratio yi: x; is greater than unity, i.e.
the rate of change of the ordinate exceeds that of the abscissa.
2.5.12. A body of mass 6g is in rectilinear motion according to
the law s=-l+ln(t+l)+(t+l) 3 (sis in centimetres and t, in
seconds). Find the kint>tic energy (mv 2/2) of the body one second
after it begins to move.
Solution. The velocity of motion is equal to the time derivative
of the distance:
v (t) = s/ = t~1 + 3 (t + 1)2.
Therefore
v(l)= 12 2I and mv
y= 26 ( 12 2I ) =468 43 (erg).
2 2

2.5.13. The velocity of rectilinear motion of a body is proporti-


onal to the square root of the distance covered (s), (as, for example,
in free fall of a body). Prove that the body moves under the action
of a constant force.
Solution. By hypothesis we have
v=si=aVs (a=const);
.~ 2.5. Applications of the Derivative 121

whence
s"11 = v1' =a ---:;p
I s'1 =a''/2 .
2 f s
But according to Newton's law the force
F = ks/1 (k = const).
Hence,
F = ka. 2/2 =canst.
2.5.14. A raft is pulled to the bank by means of a rope which
is wound on a drum, at a rate of 3 m/min. Determine the speed
of the raft at the moment when it is 25 m distant from the bank
if the drum is situated on the bank 4 m above water level.
Solution. Let s denote the length of the rope between the drum
and the raft and x the distance from the raft to the bank. By
hypothesis
s2 =x2+42_
Differentiating this relation with respect to t, find the relation-
ship between their speeds:
2ssi =2xx/,
whence
' s '
X1=xSt 0

Taking into consideration that


si=3; x=25; s=V25 2 +4 2 ~ 25.3,
we obtain
' = v2s2+ 42 3 ~ 3.03 (m/mm).
Xt
.
25

2.5.15. (a) Find the slope of the tangent to the cubic parabola
y=x3 at the point x=V3/3.
(b) Write the equations of the tangents to the curve y= 1/(1 +x 2 )
at the points of its intersection with the hyperbola y = 1/(x+ 1).
(c) Write the equation of the normal to the parabola y = x2 +4x+ I
perpendicular to the line joining the origin of coordinates with the
vertex of the parabola.
(d) At what angle does the curve y =ex intersect the y-axis?
2.5.16. The velocity of a body in rectilinear motion is determi-
ned by the formula v=3t+t 2 What acceleration will the body
have 4 seconds after the start?
2.5.17. The law of rectilinear motion of a body with a mass of
+ +
100 kg is s = 2t 2 3t l. Determine the kinetic energy (mv 2/2) of
the body 5 seconds after the start.
122 Ch. II. Diflerentiat ion of Functions

2.5.18. Show that if the law of motion of a body is s=ae1 +be- 1,


then its acceleration is numerically equal to the distance covered.
2.5.19. A body is thrown vertically with an initial velocity of
am/sec. What altitude will it reach in t seconds? Find the velocity
of the body. In how many seconds and at what distance from the
ground will the body reach the highest point?
2.5.20. Artificial satellites move round the Earth in elliptical
orbits. The distance r of a satellite from the centre of the Earth
as a function of time t can be approximately expressed by the fo-
llowing equation:
r = a [ I - e cos M - e; (cos 2M - I) J
where M = ?; (t- tn)
t =time parameter
a= semi-major axis of the orbit
e =eccentricity of the orbit
P =period of orbiting
tn=time of passing the perigee 1 ty the satellite.
Here a, e, P and tn are constants.
Find the rate of change in the distance r from the satellite to the
centre of the Earth (i.e. find the so-called radial velocity of the
satellite).

2.6. The Differential of a Function .


.4pplication to Approximate Computations
If the increment !J.y of the function y = f (x) can be expressed as:
!J.y = f (x+ !J.x)-f (x) =A (x) !J.x+a (x, !J.x) !J.x,
where
Jim a (x, !J.x) = 0,
l'J.X - 0

then such a function is called ditferentiable at the point x. The


principal linear part of this increment A (x) !J.x is called the ditfe-
rential and is denoted df (x) or dy. By definition, dx = !J.x.
For the differential of the function y = f (x) to exist it is nece-
ssary and sufficient that there exist a finite derivative y' =A (x).
The differential of a function can be written in the following way:
dy = y' dx= f' (x)dx.
1 The perigee of the satellite orbit is the shortest distance from the sate-
llite to the centre of the Earth.
2.6. Differential of a Function 123

For a composite function y = f (u), u =qi (x) the differential is


retained in the form
dy =f' (u) du
(the invariance of the form of the differential).
With an accuracy up to infinitesimals of a higher order than dx
the approximate formula dy :::::::: dy takes place. Only for a linear
function y =ax+ b do we have dy = dy.
Differentials of higher orders of the function y = f (x) are succes-
sively determined in the following way:
d 2 y=d(dy); d 3 y=d(d2 y), .. . , dny=d(d"- 1 y).
If y = f (x)and x is an independent variable, then
d y = y" (dx) 2 ; d 3 y = y"' (dx)3, ... , dny = y<tl) (dx)".
2

But if y -= f (u), wt:ere u =qi (x), then d 2 y = f" (u) du 2 + f' (u) d2 u, and
so on.
2.6. l. Find the differential of the function
y =In ( 1 + e10 x) +arc tan ex.
Calculate dy at x = O; dx = 0.2.
Solution.
_ [(I+ etox)' (ex)' ] _ Sex (2ex- I)
dy- I +eiox I +eiox dx- I +e1ox dx.
Substituting x = 0 and dx = 0.2, we get
5
dy lx=O; dx=0.2 = 2 0.2 = 0.5.
2.6.2. Find the increment and the differential of the function
y=3x 3 +x-1
at the point X= 1 at dx=0.1.
Find the absolute and relative errors allowed when replacing the
increment of the function with its differential.
Solution.
f:.y = [3 (x+ dx} 3 + (x+ dx)-l]-(3x 3 +x- l) =
= 9x2 dx + 9x dx2 + 3dx3 + dx,
dy = (9x 2 1) dx. +
Whence
dy-dy = 9x dx2 + 3dx3
At x=l and dx=0.1 we get
dy-dy = 0.09+ 0.003 =0.093,
dy =I; dy = 1.093.
124 Ch. II. Di(Jerentiation of Functions

The absolute error Ifl.y-dy I= 0.093, the relative error j !:ly-;/Y I=


= ~ :~~~ ~ 0.085 or 8.5 % .
2.6.3. Calculate approximately the increment of the function
y=x 3 -7x2 +8
as x changes from 5 to 5.01.
2.6.4. Using the concept of the differential, find the approximate
value of the function
y= Vr2-x
2 +x at
ri
X=0.15.
Solution. Notice that from fl.y = y (x+ fl.x)-y (x) we get
y (x+ fl.x) = y (x) + fl.y,
or, putting fl.y ~ dy,
y (x + fl.x) ~ y (x) + dy.
In our problem let us put x = 0 and fl.x = 0.15. Then

Y
,
=s
I V(2+x)
2-x
4 (-4)
(2+x)2'
.

y'(O)=- ! , dy=- ! 0.15=-0.03.


Hence,
y (0.15) ~ y (0) + dy= 1-0.03 = 0.97.
The true value of y (0.15) = 0.9702 (accurate to 10- 4 ).
2.6.5. Find the approximate value of:
(a) cos31; (b) log 10.21; (c) V33; (d) cot 4510'.
Solution. (a) In solving this problem we shall use the formula (*)
of the preceding problem. Putting x = n/6, fl.x = n/180, we compute:

y(x)=COS5=-2- ;
1t 3

I ( ) 1t I
y X =-Stn5=-2;

cos31 o =cos 6 + 180 (n n) Y3


~ - 2 - - 2180
In =0.851.
(c) Put x = 32; fl.x = 1. By formula (*) we get

V -33~ v- y Xx=321=2+ v-
32+ (5/"-)' 5
I
32 G 4
2+ 80I =2.0125.
2.6. Differential of a Function 125

2.6.6. All faces of a copper cube with 5-cm sides were uniformly
ground down. As a result the weight of the cube was reduced by
0.96 g. Knowing the specific weight of copper (8) find the reduction
in the cube size, i.e. the amount by which its side was reduced.
Solution. The volume of the cube v=x 3 , where x is the length
of the side. The volume is equal to the weight divided by the den-
sity: v = p/d; the change in cube's volume L1v = 0.96/8 = 0.12 (cm 3 ).
Since L1v approximately equals dv and taking into consideration that
dv = 3x2 dx we shall have 0.12 = 3 x 5 2 x L1x, whence
L1x = ~". 1225 = 0.0016 cm.
Thus, the side of the cube was reduced by 0.0016 cm.
2.6.7. Find the expressions for determining the absolute errors in
the following functions through the absolute errors in their argu-
ments:
(a) y = lnx; (b) y = logx;
(c) y =sin x (0 < x < :rt/2); (d) y = tanx (0 < x < :rt/2);
< <
(e) y = log (sin x) (0 x :rt/2};
(f} y =log (tan x) (0 < x < :rt/2).
Solution. If the function f (x) is differentiable at a point x and
the absolute error of the argument dx is sufficiently small, then
the absolute error in the function y can be expressed by the number
L1y =I y; I L1x.
(a) dy =I (In x)' Ix L1x =~xx , i.e. the absolute error of a natural
logarithm is equal to the relative error in its argument.
M
(b) dy =(log x)' dx = x dx, where M = loge= 0.43429;
(e) dy =I [log (sin x)]' I dx =MI cot x I dx;
(f) dy =I [log (tan x))' I dx = I s~~x I dx.
From (e) and (f) it follows that the absolute error in log tan x
is always more than that in log sin x (for the same x and dx)
2.6.8. Find the differentials dy and d 2 y of the function
y= 4x~-7x 2 +3,
assuming that:
(1) x is an independent variable;
(2) x is a function of another independent variable.
Solution. By virtue of the invariance of its form the differential
of the first order dy is written identically in both cases:
dy = y' dx = (20x'-14x) dx.
126 Ch. 11. Differentiation of Functions

But in the first case dx is understood as the increment of the


independent variable dx (dx = dx), and in the second, as the diffe-
rential of x as of a function (dx may not be equal to dx).
Since differentials of higher orders do not possess the property
of invariance, to find d2 y we have to consider the following two
cases.
( 1) Let x be an independent variable; then
d2 y = y" dx 2 = (80x 3 - l4) dx2
(2) Let x be a function of some other variable. In this case
d2 y = (80x 3 - l4) dx 2 + (20x 4 - l4x) d2 x.
2.6. 9. Find differentials of higher orders (x an independent va-
riable):
(a) y=4-x2 ; find d2 y;
(b) y = Vln 2 x-4; find d2 y;
(c) y = sin 2 x; find d 3 y.
l-x2
2.6.10. y=ln i+x2 ; find d2 y 1f: (a) xis an independent variable,
(b) x is a function of another variable. Consider the particular
case when x = tan t.
2.6. t 1. The volume V of a sphere of radius r is equal to
: nr 3 Find the increment and differential of the volume and
explain their geometrical meaning.
2.6.12. The law of the free fall of a material point is s=gt 2 /2.
Find the increment and differential of the distance at a moment t
and elucidate their mechanical meaning.

2.7. Additional Problems


2.7.1. Given the functions: (a) f(x)=lxl and (b) 1P(X)=lx3 [.
Do derivatives of these functions exist at the point x = O?
2.7.2. Show that the curve y=elxl cannot have a tangent line
at the point x = 0. What is the angle between the one-sided tan-
gents to this curve at the indicated point?
2.7.3. Show that the function
f (x) =Ix-a I IP (x),
where IP (x) is a continuous function and IP (a) =f= 0, has no deriva-
tive at the point x =a. Find the one-sided derivatives f:_ (a) and
f: (a).
2.7. Additional Problems 127

2.7.4. Given the function


f (x) = {. x2 sin ( l/x} at x :p 0,
0 at X=O.
Use this example to show that the derivative of a continuous
function is not always a continuous function.
2.7.5. Let
f (x)= f x2 , if x~x0 ,
l ax+b, if x > x 0
Find the coefficients a and b at which the function is continuous
and has a derivative at the point x 0
2.7.6. By differentiating the formula cos3x=cos 3 x-3cosxsin 2 x
deduce the formula sin 3x = 3 cos 2 x sin x-sin 3 x.
2.7.7. From the formula for the sum of the geometric progression
1-xn+l
I +x+x2 + ... +xn= l-x (x=F 1)
deduce the formulas for the following sums:
(a) I+ 2x+ 3x2 + ... + nxn-1;
(b) 12+ 22 x+3 2 x 2 + ... +n 2 xn- 1.
2. 7 .8. Prove the identity
cos x + cos 3x + ... + cos (2n- 1) x = -sin
2-
2nx
.- ,
smx
x =/= k n
and deduce from it the formula for the sum
sin x+ 3 sin 3x+ ... + (2n- l)sin (2n-1)x.
2.7.9. Find y' if:
(a) Y---'f(sin 2 x}+f(cos 2 x); (b} y=f(ex)eJ1x 1;
(c} y = logcp ex>' (x) (qi (x) > O; ' (x) > 0).
2. 7. IO. Is it reasonable to assert that the product F (x) = f (x) g (x)
has no derivative at the point x = x 0 if:
(a) the function f (x) has a derivative at the point x 0 , and the
function qi (x) has no derivative at this point?
(b) neither function has a derivative at the point x0 ?
Consider the examples: (1) f(x)=x, g(x)=lxl;
(2) f (x) =Ix 1. g (x) = Ix I
Is it reasonable to assert that the sum F (x) = f (x) + g (x) has no
derivative at the point x = x 0 if:
128 Ch. 11. Diflerentiation of Functions

(c) the function f (x) has a derivative at the point x 0 , and the
function g (x) has no derivative at this point?
(d) neither function has a derivative at the point x0 ?
2. 7. l l. Prove that the derivative of a differentiable even function
is an odd function, and the derivative of an odd function is an
even function. Give a geometric explanation to these facts.
2.7.12. Prove that the derivative of a periodic function with
veriod T is a periodic function with period T.
2.7.13. Find F' (x) if
x x2 x3
F (x) = I 2x 3x2
0 2 6x
2.7.14. Find the derivative of the function y=xlxl. Sketch the
graphs of the given function and its derivative.
2.7.15. Suppose we have a composite function y=f (u), where
u =qi (x). Among what points should we look for points at which
the composite function may have no derivative?
Does the composite function always have no derivative at these
points? Consider the function y=u 2 , u=lxl.
2.7.16. Find y" for the following functions:
(a) u=lxsl; (b) u={ x2sin(l/x), x=FO,
0 at x=O.
Is there y" (O)?
2.7.17. (a) f(x)=xn; show that
f (I)+ f' ~II) + fC2~~1) + + f<n~~I) = 2n.
(b} f (x} =xn- 1e11 x; show that
[f (x)]<n>= (-l)nfx~~ (n= 1, 2, ... ).
2.7.18. y=x2e-x 1a; show that
f<nJ (0) = (-l)n;~~-1) (n ~ 2).
2.7.19. Show that the function y=arcsinx satisfies the relation
( l -x2 ) y" = xy'. Find y<n> (0) (n ~ 2) by applying the Leibniz for-
mula to both members of this identity.
2. 7 .20. Prove that the Chebyshev polynomials
I
Tn(x)= 2n_ 1 cos(narccosx) (n= 1, 2, ... )
2.7. Additional Problems 12::l

satisfy the equation


(l-x 2 ) T~ (x)-xT~ (x) + n 2 T,, (x) = 0.
2.7.21. The derivative of the nth order of the function e-x has
the form
(e-x 2 )( I I ) = e-x 2 H n (x),
where H 11 (x) is a polynomial of degree n called the Chebyshev
Hermite polynomial.
Prove that the recurrence relation
H 11 + 1 (x)-2xH 11 (x)+2nH 11 _ 1 (x)=O (n=l, 2, ... )
is valid.
2.7 .22. Show that there exists a single-valued function y = y (x)
defined by the equation y~ + 3y = x, and find its derivative y:.
2.7 .23. Single out the single-valued continuous branches of the
inverse function x=x(y) and find their derivatives if y=2x 2 -x 4
I I +u . du du
2. 7 .24. u = 2 In 1 ~v; check the relation du du= 1.

2.7 .25. Inverse trigonometric functions are continuous at all


points of the domain of definition. Do they have a finite deriva-
tive at all points of the domain? Indicate the points at which the
following functions have no finite derivative:
x+I . I
(a) y=arccos-2-; (b) y=arcs111x.

2.7 .26. Show that the function y = y (x), defined parametrically:


x=2t-1tl, y=t 2 +tltl, is differentiable at f=O but its deriva-
tive cannot be found by the usual formula.
2.7.27. Determine the parameters a, b, c in the equation of the
parabola y = ax2 +bx+ c so that it becomes tangent to the straight
line y = x at the point x = 1 and passes through the point (-1, 0).
2.7 .28. Prove that the curves y 1 = f (x) (f (x) > 0) and y 2 =
= f (x) sin ax, where f (x) is a differentiable function, are tan-
gent to each other at the common points.
2.7.29. Show that for any point M (x 0 , y 0 ) of the equilateral
hyperbola x 2 -y 2 =a~ the segment of the normal from the point
M to the point of intersection with the abscissa is equal to the
radius vector of the point M.
2.7.30. Show that for any position of the generating circle the
tangent line and the normal to the cycloid x =a (t-sin t),
y=a(l-cost) pass through the highest (at, 2a) and the lowest
(at, 0) points of the circle, respectively.
5-3148
130 Ch. II. Differentiation of Functions

2.7.31. Show that two cardioids p=a(l+coscp) and p=


== a (I -cos cp) intersect at right angles.
2.7 .32. Let y = f (u), where u = cp (x). Prove the validity of the
cc;uality
d 3 y = f'" (u) du 3 + 3f" (u) du d 2 u + f' (u) d:iu.

2.7.33. Let y=f(x), where x=cp(t); the functions f(x) and


cp (t) are twice differentiable and dx =I= 0. Prove that
" d 2 ydx-dyd 2x
Yxx = dx3

where the differentials forming the right member of the relation


are differentials with respect to the variable t.
2. 7 .34. How will the expression
(l-x2) d2y -
dx 2
xdy
dx
+y
be transformed (where y is a twice differentiable function of x) if
we introduce a new independent variable t, putting x =cost?
2.7 .35. In determining an electric current by means of a tangent
galvanometer use is made of the formula
l=ktancp,
where I= current
k =factor of proportionality (depending on the instrument)
cp =angle of pointer deflection.
Determine the relative error of the result which depends on the
inaccuracy in reading the angle cp. At what position of the pointer
can one obtain the most reliable results?
Chapter 3
APPLICATION OF DIFFERENTIAL
CALCULUS TO INVESTIGATION
OF FUNCTIONS

3.1. Basic Theorems on Differentiable Functions


Fermat's Theorem. Let a function y = f (x) be defined on a cert-
ain interval and have a maximum or a minimum value at an in-
terior point x0 of the interval.
If there exists a derivative f' (x 0 ) at the point x 0 , then f' (x0 ) = 0.
Rolle's Theorem. If a function f (x) is continuous in the interval
[a, b], has a finite derivative at all interior points of this interval,
and f (a)= f (b), then inside [a, b] there exists a point E (a, b) s
such that f' (s) = O.
Lagrange's Theorem. If a function f (x) is continuous in the in-
terval [a, b] and has a finite derivative at all interior points of
s
the interval, then there exists a point E (a, b) such that
f (b)-f (a)= (b-a) f' (s).
Test for the Constancy of a Function. If at all points of a cer-
tain interval f' (x) = 0, then the function f (x) preserves a constant
value within this interval.
Cauchy's Theorem. Let cp (x) and 1jJ (x) be two functions continu-
ous in the interval [a, b] and have finite derivatives at all inte-
rior points of the interval. If these derivatives do not vanish si-
s
multaneously and cp (a) =I= cp (b), then there exists E (a, b) such that
"'(b)-'IJ (a) 'IJ' <6)
<p (b)"- <p (a) = <p' (6)

3.1.1. Does the function f (x) = 3x2 - l satisfy the condition of


the Fermat theorem in the interval [ 1, 2]?
Solution. The given function does not satisfy the condition of the
Fermat theorem, since it increases monotonically on the interval
[l, 2], and, consequently, takes on the minimum value at x= l and
the maximum one at x= 2, i. e. not at interior points of the in-
terval. Therefore, the Fermat theorem is not applicable; in other
words, we cannot assert that f'(l)=f'(2)=0. Indeed, f'(l)=6,
f' (2) = 12.
S*
1::12 Ch. 111. DifJerentiat Calculus: Investigation of Funct's

3.1.2. Do the following functions satisfy the conditions of the


Rolle theorem?
(a) f(x)=l-Vx 2 in [-1, IJ;
(b) f (x) = In sin x in [n/6, 5n/6 J;
(c) f(x)=l-lxl in [-1, lJ.
If they do not, explain why.
Solution. (a) The function is continuous in the interval [-1, IJ;
furthermore, f (-1) = f (I)= 0. Thus, two conditions of the Rolle
theorem are satisfied. The derivative f' (x) = -2/(3 Vx)
exists at
all points except x = 0. Since this point is an interior one, the third
condition of the theorem is not satisfied. Therefore, the Rolle the-
orem is not applicable to the given function. Indeed, f' (x) =I= 0 in
[-1, lJ.
3.1.3. Prove that the equation
3x + 15x-8 = 0
has only one real root.
Solution. The existence of at least one real root follows from the
fact that the polynomial f (x) = 3x + 15x-8 is of an odd power.
Let us prove the uniqueness of such a root by reductio ad absur-
dum. Suppose there exist two roots Xi < x2 Then in the interval
[x 1 , x2 J the function f (x) = 3x5 + 15x-8 satisfies all conditions of
the Rolle theorem: it is continuous, vanishes at the end-points and
has a derivative .at all points. Consequently, at some point, Xi < s < x2 ,
f' (s) = 0. But f' (x) = 15 (x 4 + 1) > 0. This contradiction proves that
the equation in question has only one real root.
3.1.4. Does the function f(x)=3x 2 -5 satisfy the conditions of
the Lagrange theorem in the interval [-2, OJ? If it does, then
find the point s which figures in the Lagrange formula f (b)-f (a)=
= f' (s) (b-a).
Solution. The function satisfies the conditions of the Lagrange
theorem, since it is continuous in the interval [-2, OJ and has a
finite derivative at all interior points of the interval. The point s
is found from the Lagrange formula:
f' (s) = 66 = , (0)- t c-2> = - s - 7 = _ 6
0-(-2) 2 '
whence s=-1.
3.1.5. Apply the Lagrange formula to the function f (x) =In x in
the interval [ 1, eJ and find the corresponding value of .
3.1.6. Ascertain that the functions f (x) = x2 - 2x+ 3 and g (x) =
= x 3 - 7x2 + 20x-5 satisfy the conditions of the Cauchy theorem
in the interval [l, 4J and find the corresponding value of .
3.1. Basic Theorems on DiOerentiable Functions 133

Solution. The given functions f (x) and g (x) are continuous eve-
rywhere, and hence, in the interval [I, 4] as well; their derivatives
f'(x)=2x-2 and g'(x)=3x 2 -14x+20 are finite everywhere; in
addition, g' (x) does not vanish at any real value of x.
Consequently, the Cauchy formula is applicable to the given
functions:
f(4)-f(I) f'(;)
g(4)-g(I) = g' Cs)'
i.e.
11-2 26-2
21-9=35 2 -145+20 ( 1 <s< 4).
Solving the latter equation, we find two values of s: =
1 2 and
s =4.
2
Of these two values only 1 = 2 is an interior point of the interval.

3.1. 7. Do the functions f (x) =ex and g (x) = 1; 2x~ satisfy the con-
ditions of the Cauchy theorem in the interval [-3, 3]?
3.1.8. On the curve y = x 3 find the point at which the tangent
line is parallel to the chord through the points A (-1, -1) and
B(2, 8).
Solution. In the interval [-1, 2), whose end-points are the abs-
cissas of the points A and B, the function y = x3 is continuous and
has a finite derivative; therefore the Lagrange theorem is applicable.
According to this theorem there will be, on the arc AB, at least
one point M, at which the tangent is parallel to the chord AB.
Let us write the Lagrange formula for the given function:
t (2)-f <-1) = t, (s) (2-(-1) 1,
or

whence
s1 = - 1, 2 = I.
The obtained values of s
are the abscissas of the desired points
(as we see, there exist two such points). Substituting 1 and 2 in
the equation of the curve, we find the corresponding ordinates:
Y1=s~= I; Y2=~=-l.
Thus, the required points are: M 1 (l, 1) and M 2 (-l, -I), of which
only the former is an interior point on the arc AB.
Note. This problem can be solved without using the Lagrange the-
orem; write the equation of the chord as a straight line passing
through two given points, and then find the point on the curve at
which the tangent is parallel to the chord.
134 Ch. 111. Differential Calculus: Investigation of Funct's

3.1.9. Taking advantage of the test for the constancy of a func-


tion, deduce the following formulas known from elementary math-
ematics:
(a) arcsinx+arccosx=n/2;
(b) sin 2 x ~-= (l -cos2x)/2;
1 x2
(c) arccosl~xz=2arctanx at o:::;;;x<oo;
:rt-2arctanxatx;:>-l,
(d) arc sin l-j~\ 2 = { 2arctanx at -1 :::;;;x:::;;; l,
-n-2arctanx at x:::;;;-l.
Solution. (a) Let us consider the function
f (x) =arc sin x +arc cos x,
defined in the interval f-1, l]. The derivative of the indicated
function inside this interval equals zero:
I
-;-r- _Q
r I -x2
(-1 < x< I).

According to the test for the constancy of a function f (x) = const,


i. e. a re sin x + a re cos x = C ( - 1 < x < 1).
To determine the constant C let us put, for instance, X= O; then
we have n/2 = C, whence
arcsinx+arccosx=n/2 (-1<x<1).
The validity of this equality at the points x = + l is verified
directly.
(b) Let us take the function
f (x) = sin 2 x+ { cos2x
defined throughout the number scale: -oo < x < oo. The derivative
of this function is everywhere equal to zero:
f' (x) = 2 sin x cos x-sin 2x = 0.
According to the test for the constancy of a function
2
sin x+ 2I cos 2x= C .

To determine C put, for instance, x=O; then we get l/2=C.


Wherefrom
sm
2
x + I2 cos 2x-
- I
2 ,
3.1. Basic Theorems on Differentiable Functions 135

or
2 I-cos 2x
Sin X= 2

(c) Let us introduce the function


l-x 2
f (x) =arc cos 1+ x2 -2 arc tan x,
determined along the entire number scale, since 1~ I. I::; ;:
The derivative of the function f (x) is zero for all x > 0:
-4x 2 4x 2
f' (x) = -. / -(I -x2) 2 (l+x2)2-1+x2=2x(l +x2) - I+x2 - O.
V I I+x 2

According to the test for the constancy of a function


l -x2
arc cos 1 +x2 -2 arc tanx= C at x > 0.
To determine C let us put, say, x = I, which gives C =arc cos 0-
- 2 arc tan I = 0.
The validity of the proved formula at x = 0 is verified directly.
l-x2
Note. At x = 0 the function arc cos 1 +x2 has no derivative. At
x < 0 its derivative is
( a re cos l-x2)'
I + x2 = - I +2x2 '
which enables us to derive the formula
l-x 2
arccos 1+x 2 = -2 arc tan x (x < 0).
The latter formula can be obtained on the strength of the fact
that arc cos : +;:
is an even function, and 2 arc tan x is an odd one.

3.t.10. As is known, (ex)' =ex for all x. Are there any more
functions that coincide with their deri\'atives everywhere?
Solution. Let the function f (x) be such that f' (x) = f (x) every-
where.
Let us introduce the function
cp (x) = f e~) = f (x) e-x.
The derivative of this function equals zero everywhere:
cp' (x) = f' (x) e-x -e-x f (x) = 0.
By the test for the constancy of a function f (x)/ex = C, whence
f (x) = Cex.
136 Ch. I II. Differential Calculus: Investigation of Func/'s

And so, we have proved that the group of functions for which
f' (x) = f (x) is covered by the formula f (x) ~ Cex.
3.1.11. Prove the inequality
arc tanx 2 -arc tan x1 < x 2 -x 1 ,
where x 2 > x 1 .
Solution. To the function f(x)=arctanx on the interval [x 1 , x 2 ]
apply the Lagrange formula:
I
arc tanx 2 -arc tanx 1 =-= l+~ 2 (x 2 -x1 ),
where x1 < s< x 2
Since

then
arc tan x 2 - arc tan x 1 <x 2 - x1
In particular, putting x 1 ~~ 0 and xt = x, we get
arc tanx <x (x > 0).
3.1.12. Show that the square roots of two successive natural
numbers greater than N 2 differ by less than l/(2N).
Solution. To the function f(x)=Vxon the interval [n, n+l]
apply the Lagrange formula:
f (n + I )-f (n) = Jl n + 1-Vn = ~/"-,
2 r 1;
where n < s < n +I.
If n > N then s > N hence 1/(2 ~1 ~) <
2, 2, l/(2N), whence
V-n + 1- J/n < l/(2N).
3.1.13. Using the Rolle theorem prove that the derivative f' (x)
of the function
f (x ) = <( x sin ~x at x > 0,
! 0 at x = 0
vanishes on an infinite set of points of the interval lO, l).
Solution. The function f (x) vanishes at points where
sin (n/x) = 0, n/x =kn, x = l/k,
k = l, 2, 3, ...
Since the function f (x) has a derivative at any interior point
of the interval [O, l], the Rolle theorem is applicable to anyone
3.1. Basic Theorems on Diflerentiable Functions 137

of the intervals [l/2, l], [l/3, 1/2]. ... , [I!(k+l), l/k],


Consequently, inside each of the intervals of the sequence, there is
a point k, l/(k + l) < k < l 1k, at which the derivative f' (sk) = 0.
And so we have shown 1.hat the derivative vanishes on an infinite
set of points (see Fig. 38).
3.1.14. The Legendre polynomial is Y
a polynomial defined by the following t
formula (Rodrigues' formula):
I dn
p n (x) = 2--,,-,
n. . d----,,
x (x
2- l)n (n = 0,
l, 2, ... ).
Using the Rolle theorem, prove
that the Legendre polynomial P n (x)
has n different real roots, all of them --=*liftt-t~~~--~-:r:
found between - 1 and + l. 0
Solution. Consider the function
f(x)=(x 2 -l)"=(x-l)n (x+l)".
This function and its n- 1 successi
ve derivatives vanish at the points Fig. 38
x = + 1 (use the Leibniz formula for
higher derivatives of the product of two functions).
It follows from f ( l) = f ( - 1) = 0 that inside the interval [.- 1, 1J
a point 1 can be found at which f' ( 1 ) = 0, i.e. x = ~ 1 will be the
root of the first derivative. Now apply the Rolle theorem once again
to the function f' (x) on the intervals [-I, ; 1 ]. rn 1 , 1). We find
that besides + 1 and - 1 the function f" (x) has two more roots
on the interval [ - 1, 1]. Reasoning as before, we will show that,
apart from + 1 and -1, fhe (n- l)th derivative has (n-1) more
roots on the interval (-1, 1), i.e. the function rn-l) (x) has all in
all n + 1 roots on the interval [ - 1, 1), which divide this interval
into n parts. Applying the Rolle theorem once again, we ascertain
that the function f(n> (x), and hence, the function P,, (x)= 2 n~! f<"> (x),
has n different roots on the interval (-1, 1).
3.1.15. Check whether the Lagrange. formula is applicable to the
following functions:
(a) f (x) = x 2 on [3, 4);
(b) f(x)=lnx on (1, 3];
(c) f(x)=4X 3 -5x2 +x-2 on [O, l);
(d) f (x) = V X 4 (x- l) on [- 1/2, 1/2].
If it is, find the values of appearing in this formula.
3.1.16. Using the Lagrange theorem estimate the value ln(l-l-t:)
i38 Ch. Ill. Differential Calculus: Investigation of Funct's

3.1.17. Using the Lagrange formula prove the inequality

1~ x < In ( 1+ x) < x at x > 0.

3.2. 'Valuation of Indeterminate Forms.


L'Hospital's Rule
I. Indeterminate forms of the type ~ , ~. If the functions f (x)
and g (x) are differentiable in a certain nt>ighbourhood of the point a,
except, may be, at the point a itself, and g' (x) =F 0, and if
Jim f(x)= lim g(x)=O or lim f(x)= lim g(x)=oo,
r- a r-a r - a
then
. f-(x)= 1lill
Ilill . - f' (x)
,-
r - a g (x) x - a g (x)

provided the limit lim


x- a g
r: ((x))
X
exists ( L'Hospital's rule). The point a
may be either finite or improper + oo or - ou.
II. Indeterminate forms of the type Ooo or 0 0 - 0 0 are reduced
to forms of the type ~ or : by algebraic transformations.
II I. Indeterminate forms of the type l"', 00 or 0 are reduced
to forms of the type O oo by taking logarithms or by the transfor-
mation [f(x)]cp<xl=&<x)lnfCxl.
3.2.1. Applying the L'Hospital rule, find the limits of the follo-
wing functions:
. eax -e-2ax . VT+2X+1
(a) }~~ In (I +x) ; ( b) Itm .r- ;
x--1 r2+x+x
. ex -e-x --2x
(c) I tm . ; (d) I. In (I +x2)
x _ 0 x-sm x trll cos3x-e-x;
x- 0
,. sin 3x2 1/x2
{e) 1111
x _ 0

In cos (2x 2 - x) '
(f) !' e - I
tm 2arctanx2 -n"
x - "'
Solution. (a) Here b0th functions f e0 x-e- 2ax and g(x)= (x) =
+
=In (1 x) are infinitesimals in the neighbourhood of zero, since
lim f (x) = 1- 1 = O; lim g (x) = In 1 = 0.
x _.,. 0 x -)> 0

Furthermore f' (x) and g' (x) exist in any neighbourhood of the
point x = 0 that does not contain the point x - - 1, and
g' (x) = 1 ~ x =F 0 (x >- 1).
3.2. Evaluation of Indeterminate Forms 139

Finally, there exists a limit of the ratio of the derivatives:


. f'(x) . aeax+2ae-2ax
ltm ---,--() = X-+0
X->OgX
ltm I/( 1 +X) = 3a.
Therefore the L'Hospital rule is applicable:
. eax_e-2ax . aeax+2ae-2ax
;1_1,110 In (I +x) = !1!.110 1/(1 +x) - = 3a. (*)

Note. When the limit of the ratio is computed according to the


L'Hospital rule the result is usually written directly as shown in(*).
Whether the desired derivatives and limits exist is ascertained in
the course of calculation. In case the ratio of the derivatives ~'. ~;~
again represents an indeterminate form, the L'Hospital rule should
be applied for a second time, and so on until the indeterminacy is
removed or until it becomes clear that the required limits do not
exist. Therefore, henceforward we write only the necessary transfor-
mations, leaving to the reader the task of checking whether the
conditions of their applicability are fulfilled.

(b) lim
v1+2x+ I
_ =
.
hm =-;
42/(3 Vo +2x) 2)

x--1 Y2+x+x x--1 l/(2V2+x)+l 9


() r sin 3x2 _ (" -6X COS 3x2 CQS (2x 2 -x) _
e .i: 1.:11olncos(2x2 -x)-/~o (4x-l)sin(2x 2 -x) -
. cos 3x2 cos (2x 2 -x) 1. x
= - 6 11m 1m .
x_ 0 4x - I x __, 0 sm (2x2 - x)

The limit of the first factor is computed directly, the limit of the
second one, which represents an indeterminate form of the type ~ is
found with the aid of the L'Hospital rule:
. cos 3x 2 cos
6 11m (2x 2 -x~ . x
- - 1Im ---=---
x_ 0 4x - I x _ 0 sin (2x 2 - x)

=-6!.:_!_lim I = 6 -1- = - 6
-Ix_ 0 (4x-l)cos(2x2 -x) -11

3.2.2. It is known that, as x-+-+ oo, the functions xk (k > O);


logax; ax (a> 1) are infinitely large quaFttities. Applying the L'Hos-
pital rule, compare these quantities.
I
-logae
.
Sol ut ton. 1. r1m loga x
-k-=
rtm xk k-i = 1ogae l'Im -I = 0;
x-+oo X x-+oo X x-+ookxk

2. r xm r mxm -1 r m! - 0
Im
X-++OO
ax = X-++OOa
Im ------X----111 = =
a
Im
x-+oo
ax (In a)m -
140 Ch. 111. Differential Calculus: Investigation of Funcf's

Hence, the power function xk (k > 0) increases more rapidly than


the logarithmic function logax(a >I), and the exponential function
ax with the base exceeding unity increases more rapidly than the
power function xm.
3.2.3. Find the limits:
(a) x-
lim (-1- - -1- ) ; (b) lim (cot x-I.);
lnx1 x-1 x- 0 x
(c) x-0
lim (I.-
x
-
eX-1
1--)

Solution. (a) We have an indeterminate form of the type 00-00.

Let us reduce it to an indeterminate form of the type ~- and then


apply the L 'Hospital rule:
. ( -I - - -
I1m I ) . x-1 - ln.x = 1tm
= 11m . I - l/x
~---
" .... 1 In x x- I x .... 1 (x-1) In x x+ I - l/x
x .... 1 In
. x-1 .
= xI1m I + = !tm
.... I X n X X - 1 x .... I

3.2.4. Find the limits:


{a) Iim xn In x (n > O);
x-o
{b) lim [In ( 1
x .... 0
+ sin 2 x) cot ln 2 (I + x)].
Solution. (a) We have an indeterminate form of the type 0. oo.
Let us transform it to : , and then apply the L 'Hospital rule:
. I . In x . I/x I . O .
Itm xn nx=. 1tm x-n = 1tm -nx-n-i - - f i 1tm xn= ,smcen>O.
:1:-0 x- 0 x-0 x-0

(b) We have an indeterminate form of the type O oo:


. [! n (I+ srn
.11m . 2 x) co ti n2 (l+ x)] = 1tm
. In(l+sin2x) 2 (I+ ) =
.x .... 0 x .... 0 1an 1n x
I . 2
. I+ sini x sin x
=hm I
x .... o 2 {I +tan2 [In2 (I +x)l} In (I +x)- 1+x

r sinx ,. cosx
= x1m I + ) = x-o
.... on 0 x
1m - 1- = I.
l+x
3.2.5. Find the limits:
(a) Jim (l/x)in x; (b) lim xt/ln(e"-ll.
x .... +o x-+-+ 0
3.2. fa.Ja/uation of Indeterminate Forms 141

Solution. (a) We have an indeterminate form of the type 00.


Let y ~ (l/x)sin x; then
In y =sin x In (l/x),
lim lny= Jim sinxln(l/x) (indeterminate form of the type 0-oo).
x-----10 x___,.+Q

Let us transform it to 00
00
and apply the L'Hospital rule:
. I . -In x
l1111 ny = 11m - -.-= 1lm. -1/x . sin2 x
. . 2 = 1lm - - = 0 .
x~+o -'~+o
1
Ls111x x~o -(cosx)/sm x x-oxcosx
Hence, Jim y = e" = I.
X - I 0

3.2.6. Find the limits:


(a) Jim (sinx)tanx; (b) Jim xx.
x - ;r/2 x- 0

3.2.7. Compute
Jim (tan x) cot x.
x - + rr/2-0

Solution. Let us take advantage of the identity


(tan X) cot x = ecot x In tan x,
but
Jim cot x In tan x = Jim 1 ~ tan x Jim _Iny =0.
x-+rr/2-0 x-+rr/2-0 anx y=tanx-+oo Y
Whence
Jim (tan x) cot x = eo = l.
x- +rr/2- o

3.2.8. Ascertain the existence of the following limits:


()I . x2 sin(l/x).
a 1m . ,
x- 0 Sll1 X

(b) Jim 2+2x+sin2x ;


x - oo (2x+ sin 2x) ern x
.
(c) Illll tan x
-- .
x _ n/ 2 sec x
Can the L'Hospital rule be applied in computing them?
Does its formal application lead to the cerrect answer?
Solution. (a) The limit exists and equals zero. Indeed,
. x2 sin(l/x)
11m . . x . . I l O O.
= 11m - . - 11mxstn-= =
x- O SI 11 X x _ O Slll X x - O X

But the limit of the ratio of the derivatives does not exist. Indeed,
. 2x sin (l/x)-cos (l/x) O 1.
Illll = - Im COS - I ,
x~0 cos x x - 0 x
142 Ch. II I. Differential Calculus: I nvestigatinn of Fu net's

but lim cos (l/x) does not exist, hence the L'Hospital rule is not
x - 0
applicable here.
(b) The limit of the ratio of the functions does not exist:
.
11m 2+2x-t-sin2x
.
x _ oc (2x + sin 2x) e 5111 x
=
.
11m
x _ en
(t , -t-
2
.
2x -1- s1 n 2x ;
\
/ e-<inx

but li;n e-in x does not exist, since the function e- ,jn traverses the
x _,. oc
values from l/e to e infinitely many times.
Now we will show that the limit of the ratio of derivatives exists:
Ii m _____2_+_2_c_o_s_2_x_ _ _ __
x - ;;; 12 + 2 cos 2x+ (2x-I- sin 2x) cos xi esi" '
-_ 11m
. , 4cos 2 x
. e-inx_
--
x- "'4 cosc x+ (2x-t- s111 :2x) cos x
.
"= I irn
4 cos x
. e-<in. x =0
,. _"' 2x+ 4 cos x+ Sill 2x '

4 cos .t
since the function e- in x is bounded, and 9 + 4 + . - > 0.
~X COS X , Sill 2X x - oo
Here cosx, which vanishes for a11 infinite ~et of values of x, has
been cancelled out. It is the presence of this multiplier that makes
the L'Hospital rule inapplicable in this case, sine<> it simultaneously
nullifies the derivatives of the functions being compared.
( ) l . tan x
c 1.
1m - - = 1m
sec 2 x 1. sec x
= 1. tan x 1m - - = un - - =
"_ 1112 sec x "_ 1112 sec x tan x x
x-rr./ 2 tan x-:rr./ 2 sec x

Here application of the L'Hospital rule gives no useful result, though


there exists a limit:
. tanx . sinxcosx . l
11m --= 1Im = 11m StnX= .
x-:rr./ 2 sec x x-:rr/ 2 cos x x _ :r/ 2

3.2.9. Using the L 'Hospital rule find the limits of the following
functions:
. In (x 2 -3) aln x_x
(a) thm 2+3--10; (b) hm I ;
- 2 x x x-1 nx
() 1. tanx-x (d) . I -4 sin 2 (:ru:/6) .
I Im
c 1m . ;
x _ 0 X-Slll X
I -x-., ,

. x-a
"- I
(e) I 1m arc sin - - cot (x-a); (f) lim (n-2arctanx)lnx;
x-a a x- + 00
.
(g) I1m (I- )tJnx ; (h) lim (a 1/x_ l) x (a> O);
x-+o x
(i) lim (cosmx) 11 tx'; (j) lim ( 2 _ _:_)tan (:rrx/( 2a));
K-0 x-.. a a
.~ 3.3. Taylor's Formula: Approximate Calculatiom 143

(k) Iirn (-1


x - 1 In x
--2-);
In x
(I) lim xt/ln (e~-1);
x- 0
(m) Jim (
x- 0
1 -cot2
2
x
x); (n) }~~ [ x- x2 In ( l + ~) J;
(o) Jim x 2 [-cosh !:--
K-+Xl X
Ij ; (p) lim (
x- o 2+ Y9+x
5 )l/sinx;

. elfx' _ 1
(q) Jim (In cot x)tan x; (r) ltm .
x .. -rO x- 00 2 arc tan x 2 - :rt

3.3. Taylor's Formula. Application to Approximate


Calculations
If the function f (x) is continuous and has continuous derivatives
through order n-1 on the interval [a, b], and has a finite derivative
of the nth order at every interior point of the interval then at
x E [a, bJ the following formula holds true:
f (x) = f (a)+ f' (a) (x- a)+ r (a) (x~-, a) + 2

+ f'" (a) (x-31a)a


.
+ ... + fln-u (a) (x(-~~)~1
n .
+ f<n> (s) !x-,a)n'
n.
where
s=a+e(x-a) and 0<0< 1.
It is called Taylor's formula of the function f (x).
If in this formula we put a= 0, we obtain Maclaurin's formula:

f (x) = f (0) + f' (0) X + f" (0) ~; + ... + f<n-u (0) (n ~~; ! +
+ f<n> (s) xnnl ' where s=ex, 0 < e < 1.
The last term in the Taylor formula is called the remainder in
Lagrange's form and is denoted Rn (x):
Rn (x ) -_f<m [a+8
nl
(x-a)] ( _ )n
x a '
accordingly, the remainder in the Maclaurin formula has the form
Rn (X ) = f<n> (8x)
n!
n
X.

3.3.1. Expand the polynomial P(x)=x&-2x'+x 3 -x2 +2x-l


in powers of the binomial x-1 using the Taylor formula.
Solution. To solve the problem it is necessary to find the value
of the polynomial and its derivatives at the point x = 1. The
144 Ch. 111. Diflerential Calculus: I nvestigaiion of Funct's

relevant calculations are given below.


P (l) ~ 0, P' (I) =0,
P"(l)=O, P"'(l)=l8,
P( 41 (l)=-=72, P (i>) (I)= 120,
pim (x) = 0 (n;;?: 6)
at any x.
Substituting the values thus found into the Taylor formula, we get
P (x ) -_ 18 (
3! x-
l )3 4f ( x- l ) + 5!
+ 72 4 120 (
x- I )o.'
P (x) =3 (x-1) 3 + 3 (x-1) 4 + (x-1) 0
3.3.2. Applying the Maclaurin formula, expand in powers of x
(up to x9 , inclusive) the function
f(x)=ln(l+x),
defined on the interval [O, 1]. Estimate the error due to deleting
the remainder.
Solution.
f (0) = In 1 = 0.
The derivatives of any order of the given function (see 2.3):
f(n) (x) = (-l)n-1 (n-1)!
(I+ x)n'
fU' 1 (0)=(-l)n- 1 (n-l)! (n=l, 2, 3, ... ).
Substituting the derivatives into the Maclaurin formula, we get
x~ x3 x9
ln(l+x)=x-2+3- ... + 9+R 10 (x),
where the remainder R 10 (x) in the Lagrange form will be written
as follows:
f\10) (~) 9! xlO
Rio (x) = l ( j l xio = - 10! (I+ ~)10 xio =- 10 (I+ )1" (0 < ~ < x).
Let us estimate t.he absolute value of the remainder R10 (x); ket'ping
in mind that 0 ~ x ~ I and > 0, we have s
I
I Rio (x) I= 10(~:~)10 I< I~
3.3.3. How many terms in the Maclaurin formula should be taken
for the function f (x) =ex so as to get a polynomial representing
this function on the interval [-1, l], accurate to three decimal
place?
Solution. The function f (x) =ex has a derivative of any order
rm (x) =ex.
3.3. Taylor's Formula: Approximate Calculations 145

Therefore, the Maclaurin formula is applicable to this function. Let


us compute the values of the function ex and its first n-1 deriva-
tives at the point x = 0, and the value of the nth derivative at the
point s=Ox(O < 8 < 1). We will have
f (0) = f' (0) = f" (0) = ... = r"- 11 (0) =I;
rll) (s) =e' =eBx.
Whence

where
x"
R"(x)=-,
n.
esx.

Since, by hypothesis, \ x I~ I and 0 < 8 < I, then


Ix I" I 3
I R,,(x) l=-,-eex
n. < -,
n. e <-,.
n.
Hence, if the inequality
3
Iii~ 0.001
is fulfilled, then the inequality
IR,, (x) I~ 0.001
will be fulfilled apriori. To this end it is sufficient to take n ~ 7
(7! = 5040). Hence, 7 terms in the Maclaurin formula will suffice.
3.3.4. At what values of x will the approximate formula
x2 x4
cos x ~ 1-21+4f
have an error less than 0.00005?
Solution. The right member of the approximate equation repre-
sents the first six terms in the Maclaurin formula for the function
cos x (the second, fourth and sixth terms are equal to zero; check iti).
Let us estimate R6 (x). Since (cosx)( 6 >=-cosx, then

I R (x) I= cos ex
x 1~ ~
I - 6
6 6! ~ 6! .

For the error to be less than 0.00005, choose the values of x that
satisfy the inequality
Ix 16
---m- < 0.00005.
Solving this inequality, we get Ix I< 0.575.
3.3.5. Compute the approximate values of:
(a) cos 5; (b) sin 20,
146 Ch. 111. Differential Calculus: lniestigation of Funct's

accurate to five decimal places.


Solution. (a) Into the Maclaurin formula
xz x4 x2n
COSX= 1-21+41- ... + (-1)" (2n)! +R2 n+2

substitute x= n/36; since


x2
21=
:n;2
2362 =0.003808, 4T =5 "T
x4
l
I , x2 ) 2
=2.4-10- 6 ,
we confine ourselves to the following terms:
cos x ~ l-x2 /2,
the error being estimated at
IR4(x)l=j co:,ex x41~ 1~!14 <2.s.10-a.
And rn, within the required accuracy
cos5=COS~= 1-0.00381 =0.99619.

3.3.6. Compute the approximate value of V83 accurate to six


decimal places.
3.3.7. Prove the inequalities:
(a) x-x2 /2 < ln(l +x) < x at x > O;
(b) tanx > x+x 3 /3 at 0 < x < n/2;
I
(c) 1+ 2 x- 8 <
x2 v--
l+x<l+ 2I x at O<x<oo.
Solution. (a) According to the Maclaurin formula with the rema-
inder R2 (x) we have
x:a
In (1 +x) =x- 2 n+;> 2 ,
where O<s<x.
According to the same formula with the remainder R3 (x) we have
x2 xa
In (I + x) = x- 2 + +;
3 (1 1) 3 , where
o < ~ 1 < x.
x2 xa
Since 2 (I +;) 2 >0 and 3 (I +; 1 ) 3 >0 at x > 0, it follows that
x-x2 /2 <In (1 +x) < x.
3.3.8. Show that sin(a+h) differs from sina+hcosa by not
more than h2 /2.
Solution. By Taylor's formula
sin (a +h) =sin a +hcosa- sin 6; h;
3.4. Taylor's Formula: Evaluation of Limits 147

whence

3.4. Application of Taylor's Formula to Evaluation


of Limits
The expression
f(x)=f (a)+/'!(~) (x-a)+ f"_<,a) (x-a) 2
. ~-
+ ... +f n>,(a)(x-a)n+
n
1

+ o(lx-al"l
is the Taylor formula with the remainder in Peano's form where
qi(x)=o['ljl(x)]meansthat,as x-.a, the function qi(x) has a high-
er order of smallness than the function 'ljl (x), i. e. Jim ~ ((x)) = O.
x- a 't' X
In particular, at a=O we have

f (x) = f (0) + 1' 1\0) X + /"2(,0) X2 + ... + fCn~:O) xn + O (IX/").

Peano's form of the remainder for Taylor's formula shows that,


when substituting the Taylor polynomial of degree n for f (x) in the
neighbourhood of the point a, we introduce an error which is an
infinitesimal of a higher order than (x-a)n as x--+ a.
The following five expansions are of greatest importance m sol-
ving practical problems:

sinx=x- ~; + ... +(-1) 11 - 2 1


2: :._-1)! +o(x2 n);
1 ,

x2 x' x2n
cosx= 1- 21 + 41 + ... +(-l)n (2n)' +o(x2n+1);

(l+ x )a= l+ ax +a(a-1)


21 x +... 2 +a(a-1) ... (a-n+I)
n! x "+ o (x "),

In (1 +x) = X - ~2 + ~3 + ... + (-1) 11 - 1 :n +o (x").


3.4.1. Expand the function f(x)=sin 2 x-xze-x in positive inte-
gral powers of x up to the terms of the fourth order of smallness
with r.espect to x.
Solution. We have
f(x) = [x- ~3 +o(x')J2-x 2 [ 1-x+ ~2 + o(x 2 )1 =
x' x 4 5
=x2 - 3 +o (x&)-x2 +x 3 - 2 + o (x') = x 3 - 6 x'+o(x 4 ).
148 Ch. 111. Diflerenlial Calculus: I nuestigation of Funct's

3.4.2. Expand the following functions:


(a) f(x)=xVI-x 2 -cosxln(l +x);
(b) f(x)=ln(I+sinx)
in positive integral powers of x up to the terms of the fifth order
of smallness with respect to x.
3.4.3. Applying the Taylor formula with the remainder in Peano's
.form, compute the limits:
(a ) 1Im. I- VT+'X2 cos x
t ;
x: ... 0 an 4 x

(b) Iim
x ... 0
Vt+3x--;x Vl+2X
. cosx-e -x:'/2
(C) IIm 4 ;
x-> 0 x
(d) 1Im. ex sin x-x.1 (I -t-x).,
x:-> o x
. ex+e-x-2
(e) hm 2
x - 0 x
Solution. (a) Retaining the terms up to the fourth order with
respect to x in the denominator and the numerator, we get

lim
I- JfT+X2 cos x = Jim 1-(1 +x2) 112 cos x =
X-+0 tan 4 x x ... 0 x4
I- [1 +_!_ x2+ 112 (-l/2) x4 +o (x4)1 [1- x2 +~+o (x5)]
lim 2 2 2 24
I( ... 0
x4
1 I I
. 4 x4 +8 x4 -24 x4 + o (x4) . [ 1 o (x4) J 1
= hm
X:-+0 X
4 = X->0
hm 3 + -4-
X
=3

3.4.4. Expand the following functions in positive integral powers


of the variable x up to the terms of the indicated order, inclusive:
(a) f (x) = e2 x-x 2 up to the term containing x5 ;
(b) In cos x up to the term containing x 6 ;
(c) ex x 1 up to the term containing x'.

3.5. Testing a Function for Menotoniclty


Let a continuous function f (x) be defined on the interval [a, b)
and have a finite 8.erivative inside this segment. Then:
( l) For f (x) to be non-decreasing (non-increasing) on ['a, b] it is
necessary and sufficient that f' (x):;;::::: 0 (f' (x) ~ 0) f~r all x in (a, b).
3.5. Testing a Function for Monotonicity 149

(2) For f (x) to be increasing (decreasing) on [a, b] it is suf fi-


cient to fulfil the condition f' (x) > 0 (f' (x) < 0) for all x in (a, b).
3.5. 1. Determine the intervals of monotonicity for the following
functions:
(a) f (x) = 2x2 -ln x;
(b) f(x)=2x:1 -9x 2 -24x+7;
(c) f (x) = x e-x;
2

(d) f (x) =Jn Ix J;


(e) f (x) = 4x"-2lx + 18x+ 20;
2

(f) f (x) =ex+ 5x.


Solution. The solution of this prob!Em is reduced to finding the
intervals in which the derivative preserves its sign. If the function
f (x) has a continuous derivative in the interval (a, b) and has in
it a finite number of stationary points x 1 , x 2 , , xn (a< x 1 <
< x2 < ... < xn < b), where f'(xk)=O (k= 1, 2, ... , n), then f'(x)
preserves its sign in each of the intervals (a, x 1 ), (x1 , x 2 ), ,
(xn-1' Xn), (xn, b).
(a) The function is defined at x > 0.
Let us find the derivative
f' (x) = 4x- l/x.
The function increases if 4x-1/x > 0, i.e. x > 1/2.
The function decreases if 4x-1/x < 0, i. e. x < 1/2.
And so, the function decreases in the interval 0 < x < 1/2 and
increases in the interval 1/2 < x < +oo.
(b) Evaluate the derivative
f' (x) = 6x 2 - l8x-24 = 6 (x2 -3x-4).
It vanishes at the points x = -1 and x = 4. Since f' (x) is a
quadratic trinomial with a coefficient at its highest-power term 6 > 0,
then f' (x) > 0 in the intervals
(- oo, -1), (4, oo), and f' (x) < 0 !I
in the interval (- l, 4). Con-
sequently, f (x) increases in the
first two intervals, whereas in
(-1, 4) it decreases.
(c) h1 this case the derivative
f' (x) = (2x-x2 ) e-x vanishes at
the points X= 0 and x= 2. In
the intervals (- oo, 0) and (2, oo) Fig. 39
the derivative f' (x) < 0 and the
function decreases; in (0, 2) the derivative f' (x) > 0 and the f unc-
tion increases (see Fig. 39).
3.5.2. Find the intervals of decrease and increase for the follo-
wing functions:
150 Ch. I II. Differential Calculus: Investigation of Funct's

(a) f(x)=cos(n/x);
(b) f(x)=sinx+cosx on [O, 2n).
Solution. (a) The function y= cos (n/x) is defined and differentiable
throughout the number scale, except at the point x= O;
, n . n
Y =---,,Sin -
x~ x
.

As is obvious, the sign of y' coincides with that of the multi-


plier sin (n/x).
(l) sin (n/x) > 0 if
2kn < n/x < (2k+ l)n (k=O, +l, +2, ... );
(2) sin ( n/x) <0 if
(2k+ l)n < n/x < 2 (k+ l)n.
Hence, the function increases in the intervals

(2k~ I' ~)
and decreases in the intervals

( 2k ~2' 2k ~ I) '
3.5.3. Investigate the behaviour of the function f (x) = 2 sin x +
+ tan x- 3x in the interval (-n/2, n/2).
Solution. The derivative
f' (x) = 2 cos X + _1_.__ 3 = (1-co_s x)(I +cos x-2 cos2 x) =
cos 2 x cos 2 x
4 sin 3 (x/2) sin (3x/2'
=
cos 2 x
is positive in the intervals (-n/2, 0) and (0, n/2) and vanishes
only at X= 0. Hence, in (-n/2, n/2) the function f (x) increases.
3.5.4. Prove that at 0 < x ~ 1 the inflqua Ii ties
x-x 3 /3 <arc tan x < x-x 3 /6
are fulfilled.
Solution. We will prove only the right inequality (the left one
is proved analogously).
The derivative of the function
x3
f (x) =arc tan x-x+ 6
is equal to
3.5. Testing a Function for Monotonicity 151

The function f (x) is continuous throughout the entire number


scale, in particular, it is continuous in the interval [O, l], and in-
side this segment f' (x) < 0. Therefore, f (x) decreases on the interval
[O, I] and, consequently, for any point x, 0 < x::::;;; 1, the inequality
f (x) / f (0) = 0 or
xs
arc tanx-x+ 6 < 0
is fulfilled, whence
arc tanx < x- xs6 .
.3.5.5. Prove the inequalities
x-x 3 /6<sinx<x at x>O.
3.5.6. Prove that for 0::::;;; p::::;;; l and for any positive a and b
the inequality (a+b)P::::;;;aP-t-bP is valid.
Solution. By dividing both sides of the inequality by bP we get

(:+1y::::;;;(:y+1
or
(l+x)P::::;;; l+xP, (*)
a
where x=IJ
Let us show that the inequality (*) holds true at any positive x.
Introduce the function
f (x) = 1 + xP - ( l + x)P; x ~ 0.
The derivative of this function
f' (x) = pxP-1-p (l + x)P-1 = p [xi~p 1
(l+x)l-p J
is positive everywhere, since, by hypothesis, 1-p ~ 0 and x > 0.
Hence, the function increases in the half-open interval [O, oo), i.e.
f (x) = l +xP-(l + x)P > f (0) = 0, whence I +xP > (1 +x)P, which
completes the rcof. If we put p = I/n, then we obtain

Va+b::::;;; }:la+ V'b (n ~I).


3.5.7. Prove that the functiony=x"+2x 3 +x increases everywhere,
and the function y = l -x3 decreases everywhere.
3.5.8. Determine the intervals of increase and decrease for the
followirag functions:
(a) f (x) = X3 + 2x--5;
(c) f (x)=cosx-x;
152 Ch. Ill. Differential Calculus: lnvestiaation of Funct's

2x
(e) f (x) = 2x
-1 ; (f) f (x) = I + x~ .
nx
3.5.9. Prove the following inequalities:
(a) tanx > x+x 3 /3, if (0 < x < n/2);
(b) ex;;;:: 1 +x for all values of x;
(c) ex >ex at x > I.
3.5.10. At what values of the coefficient a does the function
f (x) = x 3 -ax increase along the entire number scale?
:>.5.11. At what value of b does the function
f (x) =sin x-bx +c
decrease along the entire number scale?

3.6. Maxima and Minima of a Function


If a function y= f (x) is defined on the interval X, then an in-
terior point x 0 of this interval is called the point of maximum of
the function f (x) [the point of minimum of the function f (x)] if
there exists a neighbourhood U EX of the point x 0 , such that the
inequality f (x) ~ f (x 0 ) [f (x);;;:: f (x 0 )] holds true within it.
The generic terms for points of maximum and minimum of a fun-
ction are the points of extremum.
A Necessary Condition fQr the Existence of an Extremum. At
points of extremum the derivative f' (x) is equal to zero or does
not exist.
The points at which the derivative f' (x) = 0 or does not exist are
called critical points.
Sufficient Conditions for the Existence of an Extremum.
I. Let the function f (x) be continuous in some neighbourhood of
the point x 0
I. If f' (x) > 0 at x < x 0 and f' (x) < 0 at x > x 0 (i.e. if in mo-
ving from left to right through the point x 0 the derivative changes
sign from plus to minus), then at the point x 0 the function reaches
a maximum.
2. If f' (x) < 0 at x < x 0 and f' (x) > 0 at x > x 0 (i.e. if in mo-
ving through the point x0 from left to right the derivative changes
sign from minus to plus), then at the point x0 the function reaches
a minimum.
3. If the derivative does not change sign in moving through the
point x 0 , then there is no extremum.
11. Let the function f (x) be twice di1Terentiable (that is f' (x 0 ) = 0)
at a critical pGint x0 If f" (x 0 ) < 0, then at x0 the function has a
maximum; if f" (x 0 ) > 0, then at x 0 the function has a minimum;
but if f" (x 0 ) = 0, then the question of the existence of an extremum
at this point remains open.
3.6. Maxima and Minima of a Function 153

I I I. Let f' (x 0 ) = f" (x 0 ) = ... = f<"- 1 > (x 0 ) = 0, but f 111 > (x 0 ) =I= 0. If
n is even, then at f<" 1 (x 0 ) < 0 there is a maximum at x 0 , and at
f<',, (x,,) > 0, a minimum.
If n is odd, then there is no extremum at the point x 0
IV. Let a function y = f (x) be represented parametrically:
x = <p (t), y = 1jJ (t),
where the functions cp (t) and 1jJ (t) have derivatives both of the first
and second orders within a certain interval of change of the argu-
ment t, and cp' (t) =I= 0. Further, let, at t = t 0
1jJ' (t) = 0.
Then:
(a) if 1jJ" (t 0 ) < 0, the function y = f (x) has a maximum at x =
= Xo = <p (to);
(b) if 1jJ" (t 0 ) > 0, the function y = f (x) has a minimum at x =
= Xo = <p (to);
(c) if 1jJ" (t 0 ) = 0, the question of the existence of an extremum
remains open.
The points at which cp' (t) vanishes require a special study.
3.6. t. Using the first derivative, find the extrema of the follo-
wing functions:
(a) f (x) = 43 X 4 -x 3 -9x2 + 7;
(b) f (x) =X -8x +22x -24x+ 12;
4 3 2

(c) f(x)=x(x+l) (x-3) 2 ; 3

x2-3x+2
(d) f(x)=x~+2x+l
Solution. (a) The function is defined and differentiable over the
entire number scale. Therefore, only the real roots of the derivative
f' (x) = 3x3 -3x 2 -18x = 3x (x + 2) (x-3)
are critical points. Equating this expression to zero, we find the
critical points: x1 = -2, x2 = 0, X 3 = 3 (they should always be ar-
ranged in an increasing order). Let us now investigate the sign of
the derivative in the neighbourhood of each of these points. Since
there are no critical points to the left of the point X= -2, the
derivative at all the points x <-2 has one and the same sign: it
is negative. Analogously, in the interval (-2, 0) the derivative is
positive, in the interval (0, 3) it is negative, at x > 3 it is posi-
tive. Hence, at the points x 1 = -2 and x3 = 3 we have minima
f (-2) 0 = -9 and f (3) = -40+, and at the point X2 = 0, maxi-
mum f (0) = 7.
154 Ch. 111. Differential Calculus: lnuestigation of Funct's

(c) Just as in item (a), the critical points are the roots of the
derivative f' (x), since the function is defined and differentiable
throughout the number scale. Find f' (x):
f' (x) = (x + l)' (x- 3) 2 + 3x (x + 1) 2 (x-3) 2 + 2x (x + 1) 3 x
x (x-3) = 3 (x + 1)2 (x-3) (2x 2 -3x- 1).
Equating this expression to zero, we find the critical points:
=-l, X2 =(3-VT7);4, X 3 =(3+1/T7);4, X 4 =3.
X1

Let us tabulate the sigm of the derivative in the intervals be-


tween the critical points:

Intervals I
~~~~.,___~~-.,...-~~
x < x1 Ix 1 < x < x2 I
X2 <x< X3 Ix,, < x < x 4 / x~ <x

Sign of /' (x) I I I + I +


As is seen from the table, there is no extremum at the point
x 1 = -1, there is a minimum at the point x2 , a maximum at the
point x 3 , and a minimum at the point X 4
3.6.2. Using the first derivative, find the extrema of the follo-
wing functions:
(a) f (x) = 3 VX2-x 2 ;
(b) f(x)=V(x-l) 2 +V(x+l) 2
Solution. (a) The function is defined and continuous throughout
the number sea le.
Let us find the derivative:
,, <x>= 2 (v;-x).
From the equation f' (x) = 0 we find the roots of the derivative:
X= + l.
Furthermore, the derivative goes to infinity at the point x = 0.
Thus, the critical points are x1 = - 1, x 2 = 0, x3 = l. The results
of in\'estigating the sign
of the derivative in the
)' x neighbourhood of these
points are given in Fig.
Fig. 40 40. The investigation
shows that the function
has two maxima: f(-1)=2; f(l)=2 and a minimum f(O)=O.
3.6.3. Using the second derivative, find out the character of the
extrema of the following functions:
3.6. Maxima and Minima of a Function 155

(a) y=2sinx+cos2x;
(b) f (x) = 2x 3 - l5x2 -84x 8. +
Solution. (a) Since the function is a periodic one we may confine
ourselves to the interval [O, 2n). Find the first and second deri-
vatives:
y' = 2 cosx-2 sin 2x=c= 2 cos x (l-2 sin x);
y" = -2 sin x-4 cos 2x.
From the equation 2 cos x (l-2 sin x) =~ 0 determine the critical
points on the interval [O, 2n]:
x 1 = n/6, x 2 = n/2, X 3 = 5:n:/6, X 4 = 3n/2.
Now find the sign of the second derivative at each critical point:
y" (:rt/6) = - 3 < O; hence, we have a maximum y (n/6) = 3/2 at
the point x 1 = n/6;
y" (n/2) =-= 2 > O; hence, we have a minimum y (n/2) =I at the
point X 2 = :rt/2;
y" (5n/6) = - 3 < 0; hence, we have a maximum y (5n/6) = 3/2
at the point x,1 = 5n/6;
y" (3n/2) = 6 > O; hence, we have a minimum y (3n/2) = - 3 at
the point x 4 = 3n/2 (see Fig. 41).
!J

Fig. 41

3.6.4. Investigate the following functions for extrema:


_ f -2x (x < 0),
(a) f (x) - I 3x + 5 (x;;;;: O);
(b) f (x)= { 2x 2 +3(x=i=O),
4 (x = 0).
Solution. (a) Though the derivative
f' (x) = f - 2 (x < 0),
I 3 (x > O)
exists at all points, except the point x = 0, and changes sign from
minus to plus when passing through the point x = 0, there is no
minimum here:
f (0) = 5 > f (x) at -1 < x < 0.
155 Ch. 111. Differential Calculus: Investigation of Funct's

This is explained by the fact that the function is discontinuous


at the point x = 0.
(b) Here the derivative f' (x) = 4x (x =F 0) also exists at all points,
except at x = 0, and it changes sign from minus to plus when pass-
ing through the point x = 0. Nevertheless, we have here a maximum
but not a minimum, which can readily be checked.
It is explained by the fact that the function is discontinuous at
the point x = 0.
3.6.5. Find the extrema of the following functions:
50
(a) f (x) =3x4 +sxs-1sx2+60;
(b) f (x) = JI ex2 - I.
Solution. (a) Here it is simpler to find the extrema of the func-
tion f 1 (x) = 3x4 + 8x3 - l8x2 + 60. Since
f; (x) = 12x3 + 24x2 -36x = 12x (x 2 + 2x-3),
f~ (x) = 12 (3x2 + 4x-3),
the critical points are:
X1 = - 3, X2 = 0, X3 = J,
and the character of the extrema is readily determined from the
sign of the second derivative f; (-3) > O; hence, at the point x, = -3
the function f 1 (x) has a minimum, and the given function f (x)
obviously has a maximum f (-3) = - 2/3, f; (0) < O; hence, at
the point x2 = 0 the function f 1 (x) has a maximum, and f (x) a
minimum f(0)=5/6; f~(l)>O; hence, at the point X3 =l the
function f i(x) has a minimum, and f (x) a maximum f (1) = 50/53.
(b) In this case it is easier to find the points of extremum of
the radicand
f1(X)=ex2_l,

which coincide with the points of extremum of the function f (x).


Let us find the critical points of f 1 (x):
t;(x) = 2xex 2 ; f; (x) = 0 at the point x = 0. Determine the sign of
the second derivative at the point x = 0:
f; (x) = 2ex' (1 + 2x 2 ), f~ (0) = 2 > 0.
Therefore the point x = 0 is a minimum of the function f 1 (x); it
will also be a minimum of the given function f (x): f (0) = 0.
3.6.6. Investigate the character of the extremum of the function
y=coshx+cosx at the point x=O.
Solution. The function y is an even one and apparently has an
extremum at the point x = 0. To determine the character of the
3.6. Maxima and Minima of a Function 157

extremum Jet us evaluate the derivatives of this function at the


point x=O:
y' == sinh x-sin x, y' (0) = 0;
y" =cosh x-cosx, y" (0) = O;
y"' = sinh x +sin x, y"' (0) = O;
y< 4 > = cosh x +cos x; y"' (0) = 2 > 0.
Since the first non-zero derivative at the point X=O is a derivative
of an even order, which takes on a positive value, we have a mi-
nimum y (0) = 2 at this point.
3.6.7. Investigate the following functions for an extremum at
the point x = 0:
(a) y=cosx-1+~;-~;; (b) y=cosx-1+~2
x2
Solution. (a) y'=-sinx+x- 2 ; y'(O)=O;
y" =-cosx+ 1-x; y" (0) =0;
y"' =sin x- I; y'" (0) = - 1 =I= 0.
And so, the first non-zero derivative at the point x = 0 is a deri-
vative of the third order, i. e. of an odd order; this means that
there is no extremum at the point x= 0.
3.6.8. Investigate the following functions for extrema:
(a) f (x)=x 4 e-x 2 ; (b) f(x)=sin3x-3sinx.
Solution. (a) The function f (x) = x 4e-x 2 is continuously differen-
tiable everywhere. Equating the derivative
f' (x) = 4x3e-x 2 -2x 0 e-x2 = x3e-x 2 (4-2x 2
)

to zero, find the critical points:


X1 =-V2; X2 =0; X3 = V2.
Compute the values of the second derivative at the critical points~

f" (x) = 12x2e-x" -8x4 e-x" - l0x 4 e-x" +4x6 e-x" =


= 2x2e-x 2 (6-9x 2 -r- 2x );
f" (0) = O; f" (- V2) < 0; f" (V2) < 0.
Consequently, at the points X1 =-V2 and X3= + v2
the function
reaches a maximum f (+ V2) = 4e- = e~ . As far as the critical
2

point x2 = 0 is concerned, nothing definite can be said as yet, we


have to find derivatives of f (x) of higher orders (up to the fourth
order!). But this process is cumbersome, therefore \\'e will turn to
the first sufficient condition of an extremum: let us find the sig11s
158 C/z. 111. Differential Calculus: Investigation of Funct"s

of the first derivative in the neighbourhood of the critical point


X 2 =0:
f' (-1) < O; f' (1) > 0.
Hence, at the point x = 0 the function has a minimum f (0) = 0.
3.6.9. The function y = f (x) is rPpresented parametrically:
I x= cp (t) = tj-sr1 -2ot -1- 7,
\ y=\jl(t)=4f 3 -3f 2 -18t+3 (-2<t <2).
Find the extrema of this function.
Solution. We have
cp' (t) = 5t 4 - l 5t 2 -20.

In the interval (-2, 2) cp' (t) =I= 0.


Find \jl' (t) and equate it to zero:
"'' (t) = 12t 2 -6t - 18 = 0.
Whence t 1 = - 1 and t 2 = 3/2.
These roots are interior points of the considered interval of va-
riation of the parameter t.
Furthermore:
\jl" (t) = 24t -6; \jl" (-1) = - 30 < 0, 'Ii" (3/2) = 30 > 0.
Consequently, the function y = f (x) has a maximum y = 14 at
i=-1 (i.e. at x=31) and a minimum y=-17.25 at t=3/2
(i. e. at x= - 1033/32).
3.6. lO. Find the maxima and minima of the following functions:

f (x) = 4x
(a) f(x)=x 2e-x; (b) x2+4;

(c) f (x) = - x2 V (x-2) 2;


(d) f (x) =
14
x4-8x2+2;
(e) f (x)= V2x 3 +3x2 -36x;
(f) f (x) = x2 In x; (g) f (x) = x ln 2 x.
3.6.11. Investigate the following functions for an extremum at
the point x = 0:
(a) f(x)=sinx-x; (b) f(x)=sinx-x+x 3 /3;
. . x~ x4
(c) t(x)=stnx-x+ 31 - 41 ;
e1 1x if x=j=.O
{
(d) f(x)= O, 'if x=o.'
3.7. Finding the Grpa/cst and the Least Values of a Function 159

3.7. Finding the Greatest and the Least


Values of a Function
The greatest (least) value of a continuous function f (x) on an
interval [a, b) is attained either at the critical points, or at the-
end-points of the interval. To find the greatest (least) value of the
function we have to compute its values at all the critical points
on the interval [a, b), the values f (a), f (b) of the function at the
end-points of the interval and choose the greatest (least) one out
of the numbers obtained.
If a function is defined and continuous in some interval, and if
this interval is not a closed one, then it can have neither the
greatest nor the least value.
3. 7 .1. Find the greatest and the least values of the following
functions on the indicated intervals:
(a) f(x)=2x 3 -3x2 - i2x+ I on [-2, 5/2);
(b) f(x)=x 2 lnx on[!, e);
(c) f (x) = xe-x on fO, + oo );
(d) f(x)=V(l-x 2 )(1-t-2x 2 ) on [-1. l).
Solution. (a) Find the derivative f' (x):
/' (x) = 6x2 -6x-12.
It vanishes at two points: x1 = - l and x2 = 2. They both lie in-
side the indicated interval [-2, }] ; consequently both of them
must be taken into consideration. To find the extreme values of
the function it is necessary to compute its values at the points
x1 and x2 , and also at the end-points of the segment:

f(-2)=-3, f(-1)=8; f(2)=-19, t(~)=-16 ~.


Hence, the greatest value is f (-I)= 8 and the least f (2) = -
19_
(b) Find the critical points: f' (x) = x ( 1+2 ln x). The derivative
f' (x) does not vanish inside the given interval [l, e]. ThNefore
there are no critical points inside the indicated interval. It now
remains to compute the values of the function at the end-points of
the interval [I, e)
f (1)=0; f (e)=e2
Thus, f(l)=O is the least value of the function and f(e)=e 2 the
greatest.
3.7 .2. Find the greatest and the !east values of the following
fund ions on the indicated intervals:
(a) y=sinxsin2x on (-oo, oo);
160 Ch. 111. Differential Calculus: Investigation of Funct's

(b) y=arccosx 2 on [-J/2;2, J/2;2];


(c) y=x+V-x.on [O, 4].
Solution. (a) Represent the function y =sin x sin 2x in the form
cos x-cos 3x
y= 2

whence it is seen that the function is an even one and has a pe-
riod 2n. Hence, it is sufficient to seek the greatest and the least
values among the extrema on the interval [O, n]. Find the deri-
vative y':
y' = ~ (3 sin .3x-sin x).
In [O, n] the derivative vanishes at the points
x1 = 0, x2 =arc cos ;f, x3 = arc cos ( - ,
y~:f) x~ = n.

Compute the values of the function at these points:

y (0) = y (n) = 0, y [arc cos ( + ; 3 ) J= + 3 ~f.


Hence, the least value of the function in the interval (- oo, oo)
is equal to -4/(3 V3), and the greatest to 4/(3 V3).
3.7.3. The function
f (x) =ax+!!._x (a, b, x > 0)
consi~ts of two summands: one summand is proportional to the
independent variable x, the other inversely proportional to it.
Prove that this function takes on the least value at X= Vb/a.
Solution. Find the roots of the derivative f' (x) in the interval
(0, 00 ):
f' (x) =a - :2 = 0
at x = V-b/a (x > 0). Since f" (x) = 2b/x 3 > 0
for any x > 0, the f unc-
tion f (x) reaches a minimum at this critical point. This is the
only exiremum (minimum) in the interval (0, oo ). Hence, at
x = J/ b/a the function f (x) attains the least value.
3.7.4. As a result of n measurements of an unknown quantity
x the numbers x 10 x2 , , x,. are obtained.
It is required to find at what value of x the sum of the squares
of the errors
f (x) = (X-X1) 2 + (X-X2) 2 + ... + (x-x,.)2
will be the least.
.~ 3.7. Finding the Greatest and the Least Values of a Function lfi l

Solution. Compute the derivative


f' (x) = 2 (x-x 1 H- 2 (x-x 2 ) + ... + 2 (x-xn)
The only root of the derivative is
X= X1 + X2 + ... + Xn
n
Then, for all x we have f" (x) = 2n > 0. Therefore, the function
f (x) has its minimum at the point
X = X1 + X2 + ... + X,,
n
Being the only minimum, it coincides with the least \'alue of
the function (cf. Problem 1.3.8).
And so, the best (in the sense of "the principle of the minimum
squares") approximate value of an unknown quantity x is the arith-
metic mean of the values x 1 , x2 , , xn-
3.7 .5. Find the largest term in the sequence

0n = n'1 + 200 .
. x~
Solution. Consider the funct10n f(x)=x 3 + 200 in the interval
[l, oo). Since the derivative
t X (400-X3 )
f (x) = (xa +200)2

is positi\e at 0 < x < V


400 and negative at x > 400, the V
function f (x) increases at 0 < x < V 400 and decreases at x > V 400:
From the inequality 7 < V 400 < 8 it follows that the largest term
in the sequence can be either a1 or a8 Since a1 = 49/543 > a8 = 8/89,
the largest term in the given sequence is
49
01 = 543.

3.7 .6. Find the greatest and the least values of the following
functions on the indicated intervals:

(a ) f (x) = 4l x4 - 32 x'l - 23 x2 + 2 on [- 2 , 4] ;
(b) f (x) = V4-x2 on [ -2, 2];
(c) f(x)=arctanx-f lnx on [ ; 3 , V3-];
(d)/(x)=2sinx+sin2x on [o, ~ n:j;
6 -3148
162 Ch. II I. Differential Calculus: Investigation of Funct's

{e) f(x)=x-2lnx on [I, e];


(f) f (x) = ~ 2x2+!
1
2 for -2 ~ x < O; 0 < x ~ 2,
I for x =0.

3.8. Sol'Ving Problems in Geometry and Physics


3.8.1. The force of a circular electric current acting on a small
magnet with the axis perpendicular to the plane of the circle and
passing through its centre is expressed by the formula
F= Cx
(a2+x2i'!, '
where a= radius of the circle
x =distance from the centre of the circle to the magnet
(0 < x < oo)
C =constant.
At what x will the value of F be the greatest?
Solution. The derivative
F'(x)=C a2-2x2
(a2+x2)'/1

has a single positive root x=a/V2. This solves the problem.


Note. It often happens that reasons of purely physical or geo-
inetric character make it unnecessary to resort to the differential
methods in investigating a function for the greatest or the least
value at the point under consideration.
3.8.2. Determine the most economical dimensions of an open-air
swimming pool of volume 32 m 3 with a square bottom so that the
facing of its walls and bottom require the least quantity of ma-
terial.
Solution. Let us denote the side of the bottom by x and the
height by y. Then the volume V of the pool will be
V =x2 y=32, (*)
and the surface S to be faced
S=x 2+4xy.
Expressing y through x from the relation (*), we get
S=x2+4x~ =x2+ 128.
x~ x
Investigate the function thus obtained for a minimum in the
interval (0, oo ):
S' = 2x - ~8 ; 2x - 1;~ = O; x = 4.
.~ 3.8. Solving Problems in Geometry and Physics 163

The single point thus found will obviously yield the least value
of the function S, since it has no greatest value (it increases un-
boundedly as x ~ 0 and x ~ oo ).
And so, the required dimensions of the pool are: x = 4 m, y = 2 m.
3.8.3. Inscribe into a given sphere a cylinder with the greatest
lateral surface.
3.8.4. 20 m of wire is available for fencing off a flower-bed
which should have the form of a circular sector. What must the
radius of the circle be if we wish to have
a flower-bed of the greatest possible surface !J
area?
Solution. Let us denote the radius of the
circle by x, and the length of the arc by y
(see Fig. 42). Then
20 = 2x+ y, O
whence
y = 2 (10-x). Fig. 42
The area of the circular sector S= ~ xy=x(lO-x) (O~x~lO).
The derivative S' (x) = 10-2x has a root x = 5.
Since the least value S = 0 is reached at the end-points of the
interval [O, 10], the obtained value
x = 5 yields the greatest surface area S.
3.8.5. It is required to construct
an open cylindric<ll reservoir of capa-
city V0 The thickness of the material
is d. What dimensions (the base radius
and height) should the reservoir have
so as to ensure the least possible
expenditure of the material?
Solution. Figure 43 represents a lon-
gitudinal section of the reservoir,
where the radius of the base of the
inner cylinder is denoted by x and
the height of the inner cylinder,
Fig. 43
by h. The volume of the bottom
and the wall of the reservoir
+
V = n (x+d) 2 d+ n [(x+ d) 2 -x 2 ] h = nd (x+d) 2 nh (2xd +d2 ). (*)
On the other hand, by hypothesis we must have
V0 = nx 2 h
whence
h-= !i_2
nx
164 Ch. 111. DitJerenfial Calculus: lnuestigation of Funcl's

Substituting into (*), we get

V-= nd (x + d) 2 +:rev."2 (2xd + d2 ) = nd (x + d) 2 + 2Vod + v".d 2



rrx x x~

Now we have to investigate the obtained function V (x) for an


extremum at x > 0.
We have
V' (x) = 2nd (x + d)- 2~ 0 d _ 2V 0 d2 = 2d (x+d) (nx 3 -V0 )
x2 xa xa
.!/--
The only positive root of the derivative is the point x = v V0 /n.
This solves the problem:

V -~ o =X.

3.8.6. A factory D is to be connected by a highway with a


straight railway on which a town A is situated. The distance DB
from the factory to the railway is
])
equal to a, the segment AB of the
R railway equals [. Freight charges on
/ ' II
/ I the highway are m times higher than
,'
/ I
ia on the railway (m > 1).
,
/ I
I How should the highway DP be
/
/ I
I
connected with the railway so as to
-~-o-~~~Gef~~~~~~ ensure the least freight charges from
A x P 8 factory to town?
Fig. 44
Solution. First, let us make a draw-
ing (see Fig. 44). It is absolutely
clear that the highway must also be
straight (a straight line is shorter than any curve connecting two
given points!). Furthermore, the point P cannot lie either to the
left of the point A or to the right of the point B. If we denote
the distance AP by x, it will mean that 0 ~ x ~ l.
Let the freight charges on the railway (per ton-kilometre) be k,
then the freight charges on the highway will be km. The total
freight charge N for transporting loads from D to A amounts to
N =kx+km Va 2 +(L-x) 2
Hence, we have to find the least value of the function
f (x) = x+m ~1 a 2 + (x-l) 2 , O~x ~ l.
Take the derivative
m (x-1)
f' (x) = 1+ y a2+ (x-/)2
3.8. Solving Problems in Geometry and Physics 165

It vanishes only at one point:


X=l- a
Vm 2 -1
If this point lies in the interval [O, l]. i.e. if

l ;;;::: ~ or al ~ V m 2 - I,
m2-J

then it yields the least freight charge (which is easy to check).


If the indicated inequality is not observed, then f (x) increases on
[O, l) and therefore the least freight charge is obtained at x = 0.
3.8.7. In constructing an a-c transformer it is important to insert
into the coil a cross-shaped iron core of greatest possible surface
area. Fig. 45 shows the cross-section 1
of the core with appropriate dimen-
sions. Find the most suitable x and
y if the radius of the coil is equal
to a.

3.8.8. If the source of current is an


electric cell, then the effect P (watts)
obtained by cutting a resistance R
(ohms) in the circuit is expressed by
the formula
E2R
P=!R+R;)2 Fig. 45
where E is electromotive force in volts and R; the internal resis-
tance in ohms.
Find the greatest effect which can be obtained at given E and R;.
3.8.9. A tin of a given volume V has the form of a cylinder.
What must ~e the ratio of its height h to diameter 2R so as to
use the least amount of material for its manufacture?
3.8.10. In a given cone inscribe a cylinder having the greatest
lateral surface so that the planes and centres of the base circles
of the cylinder and cone coincide.
3.8.11. Given a point (1, 2) in the orthographic coordinates.
Through this point draw a straight line so that it forms, together
with the positive semi-axes, a triangle of the least area.
3.8.12. Given a point M on the axis of the parabola y2 = 2px
at a distance a from its vertex. Find the abscissa of the point
on the curve nearest to the given point.
166 Ch. 111. Differential Calculus: Investigation of Funct's

3.8.13. The expenses sustained in one hour's sailing of a ship


are expressed in roubles by an empirical formula of the form
a+ bv3 , where a and b are constants for a given ship, and v is the
ship's speed in knots (one knot is equal to 1.85 km/hr). In this
formula the constant part of the expenses a refers to depreciation
and crew's upkeep, and the second term (bv 3 ) to the fuel cost.
At what speed will the ship cover any required distance at the
lowest cost?
3.8.14. A trough is built from three boards of equal width. At
what slope should the lateral boards be placed to ensure the largest
cross-sectional area of the trough?
3.8.15. A tank with a vertical wall of height h is installed on
a horizontal plane. Determine the position of an orifice, at which
the range of a liquid jet will be the greatest if the velocity of
flow (according to Torricelli's law) is equal to V2gx, where x is
the depth of the orifice.
3.8.16. Two aircraft are flying in a straight line and in the
same plane at an angle of 120 to each other and with an equal
speed of v km/hr. At a certain moment one aircraft reaches the
point of intersection of their routes. while 1he second is at a dis-
tance of a km from it. When will the distance between the
aircraft be the least and what is that distance?

3.9. Convexity and Concavity of a Curve. Points of


Inflection
If f" (x) < 0 (> 0) on an interval (a, b), then the curve y = f (x)
on this interval is convex (concave), i.e. it is situated below (above)
any of its tangent lines.
If f" (x11 ) = 0 or does not exist but f' (x 0 ) does exist and the second
derivative f" (x) changes sign when passing through the point x0 ,
then the point (x 0 , f (x 0)) is the point of inflection of the curve
y-= f (x).
3.9.1. Find the intervals in which the graphs of the following
functions are concave or convex and locate the points of inflection:
(a) y = x 4 + x 3 - l8x2 + 24x-12;
(b) y=-"3x 4 -8x 3 +6x2 + 12;
x
(c) y = I +x2;
(d) y =x+x't,;
(e) y=4V(x-1)"+20V(x-1) 3 (x;;;=:11;
ln 2 x
(f) y=-x- (x > O);
:1.9. Convexity and Concavity. Points of Inflection 167

(g) y = x sin (In x) (x > O);


(h) y=2-lx"-l I.
Sotution. (a) Find the derivatives:
y' = 4x3 + 3x2 -36x + 24,
y"=l2x2 +6x-36= 12(x 2 +~-3),
whence y" = 0 at x1 = -2, x2 = 3/2.
Hence, y" > 0 on the intervals (-oo, -2) and (3/2, oo); y" < 0
on the interval (-2, 3/2). The sign of the second derivative deter-
mines the convexity or concavity of the curve in a given interval.
This enables us to compile the following table:

x
I x <-2 /-2 < x < ~I x> 2
3

I -
Sign of y" +
I I +
Conclusion I Concavity I Convexity I Concavity

Since the second derivative changes its sign when passing through
the points x1 = -2 and x2 = 3/2, the points (-2, -124) and
( ~ , -8 ~) are points of inflection.
(d) Find the derivatives:
= 1 + 35
Y' x'lo ' y" -
-9v 10
x
The second derivative is non-zero everywhere and loses its meaning
at the point x = 0. At x < 0 we have y" < 0 and the curve is con-
vex, at x > 0 we have y" > 0 and the curve is concave.
At the point X= 0 the first derivative y' = 1, the second deriva-
tive changes sign when passing through the point x = 0. Therefore
the point (0, 0) is a point of inflection.
(g) Find the derivatives:
y' =sin (In x) +cos (In x),
y" =+[cos (In x)-sin (lnx)] =~~sin (~-In x).
The second derivative vanishes at the points
Xk=enf 4 +kn, k=O, + 1, +2, ....

The function sin(n/4- lnx), and together with it y", changes sign
when passing through each point xk. Consequently, the points xk
168 Ch. 111. Differential Calculus: Investigation of Funct's

are the abscissas of the points of inflection. In the intervals


(e2kn-an/4, e2kn+n/4)

the curve is concave, and in the intervals


(e2kn+n/4, e2kn+an/4)

it is convex.
(h) The given function can be written in the following way:
2 - (x 0 - 1), x;:;:: 1,
{
y= 2+(x 5 - l), x <I.
Therefore
, I -5x ' 4 x >I,
Y =\ 5x 4 , x <I.
At the point x = 1 there is no derivative. Further,
f - 20x3 , x > I,
y" = l 20x 3, x < I;
y"=O at the point x=O. Hence, we have to investigate three in-
tervals: (-oo, 0), (0, I), (l, oo).
Compile a table of signs of y":

x
I
x<O I0 < x <I I x>I

Sign of y" I - + -
I I
I I
Conclusion Convexity Concavity Convexity I

The point (0, I) is a point of inflection, the point (I, 2) being


a corner point.
3.9.2. What conditions must the coefficients a, b, c satisfy for
the curve y=ax 4 +bx 3 +cx2 +dx+e to have points of inflection?
Solution. Find the second derivative:
y" = l 2ax 2 6bx 2c. + +
The curve has points of inflection if and only if the equation
6ax2 + 3bx+c = 0
has different real roots, i.e. when the discriminant 9b 2 - 24ac > 0, or
3b 2 -8ac > 0.
3.9. Com'exity and Concavity. Points of /1171ec/ion 169

3.9.3. At what values of a will the curve

y = x 4 + ax" +- ; x2 + I

be concave along the entire number scale?


Solution. Find y":
y" ~= 12x2 + 6ax + 3.
The curve will be concave along the entire number scale if y";:::: 0
for all values of x, i.e. when
4x 2 + 2ax + 1 ;:::: 0 for all x.
For this it is necessary and sufficient that the inequality 4a 2 - 16 ~ 0
be fulfilled; whence
lal~2.
x+I
3.9.4. Show that the curve y c~ x2+ I has three points of inflec-
tion lying in a straight line.
Solution. Find the derivatives:
, --x 2 -2x+ I
y-~ (x2+1)2
,, 2x"-i fix 2 -6x-2
y c__ (x 2 -t-- I)"

The second derivative becomes zero at three points, which are the
roots of the equation
x:i + 3x2 - 3x - I = 0,
whence
x,. == 1.
Let us compile the table of signs of y":

---co<x< -2-- J/:f < r< --2_[_ VT<


x
< -2-- v:i < -2 :- V3 <x< I l<x<oo

Sign of !I " -- ~~- - _[__


I

I I I I
1
Conclusion Convexity Concavity Convexity Concavity I
I I I I j

Hence, (--2-~,r3, - ~-- 1 ), (-2+V3, '+[3 ),(1, I)


are points of inflection. It is easy to ascertain that all of them
170 Ch. II I. Differential Calculus: lnuestigation of Funct's

lie in a straight line. Indeed, the coordinates of these points satisfy


th e re It . -2-3-1 (1-J/3)/4--1
a ion ,r- = .
-2+r 3-1 (1+3)/4+1
3.9.5. Investigate the curves represented by the following equa-
tions for convexity (concavity) and locate the points of inflection:
(a) y=x- i/(x-3) 2 ;
(b) y = ein x (-n/2 ~ x~ n/2).
3.9.6. Show that the points of inflection of the curve y = x sin x
+
lie on the curve y 2 ( 4 x2 ) = 4x2

3.10. Asymptotes
A straight line is called an asymptote to the curve y = f (x) if
the distance from the variable point M of the curve to the straight
line approaches zero as the point M recedes to infinity along some
branch of the curve.
We will distinguish three kinds of asymptotes: vertical, horizon-
tal and inclined.
Vertical asymptotes. If at least one of the limits of the function
f (x) (at the point a on the right or on the left) is equal to infi-
nity, then the straight line x =a is a vertical asymptote.
Horizontal asymptotes. If lim f (x) =A, then the straight line
x- 00

y = A is a horizontal asymptote (the right one as X-+ + oo and


the left one as x-... - oo ).
Inclined asymptotes. If the limits
lim f (x) = k1 , Iim [f (x) - k1 x] = b1
x-+> X X-++oo

exist, then the straight line y = k 1 x +b 1 is an inclined (right)


asymptote.
If the limits
Jim f (x) = k 2 and lim [f (x) - k 2 x] = b2
X-+-CX> X X-+-rl:J

+
exist, then the straight line y = k 2x b2 is an inclined (left) asymp-
tote. A horizontal asymptote may be considered as a particular
case of an inclined asymptote at k = 0.
3.10.1. Find the asymptotes of the following curves:
5x 3x x
(a) y=--
x- 3 ; (b) y=--
x- 1 +3x; (c) Y=-r-+
x 1 ;

I ...!.... 3x r I )
(d) Y=-x+4x 2 ; (e) y=xex; (f) y= 2 1n~e-3X ;
3.10. Asympto~es 171

(g) y=Vl+x2 +2x; (h) y=JII +x2 sinJ.;


x
(i) y=2V-x 2 +4.
Solution. (a) The curve has a vertical asymptote x = 3, since
.
I1m .
11m 5x
y= --=+oo
x-3:i=O x-3:i=ox-3

(the point x = 3 is a point of discontinuity of the second kind).


Find the horizontal asymptote:
.
I1m y= 11m -5x-3 = 5.
X - CX> X-+ CD X-

And so, the curve has a vertical asymptote x= 3 and a horizon-


tal one y= 5.
(b) The curve has a vertical asymptote x = I, since
!I
lim
X-+1-0
y= lim
x-1-0
(x 3x1 +3x) = - oo;
'V'
I
I
i
I
7

Jim y Jim ( 3\ +3xJ' =


= X-+l+O + oo. I
x-1+0

Find the inclined asymptotes:


X
,,v
I
iI
I

k = lim 1L = lim ( 3 +3)=3; ~


x-oo X X-+"' X
1 /1
I
I
b = lim (y- kx) = I
I
X-+ :::ii

= Jim ( ~ 1 + 3x - 3x) = 3.
x-oo X

Thus, the straight line y = 3x+3 is an


inclined asymptote (see Fig. 46). hg. 46
(e) The curve has a vertical asymptote x = 0, since
et
Iim y = Jim xe 1tx = Iim -= +oo
x-+O X-++0 1 t
I=--+"'
x
(see Problem 3.2.2.).
Find the inclined asymptotes:

k= lim .JL= lim eifx=eo=l


x-CI> X X-+oo r

elfx_ I . ez-1 I
b = lim (xe 1 1x - x) = lim IIm --= .
X-+IX> x:-.oo l/x l/x=z-+0 z
172 Ch. 111. Difjerential Calculus: Investigation of Fund's

Thus, the straight line y = x+ I will be an inclined asymptote


ol the curve (see Fig. 47). Note that

Jim Y= Jim xe 11x=O.


X->-0 X-+-0
I
(f) The function is defined and continuous at e- -
3x
> 0' i.e. at

J\/
I
x < 0 and x > 3e .
Since the function is continuous at
every point of the domain of definition,
vertical asymptotes can exist only on
finite boundaries of the domain of defini-
/
/ tion.
/ As x - - 0 we have
., /
/

1 _ //
_ _.,....,,-t-_..___ _ _ _ :J:
Jim y= X->Jim
X-> - 0 - 0
~In (e- 3~) =
1
=-_.!..Jim ln<e+z)=O
2 z~ + 00 z
(z=-_!_)
3x

(see Problem 3.2.2.), i. e. the straight


Fig 47
line x = 0 is not a vertical asymptote.
As x ---+ _.!.. + 0 we have
3e
.
I1rn 3
y=- .
11m x In ( e--I ) = -oo,
x~lf(3e)+0 2 x.... J/(3e)+0 3x

i. e. the line x = l/(3e) is a vertical asymptote.


Now let us find the inclined asymptotes:

.
k= ltrn
.r-+oo
x-=
y 3 J'1rn In ( e- Ix ) =2;
2 x-,.oo 3
3

!b= Jim [y-kx]


X-+00
=% Jim x [in
X-+00
(e- 3~)-11 =
-~I'
- 2 ltn
1n(1-:de)_
I
2 ( _ 3_!_) -__ 2e
- 2
_!_
x~oo _ e
x
Hence, the straight I .me y = 32x - 2Ie 1s . an .me l'me d asymp t ote
(see Fig. 48).
(g) The curve has no vertical asymptotes, since the function is
continuous everywhere. Let us look for inclined asymptotes. The
limits will be different as x __, + oo and x ____,. - oo, therefore we have
to consider two cases separately.
.'J.10. Asymptotes 173

Find the right asymptote (as x __..,. + oo ):


.r-- -./ --1-+-I +2
.
k 1 = ' 1m r l+x2 +2x .
= 11m V xi 3
= ;
X-+ + oo X x- + oo 1
b1 = lim (Vt+ x + 2x-3x) = 2
x....,.. +oo

Thus, as x - + oo the curve has an asymptote y = 3x.

Fig. 48 Fig. 49

Find the left asymptote (as x - - oo ):

k2 = hm
. Vl+ x2 + 2x
= hm
. IxI V ;2 + I+ 2x
~..:I,
X-+ - 00 X X-+ - 00 X

b1 = Jim IVt+x2 +2x-xl= lim ~ =0,


x-+-> .<-+-> I +x -x
since both summands (Vt+x2 and (-x)) in the denominator are
positive at x < 0.
And so, the curve has an asymptote y = x as x - - oo.
(h) The curve has no vertical asymptotes, since it is continuous
at x=FO, and in the neighbourhood of the point x=O the function
is bounded.
Let us find the inclined asymptotes. We have

y lxl y1+ 1sin_!_ 2


k= Jim-;= lim x x = 1-0=0.
X-+<D -<-+<D X

Then

b= lim (y-kx)= lim lxl


l-+<D .1:-+<D
V --1 1 f 1 as x-+oo,
1+2sin-=1-1 as X-+-oo.
x x \
174 Ch. 111. Differential Calculus: Investigation of Funct's

Thus, the curve has two horizontal asymptotes: y = +


1 and y = - 1
(see Fig. 49). The same result can be obtained proceeding from
symmetry about the origin and keeping in mind that the function
y is odd.
3.10.2. Find the inclined asymptote of the graph of the function
y= l~x as X-+- oo and show that in the interval (100, oo) this
function may be replaced by the linear function y = x- l with an
error not exceeding 0.01.
Solution. Find the inclined asymptote:
x2
k = !~n; x (I + x) = 1;

b= !~n; (I:x - x) = - 1.

And so, the equation of the asymptote is y = x- l.


Form the difference:
x2 I
~= I+x-(x-l)= l+x
Hence, assuming
x2
Y=i+x:::::::x-l,

for all x > 100, we introduce an error of not more than 0.01.
3.10.3. Find the asymptotes of the following curves:
x2 -6x+3
(a) y = x- 3 ; (b) y = x arc tan x;
(c) y=x+(sinx)/x; (d) y=ln(4-x2);
I
(e) y=2x-arccos-.
x

3.11. Oeneral Plan for Investigating Functions and


Sketching Oraphs
The analysis and graphing of functions by elementary methods
were considered in Chapter I ( 1.3 and 1.5). Using the methods
of differential calculus, we can now carry out a more profound and
comprehensive study of various properties of a function, and explain
the shape of its graph (rise, fall, convexity, concavity, etc.).
It is convenient to investigate a function and construct its graph
according to the following plan:
1. Find the domain of definition of the function.
2. Find out whether the function is even, odd or periodic.
8.11. Investigating Functions and Sketching Graphs 175

3. Test the function for continuity, find out the discontinuities


and their character.
4. Find the asymptotes of the graph of the function.
5. Find the points of extremum of the function and compute the
values of the function at these points.
6. Find the points of inflection on the graph of the function,
compute the values of the function and of its derivative at these
points. Find the intervals of convexity of the graph of the function.
7. Graph the function using the results of this investigation. If
it is necessary to specify certain regions of the curve, calculate the
coordinates of several additional points (in particular, the x- and
y-intercepts).
This is a very tentative plan, and various alternatives are pos-
sible. For instance, we recommend the student to begin sketching
the graph as soon as he finds the asymptotes (if any), but in any
case before the points of inflection are found. It should be remem-
bered that in sketching the graph of a function the principal refe-
rence points are the points of the curve corresponding to the extremal
values of the function, points of inflection, asymptotes.
3.11.1. Investigate and graph the following functions:
(a) y=x6 -3x4 +3x 2 -5; (b) Y=Vx-Vx+l;
2x3 l-x3
(c) y = x2_4; (d) y=X2;

(e) y=x+ln(x2 -l); (f) y =+sin 2x+ cosx;


. l-x 2
(h) y =arc sm 1+x2

Solution. (a) The function is defined and continuous throughout


the number scale, therefore the curve has no vertical asymptote.
The function is even, since f(-x)=f(x). Consequently, its graph
is symmetrical about the y-axis, and therefore it is sufficient to
investigate the function only on the interval [O, oo ).
There are no inclined asymptotes, since as X-+ oo the quantity
y turns out to be an infinitely large quantity of the sixth order
with respect to x.
Investigate the first derivative:
y' = 6x 5 - l2x3 + 6x = 6x (x 4 -2x2 + 1) =6x (x 2 - l) 2 ;
the critical points are:
X1 =-1, X2 =0, Xa=l.

Since in the interval [O, oo) the derivative y';:;;;:: 0, the function
increases.
176 Ch. 111. Differential Calrnlus: lnuestigation of Funct's

Investigate the second derivative:


y" = 30x 4 -36x~ + 6= 6 (5x -6x + 1). 4 2

The positive roots of the second derivative:


X1 = l/J/5, X2 = 1.
For convenience and pictorialness let us compile the following
table, where all the points of interest are arranged in an ascending
order:

)( 0 ( o. ;5-) vs
I
( ~; 1) I (I. 00)
!I
2
i
II
93 II
t/ 0 + 25 vs-~ 1. 7 + 0 +
I i
I
y" (j
+ 0 -- 0 -t
I I I I ! I I,'I

I i
y ~ -~ '"?"
-4 .L. 23

On the right one more additi.onal value of the function is com-


puted to improve the graph after the point of inflection.
Using the results of the investigation and the above table and
taking into consideratio_n the symmetry principle, we construct the
graph of the function (see Fig. 50). As is seen from the graph, the
function has roots X= a, where a:::::::; 1.6.
(b) The function is defined and continuous over the entire number
scale and is negative everywhere, since < 1. Vx V x+
The graph has neither vertical, nor inclined asymptotes, since the
order of magnitude of y is less than unity as x-... oo. Determine
the horizontal asymptote:
limy= Jim
x-.oo X-+oo
(Vx-Vx+l)=
. -I
= x-oo v x + v x (x+ I)+ v/(x-t-
hm 3 2 a,. a
J 1!
= 0.
Hence, the straight line y = 0 is the horiZ0ntal asymptote of the
graph.
3.11. Investigating Functions and Sketching Graphs 177

The first derivative


, I I V(x+n2-VX2
Y =3Vx -3t/(x+1)2 2 avx2tx+1)2
becomes zero at the point x2 = - ~ and infinity at the points
X1 = - 1, Xb = 0.
!f
!/
1

-f -12 0 f

-5
Fig. 50 Fig. 51
The second derivative
" I ( 2.) I I ( 2) I 2[ViX+TJii-VX&J
Y =3 -3 Vx-3 -3 V<x+l) 0 = 9Vlx<x+l)Jo
does not vanish and is infinite at the same points x1 = - 1, x 3 = 0.
Compile a table:

x -I (-1. -~) -2
I 1
I
\ -2 )
0. 0 (0, 00)

I
I

y'
I- I 00
-
I
0
I + I 00
I + 11

y" +
16 + -
l
00
+gv2 00

y
-1 3
~-~L
J -0.26

With the aid of this table, and of the asymptote y = 0 construct


the graph of the function (see Fig. 51).
178 Ch. 111. Differential Calculus: Investigation of Funct's

(c) The function is defined and continuous over the entire axis
except at the points x = + 2. The function is odd, its graph is
symmetrical about the origin, therefore it is sufficient to investigate
the function on the interval [O, oo).
The straight line X= 2 is a vertical asymptote:
2 3 . 2x3
Jim ~= - oo; hm - 2- 4-= -j-oo,
x - 2 - 0 x--4 x-2+0 X -
Determine the inclined asymptote:
y 2x 2
k= lim - = lim - - = 2
x--.+oo x x- +oo x2-4 ,

b= lim (y-2x) = lim ~=0.


X-++oo x ....... +oo X-

The curve has an inclined asymptote y = 2x, and


y-2X= ~ { >0 at X > 2,
x 2 -4 <0 at x < 2
The first derivative
, 6x2(x2-4)-4x4 2x 2 (x 2 -12)
Y = (x2_4)2 = (x2-4)?.
in the interval [O, oo) vanishes at the points
x = o, x = 2 V3 ~ 3.46
and becomes infinite at the point x = 2.
The second derivative
,, +
16x (x 2 12)
y = (x2-4)3

. becomes zero at the point x = 0 and infinite at x = 2.


Compile a table:

x 0 (0' 2) 2 (2, 2y3) 2y3 (2V3. ooJ

y' -0 - 00 - 0 +
I
I

3 V3
y" +o - 00 + --2- +
I

1j ~ ll ~
\! I
8.11. Investigating Functions and Sketching Graphs 179

Using the results of the investigation, sketch the graph of the


function (see Fig. 52).
(e) The function is defined and continuous at all values of x for
which x2 - l >0 or lxl> 1, i.e. on two intervals: (-oo, -1)

!J1L/
and (1, + oo).
I
I I /
I I /
I
I
I
I /
// !f 1
I I / I
I I // I
I I / I
I I / I
I I// I
I
I
I
4 -r
//I
I
I
I
~~~~:-t--=""""......+-~~~----x
-21 / O jZ -1-Vi I

/1/ i
Y-4
/I
I
I
I
// I I -1 0
i /\ -1
I I
I I

'I
I I
/. I I

Fig. 52 Fig. 53

We seek the vertical asymptotes:


lim y= lim [x+ln(x2 -l)]=-oo;
x- - I - 0 x- - I - 0
limy= Jim [x+ln(x2 -l)]=-oo.
x-l+O x-1+0

Thus, the curve has two vertical asymptotes:


x = - I and x = + 1.
Find inclined asymptotes:
k= . -=
IIm y .
1Im x+ln (x 2 -I)
= .
1Im rl +ln(x2 -1)]
= l,
X-+OOX x-(,) x x- x
b= Jim [y-x]= lim ln(x2 -l)= +oo.
x-oo x-oo
Hence, the curve has neither inclined, nor horizontal asymptotes.
Since the derivative
, l
y =
2x
x2-1
+
exists and is finite at all points of the domain of definition of the
function, only the zeros of the derivative
X1=- l-V2; X2=-l+J/2
can be critical points. At the point x2 = - 1 + V2 the function is
180 Ch. 111. Oiflerential Calculus: Investigation of Funct's

not defined; hence, there is one critical point Xi= - 1- J/2 belon-
ging to the interval (-oo, -1). In the interval (l, oo) both the
derivative y' > 0 and the function increase.
The second derivative
2 (x2 +I)
y = - (x2-1)2 <O,
hence, the curve is convex everywhere, and at the point Xi =
= - l-V2~-2.41 the function has a maximum

y(-t-V2)~-1-V2+111(2+2V2) ~-0.84.
To plot the graph in the interval (l,oo), where thereare no characte-
ristic points, we choose the following additional points:
x= 2; y= 2+ ln3 ~ 3.10 and x= 1.2; y= l.2+ ln0.44 ~ 0.38.
The graph of the function is shown in Fig. 53.
(f) The function is defined and continuous throughout the num-
ber scale and has a period 2n. Therefore in investigating we may
.confine ourselves to the interval [O, 2n]. The graph of the function
has no asymptote by virtue of continuity and periodicity.
Find the first derivative:
y' =cos 2x-sin x.
On the interval [O, 2n] it has three roots:
n 5n 3n
X1=5, X2=5, Xs=2
Evaluate the second derivative:
y" = - 2 sin 2x-cos x.
On the interval [O, 2n] it has four roots:

x1 =;, x2 =n+arcsin(l/4),X3 = 3; , x.=2n-arcsin(l/4).

Let us draw up a table of the results of investigation of all cri-


tical points of the first and second derivatives (the table also in-
cludes the end-points of the interval [O, 2n]).
Since in the interval ( 0, 3; ) the roots of the first and second
derivatives alternate, the signs of the second derivative in the in-
tervals between its critical points are indicated only for the last
three intervals.
The results of the investigation enable us to construct the graph
of the function (see Fig. 54).
(g) The function is defined, positive and continuous on each of
the intervals (-oo, 0) and (0, oo). The point x=O is a disconti-
3.11. Investigating Functlons and Slletching Graphs 181

I I

x
0
:rt
-
6
I~
5:rt
6 X2 (x2, 3; )
3:rt
2 (3; ,x4) X4 (X4,2:rt) 2:rt

l1

I
1/ I
0 -2 0 I_! 0 I1 _! 1
!
8
I
! I b- I

II-'~,
ii

r/' 3 V3
! 0 --
2
0 - 0 + 0 -

-~ ~r
JVJ
y I
-4-

7 \
0

\ -31/f
~ 4:rd" 16 ro
0
16
I

nuity. Since (see Problem 3.2.2.)


Jim y= Jim x2e 11x= Jim e~ =oo ( t= ~),
x-+o x-+o t-+oot

the straight line x =0 is a vertical asymptote. But


Jim y = Jim x 2 e 11 x = 0.
K-+-0 x--..-0

There are no inclined asymptotes, ~ince the function y = x2e 11x


has the second order of smallness with respect to x as x-.+oo.

0 }_TC
6
-f
Fig. 54
Let us find the extrema of the function, for which purpose we
evaluate the derivative:
y' = 2xe 11x_et/x = 2e 1/x (x- 1/2),
whence we find the only critical point x = ~ .
Since for x =F 0
y" (x) = 2el/x_f el/x + ~ et/x= ~ 2 e 11x (2x 2 -2x+ 1) > 0,
182 Ch. Ill. Differential Calculus: Investigation of Funct"s

on each of the intervals of the domain of definition the graph of


the function is concave, and at the point x = 1 / 2 the function has
a minimum

From the information obtained we can sketch the graph as in Fig. 55.
To specify the graph in the intervals (-oo, 0) and (1/2 , oo) the
!J following additional points are used:

V1
X= - 1, y=e- 1 ~ 0.37; X= 1,
y=e ~ 2.72.
(h) The function is defined and continuous
throughout the number scale, since at any x
1 II 1-x21
1l+x2 ~ 1.
Since the function is even, we may confine
Fig. 55 ourselves to the investigation of the function
at x~O.
As the function is continuous, the graph has no vertical asymp-
totes, but it has a horizontal asymptote:
Jim y =arc sin (-1) = - ~.
:C-++CD

The first derivative


, I -2x (I +x 2)-2x (I -x2 ) I 4x
Y = , I (l-x2)2 X (I +x2)2 = -2TXT X (I +.t2)2
V l-(l+x2)2
is negative for x > 0, therefore the function decreases.
The derivative is non-existent
at the point x = 0. By virtue of !f
the symmetry of the graph about
the y-axis there will be a maxi-
mum at the point y (0) = ~. No-
tice that at the point x = 0 the
right derivative is equal to -1,
and the left one to + 1. _1'
The second derivative is posi- 2
tive: Fig. 56
,, 2 (I + x2) 2x Bx
y (x) =2 (I +x2)4 =n+x2)3 >0 for all x > 0.
Hence, in the interval (0, oo) the graph of the function is concave.
Also note that the curve intersects with the x-axis at the points
X= 1.
3.12. Algebr. and Transcendent. Equations 183

Taking into consideration the results of the investigation, con-


struct the graph of the function (see Fig. 56).
3.11.2. Investigate and graph the following functions:
x4 x4
(a) y=l+x 2 - 2 ; (b) y=(l+x):i;
I x3
(c) y=-+4x
x
2 ; (d) y=-:i- ;
x - 1
(e) y = V
x2 - V
x2 - 4;
(f) y=x ln(x+2); (g) y=x3e-4x;
2

(h) y = ( x arctan
l0
+ at x =F 0,
at X= 0.

3.12. Approximate Solution of Algebraic


and Transcendental Equations
Approximate determination of isolated real roots of the equation
f (x) = 0 is usually carried out in two stages:
l. Separating roots, i.e. determining the intervals [a, ~] which
contain one and only one root of the equation.
2. Specifying the roots, i.e. computing them with the required
degree of accuracy.
The process of separation of roots begins with determining the
signs of the function f (x) at a number of points x = a 1 , a 2 ,
whose choice takes into account the peculiarities of the function f (x).
If it turns out that f (ak) f (ak+r) < 0, then, by virtue of the
property of a continuous function, there is a root of the equation
f (x) = 0 in the interval (ak, ak+ 1).
Real roots of an equation can also be determined graphically as
x-intercepts of the graph of the function y = f (x). If the equation
has no roots close to each other, then its roots are easi 1y separated
by this method. In practice, it is often advantageous to replace
a given equation by an equivalent one
'l-'r (x) = 'ljl2 (x),
where the functions 'ljl1 (x) and 'ljl 2 (x) are simpler than the function
f (x). Sketch the graph of the functions y = 'ljl1 (x) and y = 'ljl 2 (x)
and find the desired roots as the abscissas of the points of inter-
section of these graphs.
The Methods of Approximating a Root. I. Method of Chords. If
the interval [a, b] contains the only real root s
of the equation
f (x) = 0 and f (x) is continuous on the interval, then the first ap-
proximation x 1 is found by the formula
f (a)
Xi= a- t (b)--f(a) (b-a).
184 Ch. Ill. Differential Calculus: Investigation of Funct's

To obtain the second approximation x 2 a similar formula is ap-


plied to that of the intervals [a, x1 ] or [x1 , b], at the end-points
of which the function f (x) attains values having opposite signs. The
process is continued until the required accuracy is obtained, which
is judged of by the length of the last obtained segment.
2. Method of Tangents (Newton's method). If f (a) f (b) < 0, and
f' (x) and f" (x) are non-zero and retain definite signs for a~ x ~ b,
then, proceeding from the initial approximation x0 (x0 E [a, b]) for
which f (x0 ) f" (x0 ) > 0, we obtain all successive approximations of
the root by the formulas:
f (Xo) f (x1) f (Xn-1)
Xi=Xo-f'(xo)' X2=X1-f'(x1)' ' Xn=Xn-I-f'(Xn-1)"

To estimate the absolute error in the nth approximation we can


apply the general formula
t_x J~I f (xn)I
"' n ...._,, m1 '
'
where
m1 = min If' (x) j.
a<;;;x<O;b

Under the above conditions the method of chords and the method
of tangents approximate the sought-for root from different sides.
Therefore, it is usual practice to take advantage of their combination,
i.e. to apply both methods simultaneously. In this case one can obtain
the most precise approximation of a root more rapidly and the cal-
culations can be checked. Generally speaking, the calculation of the
approximations x 1 , x 2 , , xn should be continued until the decimal
digits to be retained in the answer cease to change (in accordance
with the predetermined degree of accuracy!). For intermediate trans-
formations we have to take one or two spare digits.
3. Iteration Method. The equation f (x) = 0 is first reduced to the
form x = cp (x) where I cp' (x) I~ q < 1 (q = const) for a~ x ~ b. Star-
ting from any initial value x0 E [a, b], successive approximations of
the root are computed by the formulas x 1 = cp (x0 ), x2 =cp (x1 ), , xn=
=qi (x,._ 1 ). The absolute error in the nth approximation can be
estimated by the following formulas:

1-x,.J~ I q qlxn-1-XnJ,

if the approximations xn-i and xn lie on the same side of the root,
and
I-Xn J :;:; I _;.q J Xn-1-Xn J,

if the approximations xn-i and xn lie on different sides of the root.


3.12. Algebr. and Transcendent. Equations 185

3. 12. l. Locate the roots of the equation


f (x) _ x:-6x+ 2 = 0.
Solution. Compile a table of signs of f (x) at some chosen points

x f (x) x f (x)
-- I I I
- 00 - I -
-3 -- 3 +
-I + + 00 +
0 +
From this table we draw the conclusion that the equation has
three real roots lying in the intervals (-3, --!), (0, 1) and (I. 3).
3.12.2. Determine the number of real roots of the equation
f (x) == x+ex = 0.
Solution. Since f' (x) = 1 +ex> O; f (-oo)=-oo; f (+ oo)=+oo,
the given equation has only one real root.
3.12.3. An approximate value of the root of the equation f (x) _
_ x4 -x-1=0 is x=l.22. Estimate the absolute error in this
root.
Solution. We have f(x)=2.2!53-l.22-l=-0.0047. Since at
X= 1.23
f (x) =2.2888-1.23-1=0.0588,
the root lies in the interval (1.22, 1.23). The derivative f' (x) =
= 4x 3 - I increases monotonically, therefore its least value in the
given interval is
m1 =4x1.22 3 - l =4x1.816-1 =6.264,
wherefrom we get an estimate of the error

Ix- I~ It ~~l 1 = 05_026~ ~ 0.00075 < 0.001.


3.12.4. Solve graphically the equation
x logx-1=0.
Solution. Let us rewrite the equation in the form
I
logx= -x .
186 Ch. 111. Differenfial Calculus: Investigation of Fund's

Here ' 1 (x) =log x, ' 2 (x) = -xI . There are tables for the values of
these functions, and this simplifies the construction of their graphs.
Constructing the graphs y =log x and y = J.. (see Fig. 57), we find
x
!f the approximate value of the only
root ~ 2.5.
3.12.5. Find the real root of the
equation

1
f (x) = x 3 -2x2 +3x-5 = 0
with an accuracy up to 10- 4 :
(a) by applying the method of
chords,
(b) by applying the method of
tangents.
Fig. 57 Solution. Let us first make sure
that the given equation has only
one real root. This follows from the fact that the derivative
f' (x) =3x2 -4x+3 > 0.
Then, from f (1) = - 3 < 0, f (2) = 1 > 0 it follows that the given
polynomial has a single positive root, which lies in the interval (l, 2).
(a) Using the method of chords, we obtain the first approximation:
-3
X1 = l - 4 l = l.75.
Since
f (I. 75) = - 0.5156 < 0,
and f (2) = l > 0, then I. 75 < < 2.
The second approximation:
0.5156
X2 = l.75+ 1. 5156 0.25= l.75+0.0850= l.8350.

Since f (l .835) = - 0.05059 < 0, then 1.835 < < 2.


The sequence of the approximations converges very slowly. Let
us try to narrow down the interval, taking into account that the
value of the function f (x) at the point x 2 = 1.835 is considerably
less in absolute value than f (2). We have
f (1.9) = 0.339 > 0.
Hence, 1.835 < < 1.9.
3.12. Algebr. and Transcendent. Equations 187

Applying the method of chords to the interval (l.835, 1.9), we


will get a new approximation:
-0.05059
X3 = l.835- 0 . 339 +o.05059 0.065= 1.8434.

Further calculations by the method of chords yield


X4 =l.8437, X0 =1.8438,
and since f (1.8437) < 0, and f (1.8438) > 0, then 6 ~ 1.8438 with
the required accuracy of 10- 4
(b) For the method of tangents we choose x 0 = 2 as the initial
approximation, since f (2) = 1 > 0 and f" (x) = 6x-4 > 0 in the in-
terval (1, 2). The first derivative f' (x)=3x2 -4x+3 also retains its
sign in the interval ( 1, 2), therefore the method of tangents is ap-
plicable.
The first approximation:
X1 = 2-1/7 = 1.857.
The second approximation:
f (I.857) 0.0779
X2 = 1.857- f' (1. 857 ) = 1.857 - 5 . 9275 = 1.8439.

The third approximation:


f (1.8439)
X3 = 1.8439 - f' (l. 8439 ) = 1.8438,

already gives the required accuracy. Here the sequence of the ap-
proximations converges much more rapidly than in the method of
chords, and in the third approximation we could obtain an accuracy
up to 10- 6
3.12.6. Find the least positive root of the equation tan x = x with
an accuracy up to 0.0001 applying Newton's method.
3.12.7. Find the real root of the equation 2-x-logx=O by
combining the method of chords with the method of tangents.
Solution. Rewrite the left member of the equation in the folio-
wing way:
f (x) = (2-x) + (-log x),
whence it is seen that the function f (x) is a sum of two monoto-
nically decreasing functions, and therefore it decreases itself. Con-
sequently, the given equation has a single root 6.
Direct verification shows that this root lies in the interval (1, 2).
This interval can be narrowed still further:
1.6 < s < 1.8,
188 Ch. 111. Differential Calculus: Investigation of Funct's

since
f(l.6)=0.1959 > 0; f(l.8)=-0.0553 < o.
Then
f' (x) = - 1 - ...!..
x
loge; f" (x) = ~
x
loge
and
f' (x) < O; f" (x)> 0 over the whole interval [ 1.6; I 8].
Applying to this interval both the method of chords and the
method of tangents with the initial point x0 = 1.6 we obtain the
first approximations:
X1 = l.6-(lf~l~8 ;~),'(l('.6~) = l.6--j-0. I559= 1.7559;
x; = 1.6 - /, ~\ .~) = 1.6 + 0.1540 = l. 7540.
Applying the same methods to the interval [1.7540, l.7559], we
get the second approximations:
= I 7559 (I. 7540- I. 7559) f (I. 7559) = I 75558
X2 ' f (I. 7540) - / (I. 7559) . '
' f(l.7540)
X2 = 1. 7540- I' (I. 7540 ) = 1. 75557.

Since x2 -x; = 0.00001, the root is computed with an accuracy


up to 0.00001.
3.12.8. Using the combined method find all roots of the equation
f (x) == x3 -5x+ 1=0 accurate to three decimal places.
3.12.9. Applying the iteration method find the real roots of the
equation x-sin X= 0.25 accurate to three decimal places.
Solution. Represent the given equation in the form x-0.25 =sin x.
Using the graphical method, we find that the equation has one
real root , which is approximately equal

J
to x0 = 1.2 (see Fig. 58).
Since
sin l.1=0.89I2> l.l-0.25,
sin 1.3 = 0.9636 < 1.3-0.25,
--=-o _ ___.__._____ a: the root lies in the interval (I.I, I 3).
Let us rewrite the equation in the
-D.25 f f.2 form
Fig. 58 x = cp (x) =sin x+ 0.25.
Since the derivative cp' (x) =cos x in the interval (I. I, l.3) does.
not exceed cos 1.1 < 0.46 < 1 in absolute value, the itE>ration method
is applicable. Let us write successive approximations
Xn=sinxn-i+0.25 (n= I, 2, ... ),
3.12. Algebr. and Transcendent. Equations 189

taking X0 = l.2 for the initial approximation:

X1 =Sin l.2 +0.25=0.932 +0.25= J.182;


x 2 =sin 1.182 + 0.25 = 0. 925 + 0.25 = 1.175;
X 3 =Sin J.175 +0.25=0.923 +0.25= 1.173;
X4 =sin J.173 +0.25=0.9219+0.25= J.1719;
x 0 =sin l.1719 + 0.25 = 0.9215 + 0.25 = 1.1715;
x6 =sin l.1715+ 0.25 = 0.9211+0.25 = 1.1711.

q <I, we have ~=1.171 within the


Since q=0.46 and hence, -1-q
required accuracy.
3.12. lO. Applying the iteration method, find the greatest positive
root of the equation

accurate to four decimal places.


Solution. Rough estimation gives us the approximate value of the
root x0 = 10.
We can rewrite the given equation in the lorm

X= 1000-x3 ,
or in the form
1000 I
X=---
x2 x '
or in the form
X= V 1000-x and so on.

The most advantageous of the indicated methods is the preceding


one, since taking (9, !OJ for the main interval and putting

<p (x) = V 1000-x,


we find that the derivative
-I
cp' (x) = -,a7====
3 V (l000-x) 2

does not exceed 1/300 in absolute value:

I <r' (x) I~ V 1 ~ 3~0 = q.


3 990 2
190 Ch. Ill. Differential Calculus: Investigation of Funct's

Compute successive approximations of xn with one spare digit by


the formula
Xn+i= av
1000-Xn (n=O, 1, 2, ... ),
X 0 = 10,

X1 = v
1000-10=9.96655,
X 2 =v1000- 9.96655 = 9.96666,

X3 = v
1000-9.96666 = 9.96667.
We may put 6= 9.9667 with an accuracy of 10- 4 ,
Note. Here, the relatively rapid convergence of the process of ite-
ration is due to the smallness of the quantity q. In general, the
smaller the q, the faster the process of iteration converges.
3.12.11. Applying the method of chords, find the positive root of
the equation
+ +
f (x) == X3 1. lx2 0.9x- l.4 = 0
with an accuracy of 0.0005.
3.12.12. Using the method of chords, find approximate values of
the real roots of the following equations with an accuracy up to 0.01:
(a) (x-l) 2 -2sinx=0; (b) ex-2(1-x) 2 =0.
3.12.13. Applying Newton's method, find with an accuracy up
to 0.0 l the positive roots of the following equations:
(a) x 3 +50x-60=0; (b) x 3 +x-32=0.
3.12.14. Using the combined method find the values of the root
of the equation
x3 - x - l =0
on the interval [ 1, 2] with an accuracy up to 0.005.
3.12.15. Applying the iteration method, find all roots of the equa-
tion 4x-5 lnx=5 accurate to four decimal places.

3.13. Additional Problems


3.13. l. Does the function
if x < 1
f (x) = { ~/x if x~ 1
satisfy the conditions of the Lagrange theorem on the interval [O, 2]?
3.13.2. Prove that for the function y=ax2 +~x+y the number 6
in the Lagrange formula, used on an arbitrary interval [a, b], is the
arithmetic mean of the numbers a and b: 6=(a+ b)/2.
3.13. Additional Problems 191

3.13.3. Prove that ii the equation


a0xn + a1x"- 1+ ... + an_ 1X = 0
has a positive root x 0 , then the equation
naoxn-l + (n-1) alxn- 2+ ... + an-1 = 0
has a positive root less than x 0
3.13.4. Prove that the equation x 4 -4x-1=0 has two different
real roots.
3.13.5. Prove that the function f(x)=xn+px+q cannot have
more than two real roots for n even and more than three for n odd.
3.13.6. Prove that all roots of the derivative of the given poly-
nomial f(x)=(x+l)(x-l)(x-2)(x-3) are real.
3.13. 7. Find a mistake in the followin_5 reasoning.
The function
I x2 sin ( l/x) for x =I= 0,
f (x) = \ 0 for x = 0
is differentiable for any x. By Lagrange's theorem

x2 sin+= x ( 2 sin ~ -cos i-) ,


whence
I I . I
cos-= 2c sm--x sm-
6
i:

6 x (0 < < x).


As x tends to zero ~ will also tend to zero. Passing to the limit,
we obtain lim cos(!/)= 0, whereas it is known that Jim cos ( l/x) is
~ ... 0 X-+ 0
non-existent.
3.13.8. Find a mistake in the following deduction of Cauchy's
formula. Let the functions f (x) and cp (x) satisfy all the conditions
of the Cauchy theorem on the interval [a, b]. Then each of them
will satisfy the conditions of Lagrange's theorem as well. Consequ-
ently, for each function we can write the Lagrange formula:
f (b)-f (a)= f' () (b-a), a<< b,
cp(b)-cp (a)= cp' () (b-a), a<< b.
Dividing the first expression by the second, we obtain:
f(b)-f(a) t'<s)(b-a) f'(s)
cp(b)-q;(a) = cp'(s)(b-a) = cp'(s)

3.13.9. Prove the following inequalities:


l a a-b 'f O b
a-b
(a) -a- < nb < -b- 1 < < a,
192 Ch. I II. Di!]erential Calculus: lnuestigalion of Funct's

(b)pyP- 1 (x-y)<;_xP-yP<;_pxP- 1 (x-y) if O<y<x and p>l.


3.13.10. Prove that all roots of the Chebyshev-Laguerre polynomial

are positive.
3.13.ll. Prove that if the function f(x) satisfies the following
conditions:
(I) it is defined and has a continuous derivative of the (n- l)th
order rn-1! (x) on the interval [xo, xn];
(2) it has a derivative of the nth order f(n) (x) in the interval
(xo, xn);
(3) f (x 0 ) = f (x1 ) = ... = f (x,,) (x 0 < x 1 < ... < x,,),
then inside the interval [x0 , x,,] there is at least one point ' such
that f(">()=0.
3.13.12. The limit of the ratio of the functions
. e- 2 x(cosx+2sinx) . -xl+2tanx
(Im . = 1tm e
x- 00 e-x(cosx+stnx) x- 00 I+tanx
. t en t , smce
.is non-ex ts . th e expression
. I +2 tan x . d" t" t
1 +tan x 1s 1scon muous a
the points x,,=nn+n/2 (n=O, 1, ... ), but at the same time the
limit of the ratio of the derivatives does exist:
. (e- 2 -<(cosx+2sinx)]' . -5e- 2 xsinx 5 1. -x O
I1m . = 1tm . = - tm e = .
x-oo (e-X(cosx+stnx)]' x- 00 -2e-xstnx 2x-"'

Explain this seeming contradiction.


3.13.13. Prove that the number 8 in the remainder of the Taylor
formula of the first order
f (a+h) = f (a) +hf' (a)+~; f" (a+ Sh)
tends to 1/3 as h-.. 0 if f"' (x) is continuous at x-= a and f"' (a) =r'=O.
3.13.14. Prove that the number e is an irrational number.
3.13.15. Prove that for 0 < x <;_ n/2 the function f (x) '--=(sin x)/x
decreases. From this obtain the inequality 2x/n <sin x < x for
0 < x < n/2 and give its geometric meaning.
3.13.16. Show that the function f(x)=x+cosx-a increases;
whence deduce that the equation x +cos x =a has no positive roots
for a < 1 and has one positive root for a> 1.
3.13.17. Show that the equation xex=2 has only one positive
root found in the interval (0, I).
.~ 3.13. Additional Problems 193

3.13.18. Prove that the function

f (x) = ( I
<2 x
+x 2
sin xI for x == 0,
l 0 for X= 0
is not monotonic in any interval containing the origin. Sketch the
graph f (x).
3.13.19. Prove the theorem if: (I) f (x) and qi (x) are continuous
in the interval [a, b] and differentiable inside it; (2) f (a)= qi (a);
and (3) f' (x) >qi' (x) (a< x < b), then f (x) > qi(x) (a< x < b).

3.13.20. Show that the function f (x) = ~;t~ has neither maxima,
nor minima at ad-be== 0.
+ +
3.13.21. In the trinomial x 2 px q choose the coefficients p and q
so that the trinomial has a minimum at x = 3 and that the mini-
mum equals 5.
r
3.13.22. Test the function f (x) = (x-x 0 qi (x) for extremum at
the point x:--= x 0 , where n is a natural number; the function qi (x) is
continuous at x = x 0 and qi (x 0 ) == 0.
3.13.23. Given a continuous function
at x=FO,
at x=O.
Show that f (x) has a minimum at the point x = 0, but is not
monotonic either on the left or on the right of x = 0.
3.13.24. Find the greatest and the least values of the following
functions on the indicated intervals:
(a) Y=\x\ for -l~x~ I,
(b) y = E (x) for - 2 ~ x ~ 1.
3.13.25. Do the following functions have the greatest and the
least values on the indicated intervals?
(a) f(x)=cosx for -n/2~x< n,
(b) f (x) = arcsinx for -1 < x < l.
3.13.26. Prove that between two maxima (minima) of a continuous
function there is a minimum (maximum) of this function.
3.13.27. Prove that the function
x2 sin2 (l/x) for x == 0,
{
f (x) = 0 for x = 0
194 Ch. fl I. Differential Calculus: Investigation of Funct's

has a minimum at the point x 0 = 0 (not a strict minimum).


3.13.28. Prove that if at the point of a minimum there exists a
right-side derivative, then it is non-negative, and if there exists a
left-side derivative, then it is non-positive.
3.13.29. Show that the function
y= { l/x 2 (x > 0),
3x2 (x~ 0)

has a minimum at the point x= 0, though its first derivative does


not change sign when passing through this point.
3.13.30. Let x0 be the abscissa of the point of i flection on the
curve y = f (x). Will the point x 0 be a point of extremum for the
function y=f'(x)?
3.13.31. Sketch the graph of the function y = f (x) in the neigh-
bourhood of the point x = -1 if
f (-1) = 2, f' (-1) = -1, f" (-1) = 0, f'" (x) > 0.
3.13.32. For what choice of the parameter h does the "curve of
probabilities"
y= ;it e-hx (h > 0)
have points of inflection x = cr?
3.13.33. Show that any twice continuously differentiable function
has at least one abscissa of the point of inflection on the graph of
the function between two points of extremum.
+ +
3.13.34. Taking the function y = x' 8x3 18x2 8 as an example, +
ascertain that there may be no points of extremum between the
abscissas of the points of inflection on the graph of a function.
3.13.35. Prove that any polynomial with positive coefficients,
which is an even function, is concave everywhere and has only one
point of minimum.
3.13.36. Prove that any polynomial of an odd degree n ~ 3 has
at least one point of inflection.
3.13.37. Proceeding directly from the definition, ascertain that
+
the straight line y = 2x 1 is an asymptote of the curve y =
=
2x4 +x 3 +I
Chapter 4
INDEFINITE INTEGRALS.
B<\SIC METHODS OF INTEGRATION

4.1. Direct Integration and the Method of Expansion


Direct integration consists in using the following table of integrals:

(1) 5undu= un+1 +C n+I (n=fo-1);

(2) Sd: =In Iu I+ C;

(3) 5a" du= -1 ~-a a" + C; ~ e" du = e 11


+ C;
(4) ~ cosudu=sinu+C; ~ sinudu=-cosu+C;
(5) ~ coshudu=sinhu+C; ~ sinhudu=coshu+C;
(6) .;1 --=tanu+ C;
(' du
cos 2 u
s-.- -= -cot u+C;
du
sin 2 u
(7) S du
U" a"
I
a
u
--.------+.. =-arctan-+C=
a
I
--arccot
a
u
-+C
a 1 (a> O);

C
(8) S ,r
ra2-u2
du . u
a
u
. =arcstn-+C=-arccos-+ 1 (a>O);
a

(9)
Jl .r .du
r uz a2
=1n(u+Vu 2 +a 2 )+c;

(10) s odu 2=2_!_1n1u-+a/+c.


U"-a a u a

In all these formulas the variable u is either an independent


variable or a differentiable fund ion of some variable. If

~ f (u) du = F (u) + C,
then
Sf (ax+b) dx = ~F (ax+b) +c.
The method of expansion consists in expanding the integrand into
a linear combination of simpler functions and using the linearity
196 Ch. IV. Indefinite Integrals

property of the integral:

sl~I aJ; (x) dx ='~I a;~ f1 (x) dx c~I Ia; I> 0).
4.1.1. Find the integral/= sx2 +~:- 1 dx.
Solution.

I= sx2 +~xx-l dx= s (xt.+5x't-x-'t)dx=


= ~ xt.dx+5 ~ x'tdx- ~ x-'tdx=
2xt. 5. 2
= -5-+C1 +3 x1,+c 2 -2x''+C 3 =
=2Vx(~ + 53x-t)+c.
Note. There is no need to introduce an arbitrary constant after
calculating each integral (as is done in the above example). By com-
bining all arbitrary constants we get a single arbitrary constant,
denoted by letter C, which is added to the final answer.

4 .1.2. 1 --S 6x3+x 2-2x+ 1 d


2x-I x.

s
4.1.3. I= sin2 x~os2 x .
Solution. Transform the integrand in the following way:
sin2x+cos2x I I
sin 2 xcos 2 x= sin 2 xcos 2 x = cos 2 x + sin2 x
Hence,
l=S cosd~ x +S sin
.d~ x =tanx-cotx+C.
4.1.4. I= ~ tan 2 x dx.
Solution. Since tan 2 x = sec 2 x-1, then
I=\v tan 2 xdx=S4-Cdx=tanx-x+C.
cos x J
+
4.1.5. I= ~ (x 2 5) 3 dx.
Solution. Expanding the integrand by the binomial formula,
we find
I= S(x + 15x'+75x + 125)dx= 7
6 2
x1 15x& 75xa
+-5-+-3-+ 125x+C.
4,1.6. I=~ (3x+ 5) 17 dx.
4. I. Direct I ntewation and the Method of Expansion 197

Solution. Here it is not expedient to raise the binomial to the


+
17th power, since u = 3x 5 is a linear function.
Proceeding from the tabular integral

Su 17
18
ulB
du=-+C '
we get
I =_!_. (3x+5)1s -t-C
3 18

4.1.7.
I= SYx+~x-Vx
4.1.8. I=~ cos(nx+ l)dx.
Solution. Proceeding from the tabular integral (4)
~cos udu =sin u +c,
we obtain
I=_!_
:rt
sin (nx+ l) + C.

4.1.9. I=~ cos 4xcos 7xdx.


Solution. When calculating such integrals it is advisable to use
the trigonometric product formulas. Here
I
cos 4x cos 7x = 2 (cos 3x +cos l lx)
and therefore

I = +Scos 3x dx +
+ Scos l Ix dx = ~ sin 3x + 2~ sin l lx + C.
Note. When solving such problems it is expedient to use the
following trigonometric identities:

sin mx cos nx =+[sin (m-n) x+ sin (m+ n) x];

sin mx sin nx = +I
[cos (m-n) x-cos (m +n) x];

cosmx cosnx = 2 [cos (m-n) x+ cos (m+ n) x].

4.1.10. I= ~cos x cos 2x cos 5xdx.


Solution. We have
I
(cos x cos 2x) cos 5x = 2 (cos x +cos 3x) cos 5x =
= 4I [cos 4x+ cos 6x] + 41 (cos2x+ cos8x).
198 Ch. IV. Indefinite Integrals

Thus,

J= i [S cos2xdx+ Scos 4xdx+ Scos 6xdx+ Scos8xdx] =


=~sin 2x+ 116 sin 4x+ 2~ sin 6x+ 3~ sin Bx+C.
4.1.1 l. I=~ sin 2 3xdx.
.
Sol ution. S mce . 2 3x = I-cos 6x , th en
sm 2

l=t S(l-cos6x)dx=i-x-/2 sin6x+C.


4.1.12. I=~ cosh 2 (Bx+ 5) dx.
Solution. Since cosh 2 u = cosh ;u+ 1 , then

I=~S[I +cash (16x + 10)] dx=i-x+ 312 sinh(l6x+ lO)+C.


4.1.13. I= Sx +'!:+s. 2

Solution. I= Sx +!:+s = S(x+:; + 1 =arctan(x+2)+C.


2 2

4 .1. 14 1 =S4x2~2s
4.1.15. I= s x2 +":+ 1

4.1.16. I= SJl'4='9X2
dx
4-9x 2
.
Solution. I= S~
4-
= - S~x
4/9-x2
9x 2
1
3 = 1 . 3x+c .
3 arcsm 2

4.1.17. I =Sy dx
5-x2 -4x

.
S ol ut ion. I= sY dx
5-x 2 - 4x
= sY dx
9-(x+2) 2
. x+-+c.
=arcsm- 3
2

4.1.18. l=S -v dx
xi+ 6x+ 1

4.1.19. /= S -x- x
4
dx
2 4
Solution.
l=S dx
4-x 2 -4x
='J dx
8-(x+2) 2
=-1-lnf
42
2Y2+x+21-C
2V2-(x+2) -j

4.1.20. I= S10xdx 2_ 7 .
4.2. Integration by Substitution 199

4.1.21. Evaluate the following integrals:


C dx
(a) J x 2-6x+ 13 ; (b) J dx;
rx-1
VX2
( c) s3-2 cot 2 x dx
cosl x ' (d) Sx22+3x2
(I +x2) dx.

4.1.22. Integrate:

~ )~
VJ-x2.+VI+x2d (b)s cos2x dx
v I -x4 ~ .
cos x- sin x '
(c) S2x+1_5x-1
iox dx; (d)
s (sin 5x- sin 5a) dx.

4.2. Integration by Substitution


The method of substitution (or change of variable) consists in sub-
stituting cp (t) for x where cp (t) is a continuously differentiable
function. On substituting we have:

~ f (x) dx = ~ f [cp (t)] cp' (t) dt,


and after integration we return to the old variable by inverse sub-
stitution t = cp- 1 (x).
The indicated formula is also used in the reverse direction:

~ f [cp (t)] cp' (t) dt = ~ f (x) dx, where x = cp (t).

4.2.1. /= ~ xVx-5-dx.
Solution. Make the substitution
Vx-5=t.
Whence
x-5=t 2 , x=t 2 +5, dx=2tdt.
Substituting into the integral we get

I= s (t 2 +5) i2t dt =2 s(t 4 +5t 2 )dt =2~+ 10; 3 +c.


Now return to the initial variable x:
I= 2 (x~5)'1 +IO (x;-5)'1 + C.
4.2.2. I= 1~xex S
Solution. Let us make the substitution 1 +ex= t. Whence
ex=t-1, X=ln{t-1), dx=dt/(t-l).
~uo Ch. IV. Indefinite Integrals

Substituting into the integral, we get

I= SI+ex = st(!-1)"
dx dt

But

therefore
I= 5 1 dt 1 -- 5 d: =In It- I I-In It I+ C.

Coming back to the variable x, we obtain


eX
/=In l+ex+C=x-ln(l+ex)+C.
Note. This integral can be calculated in a simpler way by mul-
tiplying both the numerator and denominator by e-x:

4.2.3. I= Sy (2x-5)
x +3
2

3
dx.

4.2.4. I= ~
(x2 - I) dx
2 1
(x4 +3x 2 +1)arctanx +
x
Solution. Transform the integrand
l-5 -
(l-1/x2)dx
[(x+ l/x) 2 + lj arc tan (x+ l/x)

Make the substitution x + _!_x = t; differentiating, we get

Whence
(1-:2 )dx=dt.

I= 5 dt
(! 2 + I) arc tan t
Make one more substitution: arc tan t = u. Then
dt
12+ I= du
and
I= Sduu-=lnlul+C.
4.2. Integration by Substitution 201

Returning first to t, and then to x, we have

I= In Iarc tan ti+ C =In arc tan ( x + I ~)I+ C.


4.2.5. I =S vaz=X2
x4
dx.
Solution. Make the substitution:
l dt
X=T; dx=-fi
Hence,

I= - S ~~~,-;; ~~ 12 dt =-St Va t I dt. 2 2-

Now make one more substitution: Va t -1 = z. Then 2 2 2a2 t dt =


= 2zdz and
I -- - ls
a2 Z2d Z -- - I a+c
3a2 Z

Returning to t and then to x, we obtain


(a2-x2)/1
I= - 3a2xa +c.
4 2 6 I= Sa 2 sin 2 x!b 2 cos 2 x
Solution.
I=~ a 2 sin 2 x+b
dx 2
cos 2 x
=_.!._i
b2 a2
I .~
cos2x
-tan
b2
2 x+ I
Make the substitution ab tan x = t; dt = ba __:!!_
2 Then
cos x
l \ dt I
I= ab J 1+t 2 =ab arc tan t + C.
Returning to x, we obtain

/=~arctan(~ tanx)+c.

4.2.7. I=~ VI+3sinxcosxdx.


Sol11t ion. Make the substitution 1 + 3 sin x = t, 3 cos x dx = dt. Then
I -- 3
.]__ s s
Vt dt -- 3J... {'la dt -- 3_.!._. ~4 t'J. + C-- (l +3 sin
4
x)'/a + C
.

4 . 2 .S. f=l sinxdx.


J V cos x
202 Ch. IV. Indefinite I nlegrals

4.2.9. 1=5 dxy .


(arc cos x) 0 I - x2
Solution. Make the substitution: arc cosx = t; - Yid~ x 2 dt. Then

I= - s ~
t0 = - st- 0 dt = __!._
4 t- 4 + C = - 1 - + C.
4 arc-
cos 4 x

4.2.10. I= iV x2+1
xa+3x+ 1
dx.

sin 2x
S
4.2.11. I= 1+ sin 2 xdx.
Solution. Make the substitution:
1 + sin 2 X= t; 2 sin xcosxdx =sin 2xdx '-=dt.
Then
I= S~' =int+C=in(l+sin 2 x)+C.

4 2 12

I= s I+ In x dx.
3+x lnx
Solution. Substitute
3+xlnx= t, (1 + In x) dx = dt
and get
I= S= In I t I+ C = In I 3 + x In x I+ C.
4.2.13. Evaluate the following integrals:

(a) Jrv1+1nx
x dx;
b
()
s dx
xlnx;

(c) 5 xdx
~; (d)
4
I
~
x11 - 1
+ dx;
x 211 a2

(e) 5 si1,x dx; (f) S(In x + h~ x) dxx.


4.2.14. Find the following integrals:
(a) 5x2 Vt-xdx; (b) r tnxdx ;
Jxr1+1nx
(c) 5cos 0 xVsinxdx; (d) Sy x l-x 2
. dx.

4.3. Integration by Parts


The formula
~ udv=uv- ~ vdu
is known as the formula for i11tegratio11 by parts, where u aBd v
are differentiable functions of x.
4.3. Integration by Parts 203

To use this formula the integrand should be reduced to the pro-


duct of two factors: one function and the differential of another
function. If the integrand is the product of a logarithmic or an
inverse trigonometric function and a polynomial, then u is usually
taken to be either the logarithmic or the inverse trigonometric func-
tion. But if the integrand is the product of a trigonometric or an
exponential function and an algebraic one, then u usually denotes
the algebraic function.

4.3. l. I= ~arc tan x.dx.


Solution. Let us put here
u= arc tanx, dv=dx,
whence
dx
du= 1+x2; V=X;

S
I= arc tan x dx = x arc tan x- S1~:2 = x arc tan x--} In( 1+x 2) +c.

4.3.2. I= ~ arc sin x dx.


4.3.3. I= ~ x cos x dx.
Solution. Let us put
U=X; dv=cosxdx,
whence
du=dx; v=sinx,
I=~ xcosxdx=xsinx-~ sinxdx==xsinx+cosx+C.
We will show now what would result from an unsuitable choice
of the multipliers u and dv.
In the integral ~ xcosxdx let us put
u =-co cos x; dv = x dx,
whence
du=-= - sin x dx;
In this case
Io= ; x 2 cos x + ; sx2 sin x dx.

As is obvious, the integral has become more complicated.


4.3.4. I= ~ x 3 In x dx.
204 Ch. IV. Indefinite Integrals

Solution. Let us put


U= lnx; dv=x 3 dx,
whence
d dx _I ,
U=x; V--q,
I= l.x'
4
lnx-..!..
4
Sx' dx
x
=..!..
4
x' lnx-..!..
4 J
r
x 3 dx = _!_x
4
4 lnx-l.x 4 +c
16 .

4.3.5. I=~ (x 2 -2x+ 5) e-x dx.


Solution. Let us put
U=X 2 -2x+5; dv=e-xdx,
whence
du= (2x-2) dx; v = - e-x;
I=~ (x 2 -2x+ 5) e-x dx = -e-x (x2 -2x+ 5)+2 ~ (x- l)e-x dx.
We again integrate the last integral by parts. Put
x-l=u; dv=e-xdx,
whence
du=dx; V=-e-x.
/ 1 =2 ~ (x-l)e-xdx=-2e-x(x-l)+2 ~ e-xdx=-2xe-x+c.
Finally we get
I =-e-x(x2 -2x+5)-2xe-x+C=- e-x (x 2 + 5)+C.
Note. As a result of calculation of integrals of the form ~ P (x) eax dx
we obtain a function of the form Q (x) eax, where Q (x) is a poly-
nomial of the same degree as the polynomial P (x).
This circumstance allows us to calculate the integrals of the in-
dicated type using the method of indefinite coefficients, the essence
of which is explained by the following example.
4.3.6. Applying the method of indefinite coefficients, evaluate
I= ~ (3x 3 - l 7) e2 x dx.
Solution. ~ (3x3 - l7) e2 x dx= (Ax 3 + Bx2 + Dx+ E) e2 x +c.
Differentiating the right and the left sides, we obtain
(3x 3 - 17) e2 x = 2 (Ax3 + Bx2 + Dx + ) e2 x + e2 x (3Ax 2 + 2Bx + D).
Cancelling e2 x, we have
3x3 - l7 = 2Ax3 + (2B + 3A)x2 + (2D+ 2B) x+(2E + D).
~~~~~~~~-"-_4_.3_._l_n~teg~r_a_tio_n--"by~Pa_rt.s~~~~~~~-2_05

Equating the coefficients at the equal powers of x in the left and


right sides of this identity, we get
3=2A; 0=2B+3A;
0=2D+2B; -17=2E+D.
Solving the system, we obtain
3 9 9
A=2; B=-4; D=4; E =-n
8 .
Hence,
S(3x -l7)e xdx= (23 x
3 2 3-
4 x2 + 4 x- 8 e2 x+c.
9 9 77)

4.3.7. Integrate:
I=~ (x 3 +l)cosxdx.
Solution. Let us put
u = x3 + l; dv= cosxdx,
whence
du= 3x2 dx; v = sin x.
/=(x3 +l)sinx-3 ~ x2 sinxdx=(x 3 +l)sinx-3/ 1 ,
where / 1 = ~ x2 sin xdx.
Integrating by parts again, we get
/ 1 = -x2 cosx+ 2/ 2 ,
where / 2 = ~ xcosxdx.
Integrating by parts again, we obtain
/ 2 = x sin x+ cosx+C.

Finally, we have:
I=~ (x 3 + 1) cosx dx=(x 3 + l)sinx+3x2 cosx-6x sin x-6 cosx+ C=
= (x 3 -6x+ 1) sin x+ (3x2 -6) cosx+ C.
Note. The method of indefinite coefficients may also be applied
to integrals of the form
~ P (x) sin ax dx, ~ P (x) cos ax dx.

4.3.8. I= ~ (x2 + 3x + 5) cos 2x dx.


Solution. Let us put
~ (x2 +3x+5)cos2xdx=
= (A 0 x2 + A 1 x+ A2 ) cos 2x+ (B 0 x 2 + B1 x+ B2 ) sin 2x+ C.
206 Ch. IV. Indefinite Integrals

Differentiate both sides of the identity:


(x2 +3x+ 5) cos2x= -2 (A 0 x2 + A 1x+ A2 ) sin 2x+
+ (2A 0x+ A1 ) cos2x+ 2 (8 0x2 +8 1x+ 8 2 ) cos 2x+(28 0x+8 1 )sin2x=
= [28 0x2 + (28 1 + 2A 0 ) x + (A 1 + 28 2 )) cos 2x +
+ [-2A 0x2 +(28 0 -2A 1 )x+(8 1 -2A 2 )] sin 2x.
Equating the coefficients at equal powers of x in the multipliers
cos 2x and sin 2x, we get a system of equations:
28 0 = 1; 2 (8 1 + A0 ) = 3; A 1 + 28 2 = 5;
-2A 0 =0; 2(8 0 -A 1 )=0; 8 1 -2A 2 =0.
Solving the system, we find
I I 3 3 9
A0 =0; 8 0 =2; A 1 =2; 8 1 =2; A2 =4; 8 2 =4
Thus,
~ (x 2 +3x+5)cos2xdx=( ~ + ! )cos2x+( f x + }x+;) sin x+C.
2 2

4.3.9. I=~ (3x 2 + 6x+ 5) arc tanxdx.


Solution. Let us put
u= arc tanx; dv = (3x 2 6x 5) dx, + +
whence
dx
du = 1 + x 2 ; v = x3 + 3x2 + 5x.
Hence,
(' x 3+3x2 +5x
I= (x 3 + 3x2 + 5x) arc tan x- J 1+x2 dx.
Single out the integral part under the last integral by dividing
the numerator by the denominafor:

/ 1 = Sx3+3x2+.5x
l+x2 dx =
s
(x+3)dx-t-
s4x-3
x2 +I
dx=
x2
=2+ 3x+ 25 x2+dx -3.sx dx+
2x
1 2 1=
x2 + 3x+21n(x2 +1)-.3 arctanx+C.
2
Substituting the value of / 1 , we finally get
I= (x3 +3x 2 + 5x.+3) arc tanx-x 2/2-3x-2 ln (x 2 + l) + C.
4.3.10. Find the integral
I= ~ e&x cos4xdx.
Solution. Let us put
e&x = u; cos 4x dx =.dv,
4.3. Integration by Parts 207

whence
5e5 " dx =du' v = J_
4
sin 4x.
Hence,
I= ~e 5 x sin 4x - ~ Se 5x sin4x dx.

Integrating by parts again, we obtain

/ 1 = J(' e x sin 4x dx = -
5
I
4 e5 xcos4x+ 4
5
Je xcos4xdx.
n
5

Thus,
I= ~ e5 x sin 4x - ~ ( - ~ e5 x cos 4x + ~ Se x cos 4x dx) ,
5

i. e.
I 5 x ( sm4x+-cos4x
l=-e . 5 ) --
25/
4 4 16
Whence
/= 441e5 x(sin4x+ ~ cos4x)+c.
4.3.11. I=~ cos(ln x) dx.
Solution. Let us put
u =cos (lnx); dv =dx,
whence
du = - sin (In x) dx; v = x.
x
Hence,
I= ~cos (In x) dx = x cos (In x) + ~sin (In x) dx.

Integrate by parts once again


u-= sin (In x); dv=dx,
whence
dx
du=cos(lnx)-;
x
V=X.

Hence,
/ 1 =~sin (In x)dx = x sin (In x)- ~cos (In x) dx.
Thus
I= ~cos (lnx)dx =X cos (lnx) +x sin (In x)-1.
Hence
I=~ [cos (lnx)+ sin (In x)] + C.
208 Ch. IV. Indefinite Integrals

S
4.3.12. I= x In ( 1 ~) dx. +
Solution. Let us transform the integrand

In(l + !)=1nx~ 1 =1n(x+l)-lnx.


Hence
I=~ xln (x+ l)dx- ~ xlnxdx=/ 1 - / 2
Let us integrate / 1 and / 2 by parts. Put
u=ln(x+ l); dv=xdx,
whence

Hence
/ 1 = Sxln(x+l)dx= 2I (x2 -l)ln(x+l)-2I s
(x2- I) dx
l+x =
=x -
- I2
2 -ln(x+l)- 2
I s(x-l)dx=--ln(x+l)- 4 x +-2 x+C.
x2 - I
2
I 2 I

Analogously,
I
/ 2 = J\ x In xdx= x2
2 Inx- 4 x2 +c.
Finally we have

I= Sx In ( 1 + ! )dx =} (x 2- 1) In (x + I)- x; In ~ + i + C.

4.3.13. I= s yxq:I [Jn<::+ 1)-2 ln x) dx.


Solution. First apply the substitution
I
1+2=t.
x
Then
2dx dx I
dt = - xa or x3 = - 2 dt.
Hence,
I= s VI+ ; 2 In x2: 1 s
~: = - ~ VT Int dt.
The obtained integral is easily evaluated by parts. Let us put
u = In t; dv = VTdt.
Then
dt
du=-
t '
V =3
-
2 t v-t.
4.3. Integration by Parts 209

Whence

- f 5VT In t dt = - ; [; t Vt In t - ; 5VT dt] =


=-} [; tVtlnt-: tVT] +c.
Returning to x, we obtain

I = - +l; ( 1 + :2 )312 In ( 1 + ~ ) - : ( 1 + :2 )3' 2


] + C=

= (x
2
+ 1 ~tX2+1 [2-31n ( i + ;2) J + c.
4.3.14. I= ~sin x In tan x dx.

4.3.15. I=~ In (Vl-x+ Vl +x)dx.


Solution. Let us put
u=ln(Vl-x+Vl+x); dv=dx,
whence
du= I
Vl-x+ Yl+x
(- I
Vl-x 2 Vl+x
2
+ I )dx=
=..!._. VT=X-Vr:+X. dx =..!._. VI=X2-I dx;
2 Vl-x+ Vl+x Yl-x 2 2 x Vl-xa
V=X.
Hence,
l=xln(Vl-x+Vl +x)--21 Sx vr-I
x l-x2
dX=

=X In (Vl-x+ Vl +x)-+ s sv~~xa


dx+i- =

=xln(Vl-x+Vl+x)-~ x+}arcsinx+C.
Note. In calculating a number of integrals we had to use the
method of integration by parts several times in succession. The
result could be obtained more rapidly and in a more concise form
by using the so-called generalized formula for integration by parts
(or the formula for multiple integration by parts):
~ u (x) v (x) dx = u (x) v1 (x)-u' (x) v2 (x) + u" (x) V8 (x)- ...
... +(-l)n- 1 u<n-u(x)vn(x}-(-l}n- 1 ~ u<n>(x)vn(x)dx,
where
V1 (x} = ~ V(x) dx; V2 (x) = ~ V1 (x) dx; ... ; Vn (X) = ~ Vn-t (x) dx.
Here, of course, we assume that all derivatives and integrals appea-
ring in this formula exist.
210 Ch. IV. Indefinite I nfegrals

The use of the generalized formula for integration by parts is


especially advantageous when calculating the integral ~ P n (x} qi (x} dx,
where Pn (x) is a polynomial of degree n, and the factor qi (x) is
such that it can be integrated successively n +I times. For example,
ekx ekx
SPn (x) ekx dx= Pn(x)7-P~ (x)-fi2+ ... +
ekx
+ (-l)n P)in> (x) kn+l +C =
=ekx IPn(x) I
_-k--k2 P'n (X )+ +(-l)n
kn+1 pcm
" (X >] +C .
4.3.16. Applying the generalized formula for integration by parts,
find the following integrals:
(a) ~ (x3 -2x2+3x-l)cos2xdx,
(b) ~ (2x 3 +3x 2 -8x+ I) V2x+6dx.
Solution.
(a) S(x -2x2+3x- l) cos2xdx= (x 2x2+3x-- l )sin2x
-
3 3-
2- -
-(3x2_4x+ 3) (-co: 2x) +(6x- 4) ( - si~2x)_ 6 cot62x +C=

= si~ 2x (2x 3 -4x2+ 3x) +co: 2x (6x 2 -8x + 3) + C;

(b) ~ (2x 3 + 3x -8x + 1) v 2x + 6 dx =


2

=(2x 3 +3x 2 -8x+ 1)( 2 x~ 6 ) 312 (6x2 +6x-8)( 2xt,~>"' 2 +


+(12x+6)(2x-f-6)1/2 12(2x-f-6)u/2 +C=
3.5.7 3.5.7.9

= ~~ (2x + 6) (70x3-45x2- 396x + 897} + C.


Evaluate the following integrals:
4.3.17. ~ln(x+Vl+x 2 )dx.

4.3.18. ~ V x(In x) 2 dx.


4 . 3 . 19 . s ar;;in xdx.
1-1-x
4.3.20. Sxcossin xdx
3 x
4.3.21. ~ 3"cosxdx.
4.4. Reduction Formulas 211

4.3.22. ~ (x 3 -2x 2 + 5) e x dx.


3

4.3.23. ~ ( l + x cos x dx.


2) 2

4.3.24. ~ (x + 2x- l) sin 3xdx.


2

4.3.25. ~ (x -2x + 3) In x dx.


2

4.3.26. ~ x 3 arc tan x dx.

4.3.27. ~ x 2 arc cos x dx.

4.3.28. Applying the formula for multiple integration by parts.


calculate the following integrals:
. 2(3 x+ l)dx;
(a ) S(3 x 2 +x- 2) sm (b) sx2-7x+ldx.
V2x+1
4.4. Reduction Formulas
Reduction formulas make it possible to reduce an integral depend-
ing on the index n > 0, called the order of the integral, to an
integral of the same type with a smaller index.
4.4.1. Integrating by parts, derive reduction formulas for calcu-
lating the following integrals:

(a) /"= S(x2+a2)n;


dx
(b) ln,-m=
s sinn x
cos"'xdx;
(c) In= ~ (a 2 -x2 ) 11 dx.
Solution. (a) We integrate by parts. Let us put
I
U= (x2+a2)n' dv= dx,
whence
2n x dx
du= (x2+ai)n+1 V=X.

I Ience,

whence
I = _I_ . x I _.!_ I .
n+J 2na 2 (x 2 i- a:l)" + 2n2n- aJ n
212 Ch. IV. Indefinite Integrals

The obtained formula reduces the calculation of the integral I n+l


to the calculation of the integral In and, consequently, allows us
to calculate completely an integral with a natural index, since

/ 1 =
dx
x~+ SI x
a2 =-arctan-+C.
a a
For instance, putting n = 1, we obtain
\ dx I x I I x I x
f2 = J (x2+ a2)2 = 2a2 x2+a2+2a2f1 = 2a2 x2+a2 + 2aa arc tan a+ C;
putting n = 2, we get

I a= S(x2~a2)s = 4~2 (x2~a2)2 + 4:2 /2 =


I x 3 x 3 x
= 4a2 (x2+ a2)2 + 8a4 x2+a2+ Ba& arc tan a +C.
(b) Let us apply the method of integration by parts, putting
u =sinn- 1 x; sin x d
dV=-- X
cosm x '
whence
I
du=(n-l)sinn- 2 xcosxdx; V=(m- l)cosm-lx (m =I= 1).
Hence,
sinn-lx n-1 5sinn- 2 xdx
In, -m = (m-1) cos"'- 1 x - m-1 cosm- 2 x =
sinn- 1 x n-1
(m-l)cosm-tx m-1'n-2.2-m (m=l=l).
(c) Integrate by parts, putting
U=(a 2-x 2)n; dv=dx,
whence
du=- 2nx (a 2 -x2 )n- 1 dx; v =X.
Hence
In= x (a 2- x2)n+ 2n ~ x 2 (a 2 -x2)n- 1 dx =
=x(a2- x2t+2n ~ (x 2 -a 2 +a 2 )(a 2 -x2 )n- 1 dx=
=X (a 2- X 2)n-2nf,. +
2na 2[ n-1"
Wherefrom, reducing the similar terms, we obtain
(1+2n) In =x (a 2 -x 2)" + 2na2 / ,,_ 1
Hence,
4.4. Reduction Formulas 213

For instance, noting that

I- 1/2 = SV a2-x2
dx =arc sin~+ C,
a

we can find successively

4.4.2. Applying integration by parts, derive the following reduc-


t ion formulas:
(a) In=~ (lnx)ndx=x(lnx)n-nln-i;
xa+l (In x)n n
(b) In= Sx"(lnx)ndx a+I -a+I ln-1 (a =f=.-1);

(c) In=~ xnexdx=xnex-n/n_ 1 ;


(d) ln= ~ eaxsinnxdx:=
_ ex . I
- a 2 +n 2 Stn
n-l ( .
x asmx-ncosx
)
+ n(n-1)
az+n 2 n- 2

4.4.3. Derive the reduction formula for the integration of In=


= (' .d:
j Sin X
and use it for calculating the integral / 3 = .d~ .
Sin' X
S
4.4.4. Derive the reduction formulas for the following integrals:
(a) In=~ tannxdx; (b) in=~ cotnxdx;
5
xn dx
(c) In= .r-.-
r xj+a
Chapter 5
BASIC CLASSES
OF INTEGRABLE FUNCTIONS

5.1. Integration of Rational Functions

If the denominator Q (x) of the proper rational fraction ~ ~;~ can


be represented in the following way:
Q (x)=(x-a)k(x-b) 1 (x 2 +ax+~)'(x 2 +yx+)s .. .,
where the binomials and trinomials are different and, furthermore,
the trinomials have no real roots, then
P(x) A1 A2 Ak
Q (x) = x-a + (x-a)2 + + (x-a)k +
B1 B2 B1
+ x-b + (x-b)2+ .. +rx=w+ ..
M1x+N1 M 2 ~+N 2 M,x+N,
+ x2+ax+~ + (x2+ax+~)2 + + (x2+ax+~V +
+ R1x+L 1R x+L 2 2 Rsx+Ls
x2+yx+ + (x2+yx+ )2 + + (x2+yx+)s +
where
A1 , A 2 , , B1 , B 2 , , M1 , N1 , M 2 , N2 , . , R1 , L1 , R 2 , L2 ,
are some real constants to be determined. They are determined by
reducing both sides of the above identity to integral form and
then equating the coefficients at equal powers of x, which gives
a system of linear equations with respect to the coefficients. (This
method is called the method of comparison of coefficients.) A system
of equations for the coefficients can also be obtained by substitu-
ting suitably chosen numerical values of x into both sides of the
identity. (This method is called the method of particular values.)
A successful combination of the indicated methods, prompted by
experience, often allows us to simplify the process of finding the
coefficients.
If the rational fraction ~ i;~ is imp-roper, the integral part should
Ctrst be singled out.
5.1. Integration of Rational Functions 215

5. t. t.
I= s 15x 2 -4x-81
(x-3) (x-j-4) (x-1)
dx.

Solution. The integrand is a proper rational fraction. Since all


roots of the denominator are real and simple, the integral will
appear in the form of the sum of three simple fractions of the form
15x2 -4x-81 A B D
(x-3) (x-j-4) (x-1) = x-3 + x-j-4 + x-1'
where A, B, D are the coefficients to be determined. Reducing the
fractions to a common denominator and then rejecting it, we obtain
the identity
15x2 - 4x-81=A(x+4) (x-1)+ B (x-3) (x- l) +
+D(x-3)(x+4). (*)
Comparing the coefficients at equal powers of x in both sides of
the identity, we get a system of equations for determining the
coefficients
3A-4B+D=-4; -4A +3B-12D=-81.
Solving the system of equations we find A= 3, B = 5, D = 7.
Hence,

t=35~+5
x-3 .)r~+15~=
x-j-4 x-1
= 3 In I x - 31 + 5 In I x + 4 I+ 7 In I x- I I+ C =
=In [ (x-3) 3 (x + 4) (x--1)11 + C.
Note. Let us use the same example to demonstrate the applica-
tion of the method of particular values.
The identity (*) is true for any value of x. Therefore, setting
three arbitrary particular values, we obtain three equations for
determining the three undetermined coefficients. It is most conve-
nient to choose the roots of the denominator as the values of x,
since they nullify some factors. Putting x = 3 in the identity (*),
we get A =3; putting x=-4, we obtain B=5; and putting x~ l,
we get D=7.
(' x 4 dx
5.1.2. I= J (2-j-x) (x2- I).

5.1.3. I
= 5x x3-x2-2x
-3x -3x-2 d
4 2
x.
Solution. Since the power of the numerator is higher than that
of the denominator, i.e. the fraction is improper, we have to single
out the integral part. Dividing the numerator by the denominator,
216 Ch. V. Basic Classes of Integrable Functions

we obtain
x'-3x2-3x-2 __ x l- x+ 2
x3 -x2-2x - + x(x2-x-2)
Hence,
I= S xs-x2-2x
x'-3x2-3x-2
dx =
s(x+ I) dx-
s (x+2) dx
x (x-2) (x+ I) .
Expand the remaining proper fraction into simple ones:
x+2 A B D
x(x-2)(x+l)=x+x-2+x+1
Hence
x+2 =A (x-2) (x+ I)+ Bx (x+ I)+ Dx (x-2).
Substituting in turn the values x1 =0, x2 =2, X 3 =-I (the roots
of the denominator) into both sides of the equality, we obtain
2 I
A=-l; 8= 3 ; D=-g
And so

x -~s~-_!_s~=
'=S(x+ t)dx+S dx 3 x-2 3 x+ I
~ 2 I
= 2 +x+lnlxl- 3 Inlx-2l- 3 Inlx+ l l+C.

S 2x2-3x+ 3
5.1.4. I= x3- 2x2+x dx.
Solution. Here the integrand is a proper rational fraction, whose
denominator roots are real but some of them are multiple:
x3 -2x2 +x =X (x-1) 2
Hence, the expansion into partial fractions has the form
2x2-3x+3 A B D
x3 -2x 2 + x=x+ (x-1) 2 + x-1'
whence we get the identity:
2x 2 -3x+3 =A (x-1) 2 + Bx+ Dx (x-1) =
=(A+ D)x2 + (-2A-D+8)x+A. (*)
Equating the coefficients at equal powers of x we get a system
of equations for determining the coefficients A, 8, D:
A+D=2; -2A-D+8=-3; A =3.
Whence A =3; 8=2; D=-1.
Thus,
I =3 s~ +2 s (x dx1)2-S xd\ =3 lnlxl-x 2 I-In lx-1 i+C.
5.1. Integration of Rational Functions :?17

Note. The coefficients can be determined in a somewhat simpler


way if in the identity (*) we put x 1 = O; x 2 = l (the denominator
roots), and x3 equal to any arbitrary value.
At x=O we get 3=A; at x~~ l we will have 2=8; at x=2
we obtain 5=A+2B+2D; 5=3+4+2D; whence D=-1.
x +1
5.1.5. I= Sx (x- 3
!) 3 dx.
xdx
5.1.6. I= S xa+ I .

Solution. Since x 3 + l =(x+ l)(x2 -x+ l) (the second factor is


not expanded into real multipliers of the first power), the expan-
sion of the given fraction will have the form
x A Bx+D
x3 + I = x+ I + x2 -x + I
Hence,
x = A (x 2 -x + I)+ (Bx+ D) (x + I) =
=(A +B)x2 + (-A+ B+ D)x+(A+D).
Equating the coefficients at equal powers of x, we get
I I I
A=-3; B=3; D=3.
Thus,
I =-3
I s dx I
x+1+3
s x+ I I I I I
x2-x+ldx=:-3 nlx+l +311.
To calculate the integral
I1 = Sx ~~ ~ 1 dx
2

let us take the perfect square out of the denominator:

x 2 -x+ l = ( x-+ r+ !
and make the substitution x-i=t. Then

I -
1 -
r 1+++1
J 12+ :
dt-r
-
~+~r~-
J12+ ! J12+ ! -
2

=+Jn ( t 2 + ! )+ V3 arc tan ; 3 + C.


Returning to x, we obtain
I
l1=2ln(x 2 -x+l)+ v-3 arctan ra+c.
~-I
218 Ch. V. Basic Classes of Integrable Functions

Thus,
x
J xa+ I dx =
(>

I=
_ I I 2 l y3 2x- I
- - 3 lnjx+ 11+ 6 In(x -x+ ) + -3-arctan y3" +c.

5.1.7. I= S(x2+ dx
I) (x2+4) .

Solution. The denominator has two pairs of different conjugate


complex roots, therefore
l Ax+B Dx+E
+ x +4'
x2 + I 2

hence
1=(Ax+8) (x 2 +4)+ (Dx+ E) (x + 1).
2

Here it is convenient to apply the method of particular values for


determining the coefficients, since the complex roots of the deno-
minator (x = + i and x = + 2i) are sufficiently simple.
Putting x = i, we obtain
38+ 3Ai = 1,
I
whence A= 0, B = 3 . Putting x = 2i, we obtain -3E-6Di = 1,
I
whence D=O, E = - 3 . Thus,

S(x2+ I)dx(x +4) = 3l s dx x+ I - 3l s dx2x+4 =


2 2

l I x
= 3 arc tan x- 6 arc tan 2 + C.

5 .t.B. I= S(x +x+2)(x


2
(x+ l) dx
+4x+5)' 2

_ (' x4+4x3 + I lx 2 +12x+B


5.1.9. I - j (x2+2x+3)2 (x+ l) dx.
Solution. Here we already have multiple complex roots. Expand
the fraction into partial fractions:
x4+4x3 + I lx2 + 12x+B Ax+B Dx+E F
(x 2 +2x+3)2(x+I) =(x2 +2x+3) 2 +x2 +2x+3+x+I'
Find the coefficients:
A= 1; B=-1; D=O; E =0; F= 1.
Hence,
I= s +
x'+4x 3 + I lx 2 12x+8 d =
(x 2 +2x+3) 2 (x+ I) X

= 5(x2+ 2x+ 3)2dx+


x-1 s dx
x+I =ln\x+ l \-f--/ 1 ,
5.2. Integration of Certain Irrational Expressions 219

Calculate / 1= (x 2 :;~ 3 ) 2 dx.


S
Since x 2+ 2x + 3 = (x + 1) 2+2, let us make the substitution
x-f- 1 = t. Then we obtain

The integral
/ 2 = S(t2 dt+ 2)2
is calculated by the reduction formula (see Problem 4.4.1 ):
I t I (' dt I t I t
/2 = 4 t2+2+-:r J t2+2=-:r12+2+ 4 V2 arc tan V2 + C.
Thus
I t I t
f1=-2u2+2) 2(t2+2) 2V2arctanv2+c.
Returning to x, we obtain
I ___ I__ x+I I x+I
1=-2(x2+2x+3) 2(x2+2x+3) 2 V"2 arc tan V"2 + C.
We finally obtain
I =Sx4 +4x3 + llx 2+ 12x+8 d =
(x 2+2x+3) 2 (x+ I) X

=!nix+ 1I
Find the following integrals:
5x3+9x 2-22x-8
5.1.10. S x3 - 4x dx.

5.1.11. S(x+ I) (x+dx2) 2 (x+3)3


5.1.12. S(x -4x+4)dx(x2-4x+5)
2

5. t.13. 5 +x)(I
dx
+x2) (I +x3)
(I
x 3 +3
5.1.14. S (x+ l)(x2+1>dx.

5.2. Integration of Certain Irrational Expressions


Certain types of integrals of algebraic irrational expressions can
be reduced to integrals of rational functions by an appropriate change
of the variable. Such transformation of an integral is called its ra-
tionalization.
220 Ch. V. Basic Classes of Integrable Functions

I. If the integrand is a rational function of fractional powers of

an independent variable x,
i. e. the function R (x, ;1,, ... , x~) ,
then the integral can be rationalized by the substitution X= tm,
where m is the least common multiple of the numbers q1 , q2 , , qk.
II. If the integrand is a rational function of x and fractional
powers of a linear fractional function of the form ~:t:, then ra-
tionalization of the integral is effected by the substitution
;:t! = tm, where m has the same sense as above.

5.2.1. I= J\x+V~+V~
x ( 1+ V x) dx.

Solution. The least common multiple of the numbers 3 and 6 is 6,


therefore we make the substitution:
x=f 6 , dx=6t 0 dt,
whence
I =6 5 ta+ t' +
(t6 t) to dt
(I -j-t2)
= 6 5to+ 1s + I dt =
J +t2

5 S
= 6 t3 dt + 6 t 2d~ 1 = ~ t4 + 6 arc tan t + C.
Returning to x, we obtain
3 2 6 -
I= 2 x 3 +6 arc tan V x+C.

5.2.2. J = j Vv-x+V~
V dx.
xo- x7

5.2.3. I =~ ~
(2x-3)1 dx

(2x-3) 3 +1
Solution. The integrand is a rational function of V2x-3, the-
refore we put 2x-3 = t 6 , whence
I I
dx=3t 0 dt; (2x-3)2 =f 3 ; (2x-3)3 =t 2
Hence,

I= 5+ 3tB
12 1 dt = 3 s (t6 - t 4 + t 2 - 1) dt +3 s+ 1
dt
12 =
t7 to t3
= 3 7 -3 5 +3 3 -3t + 3 arc tan t +c.
5.2. Integration of Certain Irrational Expressions 221

Returning to x, we get

l l
I= 3 7 (2x- 3) 6
.!_
-
I ~I
5 (2x-3) 6 3 (2x-3)
_!...
2 -

-(2x-3)} +arc tan (2x-3) ! ] + C.


5.2.4. 1~ L( H VSl)"
5.2.5. I= s(
2
2 x)
2 v~~;dx.
Solution. The integrand is a rational function of x and the ex-
.
press10n v 2 -+x,
x therefore let us introduce the substitution
2

V 2--x_ .
2+x- t,
whence

Hence
I=-
+ t3)2 t .J 2t2
S2 (I16to(l+t3)2 3
dt =-2
s dt 3
l3=4t2+C.
Returning to x, we get

'= ! V(;+;y +c.


5.2.6. '=S V 4 dx
(x- 1)3 (x +2)&
.

Solution. Since
V (x-1) 3 V
(x+2)~ =(x-1) (x+2)
x+2
x-- I

the integrand is a rational function of x and V


-x+2
x- I; therefore let
us introduce the substitution:

V x+2 =t x+2 =l'


x-1 'x-1 '
whence
222 Ch. V. Basic Classes of Integrable Functions

Hence,
'= -S (/4-1) (/4-1) 12/3d/ =-.!s~
3-3t 4 t (t'- 1)2 3 /2
=.!+c
3t

Returning to x, we obtain
4
l=3 vx-1x+2+C.

5.2.7. J
" v dx
(1-x) l - x2 "

5.2.s. Iv .
" dx
., (x+ (x 1) 2 -1) 4

5.2.9. J(x-2) v: +;dx.


5.3. Euler's Substitutions
Integrals of the form ~ R(x, V ax2 +bx+c)dxare calculated with
the aid of one of the three Euler substitutions:
(I) V ax 2 +bx+ c = t + x Va if a > O;
(2) Vax 2 +bx+c= tx Ve if c > O;
(3) Vax 2 +bx+c=(x-cx.)t if
ax2 +bx+c =a (x-cx.) (x-~).
i.e. if ex. is a real root of the trinomial ax 2 +bx+ c.
5.3.1. I= s 1+
y dx
x2+2x+2

Solution. Here a= 1>0, therefore we make the substitution


Vx 2 +2x+2=t-x.
Squaring both sides of this equality and reducing the similar terms,
we get

whence
/2-2 12 +21+2
X=m+t); dx= 2 (l+t) 2 dt;

1 + V x2 +2x+ 2= I +t- 2 ( 1 +t)= 2 (l+t)


/2-2 t2 +4t+4

Substituting into the integral, we obtain

l= S(t2+4t+4)2(l+t)
2 (I +t) (t2+2t+2)
dt =
s(t2+2t +2) dt
(l+t)(t+2)2"
2
5.3. Euler's Substitutions 223

Now let us expand the obtained proper rational fraction into par-
tial fractions:
t2+2t+2 A B D
(t+ I) (t+2) 2 t +I+ t +2+ (t +2) 2

Applying the method of undetermined coefficients we find: A= 1,


B=O, D=-2.
Hence,
[2 + 2t + 2
S s dt s dt
u+1)(t-i-2)2dt = t+i-2 u+2>2=lnlt + l l+t+2+c.
2

Returning to x, we get
l=ln(x+l+Vx2 +2x+2)+ / +c.
x+2+ x 2 + 2x+2
5.3.2. I =5 x+
y xdx--x-r- I
2
.

Solution. Since here c = 1 > 0, we can apply the second Euler


substitution
V x2 -x+ 1 = tx- l,
whence
2t-I
(2t-l)x=(t 2-l)x2; X=t2=T;
[2-
dx=-2((2_ 1') 2 dt; x+
t-L I vx -x+l=t-I"
2
t

Substituting into /, we obtain an integral of a rational fraction:

5x+Yx2--x+I dx s -2t 2+2t-2 dt


t(t-l)(t+l) 2 '
-2t2+2t-2 A B D E
t (t-1) (t+ 1) 2 =t+ t -I+ (t+ 1)2+ t +I'
By the method of undetermined coefficients we find
I 3
A=2; B=- 2 ; D=-3; E=- 2
Hence

1= 2 S7-2 t-1- 3 (t+l) 2 -2 t+1=


dt lsdt s dt 3sdt
I 3 3
=2lnjt!- 2 lnlt-l j+t+i- 2 1njt+ll+C,

where t = y x""""2-x-:-+-:-1 + I .
x
5.3.3. I= s .rdx
(l+x) r l+x-x2

224 Ch. V. Basic Clas~es of Integrable Functions

5.3.4. I =S (Y xdx
7x-10-x2) 3
.

Solution. In this case a < 0 and c < 0 therefore neither the first,
nor the second Euler substitution is applicable. But the quadratic
trinomial 7x-10-x2 has real roots a= 2, ~ = 5, therefore we
use the third Euler substitution:
V7x-10-x 2 = V(x-2) (5-x) =(x-2) t.
Whence
5-x= (x-2) t 2 ;
s+21 2 6t dt
X= 1+t2; dx= -(l+t2)2;

( 5+2t 31
2
(x-2)t= +
1 12 -2
)
t= i+i2
Hence
I= -2; s~dt=-: SC~ +2)dt= - ~ ( - ~ +2t)+c,
_ V7x-10-x 2
where t- x- 2
.
Ca lculate the following integrals with the aid of one of the Euler
subst it ut ions:
5.3.5. s y x-
dx
x2 +2x+4

5.3.6. sy dx
1-x2 - I
.

5.3.7. S dx
Y (2x-x2)3 '
5.3.8. I
~
(x+ YT+X2>10
.~
J' I+-<- dx.

5.4. Other Methods of Integrating Irrational Expressions


The Euler substitutions often lead to rather cumbersome calcu-
lations, therefore they should be applied only when it is difficult
to find another method for calculating a given integral. For r_alcu-
lating many integrals of the form
-----
~ R(x, J/ax 2 +bx+c) dx,

simpler methods are used.


I. Integrals of the form
I =J Mx-t-N
Yax 2 +bx+c
dx
5.4. Methods of Integrating Irrational Expressions 225

are reduced by the substitution x+:a=t to the form

I- M
- 1
s t dt
Vat 2+K
+N 1
s dt
Vat 2 +K'
where M 1 , N 1 , K are new coefficients.
The first integral is reduced to the integral of a power function,
while the second, being a tabular one, is reduced to a logarithm
(for a > 0) or to an arc sine (for a < 0, K > 0).
I I. Integrals of the form
(' Pm (x) dx
J Yax +bx+c2 '
where Pm (x) is a polynomial of degree m, are calculated by the
reduction formula:

5Y axPm(x)dx
+bx+c
2
=Pm_ 1(x)Vax +bx+c+KI .
J Vax +bx+c
2
2
dx , (I)

where Pm-i (x) is a polynomial of degree m- I, and K is some


constant number.
The coefficients of the polynomial Pm-i (x) and the constant
number K are determined by the method of undetermined coeffici-
ents.
III. Integrals of the form

S(x-a1)m Y:: +bx+ c 2

are reduced to the preceding type by the substitution


l
x-a1=T
IV. For trigonometric and hyperbolic substitutions see 5.7.
S.4.1. l=S Y (x+3)dx
4x 2 +4x-3
.

Solution. Make the substitution 2x+ I= t, whence


t-1 I
X=-:.1-, dx=zdf.
Hence,
I J
= .!4 (ty+12_4
5> dt -41 Vt 2- 4 + 54 In It + V t 2- 4j + C.
Returning to x, we get
I =i-V4x2 +4x-3+ ! lnj2x+ I+ V4x +4x-3l+c. 2
226 Ch. V. Basic Classes of Integrable Functtons

5.4.2. I= SV x2+2x+5
5x+ 4 dx.

I= S.~
x -x-1 3
5.4.3. dx.
r x +2x+2 2

Solution. Here Pm (x) = x 3 -x-l. Hence,


Pm- 1 (x) = Ax2 +Bx+ D.
We seek the integral in the form
I= (Ax 2 +Bx+ D) V x2 + 2x + 2 +KS ~x--.
r x2 +2x+2
Differentiating this equality, we obtain
I'= x3 -x-1 =
x 2 +2x+2
=(2Ax+B)Vx2 +2x+2+(Ax2 +Bx+D) V x+t
x2 +2x+2
+
+ K
Vx 2 +2x+2
Reduce to a common denominator and equate the numerators
x3 -x-1 = (2Ax+B) (x 2 + 2x+2)+ (Ax 2 +Bx+ D) (x+ l)+K.
Equating the coefficients at equal powers of x, we get the following
system of equations:
2A+A= I, B+4A+B+A =0;
2B+4A+D+B=-1; 2B+D+K=-1.
Solving the system, we obtain
I
D--
- 6'
Thus,
I= ( fx 2
- : x+ ! )V x + 2x + 2 + ; SV
2
x 2 +d;x + 2 ,
where

/ 1 =1 V dx =Sv dx =ln(x+1+Vx2 +2x+2)+C.


J x2 +2x+2 (x+ 1) 2 +1

5.4.4. I= SV 4x2 -4x + 3 dx.


Solution. Transform the integral to the form
l=S Y_ 4x2 - 4x+ 3
4x2 -4x+3
dx=(Ax+B)V-4x 2 -4x+3+K J dx
V4x 2 -4x+3
5.4. Methods of Integrating Irrational Expressions 227

Applying the method of undetermined coefficients, we get

l=(- x--{-)V4x -4x+3+5 y (2x-dx


1
2
2
n~+2
=
= ( +x-{) V 4x -4x+3 2 +{In (2x- l -1- Vr 4x2 -4x+3) + C.

5.4.5. 5 9x3 -3x 2 +2


Y3x 2 -2x+ I dx.

5.4.6. ~ Vx 2 +x+ I dx.


5.4.7. I= 5(x-l)(x+2)(x+4)Vx2+x+I
dx
2

Solution. Represent the given integral as follows:

5(x-l)(x+2)
(x+4)
2
f x+4
dx
Jlx2+x+I =. (x-l)(x+2) 2
dx
Jlx2+x+I'

.
E xpan d th e frac t 10n x+(x+
(x-I)
4 . t t 1 f t
2)2 mo par ta rac tons
x+4 A B D
(x-l)(x+2) 2 =x-I +cx+2) 2 + x+2
Find the coefficients

Hence,
/-5 [
-
5
9 (x-1)
_ 2
3 (x+2) 2
_ 5
9 (x+2)
J. dx _
Jlx2+x+ I -
55 dx 21 dx
=g- (x-1) Jfx2 +x+ I -3 J (x+2) 2Vx 2+x+ I
-9
5 5 (x+2)
dx
JI x2+x+ I

The first integral is calculated by the substitution x-- I=+, the


second and the third by the substitution x+2=+
We leave the solution to the reader.
5.4.8. 5 x3v6x2+ I Ix- 6 dx.
x2 +4x+3
5 4 9 5 3x;:5x 2 -7x+9 dx.
2x 2 +5x+7

5.4.10. 5 (x+ 1)6


~x2 +2x .
228 Ch. V. Basic Classes of Integrable Functions

5.4. t 1. s x~
(x2 -3x+2) x2 -4x+3

5.4.12. S(x+ dx
1) 3 V xL\-3x+2
5.4.13. sx Yl+3x +x'
(x2- I) dx
2

5.5. Integration of a Binomial Differential


The integral ~ xm (a+ bxn)P dx, where m, n, p are rational num-
bers, is expressed through elementary functions only in the follow-
ing three cases:
Case I. p is an integer. Then, if p > 0, the integrand is expanded
by the formula of the Newton binomial; but if p < 0, then we
put x= fk, where k is the common denominator of the fractions m
and n.
Case II. m+ 1 is an integer. We put a+ bxn = t 0 , where a is
n
the denominator of the fraction p.
Case Ill. m+I +Pis an integer. We put a+bxn=taxn, where a
n
is the denominator of the fraction p.
5.5. t. / = ~ Vx (2 +Vx) 2 dx.

Solution. I=~ xT(2+x+ rdx. Here p = 2, i.e. an integer; hence,


we have Case I.

/= SxTI( x+4x2I +4 ) dx= S(4 5 I)


xT +4x6 +4x3 dx=
3 24 .!..!. ~ .2_
=yx3 +nx +3x3 +c. 6

5.5.2. I= sx-f(1+x~ f 1
dx.
r-if1+Vx
5.5.3. I= J Vx 2 dx.

Solution. I= Sx - ~ ( 1+xi-)+ dx.


2 I I m+I
Here m=-3; n= 3 ; p= 2 ; -n-=
(-;+i)
1 =l, i.e. an
3
integer.
5.5. Integration of a Binomial Differential 229

We have Case II. Let us make the substitution

Hence,

I= 6 st 2 dt = 2t 3 + C = 2 ( l + x f) : + C.
5.5.4. I = sxT(2 + x{- )+dx.
5.5.5. f = S x ( l + x2 ) ~ dx.
5.5.6. l=Sx- 11 (1+x 4 )-+dx.
.
S ol u t ton. H ere p = - I 1s m+I = -11+1 = - 5 a Iso
. a f rac tion, -n-
2 4 2
a fraction, but m~- p = - ~ - ~ = - 3 is an integer, i.e. we have
I+
Case Ill. We put l+x 4 =x 4 t 2 Hence
I t dt
X= I ; dX=- 5
(t2-l)4 2 (t2-l)T
Substituting these expressions into the integral, we obtain

I I .!...!. ( t 2 ) I t dt
/=-2J(t2-l)4 /2-J -2 ~
(t2-J) 4

=-2I s t t t
(t2-1)2dt=-10+3-2+C.
0 3

Returning to x, we get

t =- 10~10 Vo +x 4) 0 + ! Vo +x
3 6 4) 3 -
2; 2 J/ l +x4 + c.

5.5.8. sx(I+dvxr
I
5.5.9. Sx 3 (l +x2 )2 dx.
(' dx
5.5.10. I .r .
J x4 y I +x2
230 Ch. V. Basic Classes of Integrable Functions

5.5.11.

5.5.12.

5.6. Integration of Trigonometric and Hyperbolic


Functions
I. Integrals of the form
I=~ sinm xcosn xdx,
where m and n are rational numbers, are reduced to the integral
of the binomial differential
n-1
I=~ tm (l-t 2 ) -2- dt, t =sin x
and are, therefore, integrated in elementary functions only in the
following three cases:
(1) n is odd (n 2 1 an integer),
(2) m is odd (mt 1 an integer),

(3) m + n is even (mt 1 +n-; 1 an integer).


If n is an odd number, the substitution sin X= t is applied.
If m is an odd number, the substitution cos x = t is applied.
+
If the sum m n is an even number, use the substitution
tanx=t (or cotx=t).
In particular, this kind of substitution is convenient for integrals
of the form
~ tann x dx (or ~ cotn x dx) ,
where n is a positive integer. But the last substitution is inconve-
nient if both m and n are positive numbers. If m and n are non-
negative even numbers, then it appears more convenient to use the
method of reducing the power with the aid of trigonometric trans-
formations:
cos 2 x = f
(1+cos2x), sin 2 x = f
(1-cos 2x)
. . 2
or smxcosx = 2I sm x.

5.6.1. I= S V coslx. x dx.


sin3
5.6. Integration of Trigonom. and Hyperbolic Funct's 231

Solution. Here m=3 is an odd number. We put cosx=f,


sin xdx= -dt, which gives
2 I 7
I=-S(l-t 2 )t-Tdt=-3t3 +; t3 +c=
=3 Vcosx( f cos x-l )+c.
2

5.6.2. I= s~~~:;dx.
5.6.3. /= ~ sin 4 xcos 6 xdx.
Solution. Here both m and n are positive even numbers. Let uis
use the method of reducing the power:
I= /6 S (2 sin xcosx) 4 cos2 xdx= 312 S sin' 2x(l + cos2x) dx = / 1 +1 2

The second of the obtained integrals is calculated by the substitu-


tion:
sin 2x= t, cos2xdx=; dt,

/ 2 = 312 S sin' 2x cos2xdx = 614 St' dt = 3~0 +c = 3 ~ 0 sin 2x+G.


We again apply to the first integral the method of reducing the
power:
/ 1 S
= 312 sin' 2x dx = 1 ~ 8 S ( l-cos4x) 2 dx=
= 1 ~ 8 ( x-+ sin 4x) + !
2 6 S (1 +cos8x) dx =

d
= 2 ~ 6 x- 2 !6 sin 4x+ 2 48 sin Bx+C.
And so, finally,
3 I . 4
I = 256x-256sm I . B
x+2048sm x
+320sm
I 5 2 +0
x

- -dx.
Scos
sin x
2
5.6.4. I= 6
x
Solution. Here both m and n are even numbers, but one of them
is negative. Therefore, we put
- 1-=l+t 2 ' ~=dt.
tanx=t' cos2x cos2x
Hence,
I=~ 12 (l+t 2 )dt=t~ +t~ +C=ta~sx+ta~5x+C.
232 Ch. V. Basic Classes of Integrable Functions

5.6.5. I= Scos 4x
-.--dJC.
sm x 2

Solution. Here we can put cot X= t, but it is simpler to integrate


by expansion:
I= S(l-sin2x)2
.
sm x
dx= S( -.-
2
I
sm x
--2+sin x ) dx= 11
2

=-cotx-2x+f S(1-cos2x)dx=

=-(cot x+ si~2x + 3;) + C.


5.6.6. I= sco~:x.
5.6.7. I= sV 3 dx
sin 11 x cos x

Solution. Here both exponents ( - 'i and - ~) are negative


numbers and their sum - ~1
fore we put
-+ = - 4 is an even number, there

5.6.8. Find the integrals of tan x and cot x.


Solution.
Stanxdx= S-dx=-lnlcosxl+C;
sinx
cos x

Scot x dx= Sc?s


smx
x dx= In I sin x I+c.

5.6.9. I=~ tan 7 xdx.


di
Solution. We put tanx=t, x=arctant; dx= 1+ 12 We get
I= st' 1 ~ t2 = s( f&
t 3 t - 1~ 12 ) dt =
fb -

t' 12 I
+
=5-4+2-2ln(l +tz)+C=
I I I
= 6 tan 6 x- 4 tan 4 x+ 2 tan 2 x+ In! cosxj +c.
5.6. Integration of Trigonom. and Hyperbolic Funct's 233

5.6.lO. (a) I=~ cot 6 xdx; (b) /= ~ tan 3 xdx.

5.6.11. I= S-.-
cos x
4
3-
Sin X
dx.
Solution. Here sin x is raised to an odd power. Let us put
cosx = t, - sin xdx= dt.
We obtain an integral of a rational function.

I= Scossin4x
4 x sin x s
dx=- (l-t2)2dt.
14

Here, it is simpler to integrate by parts than to use the general


methods of integration of rational functions (cf. Problem 4.4.1 (b)).
Let us put
t dt
dv = (l-t2)2 .
Then
du= 3t 2 dt; V = 2 (I ~t 2 )

Hence,

= - -2 x-
cos
- -2sin
3
3 sx
co +cos x I+C
+ -34 In 111-cosx
x 2
4x
5.6.12. I= Ssin-dx.
cos x
I I. Integrals of the form ~ R (sin x, cos x) dx where R is a rational
function of sin x and cos x are transformed into integrals of a rational
function by the substitution:

tan ( ~) = t (--:rt< x <:rt).

This is so-called universal substitution. In this case


. 21 l-t 2
Sin X = f=t=/2; COSX= l+t 2 ;

2dt
x = 2 arc tan t; dx= 1+12.
234 Ch. V. Basic Classes of Integrable Functions

Sometimes instead of the substitution tan ~ = t it is more advan-


tageous to make the substitution cot ~ = t (0 < x < 2n).
Universal substitution often leads to very cumbersome calculations.
Indicated below are the cases when the aim can be achieved with
the aid of simpler substitutions:
(a) if the equality
R(-sinx, cosx)=-R(sinx, cosx)
or
R(sinx, -cosx)=-R(sinx, cosx)
is satisfied, then it is more advantageous to apply the substitution
cos x = t to the former equality, and sin x = t to the latter;
(b) if the equality
R(-sinx, -cosx)=R(sinx, cosx)
is fulfilled, then a better effect is gained by substituting tanx = t
or cot x= t.
The latter case is encountered, for example, in integrals of the
form ~ R (tan x) dx.

5.6.13. I= Ssm. x (2+ cosdx x-2 sm. x) .


Solution. Let us put tan ~ = t; then we have

I= ~ ~ ITT -~) = c ( +
I+t 2
( I-t 2
2+1+t 2 1+t 2
1 f2) dt
j t (t 2 -4t+3)

Expand into simple fractions


1+t 2 -.'.i+~+__Q_
t (t-3)(t- I) - t t-3 t-1 '
Find the coefficients
l 5
A=3; B=- D=- l.
3 '
Hence
I = }__
3
s +~ s
dt
t 3
_!!!_ _
t-3
s __!!!__
t-1
=
I 5
=3lnlt1+ 3 In I t-31-lnl t-1 l+C =

=f In j tan~ J +~ In J tan~ -3 j-In j tan~ -1 J + C.


5.6. Integration of Trigonom. and Hyperbolic Funct's 235

5 6 14 I= 55+sinx+3cosx
dx

5.6.15. I= ssm. ( cos


x 2 d\ x- l)
Solution. If in the expression smx 12
. (2 cos x- I) we substitute
-sinx for sinx, then the fraction will change its sign. Hence, we
take advantage of the substitution t=cosx; dt=-sinxdx. This
gives
(' dt
I= - J (l-t 2) (2t 2 - l)
Since
(2-2t2)-(l-2t2) 2 1
( l - t 2) (l -2t 2) (l-t 2) (l-2t2) = l-2t'2- l-t2
then
r dt r
I =2 j l-2t2-J l-t2= 2ln
dt 1 ll+tV2I
1-t
1
V2 -2 In 1 1-t 1 +C=
1+1

I
= _1_ In 11 + V2 cos x + _!_ In 11 - cCJs x + C =
Jl2 I - V 2 cos x 2 1+cos x
I
=~lnl 1+ 2 cosxj+tnjtan~l+c.
V2 I- V2 cos x 2
5.6.16. l=S smx+cos
~in2xcosx dx.
x
Solution. Since the integrand does not change sign when sin x
and cos x do change their signs, we take advantage of the substi-
tution
dx
t= tanx; dt=-2-.
cos x
Hence,

I= Stan2 Xcos' x dx s t2dt


(tanx+ I) cos2x = (t+ I) (t 2+ 1) 2

Expand into partial fractions


t2 A Bt+D Et+F
(t-l-l)(t~+1)2 t+I + 12+1 +(12+1)2
Find the coefficients
I I I I 1
A=4; B=-4; D=4; E=2; F=-2
Hence,
236 Ch. V. Basic Classes of Integrable Functions

I I+t
4. 1+1 2 +C=

={-In I sin x+ cosx I-! cosx(sin x+cosx)+C.

5.6.17. I= Ssm. 2 xtanx+3


2 + 2 cos x dx. 2

Solution. Dividing the numerator and denominator by cos2 x and


substituting tanx= t; cos d~ x =dt, we obtain
dx
S 2tanx+3 5<
2 tanx+ 3> cos2x
I= sin 2x+2 cos2 x dx= tan 2 x+2 =
r2t+3 3 t
= j +
12 + 2 dt =In (t 2 2) + 2" arc tan y-2 +c =

tan x + C.
-_ In (t an2 x+ 2)+y-32 arctany- 2
5.6.18. I= SI+.
sin x
smx dx.
Solution. This integral, of course, can be evaluated with the aid
of the universal substitution tan ; = t, but it is easier to get the
desired result by resorting to the following transformation of the
integrand:
sin x sin x (I-sin x) sin x(l-sinx)
I+sin x (I+ sin x) (I-sin x) = cos 2x
sin x sin 2 x sin x t 2
= cos2x - cos2 x = cos2 x - an x.
Whence
I= j"-cos
sin 2-dx-
x
x
ssec2xdx+ s I
dx=---tanx+x+C.
cos x
s.6.19./=S cos 4 x1sm
2 dx.
x
Solution. Here the substitution tanx = t can be applied, but it
is simpler to transform the integrand. Replacing, in the numerator,
unity by the trigonometric identity raised to the second power, we
get
I= s(sin 2 x+ c~~ 2 x) 2dx =
cos4 x sm 2 x
s sin 4 x+ 2 sin 2 x c?s_2 x+ cos4 x dx =
c<-s4 x sm 2 x

= Ssin2
x
cos4xdx--j-2 scos2x
dx + s dx
sin2x = s dx
tan2x cos~x+2tanx-cotx=
I
= 3 tan 3 x+2 tanx-cot x+C.
5.7. Integrating Irrational Functions by Substitutions 237

III. Integration of hyperbolic functions. Functions rationally de-


pending on hyperbolic functions are integrated in the same way as
trigonometric functions.
Keep in mind the following basic formulas:

cosh 2 x-sinh 2 x = 1; sinh 2 x = ~ (cosh 2x -1);

cosh 2 x = + (cosh 2x + 1); sinh x cosh x = + sinh 2x.

x
If tanh 2 =t, then sm x= 1_t 2
. h 2t .
,
I+ t 2
cos h X=J-t2'

x=2Artanht=ln(:~~) (-l<t< 1); dx= 12:!!, 2

5.6.20. I= ~ cosh 2 x dx.


Solution.
I= 5+(cosh 2x+ 1) dx= -}sinh 2x+ ~ x+C.

5.6.21. I= ~ cosh 3 x dx.


Solution. Since cosh x is raised to an odd power, we put sinh x = t;
coshxdx=dt. We obtain

I= Scosh 2 x cosh x dx = 5(1 + t 2) dt = t + 1; + C=

=sinhx+ ~ sinh 3 x+C.


5.6.22. Find the integrals:
(a) 5sinh x cosh x dx;
2 2 (b) 5 sinh x+dx2 cosh x

5.7. Integration of Certain Irrational Functions with the


Aid of Trigonometric or Hyperbolic Substitutions
Integration of functions rationally depending on x and Vax 2 +bx+ c
can be reduced to finding integrals of one of the following forms:

I. ~ R (t, V p 2 t 2 +q 2 )dt;

II. ~ R (t, Vp 2 t 2 -q 2 ) dt;


III. JR(t, Vq 2 -p 2 t 2 )dt,
238 Ch. V. Basic Classes of Integrable Functions

b
where t = x + 2a; ax 2 +bx +c = + p 2 t 2 q2 (singling out a perfect
square).
Integrals of the forms I to III can be reduced to integrals of
expressions rational with respect to sine or cosine (ordinary or hy-
perbolic) by means of the following substitutions:
I. t=.i..tanz or t=Lsinhz.
p p

I I. t = ; sec z or t = ; cosh z.
II I. t = .i.. sin z or t = _!l_ tanh z.
p p

5.7.1. I= I dx
., Y(5+2x+x 2 ) 3
.

Solution. 5+2x+x 2 =4+(x+ 1) 2 Let us put x+ l = t. Then

We have obtained an integral of the form I. Let us introduce


the substitution:
2 d~
t=2tanz; dt= cos~ ; 2-.
V4+t 2 =2Vl +tan2 z =cos
-
z z
We get

I= ! scos z dz =
t
. Z + C = - I -----;::===
= -4I Stn 4
tanz +C---1
Yl+tan z
2 4 y2 12
1+-
4
+ C--
x+I +C.

5.7.2. '=S (x+ 1)2


dx
Y x2 +2x+ 2

4 Vs+2x+x 2

Solution. x 2 + 2x+ 2 = (x+ 1) 2 +1.


Let us put x + l = t; then
1-J
- t2 -V1dt2 +1
Again we have an integral of the form I. Make the substitution
t = sinh z. Then
dt = cosh z dz; Vt 2 + 1 = Vl +sinh 2 z =cosh z.
5.7. Integrating Irrational Functions by Substitutions 239

Hence,

I= s cosh z dz
sinh" z cosh z
= s ~ = - cot h z
sinh 2 z +C =
JI 1-:sinh2 z +C= VT+t2 +C=- Vx 2 +2x+2 , C
smh z t x+ l T

5.7.3. l=~x 2 Vx 2 -Idx.

5.7.4. I =S VX2+T d
x2 X.

5.7.5. l=~V(x 2 -l) 3 dx.


Solution. Perform the substitution:
x =cash t; dx = sinh t dt.
Hence

I= ~V(cosh 2 t-1) 3 sinh t dt = ~ sinh 4 t dt =


= s( ~t dt
cosh - I) 2 =

= ! Scosh 2t dt -~ Scosh 2t dt + i- Sdt =


2

= ~ S(cosh4t+ I)dt-+sinh2t+i-t=
= 3~ sinh 4t -+ sinh 2t + ! t + C.
Let us return to x:

t =Arcoshx= In (x + Vx 2 -1);
sinh 2t = 2 sinh t cosh t = 2x v-x 2 - 1;
sinh 4t = 2 sinh 2t cash 2t = 4x i1 x2 - I (2x 2 - l).
Hence

5.7.6. I=
.)l (I+ V x~xVx-xJ..
Solution. We make the substitution:
x = sin2 t; dx = 2 sin t cost dt
240 Ch. V. Basic Classes of Integrable Functions

and get
I= s 2 sin t cos t dt
o+sint) Ysin 2 t-sin 4 t
s 2 dt
I+sint
=

=2 Sl-sintdt
cos2
=2 tan t - -2-+C =
t cos t

= ;v-x 1-x
- . r 2 +c=2(~1) +c.
r 1-x 1-x

5.7.7. I=~ V3-2x-x 2 dx.

5.7.8.

5.8. Integration of Other Transcendental Functions

5.8.1. I= s':t dx.


Solution. We integrate by parts, putting
u= lnx; dv=-
dx
x2'

du=x;
dx V=-x;
I

I = - ~+sdx=-~-_!_+c.

5.8.2. I= s In x dx
Vi-x
2 x x x x

5.8.3. / = S(l+e
exdx
xp.
2

Solution. Let us put: eX=t; eXdx=dt. We get:

I= S(I +12)2.
di

Apply the reduction formula (see Problem 4.4.1):


t I (' dt
1=12= 2(t2+J) +2 j 1+1 2 ;

t I ex I
I= 2 (t 2 + I) +2 arctan t + C 2 (I +e2 x) +2 arctanex + C.
5.8.4. I=~ e-x In (eX + l) dx.
5.8. Integration of Other Transcendental Functions 241

Solution. We integrate by parts:


u =In (tr+ l}; dv=e-x dx;
ex
du=i+exdx; V=-e-x;

I = - e-" In (l +tr)+ Si!:x =- e-" In (l +tr}+ Se"i~;e" dx =


= -e-x In (l +tr} +x-ln (l +tr}+ C.

5.8.5. I =
r
J
ea arc tan x
3 dx.
(I+ x2) 2

586 I- r x arc tan x dx


... -_, v1+x2

Solution. Integrating by parts, we get


xdx
u =arctanx; dv=v
I +x2 ;
dx
du= 1+x2; V= Vl +x 2;

I= V-l+x 2 arctanx- sv- l +x i+xz


2 dx =

= V-1 + x2 arc tan x - In (x + Vx2 + l) + C.


242 Ch. V. Basic Classes of Integrable Functions

5.9. Methods of Integration


(List of Basic Forms of Integrals)

No. , Integral
I Method of integration

I ~ F [<p (x)] <p' (x) dx I Substitution <p (x) = t


I I
2 ~ f (x) <p' (x) dx Integration by parts

~ f (x) <p' (x) dx = f (x) <p (x)- ~ <p (x) f' (x) dx.
This method is applied, for example, to
integrals of the form ~ p (x) f (x) dx, where
p (x) is a polynomial, and f (x) is one of the
following functions:
ex; cos ax; sin ax; In x;
arc tan x; arc sin x, etc.
and also to integrals of products of an expo-
nential function by cosine or sine.

3 ~ f (x) q,<n) (x) dx Reduced to integration of the product


f<n> (x) <p (x) by the formula for multiple in-
tegration by parts
~ f (x) q;<n> (x) dx= f (x) <p<n-l> (x)-
-f' (x) <p<n -2) (x) + f" (x) <p<n - :11 (x)- .
...+ (- l)n-lf<n-11 (x) <p (x) +
+ (- l)n ~ f<n> (x) <p (x) dx

4 ~ ex Pn (x) dx, Applying the formula for multiple integra-


tion by parts (see above), we get
where Pn (x) is a polyno-
mial of degree n. ~ ex Pn (x) dx=
=ex [Pn (x) _ p~~x)
a a2
+ p~aa(x) _
+
+(-1)" Phm(x>J+c
an+l

5 Sx2+px+q
Mx+N d
x,
Substitution

p2 -4q <0
5.9. Methods of Integration (List of Basic Integrals) 243

No.1 Integral Method of Integration


I
6 I
n-
-5 dx
(x2+ l)n
Reduction formula is used
x 2n-3
In= (2n-2) (x2 + l)n-1 + 2n --2 In-I

7 SP(x) P(x) Integrand is ex pressed in the form of a


Q (x) dx, where Q (x)
sum of partial fractions
is a proper rational frac- P(x) A1 A2 A1
ti on Q (x)= (x-xi) +(x-x1)2+ +(x-x + 1)l
Q (x) = (x-x1 ) 1 (x-
-X2)m ... (x 2+px+ + (X-X2)
Br + B2 + + Bm +
(X-X2) 2 . . . (X-Xz)m
+q)k ... M 1 x+N 1 M 2 x+N 2
+ + x 2+px+q + (x 2 +px+q) 2 +
+ ... + Mkx+ Nk + ...
(x2+px+q)k

8 Reduced to the integral of a rational frac-


JR (x, x:, .. ., x+)dx, tion by the substitution x=tk, where k is a
where R is a rational func- common denominator of the fractions
tion of its arguments.
m r
- , ... , -
n s

9
s R [ x, (;:~~)*] dx, Reduced to the integral of a rational frac-
tion by the substitution
where R is a rational ax+b =tn
function of its arguments. cx+d

IO s Mx+N d
Vax 2+bx+c x
By the substitution x+ 2~=t the integral
is reduced to a sum of two integrals:

s Mx + N
Vax 2+ bx+c
dx = Mi s tdt
Vat 2 +m
+
+N
1
s dt
Vat 2 -t-m
The first integral is reduced to the integral
of a power function and the second one is a
tabular integral.
244 Ch. V. Basic Classes of Integrable Functions

No.1 integral Method ol integration

11 ~ R(x, Vax 2 +bx-t-c)dx, Reduced to an integral of rational fraction


by the Euler substitutions:
where R is a rational
function of x and Vax 2 +bx+c=t x Va (a> 0),
Vax 2 -j-bx+c Jfax 2 +bx+c=tx Ve (c > 0),
Vax 2 +bx+c=t (x-x1 ) (4ac-b 2 < 0).
where x 1 is the root of the trinomia 1 ax 2 +
+bx+c.
The indicated integral can also be eva lua-
ted by the trigonometric substitutions:
yiJC"TaC
2a
. t
Sin

Vb2=4ae
2a cos t (a < 0,
4ac-b 2 < 0)

yiJC"TaC
a sec
t
2
Yb 2 -4ac
2a coscc t (a> 0,
4ac-b 2 < 0)
V:rac=b2
a tan t
2
y4liC=b2
2a cot t (a > 0,
4ac-b 2 > 0)

12 SJf axPn+bx+c
2
(x) dx, Write the equality

where Pn (x) is a pol yno-


mial of degree n.
SJf axPn +bx+c
(x)dx
2
Qn-dx) Jfax 2 +bx+c+

+kS dx
Jf ax 2 +bx+c
where Qn-l (x) is a polynomial of degree
n-1. Differentiating both parts of this equa-
lity and multiplying by Vax 2 +bx+c, we
get the identity
Pn (x) = Q~-1 (x) (ax 2 +bx+c) +
I
+2 Qn-1 (x) (2ax+b)+k,
which gives a system of n+ 1 linear equa-
tions for determining the coefficients of the
polynomial Qn-l (x) and factor k.
5.9. Methods of Integration (List of Basic Integrals) 245

No.1 Integral Method of lnt(!gration

And the integral

SY ax +bx+c
dx
2
is taken by the method considered in No. JO
(M=O; N=I).

13 This integral is reduced to the above-con-


sidered integral by the substitution
I
X-X1=y

14 ~ xm (a+ bxn)P dx, This integral is expressed through elemen-


tary functions only if one of the following
where m, n, p are rational conditions is fulfilled:
numbers (an integral of a (I) if p is an integer,
binomial differential).
m+ I 1s. an m. t eger,
(2) 11 -n-

rm+!+ p 1s
(3) 1 -n-
an m t eger.

/st case
(a) if p is a positive integer, remove the
brackets (a+bxn)P according to the Newton
binomial and calculate the integrals of powers;
(b) if p is a negative integer, then the
substitution x=tk, where k is the common
denominator of the fractions m and n, leads
to the integral of a rational fraction;
2nd case
if m+
n
1 is an integer, then the substitu-
tion a+bxn=tk is applied, where k is the
denominator of the fraction p;
3rd case
+
if m !+P is an integer, then the substi-
n
tu ti on a+ bx11 = xntk is applied, where k is
the denominator of the fraction p.

15 ~ R (sin x, cos x) dx Universal substitution tan ~ =t.


If R(-sinx, cosx)=-R(sinx, cosx),
then the substitution cos x = t is applied.
I.f R (sin x, -cos x) = - R (sin x, cos x),
then the substitution sin x= t is applied.
If R (-sin x, -cos x) = R (sin x, cos x),
'then the substitution tan x = t is applied.
246 Ch. V. Basic Classes of Integrable Functions

No. j Integral Method of Integration

~ R (sinh x, cosh x) dx x
16 The substitution tanh 2 = t is used. In
this case
. h
Sill
2t
X = l - t 2 ; COS
h I
X=l-t 2 ;
+t 2 d 2dt
X=l-(Z'

17 ~ sin ax sin bx dx Transform the product of trigonometric


functions into a sum or difference, using one
~ sin ax cos bx dx of the following formulas:
sin ax sin bx=
~ cos ax cos bx dx I
= 2 [cos (a-b) x-cos (a-1-b) x)

cos ax cos bx=


1
=2 [cos (a-b) x+cos (a+b) x)

sin ax cos bx=


={[sin (a-b) x+ sin (a+b) x)

18 I ~ sinm x cosn x dx, If m is an odd positive number, then apply


the substitution cos x = t.
where m and n are inte- If n is an odd positive number, apply the
gers. substitution sin x=t.
If m + n is an even negative number, apply
the substitution tan x=t.
If m and n are even non-negative numbers,
use the formulas
sin 2 x= l-cos2x , COS"X=
., 1 +cos2x
2 2

19 ~ sinP x cosq x dx Reduce to the integral of the binomial


differential by the substitution sin x=t
(0 < x < n/2),
p and q- rational num-
~ sinP xcosq xdx= ~ tP (l-t 2)q-I dt
bers. (see No. 14).

20 Transform into an integral of a rational


function by the substitution eax = t
Chapter 6
THE DEFINITE INTEGRAL

6.1. Statement of the Problem.


The Lower and Upper Integral Sums
Let a function f (x) be defined in the closed interval [a, b]. The
following is called the integral sum:
n-1
In=~ f (Si) llxi,
i=o

where a=X 0 < X < X < .. < Xn-i < Xn=b,


1 2

!lxi=Xi+i-xi; S;E[xi,xi+il (i=O, 1, ... ,n-1).


n-1
The sum Sn= ~Mi !lxi is called the upper (integral) sum, and
i=O
n-1
sn = ~ mi !lxi is called the lower (integral) sum, where Mi=
i=O
=sup f (x) [mi= inf f (x)] for x E [xi, xi+il
The definite integral of the function f (x) on the interval [a, b] is
the limit of the integral sums
b n-1
~ f (x) dx =Jim .~ f (Si) !lxi when max I !lxi 1--r 0.
a 1=0

If this limit exists, the function is called integrable on the inter-


val [a, b]. Any continuous function is integrable.
6.1.1. For the integral
n
~sin xdx
0

find the upper and lower integral sums corresponding to the division
of the closed interval (0, n] into 3 and 6 equal subintervals.
248 Ch. VI. The De-finite Integral

Solution. Divide the closed interval [O, n] into 3 equal parts by


the points:
11 211
X0 =0, X1 =3 1 X2 =3 1 X3 =:it.

The function sinx increases monotonically on the interval [ 0, -i-] .


and therefore for this interval we have m0 =sin0=0, M 0 =
i-
=sin = ~3 . The least value of the function on the interval

l3 , 32n]
.n m1 =sm
ts 3 an d the grea t est va 1ue 1s
3n = -V2-, M1 =

=sin~= I. On the interval [ 2; , n] the function sinx decreases


monotonically and therefore
m2 =Sinn=0,
11
Since all tuk are equal to 3 ,

~ !!l.xk= 11 y3
O+- ) 11 y3
2-+ O = -
(
s3 = ....mk 3 6- ~o.907,
k=O

Sa= i:
k=o
Mk !!l.xk = ~ (~+I+ ~3) = 11 (V~ +I) ~ 2.86.
When subdividing the closed interval [O, n] into 6 equal intervals
11 11 11 211
by the points X0 =0, x1 = 6 , x2 =3. X3 =2 x,=3, Xs =
= 6,
511 l
X 6 = n, we find by ana ogy:

. 11
M -sm--- l
m0 =0, 0- 6 - 2'
11 l . 11
M i=Sln y3
m1=Sln6=2 3=-2-,
11 V3
m2 = sm3 =-2-,
. 2't y3
m 3 =Slll3=-2- , M 3 =sin ~ =I,

m4 =sm 6
511 l
M,=stn 3
211 y3
= 2 , = -2- ,
. 511 I
M 5=Sin5=2
m5 =sinn=0,
6.1. The lower and Upper Integral Sums 249

For this division we obtain

S5
1t
= "6 v- 1t
(mo+ m1 + +m 0 )=5(1+ 3)~ 1.43,

Sa =5 (Mo+ M1 + ... +M )=5(3+ v-


1t
3) 0
1t
~ 2.48.

As would be expected, the inequalities


n

S3 ~ s6 ~ \ sin x dx ~ s6 ~Sa
,)
0

hold true (the exact value of the integral is equal to 2).


6.1.2. At what 6 > 0 does the relation

I~ sinxdx- ~1 sinsk~xkl < 0.001


follow from the inequality max ~xi< 6.
Solution. Since sn < In <Sm then for the required inequality to
hold true it is sufficient that the upper and the lower integral sums
differ by less than 0.001:
0 <Sn-Sn< 0.001.
But
n-1 n-1
Sn -s n = ~ z ~X1 < c5 ~
~ (M i.-m.) ~ (M 1-tn)
, ,
i=o i=o

where Mi and mi are the greatest and the least values of the func-
tion sin x on the interval [xi, xi+ 1] (i = 0, 1, ... , n-1). Assuming
for simplicity that the point ; is chosen as one of the points of
division and taking advantage of monotonicity of the function sin x
on the intervals l 0, ; J and [ ~ , n J , we obtain
n-1
~(Mi-mi)=2(sin ~ -sin0)=2.
1=0

Consequently, the required inequality is satisfied if 2c5 < 0.001,


i.e. c5 < 0.0005.
6.1.3. Show that the Dirichlet function [see Problem l.14.4 (b)]
is not integrable in the interval [O, l].
Solution. In dividing the closed interval [O, l] into a fixed num-
ber of parts we must take into consideration, in particular, two
possible cases: (1) all points Si are rational; (2) all points Si are
250 Ch. VI. The Definite Integral

irrational. In the first case the integral sum is equal to unity, in


the second to zero. Hence, no matter how we reduce the maximum
length of subintervals, we always get integral sums equal to unity
and integral sums equal to zero. Therefore, the limit of integra 1
sums is non-existent, which means that the Dirichlet fund ion is
not integrable on the interval [O, l].
6.1.4. Find the distance covered by a body in a free fall within
the time interval from t =a sec to t = b sec.
Solution. A body moves in a free fall with constant acceleration g
and initial velocity v0 = 0. Consequently, the velocity at the instant t
is equal to the velocity increment within the time interval from 0
to t, i. e. v (t) = ~v. For a short time period M the velocity incre-
ment is approximately equal to the acceleration at the instant t
multiplied by M. But in our case acceleration is constant, there-
fore ~v =gM, and hence, v (t) =gt, since M = t-0 = t.
Let us subdivide the time interval from t=a to t=b into n
equal parts; then the duration M of each subinterval will be equal
to M = b-a. We assume that during each subinterval of time the
n
body moves uniformly with a velocity equal to its velocity at the
beginning of this interval, i.e.

V0 =ga,
b-a) ,
v1 = g ( a+ 1 -n-

V2
b-a) ,
=g ( a+2 -n-

Vn-l =g [a+(n-1) b n a].

Whence we find the distance covered by the body during the ith
V (b-a)
subinterval: ' . The entire distance covered by the body is
n
approximately equal to
b-a
S;::::: Sn =-n- (v0 + V1 + ... + Vn_ 1 ) =
=b-a
-
n
g [ n a +b-a
n
b-a
l-+2n
b-aJ
- + ... +(n-l)-
n
=

-(b
- -a ) g [ a + ""112
b-an{n-1)]
2

With n increasing the distance covered can be evaluated more accu-


6.1. The Lower and Upper Integral Sums 251

rately. The exact value of s is found as the limit Sn as n _,. oo:


s= lim sn = Jim g (b-a)
n-+r n-+
[a+ ~ (b-a) ( 1 _ _.!._)] =
n

=g(b-a) [a++(b-a)J =~ (b 2 - a2 ).

Since sn is an integral sum


n-1

Sn=LVidfi
i=o
the distance s is an integral:
b b

s= Svdt= Sgtdt=; (b -a 2 2 ).

a a

6.1.5. Proceeding from the definition, compute the integral


1

~ xdx.
0

Solution. By definition,
1 n-1

~ xdx= lim .~ i dxi as max dxi _,. 0,


o t=O
where
0=X0 <X 1 < ... <Xn=1, SiE [xi, Xi+ 1 ],
dXi=Xi+i-Xi.
1. Subdivide the closed interval [O, 1] into n equal parts by the
points xi=~ (i=O, 1, 2, ... , n).
I I
The length of each subinterval is equal to dxi = -n , and -n _,. 0
as n-... oo.
Let us take the right-hand end-points of the subintervals as the
points Si: si=Xi+i=i~l(i=O, 1, ... , n-1).
Form an integral sum:
n-1
I =S
n
=L i+1._.!_=_.!_
n i=o n n n2 (1+2+ ... +n)=2T-
n (n+ I)

As n - oo the limit of this sum is equal to


. n+I I
hm -2-=-2.
n- e n
252 Ch. VI. The Definite Integral

Hence,
I

Sxdx= ~.
0

2. Using this example, we will show that for any other choice
of points ~i the limit of the integral sum will be the same.
Take, for instance, the mid-points of the subintervals as si: Si =
i +..!..
=--2 (i=O, 1, ... , n-1).
n
Form an integral sum
n-l
~ 2i
In = ~ 2n . n =
+l l l [
2n2 1 + 3 + 5 + ... + (2n- l) Jl = 2n2
4n2 =
l
2 .
i:O

Hence

6.1.6. Proceeding from the definition, compute the integral:


b

~ xmdx (m=l=-1, 0 <a< b).


a

Solution. In this example the following points can be conveniently


chosen as points of division:

Xo =a; X1=a ( : r ,.. ,I

Xi= a ( : )
I

n , , Xn =a (: r
"

=' b.

They form a geometric progression with the common ratio

q=(!)n>l.
The length of the ith subinterval is equal to
tlxi= aqi+1_aqi = aqi (q-1).
Therefore the maximum length of the subintervals equals max tlxi =
fl-I I

= aqn-1 (q- 1) =a ( !) n [( ~) n -1 J and tends to zero with


increasing n, since lim q = 1.
fl -+ 00

Now let us choose the right-hand end-points of the subintervals


as Si: Si= xi+i = aqi+i (i = 0, 1, 2, ... , n-1).
6.1. The Lower and Upper Integral Sums 253

Form an integral sum:


n-1 n-1
ln= ~ srl1xi= "!iamq<i+umaqi(q-l)=
i=O i=O
= am+1 (q-1) qm [l qm+1 + + ... +
q<n-1) <m+t>] =
q<m+l)n_ I q-1
= am+1 (q- l)qm
qm+l-l
= (bm+t_am+l) qm --'-...,..--
qm+l-(

Let us calculate the limit of the integral sum as max Axi - 0,


i.e. as q- 1:
Jim I = (bm+1_am+1) lim qm q-1 = (bm+1_am+1) _1_.
n q ... J qm+l_I m+I
Thus,
b

Sxmdx=-1-
a
m+l
(bm+1_am+1)

6.1.7. Proceeding from the definition, compute the integral:

sd:.
I
2

Solution. Subdivide the interval [l, 2] into n parts so that the


points of division xi (i = 0, 1, 2, ... , n) form the geometric pro-
gression:

whence q = -j12.
The length of the ith subinterval is equal to
dXi = qi+t_qi =qi (q- 1),
and so maxl1xi=qn- 1 (q-l)-O as n-oo, i.e. as q - l.
Now let us choose the right-hand end-points of the subintPrvals
as the points Si i.e., Si= xi+i = qi+i.
Form an integral sum:
n-1 n-1
n
(
n1 )
In = L T.'"I dXi = L-., -iq I+-
i=O "'' i=O
1

q' (q- 1) = - (q -
q
l) = -
n
I
1 n 2 - 1
2

. I n= 1Im n (
1tm 2,i-l-I) = In 2,
n-+ex> n-oo

I
- I
since 2n -l,....,,-ln2 as n-+oo.
n
254 Ch. VI. The Definite Integral

And so,
2

Sdx-x=ln2.
I

6.1.8. Evaluate the integral


5
I=~ V25-x2 dx,
0
proceeding from its geometric meaning.
Solution. The curve u=V25-x2 is the upper half of the circle
x~ +y
2 = 25. The portion of the curve corresponding to the variation

of x from 0 to 5 lies in the first quadrant. Hence, we conclude


that the curvilinear trapezoid bounded by the lines x = 0; x = 5;
y = 0, and y = V25-x2 is a quarter of the circle x2 + y2 = 25; and
25n
its area is equal to - 4- .
Hence,
5

I= SV25-x2dx= 2~n.
0

6.1.9. Evaluate the integral, proceeding from its geometric


meaning:
5

I=~ (4x-l)dx.
I
6.1.10. Prove that

Solution. The integral


!J A x
I= ~ V a 2 -x2 dx
0

expresses the area SoAMx of the por-


tion of a circle of radius a lying in
the first quadrant (see Fig. 59).
This area equals the sum of the
areas of the triangle OMx and the
sector OAM.
xy x v-2--2
S 01ux=2=2 a -x.
Fig. 59
6.1. The Lower and Upper Integral Sums 255

The area of the sector


- I 2/
S OAM-2a '

where sin t = _.:...


a
Hence,
a2 x
SaAM= 2 arcsma,
and consequently,

6.1.11. Proceeding from the geometric meaning of the integral,


show that
2:rt I I

(a) ~ sin 3 xdx=0; (b) ~ e-x 2 dx=--= 2 ~ e-x 2 dx.


0 -1 0

Solution. (a) The graph of the function y = sin 3 x is shown in


Fig. 60. Let us show that the area situated above the x-axis is
equal to that lying below this axis. Indeed, let :rt~ x ~ 2:rt, then
x=n+x1 where O~x 1 ~:rt and sin 3 x=sin 3 (:rt+x1 )=-sin 3 x 1
Therefore, the second half of the
graph is obtained from the first one
by shifting it to the right by :rt
and using the symmetry about the
x-axis. Hence,
2:rt

~ sin 3 xdx= 0.
0 Fig. 60

6.1.12. Given the function f(x)=x 3 on the interval [-2, 3),


find the lower (sn) and the upper (Sn) integral sums for the given
interval by subdividing it into n equal parts.

6.1.13. Proceeding from the geometric meaning of the definite


integral, prove that:
:rt 2:rt

(a) ~ sin2xdx=O; (b) ~ cos 3 x dx = O;


0 0
2 3

(c) S (2x+ 1)dx=6; (d) SV9-x 2 dx= 9; .


I -3
256 Ch. VI. The Definite Integral

6.1.14. Passing to the limit from the integral sums, compute the
integral
4

I=~ x 3 dx,
I

by subdividing the interval [1, 4]:


(a) into equal parts;
(b) by points forming a geometric progression. In both cases
choose ~i as:
(1) left-hand end-points of the subintervals;
(2) right-hand end-points of the subintervals;
(3) mid-points of the subintervals [xi, xi+il

6.2. 'valuating Definite Integrals by the


Newton-Leibniz Formula
The following is known as the Newton-Leibniz formula:
b

~ f (x} dx= F (x) lb= F (b)-F (a),


a a

where F (x) is one of the antideri v atives of the function f (x), i.e.
F'(x)-f(x) (a::;;;;x::;;;;b).

6.2.1. Evaluate the integral


V3
I= S 1+x~ dx

Solution. Since the function F (x) =arc tan x is one of the anti-
derivatives of the function
formula we get
f (x) = 1 ;x 2 , using the Newton-Leibniz

V3
I= Sl+x
dx
2 =arctanx /Va
1 =arctan v-
3-arctanl= 3n - n n
4 = 12
I

6.2.2. Compute the integrals:


n n
2 2
(a) Ssin 2x dx; (b)
r sina
.l
cos x
x dx;
0 n
6
6.2. Newton-Leibniz Formula 257

6.2.3. Given the function


Ix 2 for 0 ~ x ~ I ,
f (x) = \Vx for 1 ~ x ~ 2.
2

Evaluate ~ f (x) dx.


0
Solution. By the additivity property of the integral
2 I 2 I 2

~ f (x) dx = ~ f (x) dx + ~ f (x) dx = ~ x dx + ~ Vx dx = 2

0 0 I 0 I
I 3 2 _

= ~
3
I + !3 x 21 = _!_ + 3
4 v3 2 - !3 = _!_
3
(4 v2 - I)
.
0 I

6.2.4. Evaluate the integral


2

I=\ I 1-x Idx.


0
Solution. Since
I 1_ x I = I I -- x for 0 ~ x ~ I ,
) x-1 for I ~ x ~ 2,
we obtain, taking advantage of the additivity property of the integral,
2 I 2

~ I I - x I dx = ~ (I - x) dx + ~ (x - I ) dx =
0 0 I
I 2
= - (I -x)2
2
I + (x-2 1)21 =__!__+_I_= l
2 2
0 I

6.2.5. Evaluate the integral


b

l=s~dx
x '
a
where a< b.
b

Solution. If 0 ~a< b, then f (x) = I; I =I, therefore ~ f (x) dx=


a
b

=b-a. If a<b~O, then f(x)=-1 and ~f(x)dx=-b-(-a)=


a
b
('

= a--b. Finally, if a< 0 < b, then divide the integral ~ f (x) dx


a
258 Ch. VI. The Definite Integral

into two integrals:


b 0 b

~ f (x)dx = ~ f (x) dx+) f (x) dx= b-(-a).


a a 0

The above three cases may be represented by a single formula:


b

S ~ Ldx =I b 1-1 a J.
1
a

Note. When evaluating integrals with the aid of the Newton-Leib-


niz formula attention should be paid to the conditions of its legi-
timate use. This formula may be applied to compute the definite
integral of a function continuous on the interval [a, b] only when
the equality F' (x) = f (x) is fulfilled in the whole interval [a, b]
[F (x) is an antiderivative of the function f (x)]. In particular, the
antiderivative must be a function continuous on the whole interval
[a, b]. A discontinuous function used as an antiderivative will lead
to the wrong result.
6.2.6. Find a mistake in the following evaluation:
v3
S l+x
dx
2 = 2I arc tan 2x
l-x 2 0
Iva = 2I [arc t an(-V3 )-arctanO] = -6 n
,
0

where ( f arc tan 2


1 xx2 )' = 1 ~x 2 (x == 1).

Solution. The result is a priori wrong: the integral of a function


positive everywhere turns out to be negative. The mistake is due
to the fact that the function ~arc tan 1 2xx 2 has a discontinuity
of the first kind at the point x = I:
. I 2x n . I 2x n
It m 2 arc tan 1-x2 = 4 ; ltm 2 arctan~=-4
.-- 1-0 x-1+0 X

The correct value of the integral under consideration is equal to


3
S1~x2 =arc tanx 1~ 3 =arc tan V3-arc tan O= ~ .
0

Here the Newton-Leibniz formula is applicable, since the function


F (x) =arc tan xis continuous on the interval [ 0, ~ and the equality J
F' (x) = f (x) is fulfilled on the whole interval.
6.2. Newton-Leibniz Formula 259

6.2.7. Find a mistake in the following evaluation of the integral:


n

Sl-t-2sin dx r
:rt
dx
2 x=.) cosix+3sin 2 x=
0 0

n __c!!__ In
=.\ 1+ 3 tanzx = V"3
cos 2 x I r-

arc tan (V .1 tan x) ~~o.


0
0

(The integral of a function positive everywhere turns out to be


zero!)
Solution. The Newton-Leibniz formula is not applicable here,
since the antiderivative F (x) = h arc tan (V3 tan x) has a discon-

tinuity at the point x = ~ . Indeed,

rr
Jim F (x) = Jim
x~~-or3
I
,r- arc tan ( v-3 tan x) =
x~2-0 2

I n
= ,r- arctan ( +oo) = ,r- ,
r 3 2 r 3

Jim F (x) = Jim .)_ arctan (V3tan x) =


rr rr r 3
x~ 2 +o x~ 2 +o

I n
= V3 arc tan (-oo) = - 2 V 3 .

The correct result can be obtained in the following way:


n rt

Scos dx r I dx
2 x+ 3 sin 2 x = J cot 2 x+ 3 sin 2 x =
0 0

= - ; 3 arc tan (J/3 cot x) J: V:r .


=

rt can also be found with the aid of the function F (x) =


= / 3 arc tan (l/3 tan x). For this purpose divide the interval of

integration [O, n) into two subintervals, [ 0, ~ and [ ~ , n and J J,


take into consideration the above-indicated limit values of the func-
tion F (x) as x--+ ~ + 0. Then the antideriva tive becomes a conti-
nuous function on each of the subintervals, and the Newton-Leibniz
9*
Ch. VI. The Definite Integral

formula becomes applicable:


.'l
11 2 11

Scos
0
2 x~-\ sin~x= 5
0
+ 5= 11
2

I arc tan
= -;r-
r3
( V---3 tan x ) In:T
o
+ .rr3I _ arc tan (V-3 tan x ) 1:-ti
2
=

= ;3 [(i-- 0 )+( 0 -(-~))J= V3


6.2.8. Compute the integral

Solution. V,- I+ c2os 2x = V 2 cot x =I cos x / =

= { cosx, O~x~ ~,
ll
-cosx, 2 ::::::;; x::::::;;n.
Therefore
n:
11 2 n:

JV +
0
1 c;s 2x dx = Jcos0
x dx + J(- cos x)
n:
dx =
2

=sinxl 0~ -1-(-sinx)I~ =(l-0)+(0-(-1))=2.

Note. If we ignore the fact that cos x is negative in [ ~ , n]


and put
y/1+cos2x
"I
2 =COSX,

we get the wrong result:


:rt

~ cosxdx=sinxl~=O.
0
6.2. Newton-Leibniz Formula 261

6.2.9. Evaluate the integral


IOO:rt

I= ~ V 1-cos2x dx.
0
Solution. We have
Vi-cos 2x= V2isinx1.
Since I sin x I has a period n, then
JOO:rt IOO:rt

~ Vl-cos2xdx=V2 ~ Jsinxldx=
0 0
:rt

= 100 V2 ~ sin xdx= 200 V2.


0

6.2.10. Evaluate the integrals:


-1 -2

(a) I= S (I I ~5x)a ; (b)J=S x2~!;


-2 -3
:rt
:rt 4

(c) I= S sin i- dx;


-:rt
2
(d) /= s
0
x2:1 dx;
2

(e) I=
e'

s ~~ x x ;
(f) I = r sin~ dx;
J x
I
:rt
I I

(g) f = s
0
I _;: 2 x dx; (h) I= SI +xs ;
x dx

0
3

3
(i) I
=
j' -Vx+I+
__x_d_x-Vsx+I
__
0
:rt
2
(j) I= ~ Vcosx-cos 3 xdx;
:rt
2

(k) I=
V3
I I
dx

(I+ x2)
3
2

262 Ch. VI. The Definite I nlegral

6.3. Estimating an Integral.


The Definite Integral as a Function of Its Limits
1. If f (x)~qJ(x) for a~x~b, then
b b

~ f (x)dx~ ~ qJ(x)dx.
a a
In particular,

If f (x)dx I~ f If (x) Idx.


b

2. m(b-a)~ ~ f(x)dx~M(b-a),
a
where m is the least value, and M the greatest value of the func-
tion f (x) on the interval [a, b] (estimation of an integral).
3. If the function f (x) is continuous on the interval [a, b], then
b

~ f (x) dx= f
a
m(b-a), a< s< b
(mean-value theorem).
4. If the functions f (x) and qi (x) are continuous on [a, b], and
qJ (x}, in addition, retains its sign on this interval, then
b b

~ f (x) qi (x) dx = f () ~qi (x) dx, a < s < b


a a
(generalized mean-value theorem).
x a
5. :x Sf (t) dt = f (x); :x Sf (t) dt = - f (x) at each point x of
a x
continuity of the function f (x).
6.3.1. Estimate the following integrals:
n

(a) I=
3

SV 3 -~ x~ dx;
I
(b) I= s si:
3

n
x dx;

4
2

(c) I= Sxx2+5
+ 2 dx.
2
0

Solution. (a) Since the function f (x) = V 3 + x2 increases mono-


tonically on the interval [1, 3j, then m=2, M=V30, b-a=2.
6.3. Estimating an Integral. Integral as a Funct. of Its Limits 263

Hence, the estimation of the integral has the form


3

2 2 ~ ~ V 3 + x 3 dx ~ V 30 2,
I
i. e.
3

4 ~ ~ i13 +x3 dx~ 2 V30 ~ 10.95.


I

(b) The integrand f (x) = si~ x decreases on the interval [ ~ , ~ J


since its derivative
f' (x ) = x cos x- sin x
x2
= (x- tan x) cos x
x2
< O
Hence, the least value of the function:

m= f (;) = 3 ~3- '


its greatest value being
M=f(~)= 2 ~i.
Therefore
:rt
3
~y:f (_:: _ _::) ~ r ~dx~ 2 2 (_:: _ _::)
21t 3 4 -....-:: J
:rt
x -....-:: 1t 3 4 '
4
i. e.
:rt
;J
V3
0.22 ~ - - 8 ~ v
1 sinx
-x- dx ~ -y2
6- ~ 9
0.-4.
:rt
4
6.3.2. Estimate the absolute value of the integral
19

S ~d
I +xs X.
10

Solution. Since I sin xi~ 1, for x~ 10 the inequality


I t~:s < J 10-s is fulfilled.
Therefore
19

S ~dx
I +x 8
< (19-10) 10-s < 10-~
10

(the true value of the integral ~ -10- 8 ).


264 Ch. VI. The Definite Integral

6.3.3. Which of the two integrals


I

~ Vxdx,
0
is the greater?
Solution. As is known, Vx > x 3 for 0 < x < 1. Therefore
I I

~ Vxdx > ~ x dx. 3

0 0

6.3.4. Prove the inequalities:

rex
l I
. x 7 dx I
(a) 0 < \ 3/ < 8; (b) 1 < 2
dx < e.
b V I +xs b
x7
Solution. (a) Since 0 < 3/ <x 1 for O < x ~ 1,
V l+xB
then

< .I
I I

0< SV 3 ,
x7 dx
l+x8
x1 dx =--= 8x8 11o = 8I .
0 0

(b) Since for 0 < x< 1 there exists the inequality 1 < e" 2 < e,
then
I l I

~ dx < ~ e"2 dx < ~ edx.


0 0 0

Hence the inequality under consideration holds true.


6.3.5. Prove the inequality
n
2
s
0
e-R sin x dx < 2~ (1-e-R) (R > 0).
Solution. Since the function f (x) = sinxx decreases on ( 0, ; ) [see
Problem 6.3.1 (b)), then for 0 < x <i
f (x) =sin
x
x > f (\ ~)
2
= ~
1t

Hence, on this interval sin x > ~


1t
x, therefore
2R
e-Rsir.x<e - -nx
6.3. Estimating an Integral. Integral as a Funct. of Its Limits 265

and

6.3.6. Prove that for any functions f (x) and g (x), integrable on
the interval (a, b), the Schwarz-Bunyakovsky inequality takes place:

Ii f (x) g (x) dx I~ -v I I f 2 (x) dx g 2 (x) dx. -

Solution. Consider the function


F (x) = [f (x)-J.g (x)]2,
where J. is any real number. Since F (x) ;;;:: 0, then
b
~ [f (x)-J.g(x)]2 dx :;;=:: 0,
a
or
b b b
J.,2 ~ g 2 (x) dx-n ~ f (x) g (x)dx+ ~ f 2 (x) dx :;;=:: 0.
a a a

The expression in the left side of the latter inequality is a quad-


ratic trinomial with respect to J.. It follows from the inequality that
at any A, this trinomial is non-negative. Hence, its discriminant is
non-positive, i. e.
J2
{
b
~ f (x) g (x)dx - j f (x)dx) g (x) dx ~ 0.
b
2
b
2

Hence

IIf (x) g (x) dx I~vi ff2 (x) dx j g2 (x) dx,


which completes the proof.
6.3.7. Estimate the integral from above
I
(' sin x
I= j I +x2dx.
0

Solution. By the generalized mean-value theorem we have

S
l
sin x d
+
1 x~ x =
.
s m "'
t s+
1

1
dx t
x 2 = s m "' a re an x
t =-Tsins(O <s< l).
0 0
266
~~~~~~~~~~~
Ch. VI. The Definite Integral

Since the function sin x increases on the interval [O, I] then


sin< sin I. Whence we get an upper estimate of the integral:
1

51+xx dx < T sin


2
:rt
Sin I~ 0.64.
0

It is possible to get a better estimation if we apply the same


theorem in the form

5
I
sin x
l+x2 dx = 1+ 62
I 5 . xdx =
1

Sin
I
1 + 6 (l-cos I)<
t -cos I~ 0.46.
0 0

6.3.8. Proceeding from geometric reasoning, prove that:


(a) if the function f (x) increases and has a concave graph in the
interval [a, b], then
b
(b-a) f (a) < 5f (x) dx < (b-a) f(a) t f(b);
a

(b) if the function f (x) increases and has a convex graph in the
interval [a, b], then
b
(b-a) f (a) tf (b) < 5f (x) dx < (b-a) f (b).
a
Solution. (a) Without limitation of generality we may assume
f (x) > 0. Concavity of the graph of a function means, in particu-
lar, that the curve lies below the chord
!J through the points A (a, f (a)) and
f(b) B (b, f (b)) (see Fig. 61). Therefore the
--,------ 8 area of trapewid aABb is greater than
I .,.,...-""' I
I ,...,.,. I that of the curvilinear trapezoid boun-
f(a) -- ,.. ------le ded above by the graph of the func-
A1 I
I I tion, i. e.
--t-::---'-~~~~~~.z b
0 a
Fig. 61
b St (x)dx < SaABb= (b-a) f (a)t f (b).
a

The inequality
b
(b-a)f (a)<~ f (x)dx
a
is obvious.
I

6.3.9. Estimate the integral ~Vt -j-x'dx using


0
6.3. Estimating an Integral. Integral as a Funct. of Its Limits 267

(a) the mean-value theorem for a definite integral,


(b) the result of the preceding problem,
(c) the inequality V l +x 4 < l
-- + x4
2,
(d) the Schwarz-Bunyakovsky inequality (see Problem 6.3.6).
Solution. (a) By the mean-value theorem
I

I = ~ VI +x 4 dx = Vl + 4, where 0 ~ ~ l.
0

But
1 < V 1+ 4 < V2,
whence
l <I< V2 ~ l.414.
(b) The function f(x)=Vl+x 4 is concave on the interval[O, l],
since
f"(x)=2xz(x4+3)>0 O~x~l.
(l + x4)3/2 ' -..:::::
On the basis of the preceding problem we get

t < SVt +x4 dx < 1 +{ 2 ~1.207.


0
I I

(c) t < I = SVt + x dx < S( l + ~ ) dx = I + +cr = l . t.


4

(d) Put f (x) =Vt +x4 , g(x) =land take advantage of the Schwarz-
Bunyakovsky inequality

I~ V I +
0
X 4 dx / = IVI +
0
x 4 dx = I <{ ~ (I + x4) dx. ~ l2 dx
() 0
=

= V1.2 ~ 1.095.
6.3.10. Find the derivative with respect to x of the following
functions:
x'
(a) F (x) = ~ Int dt (x > 0),
x'

(b) F(x)= ~ cos(t 2 )dt (x>O).


I
K
268 Ch. VI. The Definite Integral

Solution. (a) Write the given integral in the following way:


c x3 X3 x2

F (x) = ~ In t dt + ~ Int dt = ~ Int dt - ~ Int dt,


x2 c c c

where c > 0 is an arbitrary constant.


Now let us find the derivative F' (x) using the rule for differen-
tiating a composite function and the theorem on the derivative of
an integral with respect to the upper limit:

F; (x) =[~Int dt]' (x 3 )~-[I Int dt]' (x2 )~ =In x 3x


c ~ c x2
3 2 - lnx2 2x =

= (9x~ - 4x) In x.
c Vx
(b) F(x)=~ cos(t 2 )dt-t- ~cos(t2)dt=
I C
x
.l.
x Vx
= - ~ COS (t2) df + ~ COS (tZ)d/;
c c

F'(x) =-[f c
cos (t2)dt]~ ~ t + [v(cos(t )dt]'v __ (Vx);=
x
(
c
2

=- cos 2I ( -
x
2
x
I )
+ cosx 2 r x .r- '= 2I
I
x
cos I
2
x
+ 2.r-
I
rx
cos x.

6.3.11. Find the derivative with respect to x of the following


functions:
2~ 0

(a) F (x) = S si~ / dt; (b) F(x)=SVl+t 4 dt.


0 x
~

6.3.12. Find the points of extremum of the function F (x) = Ssi~t dt


.in the domain x > 0.

Solution. Find the derivative

F' (x) = [~ si~t dt I =-si1_:x

The critical points are:


x=n~ (n= l, 2, ... ), where sinx=O.
6.3. Estimating an Integral. Integral as a Funct. of Its Limits 269

Find the second derivative at these points:


F"(x) =xcosx~sinx;
x
I I
F" (me) =-cos
nn
(me)= -(-l)n=FO.
nn

Since the second derivative is non-zero at the points x =mt


(n= l, 2, ... ), these points are points of extremum of the function,
namely: maxima if n is odd, and minima if n is even.
6.3.13. Find the derivative of y, with respect to x, of the func-
tion represented parametrically:
I' 3

x= ~ Vz In zdz; y= ~ 22 ln zdz.
Vt
Yt
'
Solution. As is known, Yx=-,
Xt
Find x; and y;:
I' '

x;=(~ Vzln zdz) (t3);=tlnt 3 3t 2 =9t 3 lnt;


I I'

y; = (
\vr
~ 22 In z dz)'
VT
(Vt); = - t In VI
2
.~-
r t
= - ! Jlt Int;
whence
y~ = 9t31n t = -36t2 Vt (t > O).
_ _.!._ Vtln I
4
6.3.14. Find the limits:
x
'
~sin Vx dx ~ (arc tan x) 2 dx
0
(a) lim - - -x3
0 ,,-- (b) Jim
x~o x-+ oo

x
Solution. (a) At x =--= 0 the integral ~sin Vx dx equals zero; it is
0
easy to check the fulfilment of the remaining conditions that ensure
270 Ch. VI. The Definite Integral

the legitimacy of using the L'Hospital rule. Therefore

I. o
l' sin Vxdx
I'
[r
o
sin Vxdx].
x
tx2 ); .

1. 2xsinx 2
xi~ xa = x1!!fi 3x2 = xl~l~ 3X2 = 3 .

(c) We have an indeterminate form of the type 00


00
Use the
L'Hospital rule:

x
2 ~ ex 2 dx
o ex2
= Jim - - -
2
- = lim 2--=0.
ex
2 ex 2x
x~ + oo x-+ + oo

6.3. 15. Find the derivative ~~ of the following implicit functions:


y x
(a) ~ e- 12 dt + ~ sin 2 t dt = 0;
0 0
y x
(b) ~ e dt 1 + ~ sin t dt = 0;
0 0
x y
(c) ~V3-2sin 2 zdz+~costdt=O.
1T 0
2
Solution. (a) Differentiate the left side of the equation with
respect to x, putting y = y (x):

[
[x'
~y e-t 2 dt ] ' . ~~+ ~ sin 2 tdt ]' (x2 )~=0;
0 y 0 x'

e-Y 2
~~ + sin 2 x 2 2x = 0.
Hence, solving the equation with respect to ~~, we get
dy = -2xa+Y 2 sin 2 x 2
dx
6.3. Estlmatlng an lnlegral. Integral as a Funct. of Its Limits 271

(c) Differentiate the left side of the equation with respect to x,


putting y=y(x):

[ SV3-2 sin zdz]' + [ Scost dt]' :~ = 0.


:re
2

0 y
2 x
Whence
V3-2 sin 2 x
V3-2 sin 2 x+cosy:~=O;
dy
dx= - cosy

6.3.16. Find: (a) the points of extremum and the points of


inflection on the graph of the function
x
I=~ (t-1) (t-2) 2 dt;
0

(b) curvature of the line defined by the parametric equations:

(the Cornu spiral).


Solution. (a) The function is defined and continuously differen-
tiable throughout the entire number scale. Its derivative
I~= (x-1) (x-2) 2

equals zero at the points x 1 = I, x 2 = 2, and when passing through


the point x 1 it changes sign from minus to plus, whereas in the
neighbourhood of the point x 2 the sign remains unchanged. Conse-
quently, there is a minimum at the point x 1 = l, and there is no
extremum at the point x2 = 2.
The second derivative
1;=3x2 -10x+8

vanisaes at the points x 1 -= : , x2 = 2 and changes sign when pas-


sing through these points. Hence, these points are the abscissas of
the points of inflection.
(b) We have
. v-ncos 2
x,=a :rtf2 , . v-nsm. 2
y,=a :n;f2
,
272 Ch. VI. The Definite Integral

hence,
, Yt nf!. , (y_:l; Vilt .
Yx=-, = tan 9 , Yxx= -,- = 'JI./!.
Xt - Xt a cos 3 -
2
whence the curvature
K= I y" I
---'-"--'--3,-
Vrrt
a
l t + (y'J21 T
6.3.17. Prove that the function L(x), defined in the interval
(0, oo) by the integral
x

l (x) = S{, di

is an inverse of the function ex.


Solution. Let us take the derivative

l' (x) = _!_


x
(x > 0).
Since the derivative is positive, the fund ion y ~= l (x) increases
and, hence, has an inverse function
X= l - 1 (y).
The derivative of this inverse function is equal to
dx l
dy= L' (x) =X,
whence it follows (see Problem 3.1.10) that
X=CeY.
To find C, substitute x =I. Since
l(l)=O, i.e. Ylx= 1 =0,
then
I =Ce0 =C,
which proves our assertion:
x= L- 1 (y) =eY.
6.3.18. Given the graph of the functio:i y = f (x) (Fig. 62), find
x
the shape of the graph of the antiderivative I=~ f (t) dt.
0

Solution. On the interval [O, a] the given function is posi-


tive; consequently, the antiderivative increases. On the interval
6 ..J. Estimating an Integral. Integral as a Funct. of Its Limits 273

l0, i J the derivative of the given function is positive; hence, the


curve I= I (x) is concave. On the interval ~ r ,a]
the derivative
of the given function is negative; consequently, the curve I= I (x)
is convex, the point x = ~ being a point of inflection. The inter-
val [a, 2a] is considered in a similar way. The point x 1 = 0 is a
point of minimum, since the derivative /'(x)=f(x) changes its
sign from minus to plus; the point x 2 =a is a point of maximum,
since the sign of the derivative changes from plus to minus.

!I

~x 2 2
Fig. 62 Fig. 63

The antiderivative I (x) is a periodic function with period 2a,


since the areas lying above and below the x-axis are mutually
cancelled over intervals of length 2a. Taking all this into account,
we can sketch the graph of the antiderivative (see Fig. 63).
6.3.19. Find the polynomial P (x) of the least degree that has
a maximum equal to 6 at x =I, and a minimum equal to 2 at
X=3.
Solution. The polynomial is an everywhere-differentiable function.
Therefore, the points of extremum can only be roots of the deriva-
tive. Furthermore, the derivative of a polynomial is a polynomial.
The polynomial of the least degree with roots x 1 = I and x 2 = 3
has the form a(x-l)(x-3). Hence,
P'(x) =a(x-1) (x-3) =a(x2 -4x+3).
Since at the point x = 1 there must be P (I) = 6, we have
x x
P (x) = ~ P' (x) dx + 6 = a ~ (x2 - 4x + 3) dx + 6 =
I I

=aC;-2x2 +:3x-1; )-1-6.


The coefficient a is determined from the condition P (3) = 2, whence
a= 3. Hence,
+
P (x) = X 3 - 6x2 9x -I 2.
274 Ch. VI. The Definite Integral

6.3.20. Find the polynomial P (x) of the least degree whose graph
has three points of inflect ion: (-1, -1 ), ( 1, 1) and a point with
abscissa 0 at which the curve is inclined to the axis of abscissas
at an angle of 60.
Solution. Since the required function is a polynomial, the abscis-
sas of the points of inflection can only be among the roots of the
second derivative. The polynomial of the least degree with roots
-1, 0, l has the form ax(x2 - l). Consequently,
P" (x) =a (x 3 -x).
Since at the point x = 0 the derivative P' (0) =tan 60 = V3, we
have

P' (x) =
x

5P" (x) dx + V3 =a c~ x;) + V3.


0
-
Then, since P (1) = 1, we get

P (x) = SP'(x)dx+ 1 =a (~~-x: +Zo) +V3(x- l)+ 1.


I

The coefficient a is determined from the last remaining condition


60CV3-t)
P (-1) =-1, whence a= 7 . Hence,

P(x)= V3-t
7 (3x~-10x 3 ) +x
v-3.
6.3.21. Taking advantage of the mean-value theorem for the
definite integral, prove that
I
(a) 3 < ~ V q +x 2 dx < 10,
0
3t

(b) ~ <S v' 1 ++sin xdx < ~ Ji/;. 2

2n
( c) I3
s
< JO+
23t
dx
3 cos x <T
2n

0

6.3.22. Using the Schwarz-Bunyakovsky inequality, prove that

5Vl +x dx < ~s. 3 Make sure that the application of the mean-
o
value theorem yields a rougher estimate.
6.4. Chanf{ing the Variable in a Definite Integral 275

6.3.23. Find the derivatives of the following functions:


x x'
(a) F (x) = SInt dt (x > 0); (b) F (x) = ST.
dt

I 2
-x

6.3.24. Find the derivative ~~ of functions represented paramet-


rically:
I In I
z ('
(a) X= Jz-dz,
('In
y= j ezdz;
2 5
sin t 17 1

(b) x= Sarcsinzdz, y= j -
(' sin z 2
2 -dz.
ca 11

6.3.25. Find the points of extremum of the following functions:


x t'
(a) F(x)=~e-2(1-t 2 )dt;
I

(b) F (x) = s
x'

o
tz-st+4
2+et
dt.

6.4. Changing the Variable in a Definite Integral


If a function x = cp (t) satisfies the following conditions:
(1) cp (t) is a continuous single-valued function defined in [a, ~]
and has in this interval a continuous derivative cp' (t);
(2) with t varying on [a, ~] the values of the function x = cp (t)
do not leave the limits of [a, b];
(3) cp (a)= a and cp (~) = b,
then the formula for changing the variable (or substitution) in the
definite integral is valid for any function f (x) which is continuous
on the interva I [a, b]:
b (:l

~ f (x) dx = ~ f[ cp (t)] cp' (t) dt.


a a

Instead of the substitution x = cp (t) the inverse substituHon


t = 'ljJ (x) is frequently used. In this case the limits of integration
a and ~ are determined directly from the equalities a= 'ljJ (a) and
~ = 'ljJ (b). In practice, the substitution is usually performed with
the aid of monotonic, continuously differentiable functions. The
change in the limits of integration is conveniently expressed in
276 Ch. VI. The Definite Integral

the tabular form:

~
ala
b ~ .
v3-
6.4. t. Compute the integral ~ V 4-x2 dx.
-V3
Solution. Make the substitution x=2sint, assuming that
- ~ ~ t ~ ~ . The function x =qi (t) = 2 sin t on the interval
r- . ]
~ ~ satisfies all the conditions of the theorem on changing
the variable in a definite integral, since it is continuously differen-
tiable, monotonic and

And so,
x=2sint; dx=2costdt; V4-x 2 =2[cost[=2cost,

since cost> 0 on the interval [ - ; , ~ J.


Thus,
l't l't
Va a 3
~ V4-x 2 dx=4 ~ cos2 tdt=2 ~ (l+e<:>s2t)dt=
-ira l't
3
-~
3

=2 [t+-}sin2t]~= 4;+V3.
3

6.4.2. Compute the integral 5' Vx;=4 dx.


2

Solution. Make the substitution

x=2 sect; xI t
dx=2sintdt 2 0
cos 2 t '
4 .:.':__
3

On the interval [ 0, -'}] the function 2 sec t is monotonic, there-


fore the substitution is valid.
6.4. Changing the Variable in a Definite Integral 277

Hence,
n

Y4sec 2 t-4. 2 sint dt=


16 sec 4 t cos 2 t

. :''.
3
In
-
I
~ sin I
~23
3
= 4 2 t cost dt = Tii sin t
3
0 =
0

6.4.3. Compute the integrals:


a
(a) Sx Va -x dx;
2 2 2 (b)

6.4.4. Compute the integrals:


n
'"2' 2

(a)
\~
J
0
cos x dx
6-5 sin x+ sin 2 x; (b) 5
0
2+dc:s x

Solution. (a) Apply the substitution


sin x= t; ~
cosxdx=dt; 0 0
~.
2

The inverse function x =arc sin t ( 0 ~ x ~ ~ for 0 ~ t ~I) satisfies


.all conditions of the theorem on changing the variable. Hence,
n

l=S 2

c?sxdx
6-5smx+sin 2 x
=S l

6-51+12
dt =lnt-311=ln..
t-2 o 3
0 0

(b) Make the substitution t =tan ~

~
2dt
x = 2 arc tan t, dx= 1+12'
0 0
ll
2

which is valid due to monotonicity of the function tan ~ on the


278 Ch. VI. The Definite Integral

interval
:rt
l0, ~ J.
2
(' dx
J 2-t-cos x =
s -1-.!...=E
I

2
s I 2dt
I +12
1

= 2 3+12 =
di

o o I+ , 12 o

= J ~3 J 3 arc tan /: = 3 (arc tan ; 3 - arc tan 0) = 3 ;,T.


6.4.5. Compute the integral
:rt
4

Sa 2
dx
cos 2 x+b 2 sin 2 x
(a> 0, b > 0).
0

Solution. Make the substitution


tanx=t, ~
~=dt
cos 2 x '
0 0

Hence,
ll
4

Sa 2
dx
cos 2 x-f-b 2 sin 2 x =
s I
di
a11 -t-b 2 t 2 = b2
I s I

~=
dt

0 0 0 b2 +1 2

I b bt \1 I b
=--arctan-
b2 a a o ab
=-arctan-.
a

If a= b = 1, then :b
arc tan =arc tan 1 = ~ , which exactly coin- !
cides with the result of the substitution a= b = l into the initial
integral
:rt ~

4 4

S 0
dx
a2 cos 2 x+ b2 sin2 x
=Sdx=.!!:..
4
0

6.4.6. Compute the integrals:


1'3
(a) svir
I
dx;

(c) S3+V<x-2)J
2

8 V<x-2)2
dx.
3
6.4. Changing the Variable in a Definite Integral 279

6.4. 7. Compute the integral


. I= s" x sin x
1+cost x dx.
0

Solution. Reduce this integral to the sum of two integrals:


Jt

I= s
2

x sin x
l+cos 2 x
dx + s
Jt

x sin x dx =I
i+cos 2 x 1
+I 2
0 Jt
2

To the integral

I =
2
sJt

x sin x dx
I +cos 2 x
Jt
-
2

apply the substitution


X=n-f, x
dx=-dt,

Then
Jt

I = _
2
s0
(n-t) sin (n-t)
l+cos 2 (n-I)
dt = s2
(n-t) sin
l+cos 2 1
t dt =
!1 0
2
Jt
2
sin t dt-S t sin t dt
l + cos 2 t l + cos2 t
0

Hence
Jt Jt 11

I= I
1
+I = 2
s
2

0
x sin x dx
l+cos 2 x
+n s
0
2

sin t dt
I+cos 2 /
-s2

0
t sin t dt
I+cos 2 t

Since the first and the third integrals differ only in the notation
of the variable of integration, we have

sin t dt
l+cos 2 t
280 Ch. VI. The Definite Integral_ _ _ _ _ _ _ _ __

To this integral apply the substitution


u =cost, t
---;---1
I u
du=-sintdt, O
:rt
T 0
o I
(' du (' du rr~
/=-Jl j 1+u 2 =ll j 1+11 2 =4
I fJ

r> x sin x . .
Note. The mdefimte integral u\ not expressed m 1 + cos 2
x
dx 1s
elementary functions. But the given definite integral, as we have
shown, can be computed with the aid of an artificial method.
6.4 .8. Evaluate the integral

I= s
I
In (I +x)
1+x2
d
x.
0

Solution. Make the substitution


x= tant,
dt 0
dx = cos2 t' .
4
Hence,
Jt Jt
4 4

I= (' In (I+ tan t) sec2 t dt = ('In ( l +tan t) dt.


J0 sec 2 t J
0

Transform the sum l +tan t:


v~rsin (1 -/-~)
I +tan t =tan ~ +tan t = cos 1 4
Substituting into the integral, we obtain
n n n
4 4 4

I =-='S ~ln2dt+Slnsin(t+ ~ )dt-Slncostdl 0 =


0 0 ()

4 4

=+tln2t: + 51nsin(t+ ~ )dt-Slncostdt=


0 3
6.4. Changing the Variable in a Definite Integral 281

Now let us show that / 1 =/2. To this end apply the substitution
n
t= 4 -z, t I z
0 n
dt =-dz, n T
-
4 0
n
4

to the integral l 2 =Slncostdt.

Then
:rt
0 4

/ 9 =- SIn cos ( ~ -z) dz= SIn sin l ~ -( ~ -z) J dz=


~ 0
4

= S In sin ( ~ + z) dz=/ 1.
0

Therefore
I=~ In 2.

Note that in this problem, as well as in the preceding one, the


indefinite integral S
ln1 ~~2x) dx is not expressed in elementary fun-
ctions.
6.4.9. Prove that for any given integral with finite limits a and b
one can always choose the linear substitution x =pt+ q (p, q con-
stants) so as to transform this integral into a new one with limits
0 and l.
Solution. We notice that the substitution x=pt+q satisfies
explicitly the conditions of the theorem on changing the variable.
Since t must equal zero at x =a and t must equal unity at x = b
we have for p and q the following system of equations
a=pO+q,
b=pl+q,
whence p = b-a, q =a. Hence,
b 1

~ f (x)dx=(b-a) ~ f [(b-a) t+a] dt.


a o
282 Ch. VI. The De-fi,nite Integral

6.4.10. Compute the sum of two integrals


2
-5 3 ( 2)2
~ eCx+S)' dx+ 3 ~ e<J x-3 dx.
-4 I

Solution. Let us transform each of the given integrals into an


integral with limits 0 and 1 (see the preceding problem).
To this end apply the substitution x = - t-4 to the first inte-
gral. Then dx =-dt and
-5 I I

/1= ~e<x+ 5 >"dx=- ~e<-t+l>'dt=- ~e< 1 - 1 >'dt.


-4 0 0

Apply the substitution x= ~ +~ to the second integral. Then


dt
dx= 3 and
2
3 ( 2 )" I
/ 2 =3 ~ / x-3 dx = ~ e< 1- 1>'dt.
I 0
3
Hence
I I

f1 +/ 2 = - ~ eU - !)' df + ~ e( 1 -1) df = 0.
0 0

Note that neither of the integrals ~ e<x+si dx and ~ e9 ( x- ~ )' dx is


evaluated separately in elementary functions.
6.4.11. Prove that the integral
n
\' si~ 2kx dx
., smx
0
equals zero if k is an integer.
Solution. Make the substitution
x I t
X=:rt--f,
0 :rt
dx=-dt,
:rt 0

Then at k an integral number we get:


n

5 sm x
O
si~ 2kx dx = -s n
0
sin_ 2k (n-t)
sm (n-t)
dt = _ s
n

o
si~ 2kt dt.
sm t
6 .4. Changing the Variable in a Definite Integral 283

Since the definite integral does not depend on notation of the vari-
able of integration, we have

I= - I, whence I =0.

6.4.12. Compute the integral

Solution. Apply the substitution x =sin t (the given function is


not monotonic), dx =cost dt. The new limits of integration t 1 and
ta are found from the equa-
x
.
t 10ns I . t y3 . t
2 =sm ; - 2 - =Sm .
We may put t1 = ~ and ta=
:rt
= 3 , but other values may
also be chosen, for instance,
5:rt 2:rt
/1 = 6 and t 2 = 3 .
In both cases the variable Fig. 64
x =sin t runs throughout the
entire interval [ ~ , ~3 J (see Fig. 64), the function sin t being

monotonic both on [ ~ , ~ J and [ 2; , 56n] .


Let us show that the results of the two integrations will coincide.
Indeed,
Y~ n n

T,1 xVt-xi
dx = s3 cost dt =
sintcost
\~ ~ =
., sint
I
In tan!.___ 11
2
~=
~
I n n 6
2 6 6
n
=In tan-- Intan-=
n
In 2+V}""3
6 12 3

On the other hand, taking into consideration that cost is negative


on the interval [ 23n; ~n] we obtain ,
284 Ch. VI. The Definite Integral

1'3 2n 5n
-2- 3 6

.\ x
I
Y ~~x~ = S
5rt
sin ~0(~::s t) = S s~~
2n
t =
2 6 3

=
I
In Itan 2 I
t
5n
6
2n =In
t
an
n
5
12 Jt
3 tan 3

Note. Do not take t 1 = 56n t2 =; , since, with t varying on


the interval [ ~ , 56 nl , the values of the function x =sin t lie beyond
the limits of the interval [ +, ~3 ] .
6.4.13. Prove that the function L (x) defined on the interval
x
(0, oo) by the integral L (x) = S~t possesses the following properties:
I

L (X1X2) = L (X1) + L (X2),


L( :J =L (x 1 )-L (x 2 ).
Solution. By the additivity property

Let us change the variable in the second integral


t I z
t = X 1Z,
dt =X 1dz,

Then
6.4. Changing the Variable in a De'finite lntewat 285

It is also easy to obtain the other corollary L(x ~) = ~ L (x) for


any integral m and n.
Indeed, for positive m and n this follows from the relations

and for a negative exponent, from

L (1) = 0, L (x- 1 ) = L( +) =l (1)-L (x) =- L (x).


Now, taking advantage of the continuity of the integral as a fun-
ction of the upper limit, we get the general property l (xa) =al (x).
Note. As is known, L (x) =In x. Here we have obtained the prin-
cipal properties of the logarithm proceeding only from its determi-
nation with the aid of the integral.
3
6.4.14. Transform the integral ~ (x-2) 2 dx by the substitution
0
(x-2) 2 = t.
Solution. A formal application of the substitution throughout the
interval [O, 3] would lead to the wrong result, since the inverse
function x=cp(t) is double-valued: x=2Vt, i.e. the function x
has two branches: x1 = 2-Vt; x 2 = 2 +Vt. The former branch can-
not attain values x > 2, the latter values x < 2. To obtain a cor-
rect result we have to break up the given integral in the following
way:
3 2 3

~ (x-2) 2 dx= ~ (x-2)2 dx+ ~ (x-2) 2 dx,


0 0 2

and to put x=2-Vt in the first integral, and x=2+V-t in the


second. Then we get
2 0 4

11 = s
0
(x- 2)2 dx = - J
4
t2 s
~T = ~ Vt dt = ~ ,
0

12 =
3

J(' (x-2) 2 dx= J t 2


I
(' dt
VT =
Is - I
I

2 Vtdt =a
2 0 0

Hence, I= 3
8
+ 3I = 3, which is a correct result. It can be easily
286 Ch. VI. The Defi,nite Integral

verified by directly computing the initial integral:

s(x-2)2dx= <x--;2i: =f +: =3.


3 3

0 0

6.4.15. Compute the integrals:


5
(b) I - r dx
-J0 2x+V3x+l
It
3
() /-5
I

c -
dx.
1-sinx' (d) I= 5v-2x-x 2 dx;
It 0
4
It

(e) / - -5 4
sin x+cos d ,
3+sin2x X,
x
0
a

(f) / = .f x y :+; dx,


0
2 a>O;

2a I
(' dx
(g) I= 5 V2ax-x dx; 2
(h) I = J (I+ x2)2
0 -1

6.4.16. Applying a suitable change of the variable, find the fol-


lowing definite integrals:
2 a

(a) J 0
Yx+T:Vtx+l) 3 ; (b) fx+ ~;
b
2

f
(c). x(l +x');
I
dx
(d)

2
6.4.17. Consider the integral 5 !xx4 2 It is easy to conclude
-2

that it is equal to ~ . Indeed,


2

S~=_!_arc tan !_j =_!_ [~-(-~)] =~ 2


4+x 2 2 -22 2 4 4 4
-2
6.4. Changing the Variable in a Definite Integral 2b7

On the other hand, making the substitution x ~= +, we have

x I t
dt
dx= - ( i ,

2
~
-2
dx
4+x2 = -
-~2
I

I t
2 (
dt

4 +72
I)= -
r
I

dt
'412+1 =2arctan2t
'1
I 1

l-2
+ -4
I=
n

-2 -2

This result is obviously wrong, since the integrand 4 ~x 2 > 0, and,


consequently, the definite integral of this function cannot be equal
to a negative number - ~ . Find the mistake.
2n
6.4.18. Consider the integral I= 55 _g:os x. Making the substi-
o
tut ion tan ~ = t we have
2n

~ 5-2cosx- dx -i
0

(l+t 2
)
(
2dt
5 _ 2 1-ti)-
I +1 2
-O

0 0
The result is obviously wrong, since the integrand is positive.
and, consequently, the integral of this function cannot be equal to
zero. Find the mistake.
2
6.4.19. Make sure that a formal change of the variable t = xr;
2

leads to the wrong result in the integral ~ V x dx.


2 Find the
-2
mistake and explain it.
6.4.20. Is it possible to make the substitution x= sect in the
I

integral I = ~ Vx + 1 dx?2
0
1
6.4.21. Given the integral ~Vi -x2 dx. Make the substitution
0
x =sin t. Is it possible to take the numbers n and ~ as the limits
for t?
288 Ch. VI. The Definite Integral

6.4.22. Prove the equality


a a
~ f (x)dx= ~ [f (x)+f(-x)] dx
-a 0

for any continuous function f (x).


2n
6.4.23. Transform the definite integral ~ f (x) cos x dx by the sub-
o
stitution sinx=t.

6.5. Simplification of Integrals Based on the


Properties of Symmetry of Integrands
I. If the function f (x) is even on [-a, a], then
a a

-a
.\ f (x) dx = 2 \ f (x) d>:.
~

2. If the function f (x) is odd on [-a, a], then


a
~ f (x) dx = 0.
-a

3. If the function f (x) is periodic with period T, then


b b+nT

~ f (x) dx = ~ f (x) dx,


a a+nT

where n is an integer.
I

6.5. l. Compute the integral ~ Ix Idx.


-1

Solution. Since the integrand f (x) =Ix I is an even function, we


have
I I
~ Ix Idx = 2 ~Ix Idx = 2 ~ xdx = x2
I II --= 1.
0
-1 0 0

6.5.2. Compute the integral


7

Sxxs+2
sin xd
4
x.
-7

Solution. Since the integrand is odd, we conclude at once that


the integral equals zero.
6 .5. Simplification of I ntegra/s 289

6.5 .3. Evaluate the integrals


Jt

(a) ~ f (x) cos nx dx;


-:rr
"
(b) ~ f (x) sin nxdx,
-Jt

if: (1) f (x) is an even function; (2) f (x) is an odd function.


5

6.5.4. Calculate the integral S x'+x" sin x


2x + 1 dx.
2
2
-5
5
4l1
6.5.5. Compute the integral S cos' ~~2:in' xdx.
:Tl

Solution. The integrand is a periodic function with period n, since


sin 2 (x+n) sin 2x f
f(x+n)=cos4(x+n)+sin4(x+n) cos 4 x+sin 4 x= (X).
Therefore it is possible to subtract the number n from the upper
and lower limits:
5 l't

s
"
4"
S sin2xdx
cos'x+sin 4 x
=S cossin2xdx
4

x+sin x
4 4
= 2
4
tanxdx
cos 2 x(I +tan4x)
:Tl 0 0
Make the substitution

t =tanx,
dt =_!!!.._
cos 2 x'
:Tl

4 I II
2 sn coszx(l+tan'x)=
tan x dx
so l+t=arctant
2t dt 2
o=4
3t

6.5.6. Prove the equality


a a
~ cos xf (x 2 )dx = 2 ~cos xf (x2 ) dx.
-a 0

Solution. It is sufficient to show that the integrand is even:


cos (-x) f [(-x) 2 j = cosxf (x2 ).
290 Ch. VI. The Definite Integral

6.5.7. Compute the integral


Vz
5 2x7+3x 6- l0x0 -7x3 -12x2 +x+ I
x2+2 dx.
-V2
Solution.
V2
5 2x7+3x'-IOx x2+2
-7xs-12x +x+1
0
dx=
2

-V2

c= s
Vz
2x7-10xf>-7x3 +xdx+
x 2 +2
s
V2
3x2(x'-4)+1_d =
x2+2 X
_vz -V2
Vz
=0+2 S [a(x'-2x )+ x2 ~ 1 ] dx=
0
2

=~x 0 -4x 3 +-2 -arctan x_ ,v 2 =- 16 V2+~.


s 12 V2 o s 2 V2
In calculating we expanded the given integral into the sum of
two integrals so as to obtain an odd integrand in the first integral
and an even integrand in the second.
6.5.8. Compute the integral
I
2
5 cosx In: +;dx.
I
-2
Solution. The function f (x) =cos x is even. Let us prove that the
function cp (x) =In -x
+x is odd:
1
1

1-x I (l+x)-t
cp (-x) =In 1+x = n I -x = -ln l+x
I -x = -cp (x).

Thus, the integrand is the product of an even function by an odd


one, i.e. an odd function, therefore
I
T
5 cosxln :+;dx=O.
I
2

6.5.9. Prove the validity of the following equalities:


6.5. Simplification of Integrals 291

:rt I I
8 2 2
(a) ~ x 8 sinuxdx=0; (b) ~ eCOSXdX = 2 ~ eCOSX dx;
:rt I 0
-8 -2
n
(c) ~ sinmxcosnxdx=O (m and n natural numbers);
-JI

.
(d) \ sin xf (cos x) dx = 0.
-a

6.5.10. Prove the equality


h h

~ f (x)dx= ~ f (a+b-x)dx.
a a

Solution. In the right-hand integral make the substitution

x-=a+b-t, dx= -dt, I~


li__G_.
Then we obtain
h a h h

~ f (a+b-x)dx= - ~ f (t) dt = ~ f (t) dt = ~ f (x)dx.


a 11 a a

Note. The relation established between the integrals can be explai-


ned geometrically.
The graph of the function f (x), considered on the interval [a, bJ,
is symmetrical to that of the function f (a+ b-x), considered on
the same interval, about the straight line x= a;b.
Indeed, if the
point A lies on the x-axis and has the abscissa x, then the point A',
which is symmetrical to it about the indicated straight line, has
the abscissa x'=a-tb-x. Therefore, f(a+b-x')=f[a+b-(a+
-tb-x)] = f (x). But symmetrical figures have equal areas which are
expressed by definite integrals. And so, the proved equality is an
equality of areas of two symmetrical curvilinear trapezoids.
6.5. l l. Prove the equality
I I

~ f (x) g(t-x) dx = ~ g(x) f (t-x)dx.


0 0
292 Ch. VI. The Definite Integral

Solution. Apply the substitution t-x=z in the right-hand integral;


then we have
0 I

- ~ g (t - z) f (z) dz = ~ f (z) g (t - z) dz.


t 0
:rt :rt
2 2
6.5.12. Prove the equality~ sinmxdx= ~ cosmxdx and apply the
0 0
obtained result in computing the following integrals:
:rt :rt
2 2
~ cos 2 x dx and ~ sin 2 x dx.
0 0

Solution. On the basis of Problem 6.5. to we have


:rt :rt :rt
2 2 2
~ sinm x dx = ~ sinm ( ; - x) dx = ~ cosm x dx.
0 0 0

Hence, in particular,
:rt :rt
2 2
I=~ sin 2 xdx= ~ cos 2 xdx;
0 0

add these integrals:


:rt :rt
2
2/= s
0
(sin 2 x+cos 2 x)dx=
2
Sdx=~;
0
:rt
hence, != 4 .

6.5.13. Prove the equality


:rt
:rt 2
~ f (sin x) dx= 2 ~ f (sin x) dx.
0 0

Solution. Since
:rt
:rt 2 11

~ f (sin x) dx = ~ f (sin x) dx + ~ f (sinx) dx,


0 0 :rt
2
6.5. Simplification of Integrals 293

it is sufficient to prove that


n
n 2
~ f (sin x) dx= ~ f (sin x) dx.
n 0
2
In the left integral make the substitution
x
X='!t-f,
:re
dx= -dt, 2
0
Then
JI 0

~ f(sinx)dx=-~ f[sin(n-t)]dt=
n J1
2 2
J1 J1
2 2
= ~ f (sin t) dt = ~ f (sin x) dx.
0 0
6.5.14. Prove the equality
n n
5xf(slnx)dx= ~ 5f (sinx) dx.
0 0
Solution. In the left integral make the substitution

dx=-dt,
X=n-t, l~I
L_Uij.
Then we obtain
n 0
~xt(sinx)dx=- ~ (n-t)f[sin(n-t)]dt=
o n
JI J1

= ~ nf (sin t)dt - ~ tf (sin t)dt.


0 0
Whence
n :rr

2 ~ xf (sin x)dx = n ~ f (sin x) dx,


0 0

which is equivalent to the given equality.


294 Ch. VI. The Defini_te Integral

6.5.15. Using the equality

sin ( n++) x 1
- - - ' - - - - ' - = -2
. x
+cos x + cos 2x + ... +cos nx,
2 sm
2
prove that
:rr sin ( n+ ~) x
r
J . x
dx= :n:.
o sm2

6.5.16. Prove that if cp(x)=+a 0 +a 1 cosx+b 1 sinx+a 2 cos2x+


+b 2 sin 2x+ ... +an cosnx+bn sinnx, then
2:rr 2:rr

(a) ~ cp (x) dx = :n:a 0 ; (b) ~ cp (x) cos kx dx = :n:ak;


0 0
2:rr

(c) ~ cp(x)sinkxdx=:n:bk (k= l, 2, ... , n).


0

6.6. Integration by Parts. Reduction Formulas


If u and v are functions of x and have continuous derivatives,
then
b b

~ u (x) v' (x) dx = u (x) v (x)


a
1:- ~ v (x) u' (x) dx
a
or, more briefly,
b b

~ udv=uvl:- ~ vdu.
a a
I

6.6.1. Compute the integral ~ xex dx.


0
Solution. Let us put
x = u' ex dx = dv;
du = dx; v = ex'
which is quite legitimate, since the functions U=X and v=ex are
continuous and have continuous derivatives on the interval [O, l).
Using the formula for integration by parts, we obtain
I I

~ xex dx = xex j~ - ~ex dx = e- ex I~= 1.


6.6. Integration by Parts. Reduction Formulas 295

Jt
b
6.6.2. Compute the integral I= ~ eax sin bx dx.
0
Solution. Let us put
u=sinbx, dv= eax dx;
I
du= b cos bx dx, v= -eax.
a

Since the functions u=sinbx, V=_!_eax


a
together with their deri-
vatives are continuous on the interval [O, n], the formula for in-
tegration by parts is applicable:

"
I
l=-eaxsinbx
a
IJt
b --
0
bjT eaxcosbxdx=
a
0
:rt
b
=_ .!?._
a
Seaxcosbxdx= _.!?._a / 1
0

Now let us integrate by parts the integral / 1. Put

u =cos bx, dv=eax dx,


I
du= -bsinbxdx, V= -eax.
a
Then

l=-.!?._(\_!_eaxcosbxl~ +.!?._a
a
'

a 0
r
:rt

0
eaxsinbxdx
\

/
)=

( art ) (an )
=-.!:.__el) _ _!__ -~l=b eT+1 b2
-(;21.
a a a a~ a2
Hence

In particular, at a=b= l we get


1t

Sex sinx dx =+(en +l).


0
296 Ch. VI. The Definite Integral

e
6.6.3. Compute the integral ~ ln 3 x dx.
I
n'
4

6.6.4. Compute the integral ~sin Vxdx.


0

Solution. First make the substitution


Vx =t, x t
X= f 2 , 0 0
dx= 2t dt, n2 :rt
4 2
Whence
Jt2 .n
4 2
~ 3 in V x dx = 2 ~ t sin t dt.
0 0

Integrate by parts the latter integral.


Put
t=u; sin t dt =dv;
du= dt; V=-COS f.
Then

2{
0
tsintdt=2[-tcostl~ +f costdtl=2sintl~
0 0 0
=2.

I
__ \(' arc sin x
6.6.5. Compute the integral I ~ YI +x dx.
0
n
2
6.6.6. Compute the integral ~ x 2 sin x dx.
0
a
6.6.7. Compute the integral ln=~(a 2 -x2 )ndx, where n is a na-
o
tural number.
Solution. The integral can be computed by expanding the integrand
(a 2 -x2 )n according to the formula of the Newton binomial, but it
involves cumbersome calculations. It is simpler to deduce a formula
for reducing the integral In to the integral ln-i To this end let
6.6. Integration by Parts. Reduction Formulas 297

us expand the integral In in the following way:


a a
In=~ (a2-x 2)n- 1 (a 2-x 2) dx= a 2/ n- 1- ~ x (a 2 -x 2 1 x dx
0 0
r-
and integrate the latter integral by parts:

U=X; (a2 -x2)n-1 x dx = dv,


I
du=dx; v = - - ( a2 - x2 )n
2n
(n =I= 0).
We obtain

I
I n =a21 n-1 +-x(a2-x2)n
2n 0- -
a I
2n
I s
a
(a2-x2)"dx=a21 n-1 - -I
2n/ .
n
0
Whence
I 2 2n I
n=a 2n+I n-1

This formula is valid at any real n other than 0 and --} .


In particular, at natural n, taking into account that

we get
2n (2n-2) (2n-4) ... 642 a2n+1 (2n)!!
In =a 2n+i (2n+ I) (2n- I) (2n-3) ... 5.3 (2n+ I)!! '
where
(2n)!! =246 ... (2n),
(2n+ l)!! = 135 ... (2n+ 1).
6.6.8. Using the result of the preceding problem obtain the fol-
lowing formula:
C~ C~ C~ n C~ _ (2n)I!
l - 3 + 5 - 7 + ... +(-l) 2n+I -(2n+t)ll'
where C~; are binomial coefficients.
Solution. Consider the integral
I
I
n =- S(1 -X
2 )n dx
=
(2n)ll
(2n+ 1)11 '
0
298.~~~~--~~-C_h_._V_I_._T_he~D_e~fin_i_te~ln_te~g~ra_l~~~~~~~-

Expanding the integrand by the formula of the Newton binomial


and integrating within the limits from 0 to 1, we get:
I

ln= ~ (l-x 2 )ndx=


0
I

= ~ ( l -C~x 2 + C~x 4 - C~x 6 + ... + (-1 )n C~x 2 n) dx =


0

_I _ C~x3 C~x 0 _ C~x? (-l)n x2n+1 J1 _


- lX 3 + 5 7 +.'' + 2n+ I o-

c)i c~ ~~ (-l)n
= l - 3 + 5 - 7 + + 2n+l'
which completes the proof.
6.6.9. Compute the integral
n n
2 2
Hm= ~ sinmxdx= ~ cosmxdx
0 0
(m a natural number).
Solution. The substitution

tttt
sinx=t,
cosx dx=dt,

reduces the second integral to the integral


n
2 m-1 I m-1
Hm= ~ (l -sin 2 x)-2-cos x dx = ~ ( 1-t 2)_2_dt,
0 0
m-1
considered in Problem 6.6.7 with a= 1 and n = - 2 - . Therefore,
the reduction formula
m-1
Hm=~Hm_ 2 (m=t=O, m=t= 1)

is valid here, since


2 .m_-_I
2 m-1 m-1
Hm=lm-1= I /m-1 =--fm-3=--fim-2
-2- 2~+1 -2-- 1 m -2- m
2
6.6. Integration by Parts. Reduction Formulas 299

If m is an odd number, the obtained reduction formula reduces


Hm to
:rt
2
H1 =~ cos x dx = 1,
0
therefore
H = (m-1)11
m mll

If m is an even number then the reduction formula transforms


Hm into
:rt
2
H0 = Sdx= ~,
0
therefore
H _ (m-1) I!~
rn - m!I 2

6.6.10. Compute the integral


:rt
I= ~ x sin m x dx
0
(m a natural number).
Solution. Taking advantage of the results of Problems 6.5.14 and
6.5.13, we get
:rt
n n 2
I= Sxsinmxdx=~ Ssinmxdx=n S sinmxdx,
0 0 0

which, taking into consideration the result of Problem 6.6.9, gives


:n 2 (m- I)!! . .
n 2 m! 1 1f m 1s even,
{
/=~xsinmxdx= (m-1)!1 .
o rt mil 1f m 1s odd.
I

6.6.11. Compute the integral In= ~ xm (In x)n dx; m > 0, n is a


0
natural number.
Solution. First of all note that, though the integrand f (x) = xm (In x)n
has no meaning at x = 0 it can be made continuous on the interval
300 Ch. VI. The De"finite Integral

(0, 1] for any m > 0 and n > 0, by putting f (0) = 0. Indeed,

limxm(lnx)n= Jim (x:lnxY=o


X-++0 X-++0

by virtue of Problem 3.2.4.


Hence, in particular, it follows that the integral In exists at
m > 0, n > 0. To compute it we integrate by parts, putting
U=(lnx)n, dv=xmdx,
n(lnx)n- 1
du= x
dx,
Hence,
I
\
ln=~xm(lnx)ndx=
xm+l(lnx)nll
m+I
n s
I

o-m+lo xm(lnx)n-1dx=-m+1ln-1
n

The formula obtained reduces In to I n-l In particular, with a na-


tural n, taking into account that
I

lo= s
0
xmdx= m~ I,
we get
ln=(-l)n (m+n;)n+l'
I

6.6.12. Compute the integral lm,n= ~ xm(l--x)ndx,


0
where m and n are non-negative integers.
Solution. Let us put
(1-x)n =U; xm dx =dv;
du= - n (l-x)n- 1 dx;
Then
I

Im, n = l:~\ (1--x)n] ~ + m~ Is xm+1 (l -x)n-1 dx= m~ I /m+1, n-1


0

The obtained formula is valid for all n > 0, and m / - I. If n


is a positive integer, then, applying this formula successively n
times, we get
n n(n-1)
Im. n = m+ 1/ m+1,n-1 = (m+ I) (m+2) I m+2, n-2 =
_ n (n-1) ... [n-(n-1))
- (in+ I) (m+2) .. . (m+n) lm+n, o
6.7. Approximating Definite Integrals 301

But
xm+n+l I' - - - -
I

I -
m+n, o - 5xm+ndx-- m+n+ o - m+n+ I
I
I '
0
Hence,
n (n-1) (n-2) ... 32 l
Im, n = (m+ I) (m+2) .. . (m+n) (m +n+ I)
The obtained result, with m a non-negative integer, can be written
in the form
m!n!
/ m, n = (m+ n+ 1)1'
6.6.13. Compute the integrals:
I I

(a) ~arc tan V'xdx; (b) ~ (x- l)e-xdx;


0 0
Jl
3 I

(c) 5s~n~xx; ( d) ~ x arc tan x dx;


:re 0
4
:re
I 4
(e) ~ x In ( 1 + x 2 ) dx; (f) ~ ln(l+tanx)dx;
0 0
Jl
2 16
(g) ~ sin 2x arc tan (sin x) dx; (h) ~arctanyVx-ldx.
0 I
6.6.14. Prove that

0
I

5(arc cos x)n dx = n( ~ r- 1- n (n-1)


0
1

5(arc cos x)"- 2 dx (n > 1).


6.6.15. Prove that if f" (x) is continuous on [a, b], then the fol-
lowing formula is valid
b

~ xf" (x)dx= [bf' (b)-f (b)]-[af' (a)-f (a)].


a

6.7. Approximating Definite Integrals


1. Trapezoidal formula. Divide the interval [a, b] into n equal
.
parts by pomts k b-a
xk =a+ h, where h = ~. k = 0, 1, ... , n, and
apply the formula n
b

Sf (X)dx ~ b-;;a [ ~ f (x )+f (x 1) +


0 +f (xn-1)+ ~ f (xn)]
tl
302 Ch. VI. The Definite Integral

The error R in this formula is estimated as follows:


IR I~ M, l(~~a)a' where M, = sup Ir (x) I
n a<x<b
(assuming that the second derivative is bounded).
2. Simpson's formula. Divide the interval [a, b] into 2n equal
parts by points xk =a+ kh, where h = b 2na, and apply the formula
,,
Sf (x) dx ~ b6n a {f (x0 ) + f (X 2n) + 4 [f (X1 ) + f (x3 ) + ...
a
.. + f (x2n-1)] + 2 [f (x2) + f (x,) + ... + f (x2n-2)]}.
Assuming that tv
(x) exists and is bounded, the error in this formula
is estimated in the following way:

IR I~ ~~ci~;;J~)&,
n
where M 4 = sup
a<x<b
Ifv (x) I
I

6. 7. I. Approximate the integral I= S1_:x using the trapezoidal


0
formula at n = IO.
Solution. Let us tabulate the values of the integrand, the ordi
nates Y; = f (x,.) (i = 0, 1, ... , IO) being calculated within four de-
cimal places.

:q
I l+x1
I
I
!li=1 +x,
II
Xi
l l+Xi
I
I
111=1+x,

0.0000 1.0000 1.0000 0.6000 1.6000 0.6250


0.1000 1.1000 0.9091 0.7000 1.7000 0.5882
0.2000 1.2000 0.8333 0.8000 1.8000 0.5556
0.3000 1.3000 0.7692 0.9000 1.9000 0.5263
0.4000 1.4000 0.7143 1.0000 2.0000 0.5000
0.5000 1.5000 0.6667
-----
Using the trapezoidal formula, we obtain
I

I= 5I !x ~I~ ( l.OOOOt0.5000 +0.9091 +0.8333+


0

+ 0. 7692 + 0. 7143 + 0.6667 + 0.6250 + 0.5882 + 0.5fi56 +


+ o.5263) = /0 6.9377 = 0.6937 7 ~ o.6938.
6.7. Approximating Definite Integrals 303

Estimate the error in the result obtained. We have f" (x) =(I ~x)s.
Since 0 ~ x ~ 1, then If" (x) I~ 2. Consequently, we may take the
number 2 as M 2 and estimate the error:
2 I
IR I~ 12X 102 = 600 < 0.0017.
We calculated the ordinates accurate to four decimal places, and
the round-off error does not exceed -0.00005 (
1-0 - ( 1 + 9 X 1) = 0.00005 more
precisely, 0 0~~05 . 9 = 0.000045, since the ordinates Yo and y 10 are
exact numbers) . Thus, the total error due to using the trapezoidal
formula and rounding off the ordinates does not exceed 0.0018.
Note that when computing the given integral by the Newton-
Leibniz formula we obtain
I

I= Sl~x =ln(l +x) I~= In 2 ~ 0.69315.


0

Thus, the error in the result obtained does not exceed 0.0007, i. e.
we have obtained a result accurate to three decimal places.
1.5
6.7 .2. Evaluate by Simpson's formula the integral Seo.ix
x dx
0.5
accurate to four decimal places.
Solution. To give a value of 2n which ensures the required accu-
eo.1x
racy, we find f'v (x). Successively differentiating f (x) = - x , we get
ea.ix p (x)
f1v(x) =xr (0.000lx'-0.004x 3 +0.12x2 -2.4x+ 24)= X6e0 1 x,
where P (x) is the polynomial in parentheses. On the interval
[0.5, 1.5) the function cp (x) =eo,ix increases aJJd therefore reaches
its greatest value at x = 1.5: cp (1.5) = e0 10 < 1.2. The upper estimate
of the absolute value of the polynomial P (x) divided by x& can be
obtained as the sum of moduli of its separate terms. The greatest
value of each summand is attained at x= 0.5, therefore

I
p (x)
x&
I< 0.0001+0.004 + 0.12 + 2.4 + 24
x x2 xs x4
~
x& """
~ 0.0002+0.016 + 0.96 + 38.4 + 768 < 808.
And so, If 1v (x) I < 1.2 x808 < 1000. Hence, the number 1000 may
be taken as M,.
304 Ch. VI. The Definite Integral

We have to compute the integral accurate to four decimal places.


To ensure such accuracy it is necessary that the sum of errors of
the method, operations and final rounding off should not exceed
0.0001. For this purpose we choose a value of 2n (which will de-
termine the step of integration h) so that the inequality
IR I< f 0.0001=5.10- 0

is satisfied.
Solving the inequality
l&x 1 000 <5 IO-&
180 (2n) 4 X '
we obtain
2n > 19.
Let us take 2n = 20; then the step of integration h will be equal to
b-a 1
h=2n=20=0.05.
A more accurate calculation shows that at 2n = 20
IRl<35x10-&.
If we calculate Y; within five decimal places, i. e. with an error
not exceeding 10-&, then the error of the final rounding off will
also be not greater than l0- 0 Thus, the total error will be less
than 4.5x10-& < 0.0001.
eO.lX
Now compile a table of values of the function y =-x- for the va-
lues of x from 0.5 to 1.5 with the step h = 0.05. The ralculat ions
are carried out within five decimal places.

eO-IX

' I Xi
I 0.IXi
I I YI

0 0.50 0.050 1.05127 2.10254


1 0.55 0.055 1.05654 1.92098
2 0.60 0.060 1.06184 I. 76973
3 0.65 0.065 1.06716 1.64178
4 0.70 0.070 1.07251 1.53216
5 0.75 0.075 1.07788 l.4~717
6 0.80 0.080 1.08329 1.35411
7 0.85 0.085 1.08872 1.28085
8 0.90 0.090 1.09417 I. 21574 I
9 0.95 0.095 1.09966 I. 15754
10 1.00 0.100 1.10517 1.10517
11 1.05 0.105 1.11071 1.05782
6.7. Approximating Definite Integrals 305

12
I '"
1.10
I
O.lx1

0.110
I
80.1x1

1.11628
I YI

1.01480
13 1.15 0.115 1.12187 0.97554
14 1.20 0.120 1.12750 0.93958
15 1.25 0.125 1.13315 0.90652
16 1.30 0.130 1.13883 0.87602
17 1.35 0.135 1.14454 0.84781
18 1.40 0.140 1.15027 0.82162
19 1.45 0.145 1.15604 0.79727
20 1.50 0.150 1.16183 0.77455

For pictorialness sake we use the tabular data to compile the


following calculation chart:
Yi

I Xi at 1=0 and
at an odd at an even t
1=20
I 1
I
0 0.50 2.10254
I 0.55 1.92098
2 0.60 1.76973
3 0.65 1.64178
4 0.70 1.53216
5 0.75 1.43717
6 0.80 1.35411
7 0.85 1.28085
8 0.90 1.21574
9 0.95 1.15754
10 1.00 I. 10517
II 1.05 1.05782
12 1.10 1.01480
13 1.15 0.97554
14 1.20 0.93958
15 1.25 0.90652
16 1.30 0.87602
17 1.35 0.84781
18 1.40 0.82162
19 1.45 0.79727
20 1.50 0.77455
Sums 2.87709 12.02328 10.62893

Using Simpson's formula, we get


I. 5
eo.ix I
S-x-dx ~ 60 (2.87709 + 4 x 12.02328 +
0.5

+ 2 x 10.62893) = 60.
I
72.22807 = 1.2038.
306 Ch. VI. The Definite Integral

6.7.3. The river is 26 m wide. The table below shows the succes-
sive depths of the river measured across its section at steps of 2 m:

x I o I 2 I 4 I 6 \ 8 110 112 i 14 i 16 J 1s i 20 \ 22124126


y I 0.310.911.712.112.813.413.313.013.5\2.911.711.210.810.6
Here x denotes the distance from one bank and y, the correspond-
ing depth (in metres). Knowing that the mean rate of flow is
1.3 m/sec, determine the flowrate per second Q of the water in the
river.
Solution. By the trapezoidal formula the area S of the cross-sec-
tion
26

S = Sy dx ;:::::; 2 [ ~ (0.3 + 0.6) + 0.9 + 1. 7 + 2.1+2.8 + 3.4 +


0

+3.3 + 3.0+3.5 + 2.9 + 1.7 + 1.2 + 0.8 J = 55.5 (m 2 ).


Hence,
Q = 55.5x1.3;:::::; 72 (m 3 /sec).
It is impossible to estimate the error accurately in this case. Some
indirect methods of estimation enable us to indicate approximately
the order of the error. The error in S is about 3 m 2 , hence, the
error in Q is about 4 m 3 /sec.
6. 7.4. Compute the following integrals:
J1
2
(a) Ssi; x dx accurate to three decimal places, using Simpson's
J1
4
formula;
I

(b) ~ e-x2 dx accurate to three decimal places, by the trapezoidal


0
formula.
6.7.5. By Simpson's formula, approximate the integral
I. 36

I= ~ f(x)dx,
1.05
6.8. Additional Problems 307

if the integrand is defined by the following table:

x 1.05 1.10 1.15 1.20 1.25 1.30 1.35

f (x) 2.36 2.50 2.74 3.04 3.46 3.98 4.6

6.8. Additional Problems


6.8.1. Given the fund ion
1-x at O~x~ 1,
f (x) = { 0 at l < x~ 2,
(2-x) 2 at 2 < x~ 3.
Check directly that the function
x

F (x) = ~ f (t) dt
0

is continuous on the interval [O, 3] and that its derivative at each


interior point of this interval exists and is equal to f (x).
6.8.2. Show that the function

J ~ ::
at
f (x) = l 0
at
~ -1 at x= l
is integrable on the interval [O, l].
6.8.3. Can one assert that if a function is absolutely integrable
on the interval [a, b], then it is integrable on this interval?
6.8.4. A line tangent to the graph of the function 1; = f (x) at the
point x =a forms an angle ~ with the axis of abscissas and an
angle ~ at the point x =b.
b

Evaluate ~ f" (x) dx, if f" (x) is a continuous fund ion.


a
6.8.5. Prove that
x
5E (x) dx = E (x)(2(x)- l) + E (x) [x-E (x)].
0
308 Ch VI. The Definite Integral

n
6.8.6. Given the integral S1+ dxcos 2
x
Make sure that the fun-
0
ct ions
I 2 cos x I tan x
F 1 (X) = .r- arc cos V and F 2 (x) = .r-- arc tan .r-
r 2 I+ cos x
2 r 2 r 2
are antiderivatives for the integrand. Is it possible to use both an-
tiderivatives for computing the definite integral by the Newton-
Leibniz formula? If not, which of the antiderivatives can be used?
6.8. 7. For f (x) find such an antiderivative which attains the given
magnitude y =Yo at x = x 0 (Cauchy's problem).
/J

6.8.8. At what value of is the equality ~ e2 x dx=e2 ~ (b-a) ful-


u
filled? Show that
t
..,
>a+b
2 .

6.8.9. Investigate the function defined by the definite integral


x
F (x) =~Vt - t ' dt.
0

6.8.10. Show that the inequalities


I

0.692 ::::;; ~ xx dx::::;; 1


0
are valid.
6.8.11. With the aid of the inequality x~sinx~ ~ x (o::s;;x::s;;
l"I

::::;; 2Jt \) show t hat I < s


2
-x-x dx
sin < 2:n .
0

6.8.12. Using the inequality sin x ~ x- x: (x~O) and the Schwarz-


Bunyakovsky inequality, show that
n
2
1.096< \ Vxsinxdx <I.Ill.
0
6.8.13. Assume that integrable functions Pt (x), p 2 (x), Pa (x), p, (x)
are given on the interval [a, b], the function Pt (x) is non-negative,
6.8. Additional Problems 309

and the functions p 2 (x), Pa (x), p 4 (x) satisfy the itiequality


Pa (x) ~ P2 (x) ~ P4 (x).
Prove that
" b b
~ Pa (x) p 1 (x) dx ~ ~ p 2 (x) p 1 (x) dx ~ ~ p 4 (x) p, (x) dx.
u a a

6.8.14. Let the function f (x) be positive on the interval [a, b].
Prove that the expression
b h

Sf (x) dx Sf~;>
a a

reaches the least value only if f (x) is constant on this interval.


6.8.15. Prove that
l1

0
I

Sarc tan
x
x dx == ..!._
2
2
s-.
0
l - dt.
sm t

6.8.16. Prove that one of the antiderivatives of an even function


is an odd function, and any antiderivative of an odd function is
an even function.
6.8.17. Prove that if f (x) is a continuous periodic function with
a+T
period T, then the integral I= ~ f (x) dx does not depend on a.
a

6.8.18. Prove that if u = u (x}, v = v (x) and their derivatives


through order n are continuous on the interval [a, b], then
b

~ uvwi dx = [uv1n-u_u'vln-2> + ... + (-l)n-1 u<n-u v] I~+


a
b

+(-l)n ~ ulr> v dx.


a
Chapter 7
APPLICATIONS
OF THE DEFINITE INTEGRAL

7.1. Computing the Limits of Sums with


the Aid of Definite Integrals
It is often necessary to compute the limit of a sum when the
number of summands increases unlimitedly. In some cases such li-
mits can be found with the aid of the definite integral if it is pos-
sible to transform the given sum into an integral sum.
For instance, considering the points _!_, _! , ... , !!:.. as points of
n n n
division of the interval [O, l] into n equal parts of length ~x = _!__,
n
for each continuous fun.ct ion f (x), we have
1

}~~ -!z [ f ( ~ ) + f ( ! )+ ... + f ( ~ ) ] = Sf (x) dx.


0
7.1.1. Compute
lim -nn Ism
. -nn + sm. -2:rt + ... + sm. (n-1)
- -nl .
n n
ti .. "'

Solution. The numbers in brackets represent the values of the


function f (x) =sin x at the points
_ n. 2n. . (n-1) n
X,-n' X2=/j ... , Xn-1= - n - '

subdividing the interval [O, n] into n equal parts of length ~x= ~.


n
Therefore, if we add the summand sin nn = 0 to our sum, the Iat-
n
ter will be the integral sum for the function f(x)=sinx on the
interval [O, n].
By definition, the limit of such an integral sum as n - oo b the
definite integral of the function f (x) =sin x from 0 to n:
Jim -n
11 (
sm -n
11
+ sm -:lnn + ... + sm. (n-1)

n
n
+ sm. -nn)
n =
n-o"'
"'
= S sinxdx=- cos xi:= 2.
0
7.1. Computing the Limits of Sums 311

7.1.2. Compute the limit


. ( ,,,- I
I1m + .,,- _1 + ... + y 4n 1-n ) .
n-+a:ir 4n 2 - l r 4n 2 -2 2 2 2

Solution. Transform the sum in parentheses in the following way:


I
.,,-
r 4n 2- I
+ .,,-r 4n I -2 + ... + 1/
2 2 r 4n -n 2 2
-

I I I l
=n( V I I+ -.I 4
4 -fi2 v 4 - ( ~ )2)
v - ( n2)2+ ... + -.I
The obtained sum is the integral sum for the function f (x) =
= 1
1,,- on the interval [O, 1] subdivided into n equal parts.
r 4-x2
The limit of this sum as n--+ oo is equal to the definite integral
of this function from 0 to 1:

Sy 4dx-x2 =arc sm. 2x IIo =


l
:rt
= 6.
0
7.1.3. Compute

l + Vn~3+ Vn~6+ Vn~9+ + Vn+3~n-I)]'


}~m,,, ~ 1
Solution. Transform the given expression in the following way:

n + V n+a+ V n+6+ + V n+3(n-I) 1-


~ll -.In
- -.In -.I n

=~[ I
n
.V I
1+0
+-.I
V
I
I+~
+-.I
V i+~
I + ... +-./_l__ J
V i+3(n-I)
n n n

The obtained sum is the inte.gral sum for the function f (x) =
= Vr 1 !x on the interval [0, 3]; therefore, by defi.nit ion,

!~n"' ~ (l+ Vn~3+ Vn~6+ + Vn+3~n-I))=


3 3 l

= j. Vi!xdx= Jo+x)- dx=2V l+xl:=4-2=2. 2


0 0
312 Ch. VII. Applications of the Definite Integral

7.1.4. Using the definite integral, compute the following limits:


.
I1m I + +I +
( -+ I );
... +-+
n 1 n 2
(a) n-oo n n

(b) lim
n-oo n
+ 1 + -. /'I+ 2 + ... + -. /' l +
_!__ ( -. / ' l
Vn V n V
n);
n
I+ V 2+ V 3+ ... + v-~.
.
(C) IIm
n - 00
v n4
,
( d)
.
IIm
n-+oo
:re (
-2
n
l + COS -2:rtn -J- COS -2:n (n- I) :n) .
2 n -1- .. + COS - 2 n
- ,

(e) }~~ n [(n~l)2+(n~2)2+ .. + (2~) 2 J


7. 1.5. Compute the limit A= Jim _n_. .
n-oo
v' n
Solution. Let us take logarithms

In A = lim In
n-oo
;:;-;;i
n
= Jim J_ [in_!__+ In !_ + ...
n-+oo n n n
+ ln !!_l
n

The expression in brackets is the integral sum for the integral
I

~In xdx = (xln x-x)I~= -1.


0

Conseq uen t ly, lnA=-1 and Jim __n_! =e- 1


v-- n
,, .... 00

7.2. Finding Average Values of a Function


The average value of f (x) over the interval [a, b] is the number
b
I ('
= b-a j f (x) dx.
a
b _I_
The square root { b~a S[f (x)]2 dx} 2
of the average value of the
a
square ol the function is called the root mean square (rms) of the
function f (x) over [a, b].
7.2.1. Find the average value of the function f (x) = V x over
the interval [O, I].
Solution. In this case
4

=
I
1-0
5V xdx=-4- 11o =4
l
3 ,- 3x 3
-
3

c
7.2. Finding Average Values of a Function 313

7.2.2. Find the average values of the functions:


(a) f (x) = sin 2 x over [O, 2n];
I
(b) f (x) =ex+ 1 over [O, 2].

7.2.3. Determine the average length of all vertical chords of the


x2 y2
hyperbola a 2 - b2 = l over the interval a~ x ~ 2a.

Solution. The problem consists in finding the average value of


the function f(x)=2y=2!:.Vx
a
2 -a2 over the interval [a, 2a]:

2a
I r>bv--
=2- \ - x2 -a 2 dx=
a J a
a

=!~[; Vx 2 -a2 -~ ln(x+ Vx2 -a2 )J:a=b[2V3-1n(2+V3)].


7 .2.4. Find the average ordinate of the sinusoid y =sin x over
the interval [0, n].
Solution:
n n
=...!..
n
Ssin xdx =-..!.._cosx
n
=~
n
~ 0.637.
0 0

Rewrite the obtained result in the following way:


:t

n = ~ n = Ssinxdx.
0

Using the geometric meaning of the definite integral, we can say


that the area of the rectangle with the altitude=~ and the base
n
n equals the area of a figure bounded by a half-wave of the sinu-
soid y =sin x, 0 ~ x ~ n, and by the x-axis.
7.2.5. Find the average length of all positive ordinates of the
circle x 2 + y2 =I.
7.2.6. Show that the average value of the function f (x), conti-
nuous on the interval [a, b], is the limit of the arithmetic mean
of the values of this function taken over equal intervals of the
argument x.
Solution. Subdivide the interval [a, b] into n equal parts by the
points xi=a+ib-"/ (i=O, I, 2, ... , n).
314 Ch. VII. Applications of the Definite Integral

Form the arithmetic mean of the values of the function f (x) at


n points of division x 0 , x1' ... , Xn_ 1 :
n-1
n = f (Xo)+f (x1)~ .. +f (Xn-1) = ~ LJ (xi)
i=O

This mean may be represented in the following form:


I n-1
n = b-a L f (xi) ~xi,
i=O
where
b-a
~Xi=--.
n
The latter sum is the integral sum for the function f (x), the-
refore
n-1 b
lim n
n-oo
= b~-an-
Jim L f (xi) ~xi= b I ajr f (x) dx = ,
i=O a

which completes the solution.


7 .2. 7. Find the average value of pressure (Pm) varying from 2
to 10 atm if the pressure p and the volume v are related as follows:
3
pv 2 =160.
Solution. As p varies from 2 to 10 atm, v traverses the interval
[4V4. 4V100]; hence
4 v10o 3

Pm=
4
(V 100- V 4 )
1
.
f 160v -2 dv =
4V4
-_ _
(V 320
)v
-+ 14 v-
ViOO -V
_ (V40 V ),...,4.32atm.
,...,
100- v 4
3'
4 20410+ 2 4

7 .2.8. In hydraulics there is Bazin's formula expressing the velo-


city v of water flowing in a wide rectangular channel as a function
of the depth h at which the point under consideration is situated
below the open surface,
V=V 0 - 20 V HL (~ ) 2 ,
where v0 is the velocity on the open surface, H is the depth of
the channel, L its slope.
Find the average velocity vm of flow in the cross-section of the
channel.
7.2. Finding Average Values of a Function 315

Solution. We have
H

Vm = ~ S[v -20 VHL(~ )2]dh= v


0 0- VHL.
2
3
0
7.2.9. Determine the average value of the electromotive force
Em over one period, i.e. over the time from t = 0 to t = T, if
electromotive force is computed by the formula
EE .2nt
= 0 sm 7 ,
where T is the duration of the period in seconds, 0 the amplitude
(the maximum value) of the electromotive force corresponding to
the value t =0.25T. The fraction 2~t is called the phase.
Solution.
Eo
E m=y s.
T
2:nt dt
smy E0 T [ , 2:nt] T
=r.2:n -cosy 0 = .
O
0

Thus, the average value of the electromotive force over one pe-
riod equals zero.
7 .2.10. Each of the two vertical poles OA and CD is equipped
with an electric lamp of luminous intensity i fixed at a height h.
The distance between the poles is d. Find the average illumination
of the straight line OC connecting the bases of the poles.
7 .2.11. Find the average value of the square of the electromotive
force (E2)m over the interval from t = 0 to t = ~ (see Prob-
lem 7.2.9).
Solution. Since
. 2:nt
E = E 0 smy.
we have
T 4:nt
!.._

( 2) m =~
T
2o s.
2

sm 2 2:nt
T
~ 2o ('\
dt --T
l-cos-
2
2
T dt --
0 0

-
-
E~
T
[t-.!..
4n
s'1n 4:nt] ~ = E~
T o 2
7.2.12. If a function f (x) is defined on an infinite interval
[O, oo ), then its average value will be
b

= !~~ ! Jf (x) dx,


0
316 Ch. VII. Applications of the Definite Integral

if this limit exists. Find the average power consumption of an


alternating-current circuit if the current intensity I and voltage u
are expressed by the following formulas, respectively:
I = I 0 cos (wt + a);
u = U 0 cos (wt+ a+ <p),
where <p is the constant phase shift of the voltage as compared
with the current intensity (the parameters w and a will not enter
into the expression for the average power).
Solution. The average power consumption
T

Wm= Jim
T-.oo
f SJ cos(wt+a)u cos((J)t+a+<p)dt.
O
0 0

Taking into consideration that

cos a cos ~ = 2I [cos (a + ~) + cos (a - ~)],


we will get
T

wm =Jim 1~~0 S[cos (2wt + 2a.+ <p) +cos <p] dt =


T .... oo 0
_ 1. {/ 0 u 0 .sin(2ffiT+2a+cp)-sin(2a+cp)+/ 0 u0
_ tm 4w
l_l 0 u0 cosqi.
T 2 cosqil - 2
T -.oo
Hence, it is clear why so much importance is attached to the
quantity cos qi in electrical engineering.
7 .2.13. Find the average value of the function f (x) over the
indicated intervals:
(a)f(x)=2x2 +l over [O, l];
(b) f (x) =J_x over [I, 2];
(c) f(x)=3x-2x+3 over [O, 2].
7 .2.14. A body falling to the ground from a state of rest acqui-
res a velocity v1 = V2gs 1 on covering a vertical path S=S 1 Show
that the average velocity vm over this path is equal to 2 ~1
7.2.15. The cross-section of the trough has the form of a para-
bolic segment with a base a and depth h. Find the average depth
of the trough.
7.2.16. Find the average value Im of alternating current intensity
over time interval from 0 to ~
(!)
(see Problem 7.2.12).
7.3. Computing Areas in Rectangular Coordinates 317

7.2.17. Prove that the average value of the focal radius of an


ellipse p = 1 P , where p = b2 ; a, b are the semi-axes and e is
-e cos cp a
eccentricity, is equal to b.
7 .2.18. On the segment AB of length a a point P is taken at a
distance x from the end-point A. Show that the average value of
the areas of the rectangles constructed on the segments AP and
a2
PB is equal to 6 .
7 .2.19. Find the average value of the function
cos 2 x
f (x) = sin2 x+4 cos 2 x

over the interval l


0, %]. Check directly that this average, equal
I
to 6 , is the value of the function f (x) for a certain x = lying
within the indicated interval.

7.3. Computing Areas in Rectangular Coordinates


If a plane figure is bounded by the straight lines X= a, x =
=b(a<b) and the curves y=y 1 (x), y=y 2 (x), pro,lided y 1 (x)~
~ y 2 (x) (a~ x ~ b), then its area is computed by the formula
b

S= ~ [y 2 (x)-y1 (x)] dx.


a

In certain cases the left boundary x =a (or the right boundary

y
]]

A
.c
x=b) can degenerate into a point of intersection of the curves

,8
I
!I

: !f=fft(x)
I
iI
:.c x
0 a b 0 a b
(a) {b)
Fig. 65

y = y 1 (x) and y = y 2 (x). Then a and b are found as the abscissas


of the points of intersection of the indicated curves (Fig. 65, a, b).
7 .3.1. Compute the area of the figure bounded by the straight
lines x=O, x=2 and the curves y=2x, y=2x-x2 (Fig. 66).
318 Ch. VII. Applications of the Defi,nite Integral

Solution. Since the maximum of the function y = 2x-x2 is at-


tained at the point x = 1 and is equal to 1, and the function
y= 2x ~ 1 on the interval [O, 2], we have
2

S= S[2x -(2x- x 2 )] dx = 1~x2 1: -( x 2- ;') 1: = 1 ~ 2 - ~ .


0

7 .3.2. Compute the area of the figure bounded by the parabolas


X=-2y 2 , X=l-3y 2 (Fig. 67).
y
4
!I

y=2x-x 2
1

----------- -1
0 1 2
Fig. 66 Fig. 67

Solution. Solving the system of equations


X= 2y2 ;
{
X= l-3y2 ,
find the ordinates of the points of intersection of the curves y 1 =-1,
y 2 = 1. Since l -3y2 ~-2y 2 for -1 <,. y <,. 1, then we have
I

S= s[(l-3y2)-(-2y
-1
2 )] dy=2 (y-~3 ) I~=~.
7.3.3. Find the area of the
figure contained between the !f
parabola x2 = 4y and the witch
of Agnesi y = ~ x, 4 (see Fig.
68).
Solution. Find the abscis-
sas of the points A and C of ..1....---=:..o~i.::__..__ _ __.x
~--_2
intersection of the curves. For
this purpose eliminate y from Fig. 68
7.3. Computing Areas in Rectangular Coordinates 319

the system of equations

8 x2
whence x2 + 4 = 4 , or x'+4x2 -32=0.
The real roots of this equation are the points Xi= -2 and
X2 =2. As is seen from the figure, x 2 ~ 4 ~ ~
2 on the interval

[-2, 2]. (It is also p~ssible to ascertain this by directly computing


the values of these functions at any point inside the interval, for
instance, at x = 0.)
Consequently,
2

S= S (x2 ~ 4 -~2 )dx=(4arctan ;-~;)[~ 2 =2n-f.


-2

7.3.4. Find the area of the figure bounded by the parabola


y=x2 + 1 and the straight line x+y=3.
7.3.5. Compute the area of the figure which lies in the first qua-
drant inside the circle x2 + y2 = 3a2 and is bounded by the parabo-
las x2 = 2ay and y 2 = 2ax (a> 0) (Fig. !f
69).
Solution. Find the abscissa of the po-
int A of intersection of the parabola
y2 = 2ax and the circle x2 + y2 = 3a2
Eliminating y from the system of equa-
tions
{
+
xz y2 = 3a2'
y 2 =2ax,
we obtain x2 + 2ax-3a2 = 0, whence we Fig. 69
get the only positive root: xA =a. Analo-
gously, we find the abscissa of the point D of intersection of the
circle X2 + y 2 = 3a2 and the parabola X2 = 2ay; Xv= a V2.
Thus, the sought-for area is equal to
a VI
S= ~ [y2 (x)-Yi(x)]dx,
0

V2Ux for 0 ~ x ~a,


x2
where Yi (x) = -2a ' Y2
(x) __ f
- I
l
r
V 3a2 -x2 for a< x~a
v-2.
320 Ch. VII. Applications of the Definite Integral

By the additivity property of the integral


u aV2
S = 5(V 2ax- ~:) dx + 5 (V3a 2- x2 - ~:) dx =
0 a

.. - 2 ~ xa ] a [ x 3a2 x xa ] a V2
= [ V2a -x 2 - - - + -V3a2 -x2 +-arcsin--_--- =
3 6a o 2 '2 a JI 3 6a a

=2-JI 2 2 a2 3a2 (
3- a - 6 + 2
. V2 _
arcsm V 3 arcsm V 3- ) - V3 2 a+
_l 2 J__ 2 _
6 a -

-(2+3
- - 3- 2 . 3') a.2
arcsm
Here we make use of the trigonometric formula:
arcsina-arcsin~ =arcsin(a J/l-~ 2 -~ Vl-a 2 ) (a~> 0)
for transforming

arcs1n
. -.V/2-arcsm
. V3
I . ( -. /2 -. /2 I I
=arcstn V 3 V 3 - y- 3 y- 3
)
=
3
. I
=arc sin 3 .

7.3.6. Compute the area of the figure lying in the first quadrant
and bounded by the curves y 2 = 4x,
Y x2 = 4y and x2 + y2 = 5.
7.3.7. Compute the area of the
figure bounded by the lines y =
x+ I, y=cosx and the x-axis
__.c."-'-LJ.~L.f-L-'~-'-'-'-lt...L.L.UL:-~ x (Fig. 70).
-1 0 1
Fig. 70
Solution. The function

( x+ 1 if -1 ~x~O,
Y = f (x) = l cos x if 0~x~ ~

is continuous on the interval [ -1, ~ J . The area of the curvili-


near trapezoid is equal to
n n
2 0 -2
S= S f(x)dx= 5 (x+ l)dx+ 5cosxdx=(x~l) 2 [ 1 +sinxj 0
; ={.
-I - I 0

7.3.8. Find the area of the segment of the curve y 2 =x3 -x2 if
the line x = 2 is the chord determining the segment.
7.3. Computing Areas in Rectangular Coordinates 321

Solution. From the equality y 2 = x 2 (x- 1)


it follows that
x2 (x-1) ~ 0, therefore either x = 0 or ~
1. In other words, the
x
domain of definition of the implicit function y2 = x 3 -x2 consists
of the point X=O and the interval [l, oo). In computing the area
the isolated point (0, 0) does not play any role, therefore, the
interval of integration is [1, 2) (see Fig. 71).
Passing over to explicit representation y = x V x- 1, we see
that the segment is bounded above by the curve y=xVx-1 and
below by the curvey=-xVx-1. Hence,
2 2
S = ~ [ x V x- 1-(-x V x- 1)] dx = 2 ~ x V x- 1 dx.
I I

Make the sub st itut ion


!/

x-l=t 2 ,
dx= 2t dt, I 0
'2 I

Then
I

S= 4 s( +
0
f2 1) f 2 dt = 4 [~
5 + ~]
3 0I = 15.
32

7 .3.9. Determine the area of the figure


bounded by two branches of the curve
(y-x) 2 = x 3 and the straight line x = 1.
Solution. Note first of all that y, as an Fig. 71
implicit fund ion of x, is defined only for
x ~ O; the left side of the equation is always non-negative. Now
we find the equations of two branches of the curve y= x-xVX,
y =x+xVx. Since x~o. we have x+xVx~x-xVx, and
therefore
I

S= S(x+x V -x-x +x v-x)dx= 2 sx v-xdx==s-x211 =s.


I
4
5

0
4
0 0

7.3.10. Compute the area enclosed by the loop of the curve


y 2 =x(x-l) 2
Solution. The domain of definition of the implicit function y is
+
the interval 0 ~ x < oo. Since the equation of the curve conta-
ins y to the second power, the curve is symmetrical about the
322 Ch VII. Applications of the Cefinite Integral

x-axis. The positive branch y1 (x) is given by the equation

= (x>=Vx!x-ll={Vx(l-x), O~x~I.
Y Y1 Vx(x-l), x>l.
The common points of the symmPtrical branches y 1 (x) and y 2 (x) =
= -y1 (x) must lie on the x-axis. But y1 (x) = Vx Ix- 11=0 only
at x1 = 0 and at x2 = l.
Consequently, the loop is formed by the curves y=Vx(l-x)
and y = - J/x (1 - x), 0 ~ x ~ l (see Fig. 72), the area enclosed
being

S= 2 Sv-x ( 1- x) dx = 2 s' ( xT-x : ) dx = 158 .


I

0 0

7 .3.11. Find the area enclosed by the loop of the curve


y 2 = (x- l)(x-2)2

!I
7

Fig. 72 Fig.73

7 .3.12. Find the area of the figure bounded by the parabola


y= - x2 -2x+ 3, the line tangent to it at the point M (2, -5)
and the 11-axis.
Solution. The equation of the tangent at the point M (2, -5)
has the form y+5=-6(x-2) or y=7-6x. Since the branches
of the parabola are directed downward, the parabola lies below the
tangent, i.e. 7-6x~-x2 -2x+3 on the interval [O, 2] (Fig. 73).
Hence,
2 2
S= S[7-6x-(-x -2x+3)] dx= S(x 2 2 -4x+4)dx =~.
0 0
7 .3. Computing Areas in Rectanf!ular Coordinates 323

7.3.13. Find the area bounded by the parabola y=x 2 -2x+2,


the line tangent to it at the point M (3, 5) and the axis of ordi-
nates.
7.3.14. We take on the ellipse
x2 y2
Q2+1,2= 1 (a> b)
a point M (x, y) lying in the first quadrant.
Show that the sector of the ellipse bounded by its semi-major
axis and the focal radius drawn to the point M has an area
ab x
s= 2 arc cos a .
With the aid of this result deduce a formula for computing the
area of the entire ellipse.

!!

Fig. 74 Fig. 75

Solution. We have (Fig. 74):


_xy_ b
S
OMAO = S ilOJ}IB+ S MABM; S
ilOMB- :r- 2a x v-2--2.
a -x'
u a

SMABM=\ydx=
<1
" 5-Vab
a
-2--t 2 d t =b- (
2a
t va --t +a arcsin-t ) la =
2 2 2
a x
x ,

.
S mce -:rt2 -arc stn

-ax =arc cos-,
x we o bt am
.
a

S MABJ}1=2l
b [ -X v-2
a --2
X + a 2 arc COS ax 1,
Hence
SOMAO =
SMMB + S MABM + 2ab arc COS ax
324 Ch. Vil. Applications of the Definite Integral

At x=O, the sector becomes a quarter of the ellipse, i.e.


IS ah abn ab
4 elllpse=2arccos0=2 2=4n.
and consequently, Seillpse=nab. At a=b we get the area of a circle
sclrcle = na 2
x2
7.3.15. Find the area bounded by the parabolas y=4x2 , Y=g-
and the straight line y = 2.
Solution. In this case it is advisable to integrate with respect
to y and take advantage of the symmetry of the figure (see Fig. 75).
Therefore, solving the equations of the parabolas for x, we have:

X
yY-
= -2 X=3 v-y.
By symmetry of the figure about the y-axis the area sought is
equal to the doubled area SoABo:
2 :>

S=2S 0 A80 =2 S(3 V -y-21 v-)y dy=5 sv-ydy=-;j-.


20Jf2
0 0
7.3.16. From an arbitrary point M(x, y) of thecurvey=xm
(m > 0) perpendiculars MN and ML (x > 0) are dropped onto the
coordinate axes. What part of the area of the rectangle ON ML does
the area ON MO (Fig. 76) constitute?

!J y
M g=.xm 1
L

.J:

0 N
x
-1 ---y;e- --
--c<-:t

Fig. 76 Fig. 77

7.3.17. Prove that the areas S 0 , Sl' S 2 , S 3 , , bounded by the


x-axis and half-waves of the curve y = e-ax sin px, x ~ 0, form a
an
geometric progression with the common ratio q = e-tr.
Solution. The curve of Fig. 77 intersects the positive semi-axis
Ox at the points where. sin Px= 0, whence
Xn= ;'Jt' n=O, I, 2, ...
7.3. Computing Areas in Rectangular Coordinates 325

The function y = e-u sin Px is positive in the intervals (x 2 k, X 2k+i)


and negative in (x 2 k+i X2 k+ 2 ), i.e. the sign of the function in the
interval (xn, xn+i) coincides with that of the number (-l)n. Therefore
(n+ l)n (n+ l)n
-II- -,1-

S=
n ~ lyldx=(-l)n ~ e-axsinpxdx.
nn nn
lf If
But the indefinite integral is equal to
S e-q
e-x sin Pxdx = - a2 +~ 2 (a sin Px+P cos Px)+C.
Consequent Iy,

Hence
S e -a(n+l)n/ll
n+I - e-ait/ll
q = -S-
n
= e-ann/ll - '

which completes the proof.


7.3.18. Find the areas enclosed between the circle x 2 +y2 -2x+
+4y-ll=0 and the parabola y=-t2 +2x+l-2V3.
Solution. Rewriting the equations of the curves, we have:
(x-l) 2 +(y+2) 2 =l6, !f
y=-(x-1) 2 -2 V3+2.
Consequently, the centre of the
circle lies at the point C (I, -2)
and the radius of the circle equals
4. The axis of the parabola coin-
cides with the straight line x = 1
and its vertex lies at the point
B (1, 2, -2 V 3) (Fig. 78).
The area S ABDFA of the smaller
figure is found by the formula
"D
F
S ABDFA = ~ (Ypar-Ycirc1e) dx,
"A Fig. 78
326 Ch. VII. Applications of the Definite Integral

where xA and xv are determined from the system of equations


j (x-1) 2 + (y+ 2)2 =16,
\ y+2=-(x-1) 2 -2y"3 +4,
whence XA=-1, Xv=3.
Hence,
3

SABDFA = ~ [(-x 2 + 2x+ 1-2V3) + (2 + Vl6-(x- l) 2 )] dx=


-1
rl x3 ;- x I -
= - 3 +x2 +(3-2l 3)x+-; V 16-(x-1)2 +
16
+ 2 arcsin --4-
x-JJ3 = 3 -8
1
32 ;- --
~ 3 +2V12+16arcsin 2
I
=
32
=3--4 v-3 +3 8
:rt.

The area of the second figure is easy to determine.


Note. The computation of the integral can be simplified by using
the shift x-1 = z and taking advantage of the evenness of the
integrand.
7.3.19. Compute the area bounded by the curves y=(x-4)2,
y = 16-x2 and the x-axis.
7 .3.20. Compute the area enclosed between the parabolas
3
x = y2; x = 4 y2 + 1.
7.3.21. Compute the area of the portions cut off by the hyper-
bola x 2-3y2= 1 from the ellipse x2+4y2=8.
7.3.22. Compute the area enclosed by the curve y 2 =(l-x2 ) 3
7 .3.23. Compute the area enclosed by the loop of the curve
4 (y2 -x2) + xa = 0.
7 .3.24. Compute the area of the figure bounded by the curve
Vx + J/y = 1 and the straight line x+ y= 1.
7 .3.25. Compute the area of the figure enclosed by the curve
y2 = x2 ( 1- x2).
7 .3.26. Compute the area enclosed by the loop of the curve
x3+x2-y2 =0.
7 .3.27. Compute the area bounded by the axis of ordinates and
the curve x = y 2 ( 1-y).
7 .3.28. Compute the area bounded by the curve y = x 4 - 2x 3 +
+ x 2 + 3, the axis of abs<issas and two ordinates corresponding to
the points of minimum of the function y (x).
7.4. Areas with Parametrically Represented Boundaries 327

7.4. Computing Areas with Parametrically Represented


Boundaries
If the boundary of a figure is represented by parametric equations
x = x (t), y = y (t),
then the area of the figure is evaluated by one of the three for
mu las:
fl ti fl
S =- Sy (t) x' (t) dt;
u
s= sx
a
(t) y' (t) dt; S= ~ S(xy' -yx')dt,
a
where a and ~ are the values of the parameter t corresponding
respectively to the beginning and the end of the traversal of the
contour in the positive direction (the figure remains on the left).
7.4.1. Compute the area enclosed by the ellipse
x=acost, y=bsint (O~t~2n).

Solution. Here it is convenient first to compute


xy' -yx' =a cost xbcos t +b sin t xa sin t =ab.
Hence
2;c 2:n

S= i S(xy' -yx') dt
0
= ~ Sab dt =nab.
0
2 2
7.4.2. Find the area enclosed by the astroid (:) 3 +( ~) 3 = 1.
Solution. Let us write the equation of the astroid in parametric
form: x =a coss t, y =a sins t, 0 ~ t ~ 2n. Here it is also conve-
nient to evaluate first
xy' -yx' = a2 (coss t 3 sin 2 t cost+ sins t 3 cos2 t sin t) =
= 3a2 cos2 t sin 2 t.
Hence,
2:n 2:rt

S= ~ S(xy'-yx')dt= ~ a S sin 2 2 2tdt=fa2 n.


0 0

7 .4.3. Find the area of the region bounded by an arc of the


cycloid x=a(t-sint), y=a(l-cost) and the x-axis.
Solution. Here the contour consists of an arc of the cycloid
(0 ~ t ~ 2n) and a segment of the x-axis (0 ~ x ~ 2na). Let us
fl
apply the formula S=-~yx'dt.
a
328 Ch. V 11. Applications of the Definite Integral

Since on the segment of the x-axis we have y = 0, it only re-


rpains to compute the integral (taking into account the direction
of a boundary traversal):
o 2n
S=-~ a(l-cost)a(l-cost)dt=a2 ~ (l-cost) 2 dt=
~ 0
2n

= a2 S [ 1- 2 cost + ~ (1 + cos 2t) J dt = 3na2


0

7.4.4. Compute the area of the region enclosed by the curve


x=asint, y=bsin2t.
Solution. When constructing the curve one should bear in mind
that it is symmetrical about the axes of coordinates. Indeed, if we
substitute n-t for t, the variable x remains unchanged, while y
only changes its sign; consequently, the curve is symmetrical about
the x-axis. When substituting n+ t for t the variable y remains
unchanged, and x only changes its sign, which means that the
curve is symmetrical about the y-axis.

ff

t~Jf
-f~~~~~~'+--~~~~--:2:..-+-x

Fig. 79

Furthermore, since the functions x=asint; y=bsin2t have a


common period 2n, it is sufficient to confine ourselves to the fol-
lowing interval of variation of the parameter: 0 ~ t ~ 2n.
From the equations of the curve it readily follows that the va-
riables x and y simultaneously retain non-negative values only
when the parameter t varies on the interval [ 0, ~ J,
therefore at
0 ~ t ~ ~ we obtain the portion of the curve situated in the first
quadrant. The curve is shown in Fig. 79.
As is seen from the figure, it is sufficient to evaluate the area
enclosed by one loop of the curve corresponding to the variation
7.4. Areas with Parametrically Represented Boundaries 329

of the parameter t from 0 to :rt and then to double the result


:rt :rt :rt

S= 2 ~ yx' dt =2 ~ bsin 2t xacost dt =4ab ~ cos2 t sin t dt =


0 0 0

= -4ab (co;a t) 1: = f ab.


7 .4.5. Find the area of the region enclosed by the loop of the
curve
t /2
X=3(6-t); Y=g(6-t).

Solution. Locate the points of self-intersection of the curve. Both


functions x (t) and y (t) are defined throughout the entire number
scale -oo < t < oo.
At the point of self-intersection the values of the abscissa (and
ordinate) coincide at different values of the parameter. Since x=
= 3-f (t-3) 2 , the abscissas coincide at t=3 +A.. For the func-
tion y (t) to take on one and the same value at the same values
(3+A.)2 (3-A.)2
of the parameter t, the equality - 8-(3-A.)=-8-(3+A.) must
be fulfilled for A. =i= 0, whence A.= 3.
B

Fig. 80 Fig. 81

Thus, at t 1 =0 and at t 2 =6 we have x(t 1 )=x(t 2)=0, and


y (t 1 ) = y (t 2 ) = 0, i. e. the point (0, 0) is the only point of self-
intersection. Wh~n t changes from 0 to 6, the points of the curve
are found in the first quadrant. As t varies from 0 to 3, the
point M (x, y) describes the lower part of the loop, since in the
indicated interval x (t) and y (t) = 3~x increase, and then the func-
tion x (t) begins to decrease, while y (t) still keeps increasing. Fi-
gure 80 shows the traversal of the curve corresponding to increas-
ing t (the figure remains on the left).
330 Ch. V 11. Applications of the De-finite Integral

In computing the area enclosed by the loop sought it is conve-


nient to use the formula

1
S =2 s(xy -yx
6
I ') dt
=2 J
6
1 (' / 2 (6-1)2 dt
24
27
=s
0 0

7 .4.6. Find the area enclosed by the loop of the curve: x = t 8 ;


t3
y=t-3.
7 .4. 7. Compute the area enclosed by the cardioid: x =
=a cost (1 +cost); y=a sin t (I+ cost).
Solution. Since x (t) and y (t) are periodic functions, it is suffi-
cient to consider the interval [-n, n]. The curve is symmetrical
about the x-axis, since on substituting -t for t the value of the
variable x remains unchanged, while y only changes its sign, and
y ~ 0 as t varies from 0 to n.
As t changes from 0 to n the function u =cost decreases from
1 to -1, and the abscissa x=au(l+u)=a [-{+(u+ ~)2]
first decreases from x la=i = 2a to x /u=-+ = -f and then increases
_
to x ~u= 1 =0. We can show that the ordinate y increases on the
interval ( 0 ~ t ~ ~ ) and decreases on the interva I ( ~ t ~ 1- :rt) .
The curve is shown in Fig. 81, the arrow indicating the direc-
tion of its traversal as t increases.
Consequent Iy,
n n
S= ~ S(xy' - yx') dt = a2 S(1 + cos t) dt = {
2 na2
-n o
7 .4.8. Compute the area of the region enclosed by the curve
x = cos t, y = b sin 3 t.
7 .4.9. Compute the areas enclosed by the loops of the curves:
(a) X=i 2 - l , y=t 3 -f;
(b) X=2f-t 2 ; y=2f 2 - f 3 ;
t
(c) X=f 2 ; Y=3(3-t 2 ).

7 .4.10. Compute the area of the region enclosed by the curve


x=acost; y=bsintcos2 t.
7 .4.11. Compute the area enclosed by the evolute ol the ellipse
7.5. The Area in Polar Coordinates 331

7.5. The Area of a Curvilinear Sector in Polar


Coordinates
In polar coordinates the area of a sector bounded by the curve
p = p (cp) and the rays cp 1 =a and cp 2 = B is expressed by the integral

S=+Sp" (cp)dcp.
a
2

7 .5.1. Find the area of the region situated in the first quadrant
and bounded by the parabola y 2 =4ax and the straight lines
y=x-a and x=a.
Solution. Let us introduce a polar system of coordinates by
placing the pole at the focus F of the parabola and directing the
polar axis in the positive direction
along the x-axis. Then the equation of
ff
the parabola will be p = 1- coscp
P , whe-
re p is the parameter of the parabola.
In this case p = 2a, and the focus F has
the coordinates (a, 0). Hence, the equa-
tion of the parabola will acquire the
2a
form p = 1- cos <p , and those of the
straight lines will become cp= 4
Jt
and
--I-~------
O
x

cp = -i (Fig. 82). Therefore, Fig. 82


n :re
2 2
_ I ~
SFABF-2 4a2 d _ 2 2 ~ d<p
(T.:...cos <p)2 qi - a
4 sin 4 ~
:re n
T 4
Changing the variable:
qi I z
dcp =dz :rt/4 lcot (:rt/8) ,
2 sin 2 (<p/2) '
:rt/2 I 1
we obtain
cot (n/8)

SFABF=a 2
s
I
n I+ cos (:rr/4) --
or, taking into account that cot 8 = sin (n/ 4) = l + V 2,
SFABF=2a2 ( 1 +: V2).
332 Ch. VII. Applications of the Definite Integral

7 .5.2. Compute the area of the region enclosed by


(a) the cardioid p = 1 +cos <p;
(b) the curve p =a cos <p.
7 .5.3. Find the area of the regions bounded by the curve
p = 2a cos 3<p and the arcs of the circle p =a and situated outside
the circle.
Solution. Since the function p = 2a cos 3<p has a period T = ~n'
the radius vector describes three equal loops of the curve as <p va-
ries between -:rt and :rt. Permissible values for <p are those at
which cos 3<p ~ 0, whence
n 2kn n 2kn
-5+-3-~<p~5+3 (k=O, 1, +2, ... ).

Consequently, one of the loops is described as <p varies between


-i and ~ , and the other two loops as <p varies between and %
5n 7n 3n
6 , and between 6 and 2 , respect-
ively (Fig. 83). Cutting out the parts,
belonging to the circle p =a, we get
the figure whose area is sought. Cle-
arly, it is equal to the triple area
~--~~~L~~ SMLNM
2a x Let us find the polar coordinates of
the points of intersection M and N.
For this purpose solve the equation
2a cos 3<p =a, i. e. cos 3<p Between =--} .
- ~ and ~only the roots -~and
Fig. 83 6 6 9
~ (k=O) are found. Thus, the point N
is specified by the polar angle <p 1 = - ; , and the point M by <p 2 = ; .
As is seen from the figure,
SMLNIH = SOMLNO-SOMNO =

= _!_
2
s
~9

-rr/9
2
1
4a 2 cos 2 3<p d<p--
~9

.lr a d<p = a (~9 +


-rr/9
2 2 V3)
6

7 .5.4. Compute the area of the figure bounded by the circle


p V2 a cos <p and p = 3a sin <p.
= 3
Solution. The first circle lies in the right half-plane and passes
through the pole p = 0, touching the vertical line. The second circle
7.5. Tire Area in Polar Coordinates 333

is situated in the upper half-plane and passes through the pole


p = 0, touching the horizontal line. Consequently, the pole is a point
of intersection of the circles. The other point of intersection of the
circles B is found from the equation 3 V2 a cos cp = 3a sin cp, whence
B (arc tan V2, a V6). As is seen from Fig. 84, the sought-for area
!I
VJ

0 8v.?a

Fig. 84 Fig. 85

S is equal to the sum cf the areas of the circular segments OABO


and OCBO adjoining each other along the ray cp =arc tan V2. The
arc BAO is described by the end-point of the polar radius p of
the first circle for arc tan V2 ~qi~ ~ , and the arc OCB by the
end-point of the polar radius p of the second circle for 0 ~qi~
~arc tan V2. Therefore
.TT
2

SoARo = 9a2 S cos2 qi dcp = ~a ~


2 ( - arc tan V2 - r;-) ,
arc tan V2
arc tan V2
Sarno= ~ a2 S
0
sin 2cp dcp = ! a 2 (arc tan V2 - r;-) .
Hence,
SoABo + SoCBo = 2.2512 2 (n-arc tan V2 -V2).
7 .5.5. Find the area of the figure cut out by the circle p = V3 sin cp
from the cardioid p = 1 +cos cp (Fig. 85).
Solution. Let us first find the points of intersection of these cur-
ves. To this end solve the system

{ p= V3 sin qi, 0 ~ cp ~ n,
p = 1 +cos cp,
:n;
whence cpl = 3 ' cr2 = n.
334 Ch. VII. Applications of the Definite Integral

The sought-for area is the sum of twJ areas: one is a circular


segment, the other a segment of the cardioid; the segments adjoin
each other along the ray <p = ~ . The arc BAO is described by the
end-point of the polar radius p of the cardioid as the polar angle <p
changes from ~ to n, and the arc OCB by the end-point of the
1t
polar radius p of the circle for 0 ~qi~ 3 .
Therefore
3" :rt

S =-4- S.'3 sin 2 qi dqi + ~ S(1 +cos qi) dqi =


2

0 :rt
:1

sin 2<p) I ~
11
sin 2q:) 1
=-:r3 ( <p--2- I (
~ +2 .
qi+2smcp <p
+~+-4- : =

= ~ (n-V3 ).
7.5.6. Find the area of the figure bounded by the cardioid
p=a(l-cosqi) and the circle p=a.
7.5.7. Find the area of the region enclosed by the loop of the
folium of Dt scartes x3 + y 3 = 3axy.
Solution. Let us pass over b polar coordinates using the usual
formulas x =p cos <p, y = p sin qi. Then
!! A the equation of the curve is:
p3 (cos 3 cp + sin 3 (p) = 3ap2 sm cp cos q,,
or
3a sin <p cos <p
:c P = cos 3 <p + sin 3 <p
3a sin 2<p
(sincp+coscp) (2-sin 2<p)
It follows from this equation that,
1t
firstly, p=O at <p=O and at <p= 2 ,
3n
Fig. 86 and secondly, p - oo as <p - 4 and
cp-+ - ~ . The latter means that
the folium of Descartes has an asymptote, whose equation y =
= - x-a can be found in the usual way in rectangular coordinates.
Consequently, the loop of the folium of Descartes is described
as q> changes from 0 to ; and is situated in the first quadrant
(see Fig. 86).
~ 7.5 The Area in Polar Coordinates 335

Thus, the sought-for area is equal to

5 oAo = 2
l s
2
9a 2 cos 2 cp sin~ <p
+
(cosa 4 sin:i lp)2 dcp.
0

Taking advantage of the curve's symmetry about the bisector


y=x, i.e. about the ray cp= ~, we can compute the area of half

ol the loop (from cp = 0 to cp = ~) and then double it. This enab-


les us to apply the substitution

which gives
tan cp = z,
--=dz
cos 2 QJ
d<p
'
rn 0
I '

S
oAo -
_ g
a
2 s4
cos 2 cp sin" <p d _ 9 2
(cos3 cp+ sin:i <.p)2 cp - a
s
I
z2 dz
(I+ z3)2.
0 0

Still new substitution

leads to the integral


1 +Z 3

3z 2 dz =dv,
=V,

s
ttWi
1
2

S OAO = 3a2 dv
(i2 = 23 a2 .
I

7 .5.8. Compute the area of the region enclosed by one loop of


the curves:
(a) p=acos2cp; (b) p=asin2cp.
7.5.9. Compute the area enclosed by the port ion of the cardioid
p=a(l-coscp)lying inside the circle p=acoscp.
7.5.10. Compute the area of the region enclosed by the curve
p =a sin l{J cos2 cp, a> 0.
7.5.11. Compute the area of the region enclosed by the curn
fJ =a cosa q (a> 0).
336 Ch. Vil. Applications of the Definite Integral

7.5.12. Compute the area of the portion (lying inside the circle
p= 0) of the figure bounded by the Bernoulli's lemniscate p=
=aVcos2qi.
7.5.13. Passing over to polar coordinates, compute the area of
+
the region enclosed by the curve (x 2 y 2 ) 3 = 4a2x2y2.
7.5.14. Passing over to polar coordinates, evaluate the area of
+
the region enclosed by the curve x' y' = a2 (x 2 y 2 ).+
7.6. Computing the Volume of a Solid
The volume of a solid is expressed by the integral
b

V = ~ S (x)dx
a

where S (x) is the area of the section of the solid by a plane per-
pendicular to the x-axis at the point with abscissa x; a and b are
the left and right boundaries of variation of x. The function S (x)
is supposed to be known and continuously changing as x varies
between a and b.
The volume Vx of a solid generated by revolution about the
x-axis of the curvilinear trapezoid bounded by the curve y = f (x)
(f (x) ~ 0), the x-axis and the straight lines x =a and x = b (a< b)
is expressed by the integral
b

Vx=1t~!/dx.
a

The volume Vx of a solid obtained by revolving about the x-axis


the figure bounded by the curves y =Yi (x) and y = y 2 (x) [O ~Yi (x) ~
~y 2 (x)J and the straight lines x=a, x=b is expressed by the
integral
b

Vx = n ~ (yi-yi)dx.
a

If the curve is represented parametrically or in polar coordinates,


the appropriate change of the variable should be made in the above
formulas.
7 .6.1. Find the volume of the ellipsoid
x2 y2 z2
az +{ii"+ C2 = 1.
7.6. Computing the Volume of a Solid 337

Solution. The section of the ellipsoid by the plane x = const is


an ellipse (Fig. 87)
y2 z2
b2 (I - ~22)+ (I - ~22)=1
c2

with semi-axes b - { 1 - ~: ; c -{q. Hence the area of the


section (see Problem 7.4.1)

S(x)= nb -./~
V 1- 112 x c -./-x2
V l - 112 =nbc ( 1- 112
x2)
Therefore the volume V of the ellipsoid is

Snbc ( l-ax2 ) dx=nbc lx- 3x3a ] -a= 34 nabc.


a
a
V= 2 2
-a

In the particular case a= b = c the ellipsoid turns into a sphere,


and we have Vsphere = ~ :rta 3

!I
Fig. 87 Fig. 88

7.6.2. Compute the volume of the solid spherical segment of two


bases cut out by the planes x = 2 and x = 3 from the sphere
x2+y2+z2= 16.
7.6.3. The axes of two identical cylinders with bases of radius a
intersect at right angles. Find the volume of the solid constituting
the common portion of the two cylinders.
Solution. Take the axes of the cylinders to be the y- and z-axis
(Fig. 88). The solid OABCD constitutes one-eighth of the sought-
for solid.
Let us cut this solid by a plane rerpendicular to the x-axis at
a distance x from 0. In the section we get a square EFKL with
338 Ch. VI/. Application~ of the Definite Integral

=85 (aa-x~)dx
0

side EF= Va 2 -x2 , therefore S(x)=a 2 -x2 and V =


0
to
-- -(la
3 .
7.6.4. On all chords (parallel to one and the same direction) of
a circle of radius R symmetrical parabolic segments of the same
altitude h are constructed. The planes of the segments are perpen-
dicular to the plane of the circle.
Find the volume of the solid thus obtained (Fig. 89).

h ff
' c
'
x
10
I J;
I _.JI,. a
2 2
Fig. 89 Fig. 90

Solution. First compute the area of the parabolic segment with


base a and altitude h. If we arrange the axes of coordinates as
indicated in Fig. 90, then the equation of the parabola will be
y=ax 2 +h.
Determine the parameter a. Substituting the coordinates of the
. (a )
point B 2 , 0 . we get 0 =a a2 h 4/J
4 h, w enct a= - ai; hence the +
equation of the parabola is y= -~x 2 +h. and the desired area
a
a o
2 2

5
S = 2 y dx = 2
0
5(- ~ x + h) dx = ~ah.
0
2

Now find the volume of the solid. If the axes of coordinates are
arranged as indicated in Fig. 89, then in the section of the solid
by a plane perpendicular to the x-axis at the point with abscissa x
2
we obtain a parabolic segment of area S = 3 ah, where a= 2y =
= 21/ R2 -x2 Hence,
R R
S (x) = j-V R - x 2 2 hand V = 5S (x) dx= ~ h 5VR -x dx
-R 0
2 2 = ~ nhR 2
7.6. Computing the Volume of a Solid 339

7.6.5. The plane of a moving triangle remains perpendicular to


the fixed diameter of a circle of radius a: the base of the triangle
is a chord of the circle, and its vertex lies on a straight line pa-
rallel to the fixed diameter at a distance h from the plane of the
circle. Find the volume of the solid generated by the movement
of this triangle from one end of the diameter to the other.
7 .6.6. Compute the volume of the solid generated by revolving
about the x-axis the area bounded by the axes of coordinates and
I I I
+
the parabola x'i y2 =a'i
Solution. Let us find the points of intersection of the curve and
the axes of coordinates: at x = 0 y =a, at y = 0 x =a. Thus, we
have the interval of integration [O, a].
From the equation of the parabolawegety= a2-x2 ( 1 1)2
;there-
fore
a a 1 14 a 31

V= n 5y dx = n 5
2 (a 2 -x2 ) dx =n 5(a 2 - 4a2 x 2 + 6ax-
o 0 0

t 3 ) I
-4a2x2 +x 2 dx= 15 J"ta 3

7.6.7. The figure bounded by an arc of the sinusoid y=sin x,


the axis of ordinates and the straight line y = l revolves about
the y-axis (Fig. 91).

........
~~~-+-....,_~__._--.x

0 1/2 2
Fig. 91 Fig. 92

Compute the volume V of the solid of revolution thus generated.


Solution. The inverse function x =arc sin y is considered on the
interval [O, l). Therefore

Y2 1
V=:n 5x dy=n 5(arcsiny) dy.
y,
2

0
2
340 Ch. VII. Applications of the Definite Integral

Apply the substitution arc siny=t. Hence


y I t
y=sin t,
dy=cost dt, o
1
In/2o
it
2
And so, V = n St 2 cost dt. Integrating by parts, we get v= ll (ll:- 8> .
0

7 .6.8. Compute the volume of the solid generated by revolving


about the x-axis the figure bounded by the parabola y = 0.25x 2 + 2
and the straight line 5x-8y+ 14=0.
Solution. The solid is obtained by revolving the area ABCA
(Fig. 92) about the x-axis. To find the abscissas of the points A
and B solve the system of equations:
I
{ Y=4x +2,
2

5x-8y+ 14==0.
I I
Whence xA = 2 ; x 8 = 2. In our case y1 (x)= 4 x2 + 2 and y2 (x) =
= (5/8)x+ 7/4. Hence,

7 .6.9. Compute the volume of the solid generated by revolving


about the y-axis the figure bounded by the parabolas y = x2 and
8x=y2
Solution. It is obvious that x2 (y) =VY~ x 1 (y) = ~ on the in-
terval from the origin of the coordinates to the point of intersec-
tion of the parabolas (Fig. 93). Let us find the ordinates of the
points of intersection of the parabolas by excluding x from the sy-
stem of equations:

We obtain y1 = 0, y 2 = 4. Hence, V = n S( y - 64y') dy = 245 n.


0

7 .6.10. Compute the volume of the solid torus. The torus is a


solid generated by revolving a circle of radius a about an axis
lying in its plane at a distance b from the centre (b;:: a). (A tire,
for example, has the form of the torus.)
7.6. Computing the Volume of a Solid 341

7 .6.11. Compute the volume of the solid obtained by revolving


about the x-axis the figure bounded by two branches of the curve
(y-x) 2 = x 3 and the straight line x = 1.
7 .6.12. Find the volume of the solid generated by revolving
about the line y = - 2a the figure bounded by the parabola y 2 = 4ax
and the straight line x =a (Fig. 94).
!f

Fig. 93 Fig. 94
Solution. If we transfer the origin of coordinates into the point
O' (0, -2a) retaining the direction of the axes, then in the new
system of coordinates the equation
of the parabola will be !f
(y' - 2a) 2 = 4ax. a B

a a

V= n S(y~ - yi) dx = n S[(2a +


0 0
-a
Fig. 95
7.6.13. Findthevolumeofthesolid generated by revolving about
the x-axis the figure enclosed by the astroid: x =a cos 3 t; y =a sin 3 t.
Solution. The sought-for volume V is equal to double the volume
obtained by revolving the figure OAB (Fig. 95). Therefore,
342 Ch. VII. Applications of the Definite Integral

Change the variable


x=acos 3 t, x I t
dx= - 3acos2 t sin t dt,
0 n/2
y =a sin 3 t,
a 0
Hence,
0

V = 2n Sa 2 sin 6 t ( -3a cos2 t sin t) dt =


:re
2

~ 6na{ J tdt-1 t
Using the formula from Problem 6.6.9 for computing the above
sin' sin' di l
integrals, we get
V -- 6na3 (~
7
x ~5 x ~-
3
~
9
X ~7 x
5
x !)
3
= ~
105
na 3 .

7.6.14. Compute the volume of the solid generated by revolving


one arc of the cycloid x =a (t -sin t), y =a ( 1-cos t) about the
x-axis
7 .6.15. Compute the volume of the solid obtained by revolving
about the polar axis the cardioid p=a(l +coscp) shown in Fig. 81.
Solution. The sought-for volume represents the difference between
the volumes generated by revolving the figures MN KLO and OKLO
about the x-axis (which is th2 polar axis at the same time).
As in the preceding problem. let us pass over to the parametric
representation of the curve with the polar angle <p as the parameter:
x = p cos <p =a cos <p ( l +cos <p),
y =P sin <p =a sin cp (l +cos cp).
It is obvious that the abscissa of the point M equals 2a (the value
of x at <p = 0), the abscissa of the point K being the minimum of
the function x=a(l+coscp)cos<p.
Let us find this minimum:
~;=-a sin q1 (1 +2 cos<p) =0,
2
<pt = O; <p2 = 3 n.
7.6. Computing the Volume of a Solid 343

Hence, the sought-for volume is equal to


2a 0

V= n 5 Yi dx-n 5 y~ dx.
a a
-4 -4

Changing the variable x=acoscp(l +coscp), we get


x I cp x I cp
y2 = a2 ( 1 +cos cp) 2 sin 2 cp,
dx = -a sin cp ( 1+2 cos cp) dcp, -a/4 2n/3 -a/412n/3
2a 0 0 n

Thus,
0

V = :n 5a 2 (1 +coscp) 2 sin 2 cp [-a sin cp (l + 2 coscp)] dcp-


2
3n
:n

-ZT 5 a (l + coscp) sin cp [-a sin cp (1+2 cos cp)] dcp =


2
2 2 2

3:n
:n

=:na 3 5sin 3 cp(l +coscp) 2 (1 +2coscp)dcp=


0
I

= na 3 5(l-u 2) (1 + u) 2 ( 1+2u) du= f :na 3 (u =coscp).


-1

7 .6.16. Compute the volume of the solid bounded by:


x2 y2 z2
(a) the hyperboloid of one sheet 2a +-b 2 - 2c =1 and the pla-

nes z = - 1 and z = I;
(b) the parabolic cylinder z =4-y 2 , the planes of coordinates
and the plane x=a;
(c) the elliptic paraboloid z = ;: + ~; and the plane z = k (k > 0).
7.6.17. A wedge is cut off from a right circular cylinder of radius
a by a plane passing through the diameter of the cylinder base and
inclined at an angle a to the base. Find the volume of the wedge.
7 .6.18. Compute the volume of the solid generated by revolving
the figure bounded by the following lines:
(a) xy=4, X= 1, x=4, y=O about the x-axis;
(b) y=2x-x 2 , y=O :about the x-axis;
344 Ch. VII. Applications of the Definite Integral

(c) y=x3 , y=O, x=2 about the y-axis;


(d) y = siil x (one wave), y = 0 about the x-axis;
(e) x2 -y2 = 4, y = 2 about the y-axis;
(f) (y-a) 2 =ax, x=O, y=2a about the x-axis.
7 .6.19. Find the volume of the solid obtained by revolving the
ax3 -x4
curve y2 = - -a2
- about the x-axis.

7 .6.20. Compute the volume of the solid generated by revolving


about the x-axis the figure bounded by the lines y =sin x and
2
y=-X.
:n:
7 .6.21. Compute the volume of the solid generated by revolving
about the x-axis the curvilinear trapezoid bounded by the catenary

y= ~ (e~ +e- ~)=a cos h ~ and the straight lines x1 = - c, x2 =


= c (c > 0).
7 .6.22. Compute the volume of the solid generated by revolving
about the x-axis the figure bounded by the cosine line y =cos x and
9
the parabola y = 2:n: 2x2

7 .6.23. Compute the volume of the solid generated by revolving


about the x-axis the figure bounded by the circle x2 y2 = 1 and +
3
the parabola y2 = 2 x.

7 .6.24. On the curve y = x3 take two points A and B, whose


abscissas are a= 1 and b=2, respectively.
Find the volume of the solid generated by revolving the curvi-
linear trapezoid aABb about the x-axis.
7 .6.25. An arc of the evolute of the ellipse x =a cost; y = b sin t
situated in the first quadrant revolves about the x-axis
Find the volume of the solid thus generated.
7 .6.26. Compute the volume of the solid generated by revolving
the region enclosed by the loop of the curve x = at 2 , y =a t ( --i"-(I)
about the x-axis.
7 .6.27. Compute the volumes of the solids generated by revolv-
+
ing the region enclosed by the lemniscate (x2 y 2 ) 2 = a2 (x2 -y2)
about the x- and y-axes.
7 .6.28. Compute the volume of the solid generated by revolving
the region enclosed by the curve p =a cos2 cp about the polar axis.
7.7. Arc Length of a Plane Curve in Rectang. Coord's 345

7.7. The Arc Length of a Plane Curve in Rectangular


Coordinates
If a plane curve is given by the equation y = y (x) and the
derivative y' (x) is continuous, then the length of an arc of this
curve is expressed by the integral
b

[= ~
a
v 1+ lj 12 dX

where a and b are the abscissas of the end-points of the given arc.
7 .7 .1. Compute the length of the arc of the !f M(4,8}
semicubical parabola y 2 = x 3 between the points
(0, 0) and (4, 8) (Fig. 96).
Solution. The function y (x) is defined for
x ~ 0. Since the given points lie in the first qu-
a
adrant, y=x T. Hence,

y' = ; Vx and V 1 + y' 2 = l/ 1 + ~ x.


Consequently, Fig. 96

4 3 4

l= SY + !
0
1 x dx = : ; ( 1+ ~ x) 2 1

0
= :7 ( 10V10 - 1).

7 .7 .2. Compute the length of the arc cut off from the curve
y 2 = x 3 by the straight line X= -} .

7 .7 .3. Compute the arc length of the curve y =In cos x between
the points with the abscissas x = 0, x = ~ .

Solution. Since y' = - tan x, then V 1+y' 2 =V1 + tan 2 x =sec x.


Hence,
n n
T T ~
S
1= secxdx=lntan( ~+~)I =lntan3
0 0

7 .7.4. Compute the arc length of the curve y =In ex_ 1 from
ex +1
x 1 =a to x 2 = b (b >a).
346 Ch. VII. Applications of the De-fi,nite Integral

7.7.5. Find the arc length of the curve x={y 2 - ~ lny between
the points with the ordinates y = l and y = 2.
Solution. Here it is convenient to adopt y as the independent
variable; then
,
x =-2-Y-2y an
I I d v-1+x
-,
-
2_ /(_1_----r-)2 _ I
..
V 2Y+2y -2Y+2y
I
Hence,

2 2 2
7.7.6. Find the length of the astroid x3 + y3 = a3.
Solution. As is known, the astroid is symmetrical about the axes
of coordinates and the bisectors of the coordinate angles. Therefore,
it is sufficient to compute the arc length of the astroid between
the bisector y= x and the x-axis and multiply the result by 8.
3

In the first quadrant y= (a+ -x+ ) 2 and y=O at x=a, y=x


a
at t"=-,1- .
~ '
Further,

and

V I +y 2 = 1 { l + x- ~ (a ~ -x ~ ) = ( ~ ) T.
Conseq Ut'.ntly,
a I I
l=8 ~ aTx-3 dx=6a.
a
2 ;,

Note. If we compute the arc length of an astroid situated in


the first quadrant, we get the integral
a I I
~ aTx-3 dx,
0

whose integrand increases infinitely as x --+ 0.


7.7. A re Length of a Plane Curve in Rectang. Coord's 347

7.7.7. Compute the length of the path OABCO consisting of por-


tions of the curves y2 =2x 3 and x2 +y 2 =20 (Fig. 97).
Solution. It is sufficient to compute the arc lengths li5A and /.AB
since by symmetry of the figure about the
x-axis !f
/=2(/0A+/AB)
Solving the system of equations 4
x 2 + y 2 = 20,
{
y2 = 2xa,
B
we find the point A (2, 4). D 2V5 JJ
Find liJA. Here
3
y =JI 2 x2, y' =

= 3
2
v-2x, Fig. 97
Hence,
2

l iJA = \ -{ 1 + ; x dx = 2~ ( 10 VTO - 1) .
b
Since on the circle of radius v20 l AB is the length of an arc cor-
responding to the central angle arc tan 2,
/AB= V 20 arc tan 2.
Finally we have
l = :7 (10 VTO- 1) + 4 V5 arc tan 2.
7.7.8. Compute the arc length of the curve:
x~
(a) y = 2 - 1 cut off by the x-axis;
(b) y =Jn (2 cos x) between the adjacent points of intersection with
the x-axis.
(c) 3y 2 =x(x-l) 2 between the adjacent points of intersection
with the x-axis (half the loop length).
7.7.9. Compute the arc length of the curve
y= { lx V x
2 - 1- In (x + V x2 - 1)]
between
x= 1 and x=a+ 1.
348 Ch. VII. Applications of the Definite Integral

7.7.10. Find the arc length of the path consisting of portions of


the curves x 2 = (y + 1)3 and y = 4.

7.8. The Arc Length of a Curive Represented


Parametrically
If a curve is given by the equations in the parametric form x=x(t),
y = y (t) and the derivatives x' (t), y' (t) are continuous on the in-
terval [t 1 , t 2 ], then the arc length of the curve is expressed by the
integral
,.
l= ~ V x' 2 (t) + y' 2 (t) dt,
t,
where t 1 and t 2 are the values of the parameter t corresponding to
the end-points of the arc (t 1 < t 2 ).
7 .8.1. Compute the arc length of the involute of a circle x =
=a(cost+tsint), y=a(sint-tcost) from t=O to t=2n.
Solution. Differentiating with respect to t, we obtain
x; =at cost, y; =at sin t,
whence V xi 2 + y? =at. Hence,
2n

l= Sat dt = a~2 j;n = 2an2


0
7 .8.2. Find the length of one arc of the cycloid:
x=a(t-sint), y=a(l-cost).
7.8.3. Compute the length of the astroid: x=acos 3 t, y=asin 3 t.
Solution. Differentiating with respect to t, we obtain
xi= -3a cos2 t sin t;
Yi= 3a sin2 t cost.
Hence
V x?+ y?= V9a 2 sin 2 t cos2 t =3a Isin t cost I= ~al sin2t j.
Since the function Ism 2t I has a period ~ ,
n
2
l = 4 X 32a Ssin 2t dt = 6a.
0
Note. If we forget that we have to take the arithmetic value of
the root and put Vx? + y? = 3a sin t cost, we shall obtain the wrong
7.8. Arc Length of a Curve Repres'd Parametrically 349

result, since
2:rt

S
3a sin t cost dt = 3; sin 2 t 1:n = 0.
0

7.8.4. Compute the length of the loop of the curve x= V3t 2 ,


y= t-t 3
Solution. Let us find the limits of integration. Both functions
x(t) and y (t) are defined for all values of t. Since the function
x = Jl3t 2 ~ 0, the curve lies in the right half-plane. Since with a
change in sign of the parameter t, x (t) remains unchanged, while
y (t) changes sign, the curve is symmetrical about the x-axis. Furth-
ermore, the function x(t) takes
on one and the same value not
more than twice. Hence, it follows
that the points of self-intersection
of the curve lie on the x-axis. i.e., o :r:
at y = 0 (Fig. 98).
The direction in which the mo-
ving point M (x, y) runs along the Fig. 98
curve as t changes from - oo to oo
is indicated by the arrows.
But y=O at t 1 =0, t 2 , 3 =1. Since x(t 2 )=x(t 3 )=V3, the point
(V3, 0) is the only point of self-intersection of the curve. Conse-
quently, we must integrate within the limits t 2 = -1 and t 3 = 1.
Differentiating the parametric equations of the curve with respect
to t, we get x; = 2 V3t, y/ = l -3t 2 , whence
V xt2 + y? = 1 + 3t 2

Consequently,
I

l = ~ (1 + 3t 2) dt = 4.
-1
t6 t4
7.8.5. Compute the arc length of the curve x= 6 , y = 2- 4
between the points of intersection with the axes of coordinates.
x2 y2
7.8.6. Compute the arc length of the ellipse a 2 +/j2= I.
Solution. Let us pass over to the parametric representation of the
ellipse
x=acost, y=bsint, O~t~2n.
Differentiating with respect to t, we obtain
xi= - a sin t; yi = b cost,
350 Ch. VII. Applications of the Definite Integral

whence
V x; + y? = V a
2 2 sin 2 t + b2 cos2 t = a V- I - e2 cos 2 t
where e is the eccentricity of the ellipse,
c JI a2-b2
e=-=
a a
.
Thus
:re
2:rc 2
l =a ~ JI l -c: 2 cos2 t dt = 4a ~ V l -e 2 cos 2 t dt.
0 0
t
The integral ~VI -e 2 cos 2 t dt is not taken in elementary func-
o
tions; it is called the elliptic integral of the second kind. Putting
t = ~ -, we reduce the integral to the standard form:
:re :re
2 2
~ Vl-e 2 cos2 t dt = ~ Vl-e 2 sin 2 TdT= E (e),
0 0

where E (e) is the notation for the so-called complete elliptic inte-
gral of the second kind.
Consequently, for the arc length ol an ellipse the formula
I= 4aE (c:) holds good.
It is usual practice to put e= sin a and to use the tables of va-
lues for the function
1 (a)= 1 (arcsine)= E (e).
For instance, if a= 10 and b = 6, then

e=
JI 10 2 -6 2 = 0.8 = 0
sm 53 .
10

Using the table of values of elliptic integrals of the second kind,


we find l = 40 1 (53) = 40 x 1.2776 ~ 51.1.
7.8.7. Compute the arc length of the curve
t
x = t 2 , y =y (t 2 -3)
between the points of intersection with the x-axis.
7 .8.8. Find the arc length of the cardioid:
x = a ( 2 cos t -- cos 2 t),
y =a (2 sin t -sin 2t).
7.9. Arc Length of a Curve in Polar Coordinates 351

7.8.9. Find the length of the closed curve

x = 4112 a sin t; 11= a sin 2!.

7.8.10. Find the arc length of the evolute of the ellipse


c2 c2
X=-COS 3
a
t, y=--sin
b
3 t
'
C2 =a 2 -b 2

7 .8. l I. Compute the arc length of the curve

x = (t2- 2) sin t + 2t cost,


y = (2- t 2 ) cost+ 2t sin t

betwPen t 1 = 0 and t 2 =:rt.


7.8.12. On the cycloid x=a(t-sint); y=a(l-cost) find the
point which divides the length of the first arc of the cycloid in
the ratio 1:3.

7.9. The Arc Length of a Curve in Polar Coordinates


If a smooth curve is given by the equation p =P (cp) in polar
coordinates, then the arc length of the curve is expressed by the
integral:
qi,

l= ~ { p2 + p~2 dcp,
(j),

where cp 1 and cp2 are the values of the polar angle rp at t hE' end-
points of the arc (cp 1 < cp2).
7.9.1. Find the length of the first turn of the spiral ol Archi-
medes p = acp.
Solution. The first turn of the spiral is tormed as the ola1 angle
cp changes from 0 to 2n. Therefore
2n 2n

l= ~ Va 2cp 2+ a 2dc:p = a ~ Vcp 2+ l dcp =


0 0

=a [:rt V 4:rt 2 + 1+ + Jn (2n + V 4n 2 + I) 1.


7 .9.2. Find the length of the logarithmic spira I p = aem'f between
a certain poid (p 0, cp 0) and a moving point (p, cp).
352 Ch. Vil. Applications of the Definite Integral

Solution. In this case (no matter which of the magnitudes, p or


p0 , is greater!)

l= If V a2e2m'f + a2m2e2m'f dqi =


(jJ
I
-- \ .r1+m2
=a VI+ m2 em-rdqi =a " I em~-em~ol=
" m

= VI+m2
m IP - Po I= vr+m2
nz I d P I,
i. e. the length of the logarithmic spiral is proportional to the
increment of the polar radius of the arc.
7.9.3. Find the arc length of the cardioid p=a(I+cosqi)
(a > 0, 0 ~ qi~ 2n).
Solution. Here p~ =-a sin qi,
vp'' + p2= V2a (I+ cos qi)= v-4a-2
!p
2 2 -(qi-/2-)
-CO_S_ =
=2alcos( /2)1={ 2acos(qi/2), O~qi~n
qi -2a cos (qi/2), n ~qi~ 2n.
Hence, by virtue of symmetry
2n n
l = 2a S j cos : J S
dqi = 4a cos ~ dqi = 8a.
0 0

7 .9.4. Find the length of the lemniscate p2 = 2a2 cos 2qi between
the right-hand vertex corresponding to qi= 0 and any point with a
n
polar angle qi < 4 .

Solution. If 0 ~qi< T, then cos 2qi > 0. Therefore

p=a V2cos2qi; p~= a V2sin2q>,


Vcos 2q> '
VP2+p''= l/_2_a_2_(_c_o_s-2qi_+_si-n2_2_q>_) a V2 .
!p cos 2q> cos 2<p v
Hence,

1-a
- V -s
2
0
!p

dq>
Ycos2q>
=a v-s 2
!p

0
dq>
VI-2sin 2q>
.

The latter integral is called the elliptic integral of the first kind.
It can be reduced to a form convenient for computing with the
aid of special tables.
7.9. Arc Length of a Curve in Polar Coordinates 353

7.9.5. Find the arc length of the curve p =a sins j.


7.9.6. Compute the length of the segment of the straight line
p = a sec ( cp - ~) between cp = 0 and cp = ~ .

Solution. p~=asec ( cp- ~) tan ( cp-~);

V p + p~ =a sec ( cp- ~)
2 l/ 1+ tan 2 ( cp- ~)=a sec (qi-~) .
2

(The sign of the modulus in the function sec ( cp- ~) is omitted,


since on the interval [ 0, ~J this function !I
is positive.) p=asin~!J
:rt

r (
l=a 1 sec2 cp- 3 1t) dcp =4-3
3-a.
'o
7 .9. 7. Find the length of the closed
curve p=asin 4 ~.
Solution. Since the function p = asin 4 :

is even, the given curve is symmetrical about


Fig. 99
the polar axis. Since the function sin 4 : has
a period fa, during half the period from 0 to 2n the polar radius
increases from 0 to a, and will describe half the curve by virtue
of its symmetry (Fig. 99).
Further, P~'=asin 3 (cp/4)cos(cp/4) and
Vp2+ p~2 = Va 2sin 8 ( cp/4) + a 2 sin 6 ( cp/4) cos 2 (cp/4) =a sin 3 (cp/4),
if 0 ~ cp ~ 2n.
Hence,
2.'t :rr/2

l = 2a Ssin 3 (cp/4) dcp = 8a S sins t dt = 16 a (cp = 4t).


3
0 0

7.9.8. Find the length of the curve cp=f <P+ l/p) between p=2
and p=4.
Solution. The differential of the arc di is equal to

dl = vp2 + p~ dcp = vp2 dcp2 + dp2 = v,r p2 ( :: ) 2 -f i dp.


354 Ch. VII. Applications of the Definite Integral

From the equation of the curve we find :~ =f ( 1- : 2 ) . Hence,

l =
4

s
2
v
P2 . +( 1- :2) 2+ 1dp =
4

sll ~ (p2- 2 + pl2 + 4) dp =


2

-=
4

~-s l/ (r + ~
2
r dp=f (~2+lnp)1: =3+ 1~2.
7 .9.9. Find the length of the hyperbolic spiral pqi = 1 between
3 4
qi 1 = 4 and qi 2 = 3 .
7.9.10. Compute the length of the closed curve p=2a(sinqi+coscp).
7.9.11. Compute the arc length of the curve p= i+PCOS(j) from

7.10. Area of Surface of Re'Oolution


The area of the surface generated by revolving about the x-axis
the arc L of the curve y = y (x) (a~ x ~ b) is expressed by the
integral
b

P = 2n ~ y V 1 + y' 2 dx.
a

It is more convenient to write this integral in the form P = 2n ~ ydl,


L
where di is the differential of the arc length.
If a curve is represented parametrically or in polar coordinates,
then it is sufficient to change the variable in the above formula,
expressing appropriately the differential of the arc length (see 7 .8
and 7.9).
7 .10.1. Find the area of the surface formed by revolving the
2 2 2

astroid xs+ya=as about the x-axis.


Solution. Differentiating the equation of the astroid we get
2 _...!_ 2 _2_ I

3 x 3 -l--3 y 3 y = 0,
whence
...!...
Y, -__ !f_1
3

xs
7 .JO. Area of Surface of Revolution 355

Then, V 1 + y' =-. 2 r +! 1 y = a+..!.. . Since the astrnid is sym-


JI xa lxla
metrical about the y-axis, in computing the area of the surface we
may first assume x ~ 0, and then double the result. In other words,
the desired area P is equal to

Make the substitution


2 2
x 1 t
as -xs =t2,
1
2 _..!.. 0 a3
-3X 3 dx=2tdt,
a 0
1 al/3

Then P = 12na3 5 t' dt = ~2 na 2

7 .10.2. Find the area of the surface generated by revolving about


the x-axis a closed contour OABCO formed
by the curves y = x2 and x = y2 (Fig. 100). 9
Solution. It is easy to check that the 1
given parabolas intersect at the points 0
(0, 0) and B (1, I). The sought-for area
+
P = P 1 P 2 , where the area P 1 is formed
by revolving the arc OCB, and P 2 by revol-
ving the arc OAB.
Compute the area P 1 From the equation
I -,,.fli""""'------'--- ...
x=y2 we get
1 ;-::-
y=v x and y'= 2 Yx.
o 1 ...

Hence, Fig. 100

Now compute the area P 2 We have y= x2, y' = 2x and


I

P 2 = 2n ~ x2 Vt+ 4x2 dx.


0
356 Ch. VII. Applications of the De-{i.nite Integral

The substitution x =-} sinh t, dx = ~ cosh t dt gives


Arsinh 2
P.,. =~ S sinh 2 t cosh 2 tdt =;; (+sinh 4t-t) 1:rslnh 2 =
0
9 Y5n
- - 32
= -16
I nln ( 2+ v-)
5 .
Thus,
P=P1+P2= (5~-l)n + 9~5n 3~nln(2+V'5)=
= 67 !:n -i In (2+ V5)- ~ .
7 .10.3. Compute the area of the surface generated by revolving:
(a) the portion of the curve y = ~2 , cut off by the straight line
y = ~ , about the y-axis;
(b) the portion of the curve y2 = 4 + x, cut off by the straight
line x= 2, about the x-axis.
7 .10.4. Find the surface area of the ellipsoid formed by revolving
x2 y2
the ellipse az+b2= 1 about the x-axis (a> b).
Solution. Solving the equation of the ellipse with respect to y
for y ~ 0, we get
u=a:b v --
a2-x2;
b
y =-a: Ya2
I X
x2;

V--,2 - /a""""'4,---(,_a2,,__-_..,..,b2"""")"""""'x2
1 +Y - J a2 (a2-x2) '
Hence

S-ba v--
a
va'-(a2-b2)x2
P = 2n a x22-
a2 (a2-x2) dx =
-a
a
= 4:b SVa2-e2x2dx=2nab (VI= e2+arc;ine)'
0

where the quantity e= y a 2a2 b2 = ~ is the eccentricity of the


ellipse.
When b-+ a the eccentricity e tends to zero and
arcsine
11m---
e - 0 B
1'
since the ellipse turns into a circle, in the limit we get the surface
area of the sphere:
7.10. Area of Surface of Revolution 357

7 .10.5. Compute the area of the surface obtained by revolving


+
the ellipse 4x2 y2 = 4 about the y-axis.
7 .10.6. An arc of the catenary

y= 2a ( ea.!. . + e-.!....) x
a =a cosh a '
whose end-points have abscissas 0 and x, respectively, revolves about
the x-axis.
Show that the surface area P and the volume V of the solid ths
generated are related by the formula P = 2V
a
Solution. Since y' = sinh !....
a
, we have VI+ y' 2 =cash~.
a
Therefore
x x x

P = 2n Sy V l
0
+ y' 2 dx = 2an S cosh 2
0
~ dx = ! n S a cosh
0
2 2 ~ dx,
but
x x

:rr S a 2 cosh 2 ; dx = n S y 2 dx = V,
0 0
2V
hence, P = -
a
.
7.10.7. Find the area of the surface obtained by revolving a loop
of the curve 9ax2 = y (3a-y) 2 about the y-axis.
Solution. The loop is described by a moving point as y changes
from 0 to 3a. Differentiate with respect to y both sides of the
equation of the curve:
l8axx' = (3a-y) 2 -2y (3a-y) = 3 (3a-y) (a-y),
whence xx'= ( 3 a-~a(a-y). Using the formula for computing the
area of the surface of a solid of revolution about the y-axis, we have
Y2 112

P=2n S xVI+x' 2 dy=2n S Vx 2 +(xx') 2 dy=


lJ1 IJ.

= 2n
3a
r -. / y (3a-
.l V 9a
y)2 + (3a-y)2 (a-y)2 d =
36a Y 2
~
3a
s
3a
(3a2 + 2a
Y
-
6 0
-y2 ) dy = 3na2
7 .10.8. Compute the area of the surface generated by revolving
the curve 8y 2 = x 2 -x4 about the x-axis.
7 .10 . 9. Compute the area of a surface generated by revolving
about the x-axis an arc of the curve X= t 2 ; y = ~ (t 2 -3) between
the points of intersection of the curve and the x-axis.
358 Ch. VII. Applicati-Jns of the Definite Integral

Solution. Putting y = 0, find t 1 =0 and t 2 , 3 = V3, and, hence,


x 1 =0 and x2 , 3 = 3. Whence it follows that the curve intersects
with the x-axis at two points: (0, 0) and (3, 0). When the para-
meter t changes sign, the sign of the function (x) t remains unchan-
ged, and the function y (t) changes
g its sign, which means that the curve
1 is symmetrical about the x-axis
(Fig. 101).
-l.--l-1:...t---l+i-__:~:___-~ :c To find the area of the surface it
is sufficient to confine ourselves to
the lower portion of the curve OnB
-f that corresponds to the variation
of the parameter between 0 and
Fig. 101 + V3. Differentiating with respect
to t, we find
xi =2t; Yt = t 2 - l
and the linear element
dl = Vx?+y; 2 dt =<I+ t 2 )dt.
Hence,
t,
P =2n ~I y(t) IVx?+u?dt =
t,
Y3 Y3
=2tt s- 0
~ (t 2 -3)(l+t 2 )dt=- ~ 3"t s(t -2t -3t)dt=3tt.
0
5 3

7.10.10. Compute the surface area of the torus generated by re-


volving the circle x2 + (y-b) 2 = r2 (0 < r < b) about the x-axis.
Solution. Let us represent the equation of the circle in parametric
form: x= r cost; y=b+r sin t.
Hence
xi =-r sin t; yi =rcost.
The desired area is
2:rt

P= 2n ~ (b+r sin t) V<- r sin t) 2 +(r cost) 2 dt =


0
2:rt

= 2nr ~ (b + r sin t) dt = 4n 2 br.


0

7 .10.11. Compute the area of the surface formed by revolving


the lemniscate p =a V cos 2qi about the polar axis.
Solution. Real values for p are obtained for cos 2qi ~ 0, i. e. for
- ~ ~ qi~ ~ (the right-hand branch of the lemniscate), or for
! n ~ qi~ ! n (the left-hand branch of the lemniscate).
7.10. Area of Surface of Revolution 359

The linear element of the lemniscate is equal to

di = V p2+ p,2dqi = y a2 cos 2 qi + ( .r


a sin 2cp )2 dqi = .r adcp
f cos 2cp f cos 2cp

Besides, y = p sin qi= a sin qi V cos 2qi.
The sought-for surface area P is equal to double the area of the
surface generated by revolving the right-hand branch. Therefore
1!
4

P=2x2nSydl =4na S YCOS2"CPsincpdcp


2 2na2 (2-V2).
L o
y cos 2cp
7.10.12. Compute the area of the surface formed by revolving
about the straight line x+y=a the quarter of the circle x2 +y 2 =a 2
between A (a, 0) and B (0, a).
Solution. Find the distance MN from the moving point M (x, y).
+
lying on the circle x2 y 2 = a 2 , to the straight line x y =a: +
Ix+ Ya 2-x2-a I x+ Y a2 x2-a
MN= Y2 = V2

since for the points of the circle that lie in the first quadrant
x+ y ;;::=a. Further,
di = V 1 + y'2 dx = .. / 1 + (
JI x .
ya2-x2
)2 dx = a dx
ya2-x2
.

Hence,

Jx+ VaV2-x -a . ya2-x2


a
2 2adx
P= 2n =
0

= V2 na [- Va 2 -x2 +x-aarc sin :J: = ~~ (4-n).

7.10.13. Compute the area of


the surf ace formed by revolving
one branch of the lemniscate p =
=a V cos 2qi about the straight line _
:n: B :c
qi=4
Solution. From the triangle OMN Fig. 102
(Fig. 102) we find the distance MN
of an arbitrary point M of the right-hand branch from the
axis of revolution qi=~ :

MN = p sin ( ~- qi) =a V cos 2cp sin ( ~- qi) ;


360 Ch. VII. Applications of the Definite Integral

then
dl = a dqi
v cos 2qi
11/ 4
Therefore P = 2n S aV cos 2qi sin (~-qi) Va dqi
cos 2cp
= 2na2
-1!/ 4

7 .10.14. Compute the area of the surface formed by revolving


about the x-axis the arc of the curve y=; 3
between X=-2 and
x=2.
7.10.15. Compute the area of the surface generated by revolving
one half-wave of the curve y =sin x about the x-axis.
7 .10.16. Compute the area of the surface generated by revolving
about the y-axis the arc of the parabola x 2 = 4ay between the points
<'lf intersection of the curve and the straight line y = 3a.
7.10.17. Find the area d the surface formed by revolving about
the x-axis the arc of the curve x = e1 sin t; y = e1 cost between
t =0 and t = 2:rt .
7.10.18. Compute the area of the surface obtained by revolving
[3 [2
about the x-axis the arc of the curve x= 3 ; y=4- 2 between
the points of its intersection with the axes of coordinates.
7 .10.19. Compute the area of the surface generated by revolving
the curve p = 2a sin qi about the polar axis.
7 .10.20. Compute the area of the surface formed by revolving
about the x-axis the cardioid
x =a (2 cost-cos 2t),
y =a (2 sin t-sin 2t).

7.11. Geometrical Applications of the Definite Integral

7.11.1. Given: the cycloid (Fig. 103)


x=a(t-sint); y=a(l-cost); O~t~2n.
Compute:
(a) the areas of the surfaces formed by revolving the arc OBA
abowt the x- and y-axes;
(b) the volumes of the solids generated by revolving the figure OBAO
about the y-axis and the axis BC;
(c) the area of the surface generated by revolving the arc BA
about the axis BC;
7.11. Geometrical Applic's of the Defl,nite Integral 361

(d) the volume of the solid generated by revolving the figure


ODBEABO about the tangent line DE touching the figure at the
vertex B;
(e) the area of the surface formed by JJ
revolving the arc of the cycloid [see
item (d)J.
B
~

t= I
\J
Solution. (a) When revolving about ... :c
the x-axis the arc OBA generates a sur- 0 c A
face of area Fig. 103
2:7t

j'
P" = 2n y dl = 2n Sa (1- cost) 2a sin ~ dt =
L o

Ss111.
2:n:
t 64.rta2
=8a 2 n 3 -dt
2
=--
3

When revolving about the y-axis the arc OBA generates a sur-
face of area
,.,
Py= 2n Sxdl = 4na J(t-sin t) sin f dt +
2

L o
2:n: 2:n:

+4na2 J
:7t
(t-sin t) sin~ dt =4na2 J
0
(t-5in t) sin~ dt = 16n2 a 2

(b) When revolving about the y-axis the figure OBAO generates
a solid of volume
2a 2a 2a

vy = :rt ~ (x~ - xn dy = :rt ~ x: dy-n ~ x~ dy.


0 0 0
where x=x1 (y) is the equation of the curve BA, and x=x2 (y) is
the equation of the curve OB.
Making the substitution y =a (1-cos t), take into consideration
that for the first integral t varies between 2n and n, and for the
second integral between 0 and n. Consequently,
rr :n:
Vy= n ~ a (t-sin t) a sin t dt-n ~ a2 (t-sin t) 2 a sm t dt =
2 2

~ 0
0
= na 3 ~ (t-sin t) 2 sin t dt =

=Jla 3 [f t 2 sintdt 2~f


_27 2n
t(l-cos2t)dt+f sin 3 tdt]=6:rt 3a 3
2a
for computing the volume of the solid obtained by revolving
the figure OBAO about the axis BC it is convenient first to trans-
362 Ch. VII. Applications of the Definite Integral

fer the origin into the point C, which yields the following equa-
tions in the new system of coordinates
x'=a(t-n-sint); y'=a(l-cost).
Taking into account only the arc BA, we get
2a n
V=:rt ~ x' dy'=:rta ~ (t-n-sint) 2 sintdt.
2 3

O 2n

Putting t-n=z, we obtain


0 n
V = - :rta 3 ~ (z +sin z) 2 sin z dz= :rta 3 ~ (z +sin z) 2 sin z dz=
n o
lla3
=5 (9:rt2-}6).
(c) Making the above-indicated shift of the origin, we get
dl=2asin ~ ldtl=-2asin ~ dt.
Therefore
2a ll

P=.f 2nxdl=-4na2 S(t-:rt-sint)sin ~ dt=


0 2ll
n
=4:rta 2 .f (z+sinz)cos; dz=4(2n-~) :rta
0
2

(d) Transferring the origin into the point B and changing the
direction of the y-axis, we get
x'=a(t-n-sint), y'=a(l+cost).
Putting t-:rt = z, we have
x' =a (z +sin z), y' =a (I -cos z),
z changing from -:rt to :rt for the arc OBA. Hence
ll
V=:rt ~ a 3 (l-cosz) 2(l+cosz)dz=:rt2a 8
-ll
n ll

(e) P=2:rt Jydl=4:rta S


-ll
2

-ll
(1-cosz)cos ~dz =na2.
7 .11._2. Find the volume of the solid bounded by the surfaces
z2 =8 (2-x) and x2+ y2 = 2x.
Solution. The first surface is a parabolic cylinder with generat-
rices parallel to the y-axis and the directrix z2 =8 (2-x) in the
plane xOz, and the second is a circular cylinder with generatrices
7.11. Geometrical Applic's of the Definite Integral 363

parallel to the z-axis and the directrix x 2 +y 2 = 2x in the plane xOy.


2
The volume V is computed by the formula V = ~ S (x) dx. S (x) re-
o
presents the area of a triangle whose base is equal to 2y and alti-
tude to 2z:
S (x) =2y x2z = 4 V2x-x 2 Jl8 (2-x).
Hence,
2 2
V= ~ 4 Vx(2-x)8(2-x)dx=4 V8 ~ (2-x)Vx dx =
0 0
=4 V8 (~2
3
Vxa-~
5
V x&) 120 =256
15

7 .11.3. Prove that if the figure S is bounded by a simple con-


vex contour and is situated between the ordinates y 1 and y 2 (Fig. 104),
then the volume of the solid ge- !I
nerated by revolving this figure
about the x-axis can be expressed !lz i-------==---
by the formula B
y, !----''---+---~

V= 2n ~ yhdy,
y,
where
h = X2 (y)-X1 (y),
x = x 1 (y) being the equation of the !11
left portion of the contour and ~-----------,3- :c
x = x2 (y) that of the right portion. O
Solution. Let the generating fig- Fig. 104
ure S be bounded by a simple
convex contour and contained between the ordinates y1 and y 2
Subdivide the interval [y 1 , y2 ] into parts and pass through the
points of division straight lines parallel to the axis of revolution,
thus cutting the figure S into horizontal strips. Single out one
strip and replace it by the rectangle ABCD, whose lower base is
equal to the chord AD= h specified by the ordinate y, its altitude
AB being equal to !J.y. The solid generated by revolving the rectangle
ABCD about the x-axis is a hollow cylinder whose volume may
be approximately taken for the element of volume
!J.V ~ n (y + !J.y) 2 h-ny 2h = 2ny!J.yh + nh (!J.y) 2
Rejecting the infinitesimal of the second order with respect b !J.y,
we get the principal part or the differential of volume
dV =2ll yhdy.
Knowing the differential of the volume, we get the volume proper
364 Ch. VII. Applications of the Definite Integral

through integration:
y,
v =2:rt ~ yhdy.
y,

Thus, we obtain one more formula for computing the volume of


the solid of revolution.
7 .11.4. The planar region bounded by the parabola y = 2x2 + 3,
the x-axis and the verticals x = 0 and x = 1 revolves about the
y-axis. Compute the volume of the solid of revolution thus generated.
Solution. Divide the area of the figure into elementary strips by
straight lines parallel to the y-axis. The volume ~V of the elemen-
tary cylinder generated by revolving one strip is
~V =:rt (x+ ~x) 2 y-:rt x 2 y = 2:rt xy ~x+ ny (~x) 2 ,
where ~x is the width of the strip.
Neglecting the infinitesimal of the second order with respect to ~x.
we get the differential of the desired volume
dV = 2rr. xy dx.
Hence
1 1

V= ~ 2:rt xy dx = 2:rt ~ x (2x2 + 3) dx = 4:rt.


0 0

7.11.5. Compute the area of the portion of the cylinder surface


z +
x2 y 2 =ax situated inside the
sphere
x2 y2 z2 = a2.+ +
Solution. The genera trices of the
cylinder are parallel to the z-axis,
the circle ( x- ~) 2 + y 2 = ~2
serving as directrix (Fig. 105
'f--t--ttttttttr--r-~--:c shows a quarter of the sought-
for surf ace).
Subdivide the portion of the
circle shown in Fig. 105 into
small arcs ~l. The generatrices
!I passing through the points of di-
Fig. 105 vision cut the cylinder surface
into strips. H _ infinitesimals of
higher order are neglected, the area of the strip ABCD is equal to
CD~l.
If p and cp are the polar coordinates of the point D, then
p=acoscp and CD==V-a2 -p2 =asincp, and ~l=a~qi. whence we
7.11. Geometrical Applic's of the Definite Integral 365

find the element of area:


dP = a2 sin cp dcp.
Hence,
JI
2

P = 4 ~ a 2 sin cp dcp o== 4a 2


0

7.11.6. Find the area of the surface cut off from a right circular
cylinder by a plane passing through the diameter of the base and
inclined at an angle of 45 to z
the base.
Solution. Let the cylinder axis
be the z-axis, and the given dia-
meter the x-axis. Then the equa- c
tion of the cylindrical surface B
+
will be x2 y2 = a2 , and that of
the plane forming an angle of 45
with the coordinate plane xOy
will be y=z.
The area of the infinitely nar- x
row strip ABCD (see Fig. 106) will Fig. 106
be dP = zdl (accurate to infinite-
simals of a higher order), where dl is the length of the elemen-
tary arc of the circumference of the base.
Introducing polar coordinates, we get
z = y =a sin cp; dl =a dcp.
Hence dP = a sin cp dcp and
2
:rt

P = a2 ~ sin cp dcp = a 2 [-cos cp ]~ = 2a2


0

7.ll.7. The axes of two circular cylinders with equal bases inter-
sect at right angles. Compute the surface area of the solid constitu-
ting the part common to both cylinders.
7. l 1.8. Compute the volume of the solid generated by revolving
about the y-axis the figure bounded by the parabola x 2 = y- 1, the
axis of abscissas and the straight lines x = 0 and x = 1.
7.11.9. Find the area S of the ellipse given by the equation
Ax2 +2Bxy+Cy 2 =1 (o=AC-B 2 > O; > 0). c
Solution. Solving the equation with respect to y, we get
-Bx-Vc=7Jii. -Bx+ Vc=bx2
Yi= c Y2 c= c
where the values of x must satisfy the inequality
C-6x 2 :;;:::o.
366 Ch. VII. Applications of the De-finite integral

Solving this inequality, we obtain the limits of integration:

-ll~~X~l/~.
Consequently, the sought-for area is equal to
-(~ f~
S= S (y 2 -y1) dx= ~ S VC-6x dx = Vif.
2

-f ~ 0

7 .11.10. Find the areas of the figures bounded by the curves rep-
resented parametrically:
(a) X=2t-t 2 ; y=2t 2 - t 3 ;
12 t (1-t2)
(b) X = I + t2 ; Y= I + t2
7 .11.11. Find the areas of the figures bounded by the curves given
in polar coordinates:
(a) p=asin3qi (a three-leaved rose);
( b) p = I - :os cp [ ~ ~ qi ~ ~ ] ;
(c) p =3 sin qi and p = V3 cos qi.
7.11.12. Find the arc length of the curve y2 = .!
9
(2--x) 3 cut off
by the straight line x=-1.
7.11.13. Find the length of the arc OA of the curve
a2
y=a ln-
a2-
-x-
2 ,

wher-e 0 (0, O); A ( ~ , a In : ) .


2
7.11.14. Compute the arc length of the curve y2 = 3 (x-1}3con-
tained inside the parabola y 2 = ~ .
7.11.15. Prove that the length of the ellipse
x = V2 sin t; y= cost
is equal to the wavelength of the sinusoid y =sin x.
I
7 .11.16. Prove that the arc of the parabola y = 2P x2 correspon-
ding to the interval 0 ~ x ~a has the same length as the arc of
the spiral p=Pqi corresponding to the interval O~p~a.

curve y = + ( +- x)
7.11.17. Find the ratio of the area enclosed by the loop of the
Vx to the area of a circle the circumference
of which is equal to the length of the contour of this curve.
7.12. Computing Physical Quantities 367

7.11.18. Find the volume of the segment cut off from the ellipti-
cal paraboloid 2P
y2
+ 2z2q = x by the plane x =a.
7.11.19. Compute the volume of the solid bounded by the hyper-
xz z2
boloid 2a + 2y2o - c2
= -1 and the planes z =C and Z= l > c.
7.11.20. Find the volume of the right elliptical cone whose base
is an ellipse with semi-axes a and b, its altitude being equal to h.
7.11.21. Find the volume of the solid generated by revolving
about the x-axis the figure bounded by the straight lines y = x I; +
y=2x+ I and x=2.
7.11.22.Find the volume of the solid generated by revolving
x2 y2
about the x-axis the figure bounded by the hyperbola a2 - b2 = I,
the straight line 2ay-bx=0 and tht' axis of abscissas.
7.11.23. Find the volume of the solid generated by revolving the
curve p =a cos2 qi about the polar axis.
7 .11.24. Find the areas of the surfaces generated by revolving the
following curves:
(a) y=tanx(o~x~:) about the x-axis;
(b) y=x ( ~ (O~x~a) about the x-axis;
(c) x2 +y 2 -2rx = 0 about the x-axis between 0 and h.

7.12. Computing Pressure, Work and Other Physical


Quantities by the Definite Integrals
I. To compute the force of liquid pressure we use Pascal's
law, which states that the force of pressure of a liquid P on
an area S at a depth of immersion h is P = yhS, where y is the
specific weight of the liquid.
II. If a variable force X = f (x) acts in the direction of the x-axis,
then the work of this force over an interval [xtt x2 ] is expressed
by the integral
x,
A=~ f (x) dx.
x,
Ill. The kinetic energy of a material point of mass m and velo-
city v is defined as
mv 2
K=y
368 Ch. V ll. Applications of the Definite Integral

IV. Electric charges repulse each other with a force F = e;~ 2 ,


where e1 and e2 are the values of the charges, and r is the distance
between them.
Note. When solving practical problems we assume that all the
data are expressed in one and the same system of units and omit
the dimensions of the corresponding quantities.
7 .12. I. Compute the force of pressure experienced by a vertical
triangle with base b and altitude h submerged base downwards in
water so that its vertex touches the surface of the water.
Solution. Introduce a system of coordinates as indicated ii;i Fig. 107
and consider a horizontal strip of thickness dx located at an arbi-
trary depth x.
Assuming this strip to be a rectangle, find the differential of area
dS =MN dx. From the similarity of the triangles BMN and ABC
MN x h ~
we have -b-=h' whence MN=h and dS=hdx.

Fig. 107 Fig. 108

The force of pressure experienced by this strip is equal to dP =xdS


accurate to infinitesimals of higher order (taking into consideration
that the specific weight of water is unity). Consequently, the entire
force of water pressure experienced by the triangle is equal to
h h
b r I
P = j x dS = h .\ x2 dx = 3 bh2
0 II

7.12.2. Find the force of pressure experienced by a semicircle of


radius R submerged vertically in a liquid so that its diameter is
flush with the liquid surface (the specific weight of the liquid is y).
7 .12.3. A vertical dam has the form of a trapezoid whose upper
base is 70 m Jong, the lower one 50 m, and the altitude 20 m.
Find the force of water pressure experienced by the dam (Fig. 108).
Solution. The differential (dS) of area of the hatched figure is
approximately equal to dS =MN dx. Taking into consideration the
7.12. Computing Physical Quantities 369

similarity of the triangles OML and OAE, we find ~OL = 202-;;-x;


whence ML=20-x, MN=20-x+50=70-x. Thus, dS=MNx
xdx= (70-x)dx and the differential of the force of water pressure is
equal to
dP = xdS = x (70-x) dx.
Integrating with respect to x from 0 to 20, we get
20
~ l
P = .\ (70x-x 2 )dx= 11.333 3 .
0

7.12.4. Calculate the work performed in pumping the water out


of a semispherical boiler of radius R.
7.12.5. A rectangular vessel is filled with equal volumes of water
and oi I; water is twice as t:eavy as oil. Show that the force of pres-
sure of the mixture on the wall will 0 z
reduce by one fifth if the water is ;i.- !f
replaced by oil.
Solution. Let h be the depth of
the vessel and l the length of the h 1 - - - - - - - - - t
wall. Let us introduce a system of 2
coordinates as shown in Fig. 109. x - - - - - - -....
Since the oil is situated above the
water and occupies the upper half of h J
the vessel, the force of the oil pres- I
sure experienced by the upper half m
of the wall is equal to Fig. 109
h

P,= 2l s dx=nr.
2

0
xl lh 2

The pressure at a depth x > ~ is made up of the pressure of the


oil column of height ~ and that of the water column of height
h
x - 2 , and therefore

dP 2 = [ ~ x -} + (x - ~ ) J l dx = ( x - ~ ) l dx.
Consequently, the force of pressure of the mixture on the lower half
of the wa II is
h

i' 2 = \' l ( x - ~ ) dx = 1 ~ 2

h
2
370 Ch. Vil. Applications of the Definite Integral

The entire pressure of the mixture on the wall is equal to


/h2 [h2 5
p =pl+ p2 =4+T6=T6 lh 2

If the vessel were filled only with oil, the force of pressure Pon
the same wall would be

Hence,
I I
P-P=T6lh 2 = 5 P.

7.12.6. The electric charge E concentrated at the origin of coor-


dinates repulses the charge e from the point (a, 0) to the point (b, 0).
Find the work A of the repulsive force F.
Solution. The differential of the work of the force over displace-
e
ment dx is dA=Fdx= 2x dx.
Hence
/)

A =eE dx
\ 2 =eE
(I I\J.
---
X Q b I
a

eE
As b-+ oo the work A tends to -a .
7.12.7. Calculate the work performed in launching a rocket of
weight P from the ground vertically upwards to a height h.
Solution. Let us denote the force of attraction of the rocket by
the Earth by F, the mass of the rocket by mR, and the mass of
the Earth by mE. According to Newton's law
F-kmRmE
- x2 ,

where x is the distance between the rocket and the centre of the
Earth. Putting kmRmE=K, we get F(x)= ~, R~x~h+R,
R being the radius of the Earth. At x= R the force F (R) will be
the weight of the rocket P, i.e. F (R) = P = ~ 2 , whence K = P R2
PR 2
and F (x) = - x-2
Thus, the differential of the work is
PR 2
dA = F (x) dx = X2dx.
7.12. Computing Physical Quantities 371

Integrating, we obtain
R+h R+h

A= 5 F (x) dx = P R 5 ~~ = : ~hh
R
2

R
.

The limit lim A (h) = lim RP+R~ =PR is equal to the work performed
h- al h- (Z)

by the rocket engine to achieve complete escape of the rocket from


the Earth's gravity field (the Earth's motion is neglected).
7 .12.8. Calculate the work that has to be done to stop an iron
sphere of radius R rotating about its diameter with an angular ve-
locity ffi.
Solution. The amount of required work is equal to the kinetic
energy of the sphere. To calculate this energy divide the sphere
into concentric hollow cylinders of thickness dx; the velocity of the
points of such a cylinder of radius x is ffiX.
The element of volume of such a cylinder is dV = 4nx V R 2 -x2 dx,
the element of mass dM=vdV, where'\' is the density of iron, and
the differential of kinetic energy dK = 2Jt'\'ffi2 X3 V R2 -x2 dx.
Hence,
R
K=2nvffi 2 5x3VR -x dx=-
4nyRa w2R2 Mw2R2
3 - - 5- = -5- .
2 2

0
7 .12.9. Calculate the kinetic energy of a disk of mass M and ra-
dius R rotating with an angular velocity ffi about an axis passing:
through its centre perpendicular to its plane.
7 .12.10. Find the amount of heat released by an alternating si-
nu:soidal current
I= / 11 sin (2; t-cp)
during a cycle T in a conductor with resistance R.
Solution. For direct current the amount of heat released during
a unit time is determined by the Joule-Lenz law
Q = 0.24 / 2 R.
For alternating current the differential of amount of heat is
dQ=0.24/ 2 (t)Rdt, whence
'
Q = 0.24R ~ 12 dt.
t,
In this case
T

Q = 0.24 Rn 5sin 2 ( 2; t - cp) dt =


0

. c, ( -2n t-cp ) ]
sin IT
=0.I2RI~ [ t-;n ~ 0 =0.12RTI~.
372 Ch. VII. Applications of the Definite Integral

7.12.11. Find the pressure of a liquid of specific weight don a


vertical ellipse with axes 2a and 2b whose centre is submerged in
the liquid to a level h (h ~ b).
7.12.12. Find the pressure of a liquid of specific weight don the
wall of a circular cylinder of base radius r and altitude h if the
cylinder is full of liquid.
7.12.13. Calculate the work performed to overcome the force of
gravity in pumping the water out of a conical vessel with the vertex
downwards; the radius of the cone base is R and its altitude is H.
7.12.14. Compute the work required to stretch a spring by 6 cm,
if a force of one kilogram is required to stretch it by 1 cm.

7.13. Computing Static Moments and Moments of


Inertia. Determining Coordinates of the
Centre of Gra'lJity
In all problems of this paragraph we will assume that the mass
is distributed uniformly in a body (linear, two- and three-dimensional)
and that its density is equal to unity.
1. For a plane curve L the static moments Mx and My about
the x- and y-axis are expressed by the formulas
Mx = ~ ydl, My=~ xdl.
L L

The moment of inertia about the origin of coordinates

/ 0 =--= ~ (x2 + y 2 ) di.


L

If the curve L is given by the explicit equation y = y (x) (a~ x ~ b),


+
then di has to be replaced by V 1 y' 2 dx in the above formulas.
If the curve L is given by the parametric equations x = x (t),
y = y (t) (t 1 ~ t ~ t 2 ), then dl should be replaced by V x' 2 y' 2 dt in+
these formulas.
2. For the plane figure bounded by the curves y = y1 (x), y = y2 (x),
y1 (x) ~ y2 (x) and the straight lines x =a, x = b (a~ x ~ b) the
static moments are expressed by the formulas
b h

Mx =~ S(y:-yDdx; My= Sx(y -y )dx. 2 1


a a

3. The centre of gravity of a plane curve has the following coor-


Mv Mx
dinates: xc = l' Ye=-,-, where l is the length of the curve L.
7.13. Computing Static Moments and Moments of Inertia 373

M
The centre of gravity of a plane figure has the coordinates: xe = /

Ye=~', where S is the area of the figure.


7.13.1. Find the static moment of the upper portion of the ellipse
x2 1J2
a1 +Iii= l
about the x-axis.
Solution. For the ellipse
ydl = y VI+ y' 2 dx = V y 2 + (yy')" dx;
b2 b"
since y 2 =b 2 -li2x2 and yy'=-(ifx, we have

Y dl = -.VI b2 _!!:._
a2
x2 + .!!_
a
x 4
2 dx = .!!_ V a 2 -e 2 x 2 dx
a '

where e is the eccentricity of the ellipse, e = v~


a
Integrating from -a to a, we find
a a

Mx=.!!_ \ Va 2 -e2 x 2 dx= 2ab5Va 2 -e 2 x 2 dx=


a u
-a 0

= ~ (aVa -e a + ~2
2 2 2 arcsine) =b(b+'i- arcsinF).
In the case of a circle, i. e. at a= b, we shall have Mx = 2a 2 ,
.
smce e= O an d 11m
. arc sin e
--= .
I
e~o e
7.13.2. Find the moment of inertia of a rectangle with baseband
altitude h about its base.
Solution. Let us consider an elementary strip of width dy cut
out from the rectangle and parallel to the base and situated at
a distance y from it. The mass of the strip is equal to its area
dS = b dy, the distances from all its points to the base being equal
to y accurate to dy. Therefore, di x = by2 dy and
h

Ix = 5 by2 dy = b~l
0

7 .13.3. Find the moment of inertia of an arc of the circle xz yz = R 2 +


lying in the first quadrant about the y-axis.
7 .13.4. Calculate the moment of inertia about the y-axis of the
figure bounded by the parabola y 2 ~c 4ax and the straight line x, a.
Solution. We have di x = x" dS, where dS is the area of a vertical
strip situated at a distance x from the y-axis (Fig. 110):
dS = 21 y I dx = 2 v 4ax dx.
374 Ch. VII. Applications of the DefinitP Integral

Hence,
a a 5

Ix= 54x 2 Vax dx = 4 Va Sx 2 dx = ~ a


4

0 0
7 .13.5. In designing wooden girder bridges we often have to deal
with logs flattened on two opposite sides. Figure 111 shows the
cross-section of such a log. Determine the
moment of inertia of this cross-section
about the horizontal centre line.

X=a !!

__...._______._...,,_a:
0 x R

Fig. 110 Fig. 111

Solution. Arrange the system of coordinates as is shown in the


accompanying drawing. Then
di x = y 2dS, where dS =MN dy = 2x dy = 2 V-R 2-y2 dy.
Whence
h h

lx=2 ~ y2VR2-y2dy=4 ~ y2VR2-.1;2dy.


-h 0

Substituting y = R sin t; dy = R cost dt; t 1 = O; t 2 =arc sin (h/R),


wt ~et
h arc sin (h/ R>

lx=4 5y~VR 2 -y2 dy=4 5 R 2 sin 2 tRcostRcostdt=


a o
arc sin (h/ R) arc sin (h/ Rl

=4R 4 5
0
sin 2 t cos 2 t dt = R4
2
\
<I
0
(1- cos 4t) dt =

R
=T
4 h h
arc sin R+R (2h2-R2) v---
R2-h2.
When h= R, we obtain the moment of inertia of the circle
about one of its diameters: Ix= l1~ 4

7.13. Computing Static Moments and Moments of Inertia 375

7.13.6. Find the moment of inertia about the x-axis of the figure
bounded by two parabolas with dimensions indicated in Fig. 112.
Solution. Arrange the system of coordinates as shown in Fig. 112
and write the equations of the para-
bolas.
The equation of the left parabola is:
y 2 = ~: ( x+ ; ) , the equation of the
right parabola, y2 = ~: ( ; - x).
For the batched strip the moment b
of inertia is
dlx= y 2 dS =y 2 I MN ldy,
where
IM N I= X2 - X1 = 2( ; - ~~ y 2 ) =
_ 4a 2
-a-b2 y. Fig. 112
Hence,
W2 ~2

Ix = S y2 (a -
-~2
!~ y dy = 2
2) Sy2 (a -
0
!~ y2) dy = a:~ .
7 .13.7. Find the static moments about the x- and y-axis of the
arc of the parabola y2 = 2x between x = 0 and x = 2 (y > 0).
7.13.8. Find the static moments about the axes of coordinates
of the line segment : +
~ = 1 whose end-points lie on the coordi-
nate axes.
7.13.9. Find the static moment about the x-axis of the arc of
the curve y =cosx between x 1 = - ~ and x2 = ~ .
7 .13.10. Find the static moment about the x-axis of the figure
bounded by the lint!s y = x 2; y = Vx.
7.13.11. Find the moments of inertia about the x- and y-axis of
the triangle bounded by the lines x = 0, y = 0 and ~ +
~ = 1 (a> 0,
b > 0).
7.13.12. Find the moment of inertia of the trapezoid ABCD about
its base AD if AD= a, BC= b and the altitude of the trapezoid
is equal to h.
7.13.13. Find the centre of gravity of the semicircle x2+ y2 =a 2
situated above the x-axis.
Solution. Since the arc of the semicircle is symmetrical about the
y-axis, the centre of gravity of the arc lies on the y-axis, i. e. Xe= 0.
376 Ch. VII. Applications of the Definite Integral

To find the ordinate Ye take advantage of the result of Problem


7131M-2
. x - a2 'there fore Ye -- 2JIU n.
a 2 -- 2a Th us, Xe= O' Ye= 2a n.
++
7.13.14. Find the coordinates of the centre of gravity of the ca-
tenary y = (eX e-x) = cosh x between A (0, 1) and B (a, cosh a).
Solution. We have
di= vI+ y' dx= Vl + sinh xdx =COSh xdx
2 2

whence we find
a

l = ~ dl = ~ cosh x dx = sinh a.
L 0
Then
a a
MY= ~ x dl = ~ x cosh x dx = x sinh x I~
L 0
- ~ sinh x dx =
0
=a sinh a-cosh a+ I.
Hence,
asinha-(cosha-1) cosha-1 t ha
Xe= sinha =a- sinha =a- an 2'
Analogously,

=Sy dl = Scosh x dx = +S(1 + cosh 2x) dx =


a a

Mx 2

L 0 0
= _!_ (x+ sinh 2x) la= ~+sinh 2a
2 2 2 4 0
~+ sinh 2a
2 4 a cash a
Ye= sinha 2sinha+-2-

7.13.15. Find the centre of gravity of the first arc of the cycloid:
x =a(t-sin t), y =a(l-cos t) (O~ t ~ 2n).
Solution. The first arc of the cycloid is symmetrical about the
straight line x = na, therefore the centre of gravity of the arc of
the cycloid lies on this straight line and xe = na. Since the length
of the first arc of the cycloid l = 8a, we have
2n 2n

Ye=+ Sydl= 1
8a2a 2 S(I-cos t)sin ~ dt = ~ Ssin 3 ~ dt=: a.
L 0 0

7. ta.16. Determine the coordinates of the centre of gravity of the


2 ! 2

portion of the arc of the astroid x a+ ya= a~ situated in the first


quadrant.
7.13. Computing Static Moments and Moments of Inertia 377

7 .13.17. Find the Cartesian coordinates of the centre of gravity of


the arc of the cardioid p=a(l+coscp) between cp=O and cp=n.
Solution. Let us represent the equation of the cardioid in para-
metric form:
x = p cos cp = a (1 + cos cp) cos cp;
y=psincp=a(l+coscp)sincp.

As the parameter <p varies between 0 and n the running point describes
the upper portion of the curve. Since the length of the entire car-
dioid equals 8a and
di= V (x' ~) 2 + (y' '!') 2 dcp = 2a cos ~' dcp (see Problem 7 .9.3), we have

"
= 2a Scos 4 t sin ~ dcp = - ~ a cos 5 ~ 1: = ~ a.
0

Analogously,
"5
I (' xdl =4a
Yc=4aj I acoscp(l+coscp)2acos ~ dcp =
L 0
" ~

=a Scos qi cos 3 ~ dcp =a S(2cos 5 ~ - cos 3 t) dcp.


0 0

Putting t = t we get (see Problem 6.6.9)

2"
r
Ye= 2a j (2 cos" t-cos 3 t) dt = 4ct 4. 2 2 4
a.
5 . 3 -2a 3 = 5
0

4a
And so, Xc=Yc=5
It is interesting to note that the centre of gravity of the above-
considered half of the arc of the cardioid lies on the bisector of
the first coordinate angle, though the arc itself is not symmetrical
about this bisector.
7 .13.18. Find the centre of gravity of the figure bounded by the
ellipse 4x~ + 9y 2 = 36 and the circle x 2 + y 2 = 9 and situated in the
first quadrant (Fig. 113).
378 Ch. VII. Applications of the Definite Integral

Solution. Let us first calculate the static moments:


3 3

My= Sx(y2-Y )dx = J' x lV9-x


1 2- ~ V9-x 2] dx =
0 0
3

= ~ Jx V9-x 2 dx=3;
0
3 3

Mx = ~ S(y~-YD dx ={ .\' [<9-x


0 0
2) - ! (9-x 2) J dx =
3

3
!f
=i J(5- ~ x
0
2) dx =5.

2 The area of a quarter of a circle of


radius 3 is equal to 94:rc , and the area of a
quarter of an ellipse with semi-axes a= 3
3:rc
and b = 2 equa 1s 2 , therefore the area
-;o;-t------J.a-;~ x
of the figure under consideration is
Fig. 113 S = 9:rc_ 3:rc= 3:rc
4 2 4.
Thus,
My 4 Mx 20
Xc=s=n; Yc=s= 3:rc"

7.rn.19. Find the centre of gravity of the figure bounded by the


I I I
parabola x2 + y2 = a2 and the axes of coordinates.
7 .13.20. Find the Cartesian coordinates of the centre of gravity
of the figure enclosed by the curve p =a cos 3 qi (a > 0).
Solution. Since p ~ 0 in all cases, the given curve is traced
when qi changes from - ~ to ~ . By virtue of evenness of the fun-
ction cos qi it is symmetrical about the polar axis and passes through
the origin of coordinates at qi=+ ~.
Compute the area S of the figure obtained:
11 11
2
S = 2 x 2I 5p dqi = a Scos qi dqi = a
2
2 2 6 2 1 x3x5 :re
2x 4x 6 x 2
5 na 2
= 32
0 0
7.13. Computling Static Moments and Moments of Inertia 379

Now arrange the axes of coordinates as shown in Fig. 114. Then


the parametric equations of the curve are

x= p cos cp = acos 4 cp;


y= p sin qi= a sin cp cos 3 qi.

The centre of gravity of the figure lies on the x-axis, i.e. Ye= 0
by virtue of symmetry about the x-axis. Finally, determine xe:

saa ( 1 x3x5x7x9 lx3x5x7x9Xll)n 21


= (5/32) na2 2x4x6x8x 10 2X4X6X8XIOX12 2=40a.

Fig. 114 Fig. 115

7.13.21. Find the coordinates of the centre of gravity of the


figure bounded by the straight line y = ~ x and the sinusoid
y= sin x (x ~ 0) (Fig. 115).
Solution. The straight line y = _!
n
x and the sine line y =sin x inter-
sect at the points (0, 0) and (-i-, 1) . The area of the figure
bounded by these lines is

:rt
2
S= S(sinx-n-x
2 ) 4-n
dx=-
4 -.
0
380 Ch. VII. Applications of the Definite Integral

Hence,
n
2
J_
2
5(sin2 x- ~ x2) dx
:rt2
n
2

xe = 0
4-:rt
-4-
= 2
-
4-:rt -5 0
(sin 2 x-_i_x
n2
2 ) dx=

_
-4-:rt
2 [ I sin 2x
2 x - - 4 - - 3 n2 X
4 aJ I
2
n
=6(4-n)'

0
n
2
5x (sin ! x) dx
x- ~
Ye= 0
4 _n =4 4
-n 5xsmx
. dx-
-4- 0
n
2
- 8 5x2dx=-4__ n~ = 12-nz
n (4-n) 4- n 3 (4-n) 12-3n"
0
7.13.22. Prove the following theorems (Guldin's theorems).
Theorem 1. The area of a surface obtained by revolving an arc
of a plane curve about some axis lying in the plane of the curve and
not intersecting it is equal to the product of the length of the curve
by the circumference of the circle described by the centre of gravity
of the arc of the curve.
Theorem 2. The volume of a solid obtained by revolving a plane
figure about some axis lying in the piane of the figure and not
intersecting it is equal to the product of the area of this figure by
the circumference of the circle described by the centre of gravity of
the figure.
Proof. (1) Compare the formula for the area of the surface of
revolution of the curve L about the x-axis (see 7.10)
P= 2n ~ ydl
L

with that for the ordinate of the centre of gravity of this curve

Ye= ~x=+ 5 ydl.


L
Hence we conclude that
P = 2n lye= l2nye,
where l is the length of the revolving arc, and 2nye is the length
of a circle of radius Ye i.e. the length of the circle dt!scribed by
the centre of gravity when revolving about the x-axis.
7.13. Computing Static Moments and Moments of Inertia 381

(2) Compare the formula for the volume of a solid generated


by revolving a plane figure about the x-axis (see 7.6)
b

v= n ~ (yi-yD dx
a

with that for the ordinate of the centre of gravity of this figure
h

Ye= ~x = 215 S(yi-yi) dx.


a
Hence we conclude that
V=n-2Syc=S-2nye
where S is the area of the revolving figure, and 2nyc is the length
of the circumference described by the centre of gravity when revol-
ving about the x-axis.
7.13.23. Using the first Guldin theorem, find the centre of gra-
vity of a semicircle of radius a.
Solution. Arrange the coordinate axes as shown in Fig. 116. By
virtue of symmetry xc = 0. Now it remains to find Ye If the semi-
circle revolves about the x-axis, then !J
the surface P of the solid of revo-
lution is equal to 4na 2 , and the
arc length l = na. Therefore, accor- C
ding to the first Guldin theorem,
4na 2 = na 2nye; Ye= 2 !!:..
:rt

-L-----1-----'----- x
-a 0
7.13.24. Using the second Gul-
din theorem, find the coordinates Fig. 116
of the centre of gravity of the
figure bounded by the x-axis and one arc of the cycloid: x- =
=a(t-sint); y=a(l-cost).
Solution. By virtue of the symmetry of the figure about the
straight line x = na its centre of gravity lies on this straight line;
hence, xe = na.
The volume V obtained by revolving this figure about the x-axis
is equal to 5n 2 a 3 (see Problem 7.6.14), the area S of the figure
being equal to 3na 2 (see Problem 7.4.3). Using the second Guldin
theorem, we get
V 5:n: 2 a 3 5a
Ye= 2:rtS = 2:rt 3:rta 2 = 6'
7.13.25. An equilateral triangle with side a revolves about an
axis parallel to the base and situated at a distance b >a from the base.
Find the volume of the solid of revolution.
382 Ch. VII. Applications of the De-finite Integral

Solution. There are two possible ways of arranging the triangle


with respect to the axis of revolution which are shown in Fig. 117,
a and b.
The altitude of the equilateral triangle is h = a ~ 3 , the area

S= a 2 r 3 . The centre of gravity O' is situated at the point of

intersection of the medians and at a distance of b- a ~3 from the

axis of revolution in the first cas:, and b+ a~3 in the second.


y y

c c
(a} (IJ)
Fig. 117

By the second Guldin theorem


V=2:rca 4Jl3 (b- aV3)
1
2
6
=n (a bV3
2
-~)
4 '
2

v2 = 2:rca:V3 (b+ a~3) = n ( a2br3 + ~3).


7.13.26. Find the centre of gravity of the arc of a circle of radius
R subtending a central angle 2a..
7.13.27. Find the centre of gravity of the figure bounded by
the arc of the cosine line y =cos x between x = - ~ and x = ~ and
the straight line y=-}.
7 .13.28. Find the coordinates of the centre of gravity of the
figure enclosed by line y 2 = ax 3 - x 4
7 .13.29. Find the Cartesian coordinates of the centre of gravity
of the arc of the logarithmic spiral p = ae~ from <p 1 = ; to <p 2 = n.
7 .13.30. A regular hexagon with side a revolves about one of
its sides. Find the volume of the solid of revolution thus generated.
7.13.31. Using Guldin's theorem, find the centre of gravity of
a semicircle of radius R.
7.14. Additional Problems 383

7.14. Additional Problems


7 .14.1. Find the area of the portion of the figure bounded by
the curves ym = xn and yn = xm (m and n positive integers) situated
in the first quadrant. Consider the area of the entire figure depen-
ding on whether the numbers m and n are even or odd.
7 .14.2. (a) Prove that the area of the curvilinear trapezoid
bounded by the x-axis, straight lines x=a, x=b and parabola
y = Ax 3 + Bx 2 + Cx + D can be computed using Chebyshev's formula

S=b--;a [Y Ctb-;f b-;a)+Y Ctb)+Y (atb + ;f b 2 a)].


(b) Prove that an analogous area for a parabola of the fifth order
y=f (x) = Ax5 + Bx4 +Cx 3 + Dx2 + Ex+F
can be computed using the Gauss formula

S=b 9 a [st (atb-y~b 2 a)+Bt(atb) +


+st(atb + l/~ b-;a)J

7 .14.3. Show that the area of a figure bounded by any two ra-
dius vectors of the logarithmic spiral p = aem< and its arc is pro-
portional to the difference of the squares of these radii.
7.14.4. Prove that if two solids contained between parallel pla-
nes P and Q possess the property that on being cut by any plane
R parallel to these planes equivalent figures are obtained in
their section, then the volumes of these solids are equal (Cava-
lieri's principle).
7.14.5. Prove that if the function S(x) (O~x~h) expressing
the area of the section of a solid by a plane perpendicular to the
x-axis is a polynomial of a degree not higher than three, then the
volume of this solid is equal to V= ~ [s(0)+4S ( ~) +s (h)J.
Using this formula, deduce formulas for computing the volume of
a sphere, spherical segments of two and one bases, cone, frustrum
of a cone, ellipsoid, and paraboloid of revolution.
7 .14.6. Prove that the volume of a solid generated by revolving
about the y-axis the figure a~ x ~ b, 0 ~ y ~ y (x), where y (x) is
a single-valued continuous function, is equal to
b

V = 2:rt ~ xy (x) dx.


a
384 Ch. VI!. Applications of the Definite Integral

7 .14.7. Prove that the volume of the solid formed by revolving,


about the polar axis, a figure 0 ~a~ qi~~~ n, 0 ~ p ~ p (qi),
is equal to
0
V= 2n
3 5p 3 (qi) sin qi dqi.
a
7. 14.8. Prove that the arc length of the curve given by the pa-
rametric equations
x = f" (t) cos t + f' (t) sin t,
y = - f" (t) sin t + f' (t) cost
is equal to [f(t)+f"(t)J::.
7. 14.9. Find the arc length of the curve represented parametri-
cally

X--5 I
cos
- Z-z d Z, sinz
y= \ - d z
L
I

z
I I

between the origin and the nearest point from the vertical tan-
gent line.
7. 14. 10. Deduce the formula for the arc length in polar coor-
dinates proceeding from the definition without passing over from
Cartesian coordinates to polar ones.
7.14.11. Prove that the arc length l(x) of the catenary y=
= cosh x measured from the point (0, 1) is expressed by the for-
mula l (x) =sinh x and find parnmetric equations of this line,
using the arc length as the parameter.
7.14.12. A flexible thread is suspended at the points A and B
located at one and the same height. The distance between the
points is AB =2b, the deflection of the thread is f. Assuming the sus-
pended thread to be a parabola, show that the length of the thread

l = 2b ( 1 + 32 iJ:lf
2
)

at a sufficiently small ~ .
7.14.13. Find the ratio of the area enclosed by the loop of the
curve y =+( f-x) Vx to the area of the circle, whose circum-
ference is equal in length to the contour of the curve.
7.14.14. Compute the length of the arc formed by the intersection
of the parabolic cylinder
(y+ z) 2 = 4ax
7.14. Additional Problems 385

and the elliptic cone


; x2+y2-z2=0,

between the origin and the point M (x, y, z).


7.14.15. Prove that the area of the ellipse
Ax2 +2Bxy+Cy 2 +2Dx+2Ey+F =0 (AC-8 2 > 0)
is equal to
ABD
where ~ = B C E .
DEF
7.14.16. Find: (a) the area S of the figure bounded by the hy-
perbola x~-y2 =I, the positive part of the x-axis and the radius
vector connecting the origin of coordinates and the point M (x, y)
lying on this hyperbola.
(b) The area of the circular sector Q bounded by the x-axis and
the radius drawn from the centre to the point N (x, y) lying on
the circle x2 + y2 == I. Prove that the coordinates of the points M
and N are expressed respectively through the areas S and Q by
the formulas
x111 =cosh2S, y111 =sinh2S, xN=cos2Q, YN=sin2Q.
7.14.17. Using Guldin's theorem, prove that the centre of gra-
vity of a triangle is one third of the altitude distant from its base.
7 .14.18. Let 6 be the abscissa of the centre of gravity of a cur-
vilinear trapezoid bounded by the continuous curve y=f(x), the
x-axis and the straight lines x '-'-a and x ~= b. Prove the validity
of the following equality:
b h

~ (ax+ b) {(x) dx =(as+ b) ~ f (x)dx


a a
(Vereshchagin's rule).
7.14.19. Let a curvilinear sector be bounded by two radius vec-
tors and a continuous curve p = f (cp). Prove that the coordinates
of the centre of gravity of this sector are expressed by the follow-
ing formulas:
tp,
~ p3 cos cp dcp "'~' p 3 sin cp dcp
x =~<p'-"-'---- Ye = 3
2 <p I
-'-'-<r-.---
L' 3 "''
~ p2 dcp ~ p2 dcp
<p, <p,
386 Ch. VII. Applications of the Definite Integral

7 .14.20. Prove that the Cartesian coordinates of the centre of


gravity of an arc of the curve p = f (cp) are expressed by the fol-
lowing formulas:
<p, 'Pt
~ p cos 1' v p + p'
2 2 drp ~ p sin qi Jfp 2 +p'ldqi
Xe ='Pt 'Pt Ye = ---'<p'-'-'-------
<p,

~ vp2+p'2drp ~ yp2 + p'2 drp


'Pt cp,
Chapter 8
IMPROPER INTEGRALS

8.1. Improper Integrals with Infinite Limits


Let the function f (x) be defined for all x ~a and integrable on
A
any interval [a, A]. Then Iim ~ f (x) dx is called the improper
A-+ a
integral of the function f (x) in the interval [a,
+oo
+ oo] and is de-
noted by the symbol ~ f (x) dx. We similarly define the integrals
a
B +oo

~ f (x) dx and ~ f (x) dx.


-oo -oo
Thus,
+oo A
~ f (x) dx = lim ~ f (x) dx;
a A-++oo a
B B
~ f (x)dx = lim ~ f (x) dx;
-oo A--oo A
+ao C B

~ f(x)dx= Jim ~f(x)dx+ Jim \f(x)dx.


_ 00 A-+-oo A B-+oo C

If the above limits exist and are finite, the appropriate integ-
rals are called convergent; otherwise, they are called divergent.
Comparison test. Let f (x) and g (x) be defined for all x ~a and
integrable on each interval [a, A], A~ a. If 0 ~ f (x) ~ g (x) for
"'
all x ~a, then from convergence of the integral ~ g (x) dx it fol-
a
"' "'
lows that the integral ~ f (x) dx is also convergent, and ~ f (x) dx ~
a a
388 Ch. V 111. I mp roper Integrals

~ 00

~ ~ g (x) dx; from divergence of the integral ~ f (x) dx it follows that


a a

the integral ~ g (x) dx is also divergent.


a
Special comparison test. If as X-+oo the function f (x) ?- 0 is an
infinitesimal of order 'A> 0 as compared with _!_,
x
then the integral
+oo
S f (x) dx converges for 'A> I and diverges for 'A~ I.
a
Absolute and conditional convergence. Let the function f (x) be
"'
defined for all x ?- a. If the integral ~ If (x) i dx converges. then the
a
"'
integral ~ f (x) dx also converges and is called absolutely convergent.
a
In this case

00
I] f (x)dx I~~ If (x) Idx.
00

If the integral ~ f (x) dx converges, and ~If (x) I dx diverges, then


a a
"'
the integral ~ f (x) dx is called conditionally convergent.
a
The change of the variable in an improper integral is based on
the following theorem.
Theorem. let the function f (x) be defined and continuous for
x ?- a. If the function x = <p (t), defined on the interval a< t < ~
(a and ~ may also be improper numbers -oo and oo ), is monoto-
nic, has a continuous derivative <p' (t) =F 0 and lim <p (t) =a,
1~ a+O
Jim cp (t)
t .. f)-0
= + oo, then
00 ,,

~ f (x)dx= ~ f (cp(t))<p' (t)dt.


a a
Integration by parts involves no difficulties.
8.1.1. Evaluate the following improper integrals with infinite
limits or prove their divergence taking advantage of their defini-
tion
JO
"' 00

(a) Sxl~x; (b) 2


dx
Sx +2x+5'
(c) Sxsinxdx.
e' -oo 0
8.1. Improper Integrals with lnfl,nite limits 389

Solution. (a) By definition,


ao

~ -dx
e2
.
- = 1tm
xln x
3 A ... +>
s
A

e
dx- =
-.
xln 3 x
.
11m ( - -I - 'A)
A ... +:so 2 2ln xe
=

. ( I
= AI...im
+:x>
I
8-212"A
n
) I
=s
(b) By definition,
ao
(' 1.
dx
j x2+2x +5 = 8 :~ 00
sx2+ 2x+5 + A~~"'
0
dx 1. s
x +2x +5
A

2
dx
-oo B 0

(instead of the point x = 0 any other finite point of the x-axis may
be taken as an intermediate limit of integration).
Compute each of the limits standing in the right side of the
above equality:

s
ltm
B-+-oo B
0
dx .
x2 + 2x +s = ltm 2 arc tan-2-
8-+-ao
I x+ I lo = I arc tan I
2B
2 + 4re ,
A
.
ltm \ dx
= .
11m x+- J IA = -
-2I arctan- :rt I , I
A ... +oouX
2 + 2 +s
X A ... +oo
2 4 --2 arctan-
2 .0
0
Hence,

- 7J

(c) By definition,
"' A
~ x sin x dx = lim ~ x sin xdx.
0 A->+ooo

Putting u = x, dv =sin x dx and integrating by parts, we get

A ( A
lim I xsinxdx= lirn -xcosxlA+ I cosxdx)=
A ... +"'6 A ... +"' 0 6
= lim (- A cos A+ sin A).
A-+ a:i

But the last limit does not exist. Consequently, the integral
"'
~ xsinxdx diverges.
0
390 Ch. VI II. I mp roper Integrals

8.1.2. Evaluate the following improper integrals with infinite


limits on the basis of their definition:

(a)
s
<Xl

xdx
Y<x2_3)a; (b) s
"'
dx
x+x 3 ; (c)
s <Xl

xdx
V!4xz+ !)a ;
2 I 0

(d) s
<Xl

I
dx
x2 (I + x) ; (e)
-a>
<Xl

s dx
x2 -6x+ 10'
(f) S"'
0
e-x sin x dx.

Solution. (a) By definition


<Xl

SY(x -3) xdx


2 3
= Jim
A-++ "'
sY
A
xdx
(x 2 -3) 3
= Jim
A-++"'
[_!_
2
(x2_3)-1/2
-1/;i
2A] =
1

2 2

=-
A
Jim
-+ + "'
[ y I
A 2 -3
-1] = 1.

8.1.3. Prove that the integrals of the form


+oo /1

~ e-px dx and ~ ePx dx


a _.,,
converge for any constant p > 0 and diverge for p < 0.
8.1.4. Test the integral
"" dx
S I+ 2~-x~
2 +-3x~
4
0
for convergence.
Solution. The integrand
I
f(x)= 1+2x2+3x4
is positive and is an infinitesimal of order A.= 4 as compared with
_!_x as x-+ oo. Since 4 > 1, the integral converges according to the
special comparison test.
8.1.5. Test the integral

S"' x+sin
dx
2 x
I
for convergence.
Solution. The integrand f (x)= x + Sin
1.
2
X
is continuous and posi-
tive for x ~ 1.
As X-+ oo the function f (x) is an infinitesimal of order 'A= 1
as compared with ~ , according to the special comparison test the
integral diverges.
8.1. Improper Integrals with Infinite limits 391

8.1.6. Test the following integrals for convergence:

(a) ~In (x"+


I x
IJ dx; (b) i ' ";_
I I +x x
dx;

I
oc oo I 00

~
3+arc sin -
d d (' arc tan x d
(c)
I
2+cos x
,;-
r x
dx; ( )
2
,r
I +x r x
x
x; (e) J-x- x.
I
8.1.7. Test the integral
~

\
, (x+ lfx+l) dx
x 2 +2
I
V x4+ I
for convergence.
Solution. The integrand is continuous and positive for x ~ 1.
Determine its order of smallness A. with respect to _!_ as x---+ oo;
x
since
i+-. I J..+-~
x+ Yx + I
v
---'---'----'--=-
x2 + 2 x4 + I
I
x
V x x~
1+2
v
x ~---=---=======
I
6+10
x x
I ,

the order of smallness 'A= I. According to the special comparison


00

test the integral C x+ (x+T dx diverges.


J x2+2 }/ x4+ I
I
8.1.8. Test the integral
00

SYx(x-l)(x-2) dx

3
for convergence.
Solution. Since the function
I I
3 (=1-=;=)(=1-=;==-) = -3- x -V-;=::(=1-=~=)(=1-=f=-)
t (x) = -y~x=
is an infinitesimal of order 'A= ~ with respect to ; as X--> oo, +
according to the special comparison test the integral converges.
8.1.9. Test the integral

for convergence.
392 Ch. V 111. Improper Integrals

Solution. The integrand is continuous and positive for x ~ 2.


Determine its order of smallness with respect to -x as x-+ + oo:

V~ I
V +X2 -3
2
a'
V x3--I
=-1-1Xv
- 1--1-
xas xa

Since the second multiplier has the limit {/2 as X-+ + oo, we
have /.. = !~ < l. Consequently, the given integral diverges.
8.1.10. Test the integral
"'
S(I -cos ~) dx
I
for convergence.
Solution. The integrand
f(x)= l-cos~=2sin 2 J..
x x
is positive and continuous for x ~ l. Since 2 sin 2 ...!...
x
,..., 2
x
2
=~,
x
(J..)
the given integral converges (by the special comparison test).
8.1.11. Test the integral

J
"' I

In e7 +<n-I) dx, n >0


I n
for convergence.
Solution. Transform the integrand:

f(x)=ln e++!n-1) =ln[l+ e!n-1].


I

Since the function is an infinitesimal as X-+ oo, then


n
I
ex - I I
f (x),..., lim 11<1x) = 2.. . According to
,..., -nx . In other words,
n X n X-+00

the special comparison test the given integral diverges.


8.1.12. Test the integral
"'
S xs+Vx
sinl-4 2x dx
I
for convergence.
8.1. Improper Integrals with Infinite Limits 393


Sol u t ton. The f unc t'10n f (x ) = V
l - 4 sin 2x c h anges 1't s sign
t oge th er
x3 + x
with the change in sign of the numerator. Test the intt>gral
r I I-
00

4 sin 2x I d
i
'1 xs+
v3
x
x

for convergence. Since 11 - 4 sin 2x I <~ and the integral


xs+V x xs '
00

SX3 converges,
5dx .
the integral
s
00

11-4 sin 2x I
xs+ V x dx converges as well (ac-
t I
cording to the comparison test). Thus, the given integral converges
absolutely.
8.1.13. Prove that the Dirichlet integral
00

I=\ sinx dx
v x
0
converges conditionally.
Solution. Let us represent the given integral as the sum of two
integrals:
11

I=
00

r sinx dx =
.J x
2
s sinx dx
x
+ s
00

sinx dx .
x
o O n
2
The first is a proper integral (since lim sinx=
x- 0 x
t). Applying the
method of integration by parts to the second integral, we have
oo A
j,. -sinx
X
.
dX= 1Im
A-oo
j" -sinx dX=
x
n n
2 2

=Jim [-cosxJA
x n
-j~cosxdxl
x2
= - 5co~x dx.
x2
00

A-oo -
2 n n
2 _J 2
00

(' cos x
But the improper integral J -:t2 dx converges absolutely, since
n
2
00

I cos2 x I & _..!_2


x __,,x '
and the integral
Jr d:
x
converges.
n
2
394 Ch. Vlll. Improper Integrals

Therefore, the mtegral . s


"' sin-x
-x-dx converges.
0
Reasoning in a similar way it is easy to prove that the integral
<)

(' cosx
\ --dx also converges. Now let us prove t hat the integral
u x
n
2

sI si~
"-

it
x I dx diverges. Indeed,

2
I sin x I ~ sin 2 x _ I - cos 2x
-x-~-x-- 2x '
but the integral
"'
SI - cos 2x d _ 1.
2x
X - Im -I
A .... <J 2
s
A
dx - -
-
x
I
2
s
"'
cos 2x dx _
--
x
-
n n n
2 2 2
I hm
=- . In
2 A .... oo 2 2 2
s
A - -I Inri- - -I "' --dx
cos 2x
x
n
2
"' 2
diverges, since lim In A= oo, and the integral
A~~
S cos x dx
X
converges.
n
2
8.1.14. Prove that the following integrals converge
oc "' <)

(a) ~ sin(x 2 )dx; ~ cos(x 2 )dx; (b) ~ 2xcos(x 4 )dx.


0 0 0

Solution. (a) Putting x =Vt, we find

J J~n/ dt.
CX) <)

sin (x )dx=-}
2

0 0
Let us represent the integral on the right side as the sum of two
integrals:
n
~ 2 "'
S~n/ dt =S ~n/ dt+ J~~ dt.
0 O n
2
Th e fi rs t summan d 1s t egra I, s111ce
a proper m sin t = 0. Le t
I'1m .r-
r I
1 .... +o
8.1. Improper Integrals with Infinite Limits 395

us apply to the second summand the method of integration by


parts, putting
u= 1/Vt, sintdt=dv,
I"' __!_ scost s
<Xl <Xl <Xl

f
. Vt
11/2
sin t dt = _ cost
Vt 11/2 2
11/2
t3/2
dt = _ _!_
2
11/2
cost
[3/2
dt.

. I cost I I
The last integral converges absolutely, smce ~:::;; t3f2, and

I ::
<Xl

the integral 12 converges. We can prove analogously that the


n
2
<Xl

integral ~ cos (x2 ) dx is convergent. The integrals considered are cal-


o
led Fresnel's integrals. They are used in explaining the phenome-
non of light diffraction.
(b) By the substitution x 2 = t this integral is reduced to the
<Xl

integral ~cos (t 2 ) dt. The latter integral converges as has just been
0
proved.
Note. Fresnel's integrals show that an improper integral can con-
verge even when the integrand does not vanish as x-+ oo. The last
convergent integral considered in item (b) shows that an improper
integral can converge even if the integrand is not bounded. Indeed,
at x = V nn (n = 0, I, 2, ... ) the integrand attains the values
+ V nn, i.e. it is unbounded.
8.1.15. Evaluate the improper integral
<Xl

5
0
dx
(I +x2)n
n natural number.

Solution. Make the substitutionx=tant, where o:::;;t<;. Then


x =0 at t =0, x-++ oo as t-+ n2 -0 and x; = Conse- __;....t =#= 0.
cos
quently, by the theorem on changing a variable in an improper
integral
n n
<Xl 2 2

S(I : : 5se: nt X sec


0
2 )n
0
2
2 t dt = 5cos
0
2 n- 2 t dt.

On changing the variable we obtain the proper integral which


was computed in Problem 6.6.9.
396 Ch. VI II. I mp roper Integrals

Therefore,
J n/2, n = I,
5-~x~ d
00

= 135 ... (2n-3) n I


0(I +x 2
)n \ 246 ... (2n-2)' 2' n>

8.1. t 6. Compute the integral I = S"' 1~ x4 dx.


0
Solution. Apply the substitution
X= 1/1; dx=-(1// 2 )df; 11 =oo, 12 =0;

If another integral I is added to the right and left sides then we


get
I + 12 sI112 + I
00

2/= S 1+t4 di= 12+11t2dl.


00

0 0
Make the substitution z=t-1/t, (l+I/t 2 )dt=dz. Then, as
t-+o, z--+-oo and as t-+ oo, z-+ oo. Hence,

I =2
15 2+2
dz =2I[ltm \ 2+2
00

Z
dz + ltm
. 8-.-oo Z
0

A-.+oo
SA
Z
dz] =
2+2
- 00 8 0

= - I
.r- l'1m arc t an .r-
B + I
.r- l"1m arc t an A v-2 =
2 I' 2 8-.-oo I' 2 2 I' 2 A -.+oo

=2~2 (;+~)=2;2.
8.1. 17. Evaluate the following improper integrals:

(a)
Cf)

('
J 1+x2
0
Jn x
dx; (b) se-x
00

0
x2m+1 dx.

8.1.18. Compute the integral

accurate to two decimal places.


Solution. Represent the given integral in the form of a sum of
two integrals
8.2. I mp roper Integrals of Unbounded Functions 397

C.Ompute the former with the required accuracy, using Simpson's


formula, and estimate the latter. Since for x ~ 1 we have
0 < V xa -x2+ I < _x_
a/2
=x-7/2
x+x 2 + I x '
then
00

0< 12 = 5x- 7 t2 dx=; N- 0 12.

r, <
N

At N = 7 we get the estimate 1 2 < 52 x 49 7


0.0031.
C.Omputation of the integral
7
(' V x 3 -x2 + I
I 1 = .) x + x2 + I dx
I

by Simpson's formula for a step h = 1 gives

. h
s. = 0.2155,
and for a step 2 = 0.5
S 0 . 0 =0.2079.
Since the difference between the values is 0.0076, the integral / 1
gives a more accurate value S 0 0 = 0.2079 with an error of the order
0 ~76 '.:::'. 0.0005.
C.Onsequently, the sought-for integral is approximately equal to
I~ 0.208
with an error not exceeding 0.004, or I= 0.21 with all true deci-
mal places.

8.2. Improper Integrals of f:Jnbounded Functions


If the function f (x) is defined for a~ x < b, integrable on any
interval [a, b -e]. 0 < e < b-a and unbounded to the left of the
point b, then, by definition, we put
b b-e
~ f (x) dx = lim ~ f (x) dx.
a e~+o a
If this limit is existent and finite, then t.he improper integral is
said to be convergent. Otherwise it is called divergent.
Analogously, if the function f (x) is unbounded to the right from
the point a, then
b b

~ f (x) dx = Jim ~ f (x) dx.


a 8->-tOa+e
398 Ch. V 111. I mp roper Integrals

Finally, if the function is unbounded in the neighbourhood of an


interior point c of the interval [a, b], then, by definition,
b c b

~ f (x) dx = ~ f (x) dx + ~ f (x) dx.


a a r

Let the function f (x) be continuous on the interval [a, b] except


at a finite number of points. If there exists a function F (x) conti-
nuous on [a, b] for which F'(x)=f(x) except at a finite number
of points, then the Newton-Leibniz formula
b

~ f (x) dx= F (b)-F (a)


a
holds good.
Sometimes the function F (x) is called a generalized antiderivative
for the function f (x) on the interval [a, b].
For the functions defined and positive on the interval a~ x < b
convergence tests (comparison tests) analogous to the comparison
tests for improper integrals with infinite limits are valid.
Comparison test. Let the functions f (x) and g (x) be defined on
the interval a~x<b and integrable on each interval [a, b-e],
0 < e <b -a. If 0 ~ f (x) ~ g (x), then from the convergence of the
b b

integral ~ g (x) dx follows the convergence of the integral ~ f (x) dx,


a a
" b
and ~ f (x) dx ~ ~ g (x) dx; from the divergence of the integral
a a
b b

Sf (x) dx follows the divergence of the integral 5g (x) dx.


a a
Special comparison test. If the function f (x) ~ 0 is defined and
continuous on the interval a~ x < b and is an infinitely large
quantity of the order /.. as compared with b 1 x as x -b-0, then
b

the integral 5f (x) dx converges for A.< 1 and diverges for /.. ~ 1.
a
In particular, the integral
b

S<b-x)'
dx

< 1 and diverges for A.~ I.


converges for 'A
Absolute and conditional convergence. Let the function f (x) be
defined on the interval a~ x < b and integrable on each interval
8.2. Improper Integrals of Unbounded Functions 399

/J

[a, b-e]; then from the convergence of the integral ~If (x) / dx
a
b

follows the convergence of the integral ~ f (x) dx.


a
b

In this case the integral ~ f (x) dx is called absolutely convergent.


a
b b

But if the integral ~ f (x) dx converges, and the integral ~ / f (x) / dx


a a
b

diverges, then the integral ~ f (x)dx is called conditionally convergent.


a ,,
Analogous tests are also valid for improper integrals ~ f (x) dx,
a
where f (x) is unbounded to the right from the point a.
8.2.1. Proceeding from the definition, evaluate the following
improper integrals (or prove their divergence):
:rt
e 2
(a) r dx r
(b) j cos x ;
dx
t x Vlnx ' 0
3 2

(c) SJf4x~x2 -3;


I
(d) SYI ld~x2 I '
0

(e). f
0
l xa+Vx-2
5/xa
v
dx; (f)
SI d
0
l~x:f'

Solution. (a) The integrand f (x) =


x
V1 lnx
3 is unbounded in the
neighbourhood of the point x = 1. It is integrable on any interval
(I+ e, e], since it is a continuous function.
Therefore

Se - dx - = 1.1m Se
-
xVlnx 8-++0
dx
xVlnx
= 1.1m [ -3
8-++0 2
v-1
n x le
2
lte
J=
I 1+8

=um 8 ... +O
f~-~V1n
_2 2
2 <t+e)j=~.
2
(b) The integrand f (11:) = - 1
- is unbounded in the neighbourhood
cos x
of the point x = ; and integrable on any interval [ 0, ~- e j as
400 Ch. VI/ I. Improper Integrals

a continuous function. Therefore


:rt :rt
2 2-e
S ~=
cos x
lim
8-++0
(' .!!=_=
J cos x
0 0

= lim
e ... +o
lntan(i+~)J~-e
o
= lim lntan
..... +o
(~-;) = oo.

Hence, the given integral diverges.


(c) The integrand is unbounded in the neighbourhood of the points
x = 1 and x = 3. Therefore, by definition,
3 2 3

S Jf4x-x 2 -
~ 3
=S Y ~
4x-x 2 -3
+S ~
V4x-x 2 -3
I I 2

(instead of the point x = 2 we can take any other interior point of


the interval [ 1, 3)). Let us now compute each summand separately:
2 2

S
l
-r:===
dx = 1tm \'
Jf4x-x -3
2 e ... +o u
I+ e
Yl-(x-2) 2
dx =
e-+o
. (x- 2) 12
1tm arc sm
l+e
=

= Jim [0-arc sin (e-1)]


e ... +O
=i;
3 3-

S -r:= .
dx= = = 1un \' -;:::=====
dx hm .
arcsm(x-2) 13- =
.2 Y 4x- x2 -3 e-+o ,, YI- (x-2) 2 e... +o
2
2

. ( 1-e ) - OJ = 2n .
Jim [arcsm
= e-+o

Hence,
3

SY;;:::4=x""'~""x:r;i2""-~3 = ~ + ~ = n.
I

(d) The integrand f (x) = V 1 -. is unbounded in the neigh-


! I -x 2 1
'bourhood of the point x= I, which is an interior point of the
inkrval of integration. Therefore, by definition,

0
2

SYI _d_x
l-x2 1-
-s0
I

Yi
dx
l-x2 1
+S YI
I
2
dx
l-x 2 1'
8.2. Improper Integrals of Unbounded Functions 401

Evaluate each summand separately. If 0 ::s;;;;x < l, then


I I 1-e

SYi dx r dx lim r dx
1-x2 1 = J0 YI -x 2 s-++O J YI -x2 =
0 0

= lim arcsinxl 1 - 0 = lim [arcsin(l-e)-0]=~.


E-++0 0 8-++0

If I < x ::s;;;; 2, then


2

S--;=:=== = f
dx
Yil-x2 1 ,
2

Yx 2
dx
I
= 11n1.
~ .... +o
s Yx
2
dx
2 I
=
I I I +e

lim In (x + V x2 - 1) j2 =
= 8-++0 l+t

= lim [ln(2+V3)-ln(l+e+V(l+e) 2 -l)]=ln ~2


e .. +o
+ V3).
Hence,
2

Sy I 1-x
dx = :rc
2 +In (2 + V3).
2 \
0

(e) Represent the given integral as a sum of three items, divid-


ing each term of the numerator by V x3 ,
I I I I

(" xa+.Vx-2 dx= (' x12t dx+ (" ~-2 (" ~.


J0 V xa J0 J0 x4/IO J xa/&
0

The first summand is a proper integral evaluated by the Newton-


Ldbniz formula:
I

Sx12t dx--x111
-175 I'
0
=-
5
17.
0
The second and third summands are unbounded to the right of the
point x = 0. Therefore,
I I

r ~=
.Jx4/
lim (' ~= lim ~ xll/101 =
e .... + o J x'I u
JO e .... + 0 11 e
1 15 ;
11
o e
analogously,
. s- = 1tm / I' =-.
I I

j,. -x = 1tm
dx
310+o x
. -x2
5
+o 2
5
2
e ....
dx
310 e .... s
O e
402 Ch. V 111. I mp roper Integrals

Hence,

(f) Represent the integrand f (x) =I~ x3 in the form of a sum


of partial fractions;

I I I [ I x+2 ]
f (x)= I-x3 = (1-x) (I +x+x2) =3 1-x +I +x+x2

I I I

Then S 1 dxx 3 = ~ S 1 d\+~S 1 ~;~x2 dx. Since


0 0 0
I
dx 1
S -1 - = 11n
-x e-+o
s
1-8
dx = - 1tm
-1 -
-x e-+o
Int1-x) 1 -e =oo,
0
0 0

the given integral diverges. There is no need to compute the second


summand representing a proper integra I.
Note. Evaluation of the improper integrals from Problem 8.2.1
(a to f) can be considerably simplified by using a generalized anti-
derivative and applying the Newton-Leibniz formula. For instance,
in Problem 8.2.1 (a) the function F (x) = ; Vln 2 x is continuous on
the interval [ 1, e] and differentiable at each point of the interval
1 <x~e, and F'(x)=f(x) on this interval. Therefore
e
r~_d_x_=~
JI xV In x 2
v Jn2xle = ~. 1 2

8.2.2. Proceeding from the definition, compute the following


improper integrals (or prove their divergence):

(a)
J
Ja
2xdx
(x2-a2)2/3; (b) s.
2/;r.
S i nI - dx
x
-
x2 '
0 0

(c) JI
n
COSI-x'(I-x)2;
dx (d)
6

s v(::.x) 2 ;
0 2

(e) s
-2

-1
dx
xJfx2 - l
(f) 5
I
2

X
dx
JnP X
8.2. Improper Integrals of Unbounded Functions 403

8.2.3. Evaluate the following improper integrals:


3 2 --

(a) ,
\
~ x 2 dx
Y9- x 2 ;
(b) .f y;+;dx.
-3 0

Solution. (a) Find the indefinite integral


S~2~xx2 = ~ ( 9 arc sin ~ -x V9-x 2) + C.
The function F(x) = +( 9 arc sin~ -x V9-x 2) is a generalized anti-
derivative for f (x) = V x2 . on the interval (-3, 3], since it is
9-x 2
continuous on this interval and F' (x) = f (x) at each point of the
interval (-3, 3). Therefore, applying the Newton-Leibniz formula,
we get
3

,rl _!_
2
dx = _!_ ( 9
y 9- xz 2
arc sin .:.._x V9-x 2 )
3
1
3
- 3
= -i-2 n.
-3
(b) Transform the integrand
f (x) = J/-2+x = 2+x = 2 + x
2- x V4 -xz V4 - xz V4 - xz
The indefinite integral is equal to

S(;+~ dx=2 arc sin~ -V4-x 2 +C.


The function F (x) = 2 arc sin ~ -V4-x2 is a generalized anti deri-
vative for f (x) on the interval [O, 2], since it is continuous on
this interval and F' (x) = f (x) on the interval [O, 2).
Therefore, applying the Newton-Leibniz formula, we get
2

\ {;:; dx = ( 2 arc sin ;-V4-x2 )/: =n+2.


'6
8.2.4. Test the integral
1

SxVx dx

-1
for cm1vergence.
Solution. At the point x = 0 the integrand goes to infinity. Both

integrals
0

Sx v
-1
:>- x
and \'
.xVx
0
1

:x__ diverge, since 'A=~ > I. Consequent-


404 Ch. VIII. Improper Integrals

ly, the given integral diverges. If this were ignored, and the New-
ton-Leibniz formula formally applied to this integral, we would
obtain the wrong result:
I

5xvx=(-vx)[I =-G.
-I

And this is because the integrand is positive.


8.2.5. Test the following improper integrals for convergence:
I I

(a) \ .r dx;
ex b) \sinx+cosxd
v x.
. r I - cos x 0 I- xa
0 0
Solution. (a) The integrand is infinitely large as x __. + 0. Since

V 1-cosx= v-2sin 2 ,...,, 2V2 x as x--+o,
x

the integrand has the order /... = I as compared with ~ . According


to the special comparison test the given integral diverges.
(b) Rewrite the integrand in the following way:
f(x)=:inx+cosx ~.-,-'-
Vt +x +x v1-x 2

This function is infinitely large as x ~I, its order is equal


to /... = ~ as compared with 1 1 x, since the first multiplier tends
to 1 as X-> 0. Therefore, by the special comparison test, the given
integral converges.
8.2.6. Test the following improper integrals for convergtnce:
2 2
+ i/X:l) +I dx;
\ 3 Yx
In (1 2
(a) .\ esin x
- 1 dx; (b)
~ V l6-x 4
0 I
I
I"
(c) \ cos x d~ .
,0 tf x -StnX

Solution. (a) The integrand f (x) =In (1


e"
t Vxa)
x_ l
is positive in the
interval (0, 2) and is not defined at x =0. Let us show that
Jim f (x) = oo. Indeed, since
a: .... +o
8.2. Improper Integrals of Unbounded Functions 405

we have
( vxa)
Jim In 1+
x-+ o em x - I
=Jim
x -+ O
Lx.x3
5
= lim vl-= oo.
x -+ o X2

At the same time we have shown that f (x),...., V~x2 as x-.. 0, i.e.

that f (x) is an infinitely large quantity of order A.= { < l as com-


pared with _.!._ Consequently, by the special comparison test, the
x
given integra 1 converges.
(b) Determine the order of the infinitely large function f (x) =
= ;X2+1 16-x4
in the neighbourhood of the point x = 2 with respect
l
to 2 _x. To this end transform the expression for f (x):
Yx2+T
f (x) = vVX2+1x'- v
16- 4+ x2 v2+x v2-x . l

Hence it is obvious that the function f (x) is an infinitely large


f
quantity of order A.= < l as x-.. 2. According to the special
comparison test the given integral converges.
(c) The integrand
. f (x) = V x - Sill
.X oosx is unbounded in the neigh-
bourhood of the point x = 0. Since
cos x cos x l
f(x) =V x-sinx Vx (1-sV;) ,. ., Vx (x-- + O),

as x __. + 0 the function f (x} is an infinitely large quantity of order


A.= { < l as compared with ~ and, by the special comparis0n test,
the integral converges.
8.2.7. Investigate the following improper integrals for conver-
gence:

I I'
I I
ex dx x 2 dx
(a). V1-x3; (b) ,, V(l-x2)o'
0 0

J.l-x3+x
I

(c) ( vi- ~ l x' dx; (d)


dx
0 '

ii 0

(e)
I
r dx
J0 x-sinx'
. (f) r v; +
.I
In (
etan x -1
1) d
X.
0
406 Ch. VII I. Improper Integrals

8.2.8. Prove that the integral

~ _x_dx
I . I
sm-

0 Vx
.converges.
Solution. For 0 < x::::;;; 1

.f ;x converges,
I

But the integral therefore, by the comparison


0
I

test, the integral SI sin~x) Idx also converges, and consequently


0
the given integral converges absolutely.
8.2.9. Prove the convergence of the integral
2"
l=~lnsinxdx
0
and evaluate it.
Solution. Integrate by parts, putting u =In (sin x), dx = dv:
;r n: 31
:rt -

0
2

1r lnsinxdx=xlnsin x 1
-


2
-f
0
2

x sm
c?s xx dx = - s
2

0
-xt dx.
an x

Since Jim _x_ = l, Jim -x =


1 0, the last integra I is a proper
x-+otanx ., anx
x- 2-0
one. Consequently, the initial integral converges.
Now make the subst ltution x = 2t in integral /. Then dx = 2dt;
x = 0 at / 1 = O; x = i-
at t 2 =-=;;.. On substituting we get:
n/2 :rt/4 n/4
~ lnsinxdx=2 ~ lnsin2tdt=2 ~ (ln2+lnsint+lncost)dt=
0 0 0
:rt/4 '1/4

= 2t In 2 j~14 + 2 ~ In sin t dt + 2 ~ In cost dt =


0 0
'1/4 :rt/4

= ~ In 2 + 2 S In sin t dt + 2 S In cost dt.


0 0
8.2. Improper Integrals of Unbounded Functions 407

In the last integral make the substitution t =n/2-z. Then


dt = - dz; t = 0 at z1 = n/2; t = n/4 at 2 2 = n/4. Hence,
N4 ~4 ~2

2 S lncostdt ~=-2 S lncos(;-z)dz=2 5 lnsinzdz.


O N2 ~4

Thus,
:rt/2 :rt/4 :rt/2

I= 5 lnsinxdx=iln2+2 S lnsintdt+2 S lnsinzdz=


o o n/4
:rt/2

= ; ln2 + 2 S In sin t dt = -i In 2 + 21.


0
Whence
n:/2
I= S In sin xdx = - ; In 2.
0

8.2. IO. Compute the integral


I

Jr VI_
xn dx
x 2 (n a natural number).
0
Solution. The integrand is an infinitely large quantity of order
A.={ with respect to dx
as x---.l-0. Therefore, the integral con-
verges.
Make the substitution x =sin t in the integral. Then dx =cost dt,
x = 0 at t = 0, x = l at t = n/2. On substituting we get
N2 ~2
t
1'_xndx
.) y J -x~
=j' sinnt.costdt=
cos t
r sinntdt .
.)
0 0 0

The last integral is evaluated in Problem 6.6.9:


:rrJ/,2 .
sm" t dt =
{ n n 1 ~=~
n-1 n-3
... - } ,
2
~ n even,
o
n- 2 -3 ,
-- --
n n odd.
Consequently, the given integral is also computed L>y the same
formula.
8.2.11. Evaluate the following improper integrals (or prove their
divergence):
2 1

(a) J dx
xtnx; ( )
b j' x Vlnx
dx
; {c)
C3x2 +2
J VX'i dx.
1 0
408 Ch. VIII. Improper Integrals

8.2.12. Compute the improper integral


1

In=~ xm innxdx (n natural, m > -1).


Solution. At n = 0 the integral is evaluated directly:
1

lo= Sxmdx=m+I+1 11o=m+I'xm I

For 11 > 0 integrate In by parts, putting


u =inn x; dv =Xmdx;
xm+1
du - 11 lnn-i xdx
- X' V=m+I"
w~ get

This gives a formula by means of which one can reduce In to / 0 for


any natural n:
n n(n-1) (-l)nn!
In= - m+ If n-1=+(m+1)2 I n-2 = = (mf- J)n f o
And finally,
(-l)n n!
fn=(m+l)n+l'

8.2.13. Compute the integral


2.0

!-
- S v2+x-x2
e-x dx

0.3

accurate to 0.03.
Solution. The integral has a singularity at the point x= 2, since
2+x-x2 =(2-x)(l+x). Let us represent it as the sum of two in-
tegrals:
2-e
I=
1
S v2+x-x2
e-x dx
,
0. 3

Now compute the first integral to the required accuracy, and estimate
the second one. For e ~ 0.1 we have

O<l <v- s
2 3 :J
e- 1 9 dx 4 4 4
2 v==0.115x 3 e =0.153e,
2.9 2-x
2-e
8.3. Geometric and Physical Applic's of lmprop. lntegr's 409

Putting 8=0.1, we get the estimate / 2 < 0.028. Evaluation of the


integral
1.9
e-"dx
I -
1-
Sv2+x-x2
o.a

by Simpson's formula with a step h = 0.8 gives


so.8 = o.519,
and with a step h/2 = 0.4,
S 0 4 =0.513.
And so, integral I 1 gives the more accurate value, 0.513, with an
error not exceeding 0.001. Taking into consideration that integral / 2
is positive, we round off the obtained value to
I ::::::::0.52
with an error not exceeding 0.03.
Note. By putting 8=0.01, we get the estimate / 2 < 0.005, but the
computation of the integral
I -
1-
s
1.99
e-x dx
v2+x-x2
0.3

would involve much more cumbersome calculations.


8.2.14. Investigate the following integrals for convergence:

f
1
dx
(a) .r . ;
y stnx
0
6

(c) scos xdx.


1
2
(l-x) 2 '
(d) sYI
1
tan xdx .
x2 '
(e)
5

SYll-x
sin xdx
12
0 0 1
2

8.3. Geometric and Physical Applications of Improper


Integrals
8.3.1. Find the area of the figure bounded by the curve y = 1-2
1 +x
(the witch of Agnesi) and its asymptote.
Solution. The function y= 1 ~x 2 is continuous throughout the en-
tire number scale, and Jim y = 0. Consequently, the x-axis is the asymp-
x-oo
tote of the given curve which is shown in Fig. 118. It is required
to find the area S of the figure that extends without bound along the
410 Ch. V 111. I mp roper Integrals

x-axis. In other words, it is required to evaluate the improper integ-


oo

ral S = 5 dx
1 +x~ . By virtue of the symmetry of the figure about the
- 00

y-axis we have
00 00

S=l 1 ~xx 2 =2l 1 ~~x 2 =2 Jim arctanx!A=2~=Jt.


J I -
J
00 0
I A-oo 0

8.3.2. Find the surface area g(~nerated by revolving about the


x-axis the arc of the curve y = e- x between x = 0 and x = oo. +
Solution. The area of the sur-
y face is equal to the improper in-
1 tegral
+ 00
S=2:n: S rxVt+e- 2 "dx.
-1 0
Fig. 118 Making the substitution e-x = t.
dt=-rx dx, we get x=O at
t = t, x = oo at t = O; hence
I

S = 2:n: SVt+ fi dt = 2:n:-{ [t Vt+ t2 +In (t +Vt+ t 2 ))~ =


0

=n rv2+ In(l + V~2)J.


8.3.3. Compute the area enclosed by the loop of the folium of Des-
cartes
x 3 y 3 -3axy = 0. +
Solution. The folium of Descartes is shown in Fig. 86. Let us re-
present the curve in polar coordinates:
x = p cos cp; y = p sin cp.
Then p3 cos 3 cp + p:i sin 3 cp- 3a p2 cos cp sin cp = 0, whence, cancel I ing p\
we get
3a cos <p sin <p
P = cosa <p+ sina <p

Since the loop of the curve corresponds to the variation of cp between


0 and i the sought-for area is equal to
n n

S=+s
2
2
P dcp - 2
_ 9a 2 5(sinasin2<p <p+cos
2
cos2 <p
a <p)2 dcp.
0 0
8.3. Geometric and Physical Applic's of lmprop. lntegr's 411

To evaluate the obtained proper integral make the substitution


d~ rr
tan q.> = t; -.,-
cos- <p
= dt; q.> = 0 at t = 0, c:p = -2 at t = oo. Thus we get

3a 2 [ I ] A 3 ,
-2 lun
A ... oo
1+ a
t o
=-2 a-.

8.3.4. Find the volume of the solid generated by revolving the cis-
soid y 2 = -2 xa about its asymptote x = 2a.
a-x
Solution. The cissoid is shown in Fig. 119. Transfer the origin of
coordinates to the point 0' (2a, 0) without changing the direction of
the axes. In the new system of coordinates
X = x-2a, Y = y the equation of the cissoid !I
has the following form:
y 2 = (X +2a) 3
-X
The volume of the solid of revolution about
the axis X = 0, i. e. about the asymptote, is
-expressed by the integral 0 2a1
00 "'
I
I
v = :n; ~ xdY = 2n ~ x dY.
2 2
I
I
- "' I
Let us pass over to the variable X. For this I
I
purpose we find dY = Y' dX. Differentiating the I
I
equation of the cissoid in the new coordinates
.as an identity with respect to X, we get Fig. 119

2 yy' = - 3 (X+2a) 2 X-(X+2a)a = - 2 (X+2a)2 (X-a)


x2 x2
whence for Y >0 we have
Y' __ (X+2a) 2 (X-a) _ _ (X+2a) (X-a)
- X Y 2 - x2 v
-(X +2a)JX
Hence,
V=-2n f
0

, V-
(X+2a)(X-a) dX.
(X+2a)/X
-2a

Make the substitution (X + 2a)/X = - t 2 ; X = -2a at t = 0, X =0 at


l=oo. Then:
2a 4at 2at 2
X=- 1+ 12 ; dX=(l+t 2 ) 2 dt; X+2a= l+t 2 ;

3a+at 2
X-a=- 1+12 '
412 Ch. V 111. Improper Integrals

whence
00

r +
2at 2 (3a at 2 ) 4at di
V = 2n j t (I+ 12) (I + 12)( I 12)2 + =
0

I. +
00 00

/2 In /4
= 48:rta 3 (I 12 ) 4 dt + l 6na 3 (I+ 12 ) 4 dt.
0 n

Putting t=tanz, dt=sec 2 zdz, Wl' get t=O at z=O, t=oo at


z = n/2. Hence,
N2 N
V = 48na ~ sin z cos z dz+ 16na ~ cos 2 z sin 4 z dz=
3 2 4 3

0
JT/2 JT/2

= 48:rca ~ cos z dz - 48na ~ cos 6 z dz+


3 4 3

0 0
1t/ 2 1t/ 2

+ 16na 3 ~ sin zdz-16na ~ sin 6 zdz.


4 3

0 0

1t/2

Using the known formulas for the integrals ~ sin" xdx,


JT/2

~ cosn xdx (see Problem 6.6.9), we get


0

V 64 3 n I x3 3 n I x3x5
na 2 2x4- 64 na 2 2x4x6 = 231 a
2 3
=

8.3.5. Prove that the area of the region bounded by the curve
y= V l-x
1
2
, the axis of abscissas, the axis of ordinates and the

asymptote X= 1 is finite and equals ~ .

8.3.6. Prove that the area of the region bounded by the curve
y= V1x-., ,
3 the axis of abscissas and the straight lines x = 1 is
fi n i te and equals 6, and the area of the region contained between
I
the curve y= 2 , the axis of abscissas and the straight lines x= +1
x
is infinite.
8.3.7. Find the volumes of the solids enclosed by the surfaces
generated b.y revolvi'ng the lines y = t-x, x = 0, y = 0 (0 ~ x < oo ): +
(a) about the x-axis,
(b) about the y-axis.
8.3. Geometric and Physical Applic's of lmprop. lntegr's 413

x3
8.3.8. Compute the area contained between the cissoid y 2 = 2a-x
and its asymptot<:>.
8.3.9. Compute the area bounded by the curve y = e- 2 x (at x > 0)
and the axes of coordinates.
8.3.10. Find the volume of the solid generated by revolving, about
the x-axis, the infinite branch of the curve y = 2 ( ~ - : 2 ) for x ~ l.
8.3.11. Let a mass m be located at the origin 0 and attract a
material point M found on the x-axis at a distance x from 0 and
having a mass of I, with a force F = ; (according to Newton's
law). Find the work performed by the force F as the point M moves
along the x-axis from x= r to infinity.
Solution. The work will be negative, since the direction of the
force is opposite to the direction of motion, hence
oo N

A= f - ~ dx = lim f - ~ dx = - .!?::.
J
,
X N-+ooJ
1
x r

During the reverse displacement of the point M from infinity to


the point X= r the force of Newtonian attraction will perform posi-
tive work !!!_.
r
This quantity is called the potential of the force
under consideration at the point x = r and serves as the measure of
potential energy accumulated at a point.
8.3. 12. In studying a decaying current resulting from a discharge
"ballistic" instruments are sometimes used whose readings are pro-
oo

portional to the "integral current intensity" g= ~I dt or the "inte-


o
00

gral square of current intensity" S = ~ / 2 dt and not to the instan-


o
taneous value of the current intensity I or to its square / 2 Here t
is time measured from the beginning of the discharge; I is alterna-
ting-current intensity depending on time. Theoretically, the process
continues indefinitely, though, practically, the current intensity be-
comes imperceptible already after a finite time interval. To simplify
the formulas we usually assume the time interval to be infinite in
all calculations involved.
Compute g and S for the following processes:
(a) I= ! 0rkt (a simple aperiodic process); k is a constant coeffi-
cient, which is greater than zero.
414 Ch. VI II. Improper Integrals

(b) I= I 0r k t sin ffit (simple oscillating process); coefficients k and U>


are constant.
Solution.
"' A
(a) g=S l 0r k 1 dt= lim t 0r k 1 dt=l 0 lim r r-~-k 1 ] 0A=l 0 /k;
0
A-+oo.J0 A-+oo

. 00

S= j 12e-2ktdt=-o .
/2
0 2k ,
0
oo A

(b) g= ~ I,rk sinffitdt= lim \ ! 0r k 1 sinffitdt=


1
o A-+oo<~

_ Io . t k srnwt)e
1 [( wcosrn+ -kt] A_ IoUJ
-(f) 2 +k 2 1m 0 -(f)~+k 2 ,.
A ~ oo
oo A

S= l /~e-2"1 sin2 cot dt = Jim (' /~ e-2kt 1-c~s 2 @I dt =


J A-+ooJ0

:! )~m00
0

= - [ 1- UJ 2 ~k 2 (k 2 cos 2ffit + wk sin 2wt) J e- "1 1: =


2

l~w 2

8.3.13. Let an infinitely extended (in both directions) beam lying


on an elastic foundation be bent by a concentrated force P. If the
x-axis is brought to coincidence with the initial position of the axis
of the beam (before the latter is bent) and the y-axis is drawn
through the point 0 (at which the force is applied) and directed
downwards, then, on bending, the beam axis will have the follo-
wing equation
y= ~; e-a1x1(cosax+ sin a J x I),

where a and k are certain constants. Compute the potential energy


of elastic deformation by the formula
00

W =Ee ~ (y") 2 dx (, e canst).

Solution. Find y":

y" = p: e-ax [(cos ax+ sin ax)-2 (-sin ax+ cos ax)+
p
+ (-sill ax-cos ax)]= Te-ax (sin ax- cos ax).
3
8.4. Additional Problems 415

Hence,
"'
P2a.6eS 2
W =-k-2 - e- ax (l-2 sin ax cos ax) dx=
0

=
p2a,sEe [ I
- k 2--
2a.
2a.-4a. 2 +4a. 2
J p2a,5e
=~.

8.3.14. What work has to be performed to move a body of mass m


from the Earth's surface to infinity?
8.3.15. Determine the work which has to be done to bring an
electric charge e2 = l from infinity to a unit distance from a
charge e1

8.4. Additional Problems


8.4.1. Prove that the integral

S"' dx
xP lnq x
I

converges for p > l and q < l.


8.4.2. Prove that the integral
00

~ x;P sin xq dx, q =I= 0

converges absolutely for -1 < (p+ l)/q < 0 and converges condi-
tionally for 0 ~ (p + l )/q < l.
8.4.3. Prove that the Euler integral of the first kind (beta func-
tion)
I

B (p, q) = ~ xP- 1 (l-r)q- 1 dx


0

converges for p > 0 and q > 0.


8.4.4. Prove that
T

lim
r ... oo
-j,-J'sinax-sin~xdx=O,
0
if Ia I=1= I~ I
8.4.5. Prove that
"'
I= Se-x' x 2 n+ 1 dx = ~I (n natural).
0
416 Ch. VI II. Improper Integrals

J ~x)
00

8.4 .6. Prove that if the integra I f dx converges for any posi-
a
tive a and if f (x) tends to A as x--+ 0, then the integral
00

\ f (ax)-f (~x) dx (ex> 0, p > 0)


C' x
0

converges and equals A In (P/a).


8.4.7. Prove that
00

j' cos
Oh

M_e-~x dx = -cos~x dx =In 1_.


S e-
x
ax
x a
0 II

JT/ 2

8.4.8. At what values of m does the integral j. I -cos


xm
x dx
con-
verge?
:rt

8.4.9. Prove that the integral J(s~:.t?


0
converges if k < I, and
diverges if k ;::, I.
00

8.4. 10. Prove that the integral Ssin x ( lx-s cos x) dx converges if
0
0 < s < 4, and converges absolutely if I < s < 4.
8.4. 11. Suppose the integral
+ 00
~ f (x) dx (I)
a
converges and the function c:p (x) is bounded.
Does the integral
+ 00
~ f (x) c:p (x) dx (2)
a
necessarily converge?
What can be said about the convergence of integral (2), if integ-
ral (I) converges absolutely?
8.4.12. Prove the validity of the relation
f (x) = 2f (n/4 +x/2)-2f (n/4-x/2)-x In 2,
x
where f (x) = - ~ In cosy dy.
0
8.4. Additional Problems 417

Compute with the aid of the relation obtained

J
2

f(~ )= - In cos y dy.


0

8.4.13. Deduce the reduction formula for the integral


lT
2

In=~ lncosxcos2nxdx (n natural)

and evaluate this integral.


ANSWERS AND HINTS

Chapter I
1.1.5. (b) Hint. Prove by the rule of contraries, putting 2=p~,
q-
wherepandq
are positive integers without common multipliers.
s 2 -2
1.1.8. Hint. You may take k=~.

1.1.9. (b) x;?4, x~O; (c) -4~x~2.


I.I.I I. (a) x < - I or x;? I. Hint. The equality is valid for those values
x-1
of x for which x+ 1 ? 0; (b) 2 ~ x ~ 3. Hint. The equality hold;lrue for those
values of x for which x 2 -5x+6~0.
1.1.13. (a) x < ; or x > 8; (b) x <0 or 0 < x < 5. Hint. The inequality
I a-b \ > I a 1-1 b I holds good when a and b are opposite in sign or when
la I< b I
O a+ 2 . ( 3 ) 3 I)
1.2.3. ' [a (a2+3a+3)]' a +a (a - .
? 2. (a+h)3
1.2.4. b-+ab+a, - 8- - 1 . 1.2.6. 4
.r-
r 2+ I,. -
2+ 1 .
-2- , 2 VTO-s.
1.2.11. f(x)=I0+5x2x.
1213 ,( 3 ) = 45x 2+1. f( 3 )=5x 6 +1.
x 2 - 3x ' x 2 -- x3 '
3f(x)=l5x 2 +3; [f(x)]3=125x 6 +75x4+15x2+1.
2-x 8-12x+6x2-x 3

l.2.14.f(2)=5;f(0)=4;f(0.5)=4;f(-0.5)= ~3 ; f(3)=8.
1.2.15. Hint. Fromxn+ 1 =xn+d it follows that Yn+i=axn+=axn+d=axnad.
1.2.16. X= 2; 3. 1.2.17. f (x)=x 2 -5x+6. 1.2.18. f (x) ~~ 23; qi (x)=527.
1.2.19. x~-1 or x;? 2. 1.2.20. P=2b+2 (I-~) x; S=b ( 1- ~) x.
:n:
1.2.21. (b) (2, 3); (c) (-oo, -1) and (2, oo); (d) x= 2 +2k:n:(k=O, 1,
2, ... ). Hint. Since sin x ~ I, the function is defined only when sin x =I;
(g) (-oo, 2) and (3, oo); (h) [I, 4); (i) (-2, 0) and (0, I); (j) - ; +2k:n: <x<
:n:
< 2 +2k:n: (k=O, I, 2, ... ).
Answers and Hints to Ch. I 419

1.2.22. (d) The function is defined over the entire number scale, except the
points x= 2.
1.2.24. (a) (-oo, oo); (b) (3-2:n:, 3-:n:) and (3, 4); (c) [-1, 3];
(d) (-1, 0) and (0, oo ). 1.2.25. (b) 5.;;;;; x.;;;;; 6.
1.2.26. (a) 2kn.;;;;x.;;;; (2k+ 1) :n: (k=O, l, 2, ... ); (b) [ - ~ , -1 J.
1.3.3. (b) Hint. Consider the difference ~-~.
1+x2 I +xi
5
1.3.4. (b) It increases for - ;+k:n: < x < i+kn (k=O, 1. 2, ... )
and decreases on the other intervals.
1.3.7. The function decreases on the interval 0 < x.;;;;; ~ from + oo to 2 and

increases on the interval -';f.;;;;; x < ..:;. from 2 to + oo.


1.3.9. (c) The function is neither even, nor odd, (d) even.
1.3.10. (a) Even; (b) odd; (c) odd; (d) neither even, nor odd; (e) even.
1.3.12. (a) IA [=5, co=4, cp=O, T=~; (b) I A 1=4, co=3, cp= ~,

T= 2; ; (c)JA[=5,co=~, cp=arctanf, T=4n. Hint.3sin ~ +4cos ~ =

=5sin (~+cp). where coscp=~, sincp=:. l.3.13.(b)T=2:n:;(c)T=l.


1.3.16. The greatest value f (l) =2. Hint. The function reaches the greatest
value at the point where the quadratic trinomial 2x2 -4x+3 reaches the least
value.
1.3.17. (a) Even; (b) even; (c) odd; (d) even.
1.3.18. (a) T=n; (b) T=6n.
1.3.19. Hint. (a) Assume the contrary. Then
x+T+sin (x+T)=x+sin x,

whence cos ( x+ ~ ) =- ~ T , which is impossible for any constant T,


2sm 2
since the left side is not constant; (b) suppose the contrary. Then
cos Vx+T=cos Yx, whence either Yx+T+ Vx=2nk, or T y y
x+T+ x
= 2:n:k (k=O, I, 2, ... ),which is impossible, since the left-hand members
of these equalities are functions of a continuous argument x.
l +arcsiny 1 1
1.4.6. (a) X= 3 ; (b) X= 3 sin y; (c) X= y 0 g 5 (Y > O); (d) x =
log 2 Y log Y (0 < y < 2 or 2 < y < oo ).
log2y-l =log Y
2

t. 5. 3. (a)
V3 V3
-2- -2-,
V3
0,--2-, ... ; (b) -2 4' -3 16' ... ,
IO 113
(c) 2; 2.25; 2 27 ; 2 256 , ....

1.6.9. Hint. The inequality 2n+ 3 _2,<e is satisfied for n>N=


1 n+l
420 Answers and Hints

=(~-I). At e=O.I the inequality is fulfilled beginning with n=IO, at


e=0.01 beginning with n= 100, at e=0.001 beginning with n= 1000.
1.6.10. Hint. Verify that the sequence {x2n-i} tends to I as n--+ oo,
and the sequence {x2n} tends to 0 as n--+ oo.
1.6.12. (a) It has; (b) it does not have; (c) it has; (d) it does not have.
1.6.14. Hint. (a) lxnl.;;;;~;
n n
(b) Ix,,!.;;;;_!_.
1.6.19. Hint. For a> I put Va=
I +an (a,,> 0) and, with the aid of the
inequality a= (I +
an)n > na,,, prove that an is an infinitesimal. For a < I put
n/(i = - 1 > 0) and make use of the inequality _!_=(I +a,,)n >nan.
V 1 - -(an
+~ a
5 I I
1.7.1. (b) 4 , (c) 0; (e) 2 . 1.7.2. (b) 16 ; (e) I; (f) I.

-+;
1.7.4. (b) I; (f) 0. Hint. Multiply and divide by imperfect of a sum, square
4
and then divide by n 3 ; (g) (h) I. Hint. Represent each summand
of Xn in the form of the difference
I I I I I . I
Ix2= 1-2 2x3=2-3; ... 'n(n+I) n-n+I'
I
which will bring Xn to the form Xn= I - n+ 1 .

1.7.5. (a)}; (b) I; (c) 0, (d) -+. Hint. The quantity


4
2~ is an in-

finitesimal, and cos n 3 is a bounded quantity; (e) O; (f) 3.


1.8.6. (b) Hint. The sequence is bounded due to the fact that n! =I X2X
x3x ... xn ~ 2n- 1 and therefore
x,,.;;;;2+}+(f Y++( +
1.8.7. (b) 0. Hint. Take advantage of the fact that Xn+i = +
r-1
2 < I.
=3-U r-1 <3.
Xn n 3
1.8.9. Hint. For all n, beginning with a certain value, the inequalities
*<a< n are fulfilled; therefore i)n < fla < vn. and lim vn=
=lim ~Ii= I.
1 1
. 21+1 2nx2
1.8.10. flint. The sequence { y,, } d ecreases, since Yn+ 1 =a =a =
= Vi,; (Yn > I).
The boundedness of the sequence from below follows from a > I. Denote
Jim Yn by b and from the rel at ion y,, +I= vi,; find b = I.
fl~ (XI

1.8.11. Hint. Ascertain that the sequence increases. Establish the bounded-
ness from the inequalities
I I
n~ < n(n-l) n-1 n (n~ 2);

x,,<1+(1--})+(-}-f)+ ... +(n 11-~)=2-~,


~~~~~~~~~_A_n_s_Wt_'r_s_a_n_d_l_f_in_t_s~t~o~C~h~._1~~~~~~~~421

1.8.12. Hint. Transform Xn into the form Xn = 2n and take advan-


V n2 + l + n
tage of the inequalities
2n 2n
--< <I.
2n I+ y n2 + I+ n
1.8.13. Hint. See Problem 1.8.7 (a).
1.8.14. Hint. Establish the boundedness of the sequence by comparing Xn
with the sum of some geometric progression.
1.9.2. (b) Hint. Choose the sequences
I , I
x.,=n and Xn=-n (n=I, 2, . )

and ascertain that the sequences of appropriate values of the function have
different limits:
l
I X~
Um 2xn = + oo, lim 2 =0.
1.9.3. (e) Hint. Take advantage of the imquality

~-arctanx<
2
tan (~-arctanx)=_!.._
2 x
(x > 0).
(f) Hint. Transform the difference
. I . . n
smx- 2 =smx-sm 6

into a product and apply the inequality \sin a!..;;;;;; I a\.


1.10.1. (d) ~ ; (e) ~ ; (f) - 1~. Hint. Multiply the numerator and de-

nominator by i~perfect trinomial square ( V IO-x+ 2): (g) ~~; (h) log0 6.

.
Hint. . [ log 0 . r
hm x-3
- - =log 0 J [ !rm. (x-3) (VX-t6+3)]
_ 3 =log 0
. 2;
6;(1)-
3
x-3 r x+6-3 x-3 X

(j) 172.

1.10.2. (e) ; . Hint. On removing the irrationality to the denomim1tor divide


the numerator and denominator by x.
1.10.3. (b) 32. (c) ; . Hint. Put x= zU; (f) oo. Hint. Put ~ -x= z;
1t 1t
X=2-z: z----+- 0 as X-+-2; (g) -3. Hint. Put sin x=y.
I
I
1.10.5. (b) e 3 ; (c) e- 1 ; (d) emk; (f) 4; (g) -
a ' (h) 2.
I I cot a
1.10.7. (b) 4 1.10.8. (b) I; (c) - ;
e
(d) e
I 3 I 2
1.10.11. (a) 2; (b) -4 (c) 2; (cl) 5: (e) O; (f) - I.
I :rt I
1.10.12. (a) 20; (b) -2; (c) 2; (d) !i'; (e) -24.
42:2 Answers and Hints

1.10.13. (a) e4; (b) -I; (c) 2 ln a; (d) ea; (e) e 2 ; (f) e-1; (g) l;
(h) I; (i) 9; {j) l; (k) a-~. Hint.
lim eX-e~x lim e;ixe<-~>x-1 a-B.
x-o x x-o x
1.10.14. (a) Jl2. Hint. Replace arc cos (l-x) by arc sin V 2x x 2 ; (b) l; (c) a,
1.11.5. (b) It is of the third order of smallness. Hint.
lim tan a- sin a I
a- o a3 2"
t.11.6. (b) They are of the same order; (c) they are equivalent.
1.11.8. (a) IOOx is an infinitesimat of the same order as x; (b) x2 is an
infinitesimal o! an order higher than x; (c) 6 sin x is an infinitesimal of the
same order as x; (d) sin 3 x is an infinitesimal of an order higher than x;
(e) Vtan~ x is an infinitesimal of an order of smallne~s lower than x.
1.11.9. (a) It is of the fourth order of smallness; (b) of the first order of
smallness; (c) of the third order of smallness; (d) of the third order of
smallness; (e) of the first order of smallness; (f) of the order of smallness
I
2 , (g) of the first order of smallness; (h) of the first order of smallness;
(i) of the ~econd order oi smallness. Hint. Multiply and divide the difference
cos x- v~ by imperfect trinomial square; (j) of the first order of smallness.
1.11.10. The diagonal d is of the first order of smallness; the area S is of
the second order of smallness; the volume V is of the third order of smallness.
1.12.3. (b) 4; (f} 3; (g) -} ; (i) 2. 1.12.6. (a) I; (b) 2.
I 3 4 3 3 2
1.12.7. (a) I; (b) 3 . 1.12.8. (a) 5 ; (b) 5 ; (c) -2 ; (d) 2 ; (e) g;
3
(f) 4 ; (g) -2; (h) I. 1.12.9. 10.14. Hint. 1042= 103 X (I +0.042).
1.13.1. (b) f(l-0)=-2, f (1+0)=2; (f} f (2-0)=- oo;
f (2+0)= + 00.
I
1.13.3. (a) f (-0) = 2 f( +0) =0; (b) f (-0) =0,
(c) f (-0)=-l, f(+O)=i.
1.14.2. (b) The function has a discontinuity of the first kind at the point
x=3. The jump is equal to 27.
1.14.3. (c) The function is continuous everywhere; (e) the function has
a discontinuity of the first kind at the point x=O; the jump equals n. Hint.
n n
arc tan (-oo)= - 2 , arc tan ( + oo)= + 2 -.
1.14.6. (b) At the point x 0 =5 there is a discontinuity of the first kind:
/(5-0)=-"i {(5+0)=~; (c) at the point x 0 =0, a discontinuity of the

first kind: f(-0)=1, f(+0)=0; (d) at the point x 0 = ~, an infinite dis-


continuity of the second kind:

t(~-o)=+oo, t(~+o)=-oo.
1.14.7. (a) At the point x=O there is a removable discontinuity. To remove
the discontinuity it is sufficient to redefine the function, putting f (0) = 1; (b) at
Answers and Hints to Ch. I 423

the point x~~o there is a removable discontinuity. To remove the discontinuity


it is sufficient to extend the function putting f (0)= I; (c) at the point x=O
there is a discontinuity of the second kind: f (-0) = 0, f ( 0) = oo; (d) at + +
the pointsx=(2k t-1)-i (k=O, 1, 2, ... ),removable discontinuities, since
. . .,,, { 0 if I sin x / < 1,
f(x)= hm (smx)- =
n -+ oc 1 I.f I. _
Sin X 1-
1.
,

(e} at the points x=kn (k~-0. I, 2, ... ), discontinuities of the 1st kind,
since
f (x) = I si_n x I J I if sin x > 0,
smx ) -1 if sinx < O;
(f) at the points x=n=O, I, 2, ... removable discontinuities, since
(x) = { -1 ~f x=: n,
f
0 1f x ""n.
1.14.8. (a) At the point X= 1 there is an infinite discontinuity of the second
kind; (b) at the point x=-2, a discontinuity of the first kind (the jump
being equal to 2); (c) at the point x=O, an infinite discontinuity of the second
kind, at the point x= l, a discontinuity of the first kind (the jump being equal
to -4); (d) at the point X= l, an infinite discontinuity of the second kind.
3 1
l.14.9. (a) f(O)=l; (b) f(0)=- 2 ; (c) f(0)= 2 ; (d) f (0)=2.
1.15.2. (b) The function is continuous on the interval (0, oo ). +
1.15.3. (b) The function is continuous everywhere. At the only possible point
of discontinuity x=O we have
lim y = lim u 2 = 1; lim y= lim u 2 = 1;
~--0 U-+1

(c) at the points x = ~ + mt (n = 0, I, 2, ... ) there are removable dis-


con!inuities, since limy= lim IJ=-1.
3t
x--
u-oo
2
1.16.2. Yes. 1.16.12. 1.53. 1.16.13. No. For instance, the function y=x~
on the interval [-1, 1].
1.17.1. (a) Hint. Multiply the obvious inequalities:
vf:li< n11;
.
rr - - n-+-l
2(n-I)< - 2- ;

,r;-::---
... (ri-1)2 < n+ I
-2-;

.r-
r n-1 < -n+l
2- .
1 3 5 2n-I
(b) Hint. Le! A-2X-,:rX5X- .. X~.
2 4 6 2n
B -~ 3 X 5 X 7 X ... x 2n t- I .
. 2n-I 2n 1
Then A< B since~< 2n+l and A 2 < AB~ 2n+l"
424 Answers and Hints

1.17.2. (a) Hint. Extract the lOlst root from both sides of the inequality
and reduce both sides by 101 2
(b) Multiply the obviou5 inequalities:
99 X IOI < 100 2 ,
98 x 102 < 1002,

2 x 198 < 100 2 ,


I x 100 X 199 X 200 < 100'.
1.17.3. (a) -3< ~<-I or I <x<3; (b) x<- 3I or x>
5
3 : (c) the
inequality has no solutions, since it is equivalent to the contradictory system
x-2 > 0, x (4x 2 -x+4) < 0. 1.17.4. Yes. 1.17.5. (a) No; (b) Yes.
1.17.7. Hint. Apply the method of mathematical induction. At n= I the
relation is obvious. Supposing that the inequality
(I +x1 ) (I +x 2 ) (I +xn- 1 );;;,: I +x 1 +x 2 + ... +x,,_ 1
holds true, multiply both its sides by I +xn and take into consideration the
conditions l+xn > 0, x;x,. > 0 (i=I, 2, .. ., n-1).
1.17.8. (a) [I, + oo); (b) (2n:rc)2 ,,;;;;x,,;;;; (2n+ 1) 2 n 2 (n=O, I, 2, ... );
(c) x=O, I, 2 .... ; (d) (-oo, 0) for f (x); g (x) is nowhere defined;
:re
(e) [-4, -2) or (2, 4); (f) x= (2n+ I) 2 (n=O, I, 2, ... ).
1.17.9. (a) No: cp(O)=l, and f(O) is not defined; (b) No: f(x) is defined
for all x :j:. 0, and <p (x) only for x > 0; (c) No: f (x) is defined for all x, and
cp(x) only for x;;;.O; (d) Yes; (e) No: f (x) is defined only for x > 2, andcp(x)
for x > 2 and for x < I.
1.17.10. (a) (0, oo); (b) [I, oo). 1.17.11. V=B:rc (x-3) (6-x), 3 < x < 6.
1.17.12. (a) X=5. Hint. The domain of definition is specified by the inequa-
lities x+2~0, x-5;;;.0, 5-x;;;.O, which are fulfilled only at the point
x=5. Verify that the number x=5 satisfies the given inequality. (b) Hint.
The domain of definition is speci tied by the contradictory inequalities x-3 > O;
2-x > 0.
2 x ax -La- x ax - a - x
l.17.17.(a)f(x)= 1 +x 2 1 +x 2 ; + (b)ax= 2 2 +
(seePro-
blem 1.17.16).
1.17.18. An even extension defines the function
f(x)=x 2 +xfor O.;;;;x,,;;;;3,
<p (x) = { f (-x)=x 2 -x for -3,,,;;;; x < 0.
An odd extension defines the function

(x)={ f (x)=x 2 +x for O.;;;;x,,;;;;3,


'll -f(-x)=-x2 +x for -3.;;;;x<O.
1.17.21. Hint. If the function f (x) has a period Ti. and the function cp (x)
has a period T 2 , and T 1 = n 1d, T 2 = n 2d (n 1 , n 2 positive integers), then the pe-
riod of the sum and the product of these functions will be T = nd, where n is
the least common multiple ol the numbers n 1 and n 2
1.17.22. Hint. For any rational number r

A (x + r) =A (x) = { I for ~a!io.nal x,


0 for 1rrat10nal x.
But there is no least number in the set of positive rational numbers.
Answers and Hints to Ch. I 425

1.17.23. Hint. II we denote the perioc1 of the function f (x) by T, then from
f (T) = f (0) = f (- T) we get
sin T +cos aT= I= sin (-T)+cos (-aT),

whence sin T = 0, cos aT = I, and hence T =kn, aT = 2nn, a= 2; is rational.


t.17.25. The difference of two increasing functions is not necessarily a mono-
tonic function. For example, the functions f (x) = x and g (x) = xi increase for
x~O. but their difference f(x)-g(x)=x-x 2 is not monotonic for x~O: it

increases on [ 0,
1.17.26. Example:
+J and decreases on [ ~ , oo) .

x if x is rational,
{
Y= - x if x is irrational.
I I+y
1.17.27. (a) x= 2 ln l-y (-I< y < l);
(b)
(y _ for - oo < y < I. !J ~<S> /
x = { Vy for 1 ,,,;;;,. y ,,,;;;,. 16, ~/
/
I log 2 y for 16 < y < oo. /
t.17.28. Hint. Thefunctionsy=x 2 +
+2x+I (x ~-I) and u=- 1+vx
(x ~ 0) are mutually inverse, but the
equation y=x, i. e. x 2 +2x+ I =X has f{ff:c11
no real roots (see Problem 1.4.4). r '/J
1.17.30. (c) Hint. If Eis the domain
of definition of the function f (x), X
then the function y=f [f (x)] is deli- f(X) .X
ned only for those xE E for which
f(x)EE. How the points of the desired Fig. 120
graph are plotted is shown in Fig. 120.
1.17.32. Hint. The quantity T=2 (b-a) is a period: from the conditions of
symmetry f(a+x)=f(a-x) and f(b+x)=f(b-x) it follows that
f [x+2 (b-a)] =f [b+(b+x-2a)] =f (2a-x)=f [a+(a-x)] =f (x).
1.17.33. (a) It diverges; (b) it may either converge or diverge. Examples:
I . [l+(-l)n)
X11 =n Yn= 2 : 11 I~rn,,, (x11y11 )=0,
1
X11=- Y11=n2; Jim (x 11 y11 ) = oo.
n
1.17.34. (a) No. Example: x 11 =n; y 11 =-n+ l; (b) No.
1.17.35. a.11 =
n0-~
(n=3, 4, ... ).
.
1.17.36. Hint. Take mto account
n
that llx11 l-lalj.,;;;,.lxn-al. The converse is incorrect. Example: x11 =(-J)11+1.
1.17.38. Hint. The sequence a.n may attain only the following values:
0, I, ... , 9. If this sequence turned out to be monotonic, then the irrational
number would be represented by a periodic decimal fraction.
1.17.39. Hint. If the sequence ~11 increases, then
II
a; a11+1
-b <-b-' i. e b,.+1a; < a11 + 1 IJ; (i=I, 2, .. , n),
i 11+1
426 Answers and Hints

whence it follows that


b11 +1 (a1 +a 2+ ... +a,,)< a,,+ 1 (b 1+b 2+ ... +b,,),
and hence
a 1+a 2 + ... +a,,+ 1 a1+a 2+ ... +a,,
u1+bd- ... + b,,+ 1 - b1+b 2 + ... +b,,
a,,+ 1 (b 1+b 2 + ... +b,,)-b,,+ 1(a1 +a 2+ ... +a,,)
(b1+b2+ ... +bn+1)(b1+b2+ ... +b,,) > O.
1.17.40. (a) 2; (b) O; (c) 0. 1.17.41. Hint. From the inequalities
. f
nx-1<E(nx),;;;;;;nx1t allows that x-
l E (nx)
< x-nI < -n-,;;;;;;x.
1.17.42. Hint. From the inequalities
n n n
~ (kx- I),;;;;;; ~ E (kx),;;;;;; ~ kx,
k=I k=I k=I
it follows that

x
n+ I I
2n - n,;;;;;; n2
I
L; n+ I
E (kx) ,;;;;;; x 2n.
k=I

1.17.43. Hint. Take advantage of the fact that Jim a" = Jim V~ =I (see

Problem 1.6.19), Jim a


n .... oo
n =
Jim
1 v-= a
I, and for a> 1, I h I< _!_
n
the ine-

fl - 00
1

qualities a n -1 <ah- I < an - I take place.


1.17.45. Hint. Divide the numerator and denominator by xm.
1.17.46. (a) a= I; b=- 1; (b) a= I; b=-}. Hint. To find the coefficient
a divide the expression by x and pass over to the limit.
1.17.47. (a)
f (x) = { I for 0,;;;;;; x,;;;;;; 1,
x for x > I.
(b)
0 for x =I= ; +mt,
f(x)= { :rt (n=O, I, 2, ... ).
I for x= 2 +nn

1.17.48. Hint. Take advantage of the identity


(1-x) (1 +x) (I +x 2 ) (I +x2")= l-x 2".
1.17.49. Generally speaking, one can't. For example,
Jim In (I +x)+ In (1-x) = lim In (l -x 2 ) = - 1
x - o x2 x - o x:l '
and if we replace ln(l+x) by x and ln(l-x) by -x we will get the wrong
result: lim x-x =O.
x .... o x2
Answers and Hints to Ch. I 427

1.17.50. -} . Hint. If a is a central angle subtended by the arc under con-

sideration, then the ct-:ord is equal to 2R sin ~ - Ra, and the sagitta to
a2
R (I -cos a) - R 2 .

1.17.51. 2. Hint. The difference of the perimeters of a circumscribed and


inscribed regular ngons is equal to

n - sin. n:re) = 2n Rtana-sina


2R n (t an :n a -
R
n a
2

where a=~, and the side of an inscribed n-gon is


n

2Rsin ~=2R sin a,..., 2Ra.


n

1.17.52. On the equivalence of (l+a) 3 - l and 3a as a---+0.


In (I +x) x
1.17.53. No, log(l+x)= In 10 - 1n 10 as x---+0.
1.17.54. (a) Yes. Hint. If the function <p(x)=/(x)+g(x) is continuous at
the point X= x0 , then the function g (x) = <p (x)-f (x) is also continuous at this
point; (b) No. Example: f (x)=-g(x)=signx (see Problem 1.5.11 (p)); both
functions are discontinuous at the point x= 0, and their sum is identically
equal to zero, and is, hence, continuous.
1.17.55. (a) No. Example: f (x) =xis continuous everywhere, and g (x) =sin.::_
x
for x :f. 0, g(O)=O being discontinuous at the point x=O. The product of these
functions is a function continuous at x= 0 since Jim x sin.::_= 0; (b) No. Exam-
x ... o X

pie: f (x) = - g (x) = { 1 ffor x ;;::=. 0 both functions are discontinuous at the
- 1 or x < 0;
point X= 0, their product f (x) g (x)=-1 being continuous everywhere.
1 if x is rational, .
1.17.56. No. Example: f (x)= { - 1 1'f x 1s . . t
1rra 10na 1.
We may write
f (x) = 2A. (x)- 1, where A. (x) is the Dirichlet function (see Problem 1.14.4 (b)).
1.17.57. (a) x=O is a discontinuity of the second kind, x= I is a disconti-
nuity of the first kind; (b) x= I is a discontinuity of the first kind: f ( 1-0) = 0,
f (I +o) = 1; (c) <p (x) is discontinuous at all points except x = 0.
1.17.58. (a) x=n=O, 1, 2, ... are discontinuities of the first kind:
Jim y= I, Jim y=y lx=n=O. The function has a period of l; (b) x= Vn
X-+n-0 X-+n+O
(n= 1, 2, . ) are points of discontinuity of the first kind:

Ii~ y=2n-I; Ii~ y=y lx=Vn=2n.


x-Vn-0 x_.,Jln+O

The function is even; (c) x=Vn (n=l, 2, ... )are the points of
discontinuity of the first kind; at these points the function passes over from the
value 1 to -1 and returns to I. The function is even;
428 Answers and Hints

(d)
( x if I sin x I < ~ , i.e. - ~ + :nn < x < ~ +:rtn,
y= ~ Ix2 "f
1
I sm
. x I = 2I , .
1.e. x= 6:rt+ :rtn,
Il 0 1"f I sm
. x I > 2I , .
1.e. 6:rt + :rtn < x < 65:rt + :nn.

x= ~ +:rtn are discontinuities of the first kind.

1.17.59. The function f (g (x)] has discontinuities of the first kind at the
points x= -1; O; +I.
The function g If (x)) is continuous everywhere. Hint.
The function f (u) is discontinuous at u = 0, and the function g (x) changes sign
at the points x=O, 1. The function g If (x) I= 0, since f (x) attains only the
values 0, 1.
1.17.61. Hint. Write the function in the form
x+ I for -2 ~ x < 0,
f (x) = { 20 for x = 0,
(x+l)2 x tor 0 < x~2.

Make sure that the function increases from - I to I on the intE'rval r-2, 0)
and from 0 to ~ on the interval (0, 2]. Apply the intermediate value theorem
to the intervals [-2, -I) and (0, 2]. The function is discontinuous at the point
x=O: f (-0)=1, f (+0)=0.
1.17.62. Hint. Suppose e > 0 is given and the point ~oEla, b] is chosen. We
may consider that
e,.;;;;min[f(x 0 )-f(a), f(b)-f(xo)].
Choose the points x 1 and x 2 , x1 < x0 < x2 so that
f (x1)=f (xo)-e, I (x 2)=/ (xo)+e,
and put ll=min(x 0 -X1, x 2 -x0 ).
1.17 .63. Hint. Apply the intermediate value theorem to the function
g(x)=f(x)-x.
1.17.64. Hint. Apply the intermediate value theorem to the function f (x) on
the interval [x1 , Xn], noting that
min [f (x1), ... , f (xn))..;;;;;; _!_If (x1) + f (x2) +. . +f (xn)]..;;;;;; max lf (x1) .. , f (xn)J.
n
1.17.65. Hint. Apply the intermediate value theorem to the function f! (x) =
= 2x- ! on the interval [{, 1].
1.17 .66. Hint. At sufficiently large values of the independent variable the
values of the polynomial of an even degree have the same sign as the coefficient
at the superior power of x; therefore the polynomial changes sign at least twice.
1.17.67. Hint. The inverse function
-Jf-y-1 for y<-1,
x= { 0 for y=O,
Jt'Y=T for y > I
is continuous in the intervals (-oo, -I) and (I, oo) and has one isolated point
y=O.
Answers and Hints to Ch. I I 429

Ct1 apter I I
20
2.1.1. (b) -21. 2.1.2. (b) y' = IOx-2. 2.1.5. Vav=25 m/sec.

2.1.6. (a) y'=3x 2; (b) y'=- ~. 2.1.7. The function is non-differentiable at


2 _-2._ 4 _.!....
the indicated points. 2.2.1. (b) y'=- 3 ax 3 + 3 bx 3 . 2.2.2. (c) y' =

= 2x arc tan x+ I. 2.2.3. (b) -9000. 2.2.4. (a) y'=6x2 +3; (b) y'= .~- -
2 r x
----
I +
x'
9 ()
c y -
, _ -3x 2 +2x+2.
'
(d) ,_ 3
y --
Vx-+B Vx+2 vT/X.
2x Vx (x:i-x+ 1) 2 6 (x-2 V x)
() ,_cos(f'-sincp-l, (f) y'=2ex+~ (g) y'=2excosx; (h) y'=
e Y - (I-cos cp) 2 ' x '
x (cos x-sin x)- sin x-ex
x2ex
2.2.5. (f) 30ln 4(tan 3 x)~ ; (g) sin ,,,- 2 6
SIO X r 1-x _3
2(1-x) 2
2 2 cos x 2 sin x
2.2.6. (b) y'=-3(3-sinx) cosx; (c) y' +-
3 sin x sin2 x cos3X"' V
2ex + 2x In 2 5 ln 4 x
(d) ,_
y - 3 V (2ex - 2x + I):! +--
x '
(e) 11' = 3cos3x - ~ sin ~ +

I .r- a a
+ .r- sec:! r x; (f) t/=(2x-5)cos(x 2-5x+I)- 2 sec 2 - ; (h) y'=
2 r x x x

= x '1+1n2x
I + arctanx
I + I+x:i;
I (')
1
I
11 =
21 t x
narc an3 t
I
x 9+x2
3
t' arc an 3

2 28
b
( )
,__
I
11 -
2 I +
2
sinh:! (tan x) sec x cosh 2 (cot x) cosec x, (
d) I 3
Y = x X
X (x sinh 2x3 +cosh x2sinh 2x2); (e) y' = esinh ax ix<acosh ax+b>,
2.2.9. (c ) y ,_av2I-x
- x 1 +x:i sm a xcos 2 x (2x - l-x
3
I - l+x2x 2 +3 cox-
t 2t anx ),
(x+ I)
(d) y' =(tan x)_2 _ (-}In tan x + :i~2~).
2.2.13. (a) f' (x) = ~ ( cosh ~ +sinh ; ) ; (b) f' (x) = tanh x; (c) f' (x) =

= Vcoshx+I; 1h-;
(cl) f'(x)=- (e) f'(x)=4sinh4x; (f)f'(x)=(a+b)eaxx
cos x
X (cosh bx+sinh bx)=(a+b) e<a+b>x.
2.2.14. (a) y'=(cosx)' 1"x (cos x In cos x-tan x sinx);
b I COS 3x
( ) Y = V
sin 2 3x(l-sin 3x)4;
, 5x 2 +x-24
(c) Y = I 5 5 '
3(x-1) 2 (x+ 2)3 (x +3i'2
430 Answers and Hints

17 ) , In 3 tan Yarc sin 3- 2 x.


2.2. . (a Y = .rs1x-1
r V arc sin 3-2x ,
. I a I 2
(b) y' = - sm n X n x .
5x V cos ln x ( 1 +
4 3 V
cos 2 ln 3 x) V
(arc tan V cos Ina x) 2
2.3.1. (b) knekx; (e) 2n-1 sin ( 2x+n ; ) ; (f) {sin ( x+n ; ) +

+ 32n sin ( 3x+n;) +54n sin ( 5x+n;).


2.3.4. (b) ex(x 2 +48x+551); (c)ea.x{sinBx [a.n-n<7:; 1>a.n- 2B2+ ... ]+

+cos Bx [ na.n-1B - n J
(~~ 1 ~;- 2 > an-aBa+ ... J}.
2_3 _6_ (a) 2x2+3x ; (b) (1 +2x2) arc sin x +~; (c) 2e_x 2 x
(1 +x2) Vl +x2 ~ (l-x2)2
(l-x2) 2
x (2x 2 -l).
(a) x3 sin x-60 x 2 cos x-1140 x sin x+8640 cos x;
2.3.8. (b) 2e-x x
X (sin x+cos x); (c) ex [3x2+6nx+3n (n-1)-4); (d) (-l)n [(4n 2+2n +
+ I -x2) cos x- 4nx sin x].

2.3.9. (a)
1001 [ I
(x-2)101
I
(x-1)101 ;
J
(b)tx3x5x ... xt97x(399-x)
201
2100 (l -x)_2_
I I
Hint. y=2 (l-x)- 2 -(I -x)2.
" 4 cos x
2.4.1. (b) Xyy=- (6+ sin x)a.
k-1
2.4.3. (b) y~= - cot - 2 - t; (d) y~=- 2e- 2 ct.

2 4 4 (b) y~ 3 (t2~ 1)3; 1


(c) u:x= at cos 3 t
2.4.5. (b) Y~':x=-3sintsec 2 t.
,_ y
2. 4. 6 (b) Yx-x +e --xy ., (c) Yx-y_
, _ 2-x .,
5 x (d) u=-
x
Vy.
,, _(ex-eY) (1-ex+Y). 4ex-Y 4 (x+y)
2.4.7. (b) Yxx- (I +eY)a ' (c) Y:x= (ex-Y 1)3 (x+y+ 1)3. +
2a-2x-y x+y ex sin y+e-Y sin x I
2.4.9. (a) + 2 ; (b) - ; (c) -excosy+e-Ycosx; (d) -7
x y- 2a x-y
4 10 ( - 2y2 + 2 . (b) !!.!. 2.4.11. (a)
c sin t t
2. . . a) y5 ' 256 . a (b+cos t) ; (b) 2;

t 2 +1
(a cos t-b sin t) cos 3 f
(c) 4t3; (d)
2t (2t 2 +2t+ 1);
(e)
. t
4 sm
2
.. /1-4t 2
(f) - V 2-t2 ; (g) - 1-12.
2.5.1. (b) 6x+2y-9=0; 2x-6y+37=0.
Answers and Hints to Ch. 11 431

2 IO
2.5.2. (c) M 1 ( - y3", 5+ 3 y3" ) , M2
( 2
'ya , 5- 3 IO
y:f )
2.5.3. (b) qi=arctan2Y2. 2.5.8. (b) x+y-2=0; y=x.
:rt 39 2 ( 5 \
2.5.15. (a) 4 ; (b) y= I, x+2y-2=0; (c)Y+f6= -3 x+ 4 };
:rt
(d) 4". 2.5.16. 11.

2.5.17. 26,450. gt2


2.5.19. s=at- 2 ; v=a-gt; Smax=S
I
I=-
a
a2
=2.
g
g
2.5.20. v = r/ = 2 7: 8 sin M (I + 2e cos M). 2.6.3. !J.y ~ dy = 0.05.
2.6.5. (b) log 10.21 ;::: 1.009; (d) cot 4510' ~ 0.9942.
2.6.7. (c) 11y=lcosxl/J,,x; (d) 11y=(l+tan 2 x)l1x.
4 ln x-4-Jn3 x
2.6.9. (a) d 2y = 4-x 2 2 In 4 (2x 2 In 4- I) dx 2 ; (b) d 2 y dx2;
x 2 Y(ln 2 x-4)3
(c) d3 y=-4 sin 2xdx 3
2610 ()d2 =-4(1+3x')d2. (b)d2 =-4(I+3x')dx2-~d2 .
.. . a Y (l-x')2 x, Y (l-x')2 l-x4x,
in particular at x= tan t, d 2 y= - ~ dt 2
cos 21
2.6.ll. 11V=4:rtr 2 M+4:nrM 2 +: :nMa is the volume contained between
two spheres of radii r and r+M; dV=4:rtr 2 !J.r is the volume of a thin layer
with a base area equal to the sphere's surface area 4:rtr 2 and a height !J.r.
2.6.12. !J.s=gt M++ g M 2 is the distance covered by a body within the
time M; ds=gt M=vdt is the distance covered by a body which would move at
a velocity v=gt during the entire interval of time.
2.7.l. (a) It does not exist; (b) it exists and equals zero.
2.7.2. 90. Hint. Since
ex, x;;;=. 0
Y= { e-x, x < 0,
f'_ (0) = -1, 1:
(0) = l.
2.7.3. f'_ (a)= -qi (a); 1:
(a)= qi (a).
2.7.4. Hint. For x t= 0 the derivative

f' (x) = - cos ( !) +2x sin ( ! ).


At x=O the derivative equals zero:
/J.x 2 sin J_
_ _ _!J._x__ o
f' (0)= lim !J.x - .
l::,,.x -? 0
Thus, the derivative f' (x) exists for all x, but has a discontinuity of the
second kind at the point x = 0.
2.7.5. a=2x 0 , b=-x~. 2.7.7. Hint. The formula for the sum of a geo-
metric progression represents an identity with respect to x. Equating the deri-
vatives of both sides of the identity, we get
nxn+l-(n+ I) xn+ I
I +2x+3x 2+ ... +nxn- 1 = (l-x)2 ;
432 Amwers and Hints

multiplying both sides of this equality by x and differentiating again, we get


12+ 22x+ +
+n 2xn-= I +x-(n 1) 2 x"+ (2n!+2n- I) x 11 + 1 -nx11+2
(l --x1"
2.7.8. sin x+3 sin 3x+ ... +(2n- l) sin (2n-1) x~
(2n-l- I) sin (2n- l) x- (Zn- I) sin (2n+ I) x
4 sin 2 x
Hint. To prove the identity multiply its left side by 2 sin x and apply the
formula 2 sin a sin~= cos (a-~)- cos (a+~). To deduce the desired formula
differentiate both sides of the identity and equate the derivatives.
2. 7.9. (a) sin 2x [f' (sin 2 x)- f' (cos 2 x) ); (b) el <x> [ext' (ex)+ f' (x) f (ex));
"ljl' (x) I <p' (x) In 1jJ (x)
(c) 1jJ (x) ' In <p (x) - <p (x) ' ln 2 qi (x)
2.7.10. (a) No; (b) No; (c) Yes; (d) No.
2.7.11. Hint. Differentiate the identity f (-x)=f (x) or f (-x)=-f (x).
This fact is easily illustrated geometrically if we take into consideration that
the graph of the even function is symmetrical about the y-axis, and the graph
of the odd function about the origin.
2.7.12. Hint. Differentiate the identity f (x-j- T) = f (x).
2.7.13. F' (x) = 6x 2 2.7.14. y' = 2 Ix I. 2.7.15. The composite function
f [qi (x)] may be non-differentiable only at points where qi' (x) does not exist
and where qi (x) attains such values of qi (x) = u at which f' (u) does not exist.
But the function y = u 2 =Ix1 2 has a derivative y' = 0 at the point x= O,
though at this point the function u =Ix I has no derivative.
2.7.16. (a) y"=6ixl; (b) y"=2sin_!_-~cos_!_-J.sin_!_
xx xx x
at x;i:O,
y" (0) does not exist, since y' (x) is discontinuous at x = 0.
2.7.17. Hint. (a) Verify that f<k> ~ 1 =C~ (k=O, 1, .. ., n) and take advan-
tage of the property of the binomial coefficients. (b) Designate: f (x) = un;
show that u~= (n-1) Un-i-Un- 2 and use the method of mathematical
induction.
2.7.18. Hint. Apply the Leibniz formula for the nth derivative ol the pro-
x
duct of the functions u=e a and v=x 2
0 at n = 2k
2.7.19. y<n> (0)= { [I X3X ... X(2k-1)) 2 at n=2k-j- I
(k= 1, 2, ... ).
Hint. Differentiate the identity n-2 times and, putting x=O, obtain
y<n>(O)=(n-2) 2 y<n- 2 l(O) (n;;;;,:2).
2.7.21. Hint. Take advantage of the definition
e-xa H n+t (x) = (e-x2)<n+ > = (-2xe-x2)1nl
and the Leibniz formula for the nth derivative of the product u = e- x2 and
V=-2X. 2.7.22. y~= 3 (y 2 ~ I).
2.7.23. x1 2= V 1+ YI-y (-oo < y.;;;;, l),
xa.4=V1-v1-y <O.;;;;,y.;;;;,1).
xj= 4x;(/-xl) (i=l, 2, 3, 4) for x;;t:O, 1.
Anszeoers and Hints to Ch. 111 433

flint. Solve the biquadratic equation x~-2x2+y=O and find the domains
of definition of the obtained functions x; (y).
2.7.25. (a) Xi=-3; X2=l; (b) X=I.
2.7.26. Hint. Note that the function x=2t-ltl={ 3:: ~~g has no deri-
x, f x~o.
vative at t = 0. But t = 1 x 0 therefore we can express y = t 2 + t It I=
\ 3' x < ,
= { ~~ 2 : ~ g through x: y= { ~~ 2 ' ;7 g'. This function is differentiable eve

rywhere. 2.7.27. a=c={; b=-}. 2.7.28. Hint. The curves intersect at the
points where sin ax= I. Since at these points cos ax= 0,
y~=f' (x) sin ax+f (x)acosax=f' (x)=y;,
i.e. the curves are tangent.
2.7.30. Hint. For t :j:: :rcn the equations of the tangent and the normal are
reduced to the form:
t l
y=cot 2 (x-at)+2a; y=-tan 2 (x-at),

respectively. For t=n ('lk-1) (k= I, 2, ... ) the tangent line (y=2a) touches
the circle at the highest point, and the normal (x=at) passes through the high
est and lowest points; for t = 2kn (k = 0, I, ... ) the tangent line (x =at) pas-
ses through both points, and the normal (y = 0) touches the circle at the lowest
d2y . M 2d<p
point. 2.7.34. dt 2 +Y
2.7.35. The relative error i'J=T::::J sin 2<p. The most
reliable result, i.e. the result with the least relative error, corresponds to the
value <p=45.

Chapter III
3.1.2. (b) Yes; (c) No, since the derivative is non-existent at the point 0.
3.1.5. s=e-1. 3.1.7. No, since g (-3) =g (3). 3.1.9. (d) Hint. Consider
the functions
f(x)=arcsin l~x2 +2arctanx for lxl >I,

g(x)=arcsin l~x2 -2arctanx for I xi< I.

1
3.1.15. (a) s= 2 ; (b)
2
~= 1 n 3 ;
10 rs2.,
(d) it is not appli-
(c) =
24
cable, since the function has no derivative at the point x=O.
3.1.16. 1.26 < In (I +e) < 1.37. Hint. Write the Lagrange formula for the
function f (x) =In x on the interval (e, e+ I] and estimate the right-hand side
in the obtained relation: In (I J-e) =I +i- (e < < e+ I).
3.1.17. Hint. Apply the Lagrange formula to the function f (x)= In x on the
interval [I, I +xJ. x > 0, and estimate the right-hand side in the obtained re-
lation In (I +x)=; (I<< I f-x). 3.2.1. (c) 2; (d) O; (f) - ; .
I x-tanx I
3.2.3. (b) 0. flint. Represent cot x--~=---- (c) -2 3.2.5. (b) e1 =e.
x x tan x '
434 Answers and Hints

3.2.6. (a) l; (b) I. 3.2.9. (a)


4
1 ; (b) In a- I; (c) 2; (d) n 3
6 '
. (e) _!a ;
-m2~
(f) O; (g) I; (h) Ina; (i) e .
2.
(j)
2
-
n
; (k) -I; (1) e; (m) 3;
2

1
I a2 I
(n) 2; (o)
2 (p) e 30 ; (q) I; (r) -2 3.3.5. (b) 0.34201.
1

3.3.6. V 83 ~ 3.018350. Hint. V 83 = V 81 +2 = 3 (I+ 821) 4


Apply the bi-
nomial formula and retain four terms.
3.3.7. Hints. (b) Write the Maclaurin formula for the function f (x)=tanx
with the remainder R4 (x); (c) write the Mnclaurin formula for the function
t

f (x) =(I + x) 2 with remainders R2 (x) and R3 (x).


I I I x2 x:ix~
3-4,2. (a) f (x)= 2 x 2 - 3 x3 - 5 x6 +o (x6); (b) f (x)=x - 2 +6- 12 +
x6
+ 24 +o (x6).
I I
3.4.3. (b) - 2 ; (c) 12 ; (d) 3 ; (e) I.
2 5 I
xt xt
3.4.4.(a) 1+2x+x2 - 3 x3- 6 x4- 15 x6; (b)- 2 - 12 +45x ;
6
(c) 1-2+
x

x2 x4
+12-120
3.5.1. (d) The function decreases on the interval (-oo, 0) and increases on
(0, oo ); (e) the function increases on the intervals ( -oo, ~ ) and (3, + oo)

and decreases on ( ~ , 3} (f) the function increases over the entire number
scale.
3.5.2. (b) The function increases on the intervals ( 0, ~) and ( 5: , 2:rc)

and decreases on ( ; , 5: ) .
3.5.8. (a) The function increases throughout the number scale; (b) the func-
tion increases on the interval (-1, 0) and decreases on (0, I); (c) the function
decreases throughout the number scale; (d) the function increases on both
intervals (-oo, 0) and (0, oo) where it is defined; (e) the function decreases
on the intervals (0, I) and (I, e) and increases on (e, + ao); (f) the function
decreases on the intervals (-oo, I) and '(I, oo), increases on (-1, I).
3.5.10. a,,;;;;O. 3.5.11. b;;?: I. 3.6.1. (b) The minimum is f (l)~f (3)=3, the
maximum f (2) =4; (d) the minimum f ( ~ ) =- ; 4 . 3.6.2. (b) The mi.nima
are f (I)= V3; the maximum f (0) = 2.
3.6.3. (b) The maximum is f (-2) = 160; the minimum f (0) = 2.
3.6.7. (b) The minimum is f (0)=0.
3.6.8. (b) On the interval (0, 2n]: the minimum is f ( ; ) =-4; the maxi-

mum 3:rc) =4. 3.6.10. (a) The minimum is f (0) = 0, the maximum f (2) = 4e-~;
f(
2
(b) the minimum is f (-2)=-l. the maximum f (2)= I; (c) the maximum i~
Answers and Hints to Ch. /JI 435

/ (0)=0, the minimum f ( 35) = - 9


25 v T
9;(d)themaximumisf(2)=-I,
the minimum f(0)=7; (e) the maximum is f (-3)=3V3, the minimum
3 ,-
' (2)= -v 44.
3.6.11. (a) There is no extremum; (b) there is no extremum; (c) the ma
xi mum is f (0) =0; (d) the minimum is f (0) =0.

greatest value is f ( ; ) = :s ,
3.7.1. (c) The greatest value is f (I)=_!_, the least value f (0)=0;
e
the least value f (I)= 0.
(d) the

3.7.2. (b) The greatest value is y (0) = ; , the least value y ( ~ 2 ) = ~;


(c) the greatest value is y (4) =6, the least value y (0) = 0.
. 16 37
3.7.6. (a) The greatest value 1s f (-2)= 3 , the least value f (3)=- 4 :
(b) the greatest value is f (0)=2, the least value f (2)=0; (c) the greatest
value is f ( ;-3 ) = ~ +o.25 ln 3, the least value f ( 3) = ~ -0.25 In 3; (d) the

greatest value is f ( ~) = 3 ; 3,
the least value f ( 32:rt) =-2; (e) the great-
est value is f (I)= I, the least value f (2) = 2 (I -Jn 2); (f) there is no great-
est value, the least value is f (0) =I.
3.8.3. H = R V2, where H is the height of the cylinder, R is the radius of
the sphere. 3.8.7. x=a sin a, y=a cos a, where a=0.5 arc tan 2.
Hint. The problem is reduced to finding the greatest value of the function
S = 4xy+4x (y-x) = 4a2 (sin 2a-sin 2 a)
:rt 2
in the interval 0 <a< T. 3.8.8. Prnax = 4W. at W = Wi. 3.8.9. h=2R=
I

= 2 V . ~: 3.8.10. The radius of the cylinder base is r = ~ , where R


is the radius of the cone base. 3.8.11. The equation of the desired straight
.
1me .1s x+y
2 -.r= .
I
3.8.12. x=a-p for a> p and x=O for a~p.

3.8.13. v= V ;b . Hint. It will take ~ hours to cover one knot. The


. a+hv3 a
appropriate expenses are expressed by the formula T=--=-+bv2.
v v
3.8.14. <p= ~. Hint. At Hie board width a the cross-sectional area of the
trough is equal to a2 (I +cos cp) sin <p. where <p is the angle of inclination of the
walls to the bottom.
3.8.15. ~ . Hint. The point of fall of the jet is at a distance of v ~from
the tank base, where H = h-x is the height at which the orifice should be
located, v is the rate of flow; therefore the length of the jet is determined by
the expression
436 Answers and Hints

a a
3.8.16. After 2v hours the least distance will be equal to 2 km.
3.9.1. (b) The intervals of concavity are ( - oo, ~) and (I, oo), of conve-

xity ( ; , I); the points of inflection are (; , 12 ~~) , (l, 13); (c) the in-
tervals of concavity are ( - y'3, 0) and ( y'3, oo ), of convexity ( - oo, - y3)
and (0, y3); the points of inflection are ( - y3, - ~;). (0, 0), ( 3,

~) ; (e) the curve is concave everywhere; (f) the intervals of concavity are
3-Vs 3+ vs
(0, x 1 ) and (x 2 , oo ), of convexity (x1 , x 2 ), where x 1 = e-2- x 2 = e-2- the
points of inflection are (x 1 , y 1 ), (x 2 , y 2 ), where

Y1= (3-5)2
2 -2
e
V5-3
, Y2= (
3+
2
5) 2
e
3+V5
--2-
.

3.9.5. (a) The point of inflection is (3, 3); the curve is convex for x <3 and
concave for x > 3; (b) the abscissa of the point of inflection X= arc sin ~-I ;
th e curve 1s . ( - n
. concave m
2 , .
arc sm YS-1)
2 , an d convex . ( arc sm
m 5-1
. - -
2- ,

~).
3.10.1. (c) y=O; (d) x=O; (i) y=2X as x-++ oo and y= -2x as
X-+-00. 3.10.3. (a) x=3, y=x-3; (b)u= 2nx -1; (c) y=x;
n
(d) x= 2;(e)y=2x-2.
3.11.2. (a) The function is defined everywhere, it is even. The graph is sym-
metrical about the y-axis and has no asymptotes. The minimum is y (0) =I,
maxima y (I)= y (-1) = : . The points of inflection are ( ~"3, ~~} (b) the
function is defined in ( - oo, - I) and ( -1, +
oo ). The graph has a vertical
asymptote x= - I and an inclined asymptote y=x-3. The minimum is y (0)=0,
256
maximum y (-4) = - 27 . .
The points of inflection are ( -6, - 3296) and
125

( 2, ~~} (c) the function is defined in ( - oo, 0) and (0, + oo). The graph

has a vertical asymptote x=O. The minimum is y ( ~) =3. The point of in flee-

. . v2 0) .,
t1on 1s ( - - 2- , (d) the function is dtfincd in the intervals ( - oo, - 1),
(-I, I) and (I, oo); it is odd. The graph is symmetrical about the origin, has
two vertical asymptotes x= I and an inclined asymptote y=x. The minimum
is y ( y3) = +3 ~"3, the maximum y (- Jl3) = - 3 1:3 . The point of
inflection is (0, O); (e) the functioa is defined everywhere, it is even. The
Answers and Hints to Ch. /// 437

minimum is y (0) = the maxima y ( y2) = 2 v2.


graph is ~ymmetrical about the y-axis and has a horizontal asymptote y=O. The
V4. The points of inflec-
tion are ( 2, V4); (f) the function is defined in ( -2, + oo ). The vertical
asymptote isx=-2. The minimum isy(O)=O, the maximum y(-0.73)~0.12.
The point of inflection is (-0.37; 0.075); (g) the function is defined everywhere.
The horizontal asymptote is y=O as x -++ oo. The maximum is y ( !) = ( ~ )3.
The points of inflection are (0, 0),
v'3'
( 3-4 (3- 3)
4
3
va- 3)
e
(3+ 3
'4'
(3+
4
3
3)
e
-3-va).
'
(h) the function is defined and continuous everywhere. The horizontal asymptote
is y =I. The minimum is y (0) = 0, the point (0, 0) being a corner point on the
, n , 1t
graph: y_ (0) = -2. Y+ (0) = +2.
3.12.6. 4.4934. 3.12.8. X1 =-2.330; X2=0.202; X3 =2.128. 3.12.11. 0.6705.
3.12.12. (a) 0.27; 2.25; (b) 0.21. 3.12.13. (a) 1.17; (b) 3.07. 3.12.14. 1.325.
3.12.15. 0.5896 and 2.2805. Hint. To approximate the smaller root more precisely
write the equation in the form x=eo.sx-1, to find a more accurate value of the
larger root represent it in the form X= 1.25 (I+ In x).
3.13.1. No. Hint. Show that at the point x= I the derivative is non-exis
tent: (.(l)=l; f~(l)=-1.
3.13.2. Hint. Check the equality f (b)-f (a)=(b-a) f' (atb).
3.13.3. Hint. Apply the Rolle theorem to the function f (x) =a.,xn + .
. . . +an-I (x) on the interval (0, x0 ).
3.13.4. Hint. Make sure that the derivative f' (x) = 4 (x3 -1) has only one
real root, x= I, and apply the Rolle theorem.
3.13.5. Hint. The derivative f'(x)=nxn-i+p has only one real root at an
even n and not more than two real roots at an odd n.
3.13.6. Hint. The derivative is a polynomial of the third degree and has
three roots. Take advantage of the fact that between the roots of the polyno-
mial lies the root of its derivative.
3.13.7. Hint. From the correct equality lim cos~=O (0 < 6 < x), where 6 is
x- 0 l::i
I
determined from the mean value theorem, it does not follow that lim cos -=0,
x - 0 x
since it cannot be asserted that the variable 6 attains all intermediate values
in the neighbourhood of zero as x-+ 0. Moreover, 6 takes on only such a sequ-
ence of values E for which lim cos ~ =0 <6E).
3.13.8. Hint. The mistake is that in the Lagrange formula one and the same
point 6 is taken for f (x) and qi (x).
3.13.9. Hint. Apply the Lagrange formula to the function In x on the interval
(b, a]; (b) apply the Lagrange formula to the function zP on the interval [y, x].
3.13.10. Hint. With the aid of the Leibniz formula ascertain that the
coefficients of the Chebyshev-Laguerre polynomial alternate in sign, the odd
powers of x having negative coefficients. Whence deduce that L,, (x) > 0 for
x < 0.
3.13.11. Hint. Using the Rolle theorem, show that inside the interval l_x 0 , x,,j
there are at least n roots of the first derivative, n-1 roots of the second deri-
vative, and :o on.
3.13.12. Hint. The L'Hospital rule is not npplicable here, since the derh'<l
tive~ of both the numerator am! <lenond1:ator vanish at all points where the
438 Answers and Hints

factor sin x (which we cancelled in computing the limit ol the ratio of deriva-
tives) vanishes.
3.13.13. Hint. Write the Taylor formula with the remainder R 2 :
h2 h3
f (a+h) = f (a)+hf' (a) + 21 f" (a) + 31 f'" (a +8 1h).
Comparing it with the expansion given in the problem, get the equality
f" (a+Sh)-f"
h
(a) I
3f'"(a+8 1 h) and pass over to the limit as h--+0.

3.13.14. Hint. Prove by using the rule of contraries. Suppose that e=.!!...,
q
where p and q are natural numbers, p > q > I, and, using the Taylor formula,
get for n > p
p I I I I ( p )B
(/= 1+rr+2f+ ... +Ii!+ <n+ I)! (/ (O < e< 1).
Multiply both sides of this equality by n!, .!!... n! and

r
and noting, that
q
(1+fi-+ ... + ~!) nl are positive integers and n~l (~
nll < :<I,
obtain a contradictory result.
3.13.15. Hint. Verify that the function
J sinx :re
f(x)=i -x-' O < x~2' is continuous on the interval ~]. lo,
l l, X=O
Ascertain that the derivative f' (x) < 0 is inside the interval.
3.13.16. Hint. Show that f' (x) ~ 0. Ascertain that
f (O) = I -a { > 0 for a < I,
<0 for a> I,
and take advantage of the fact that the function increases.
3.13.17. Hint. Show that the function f (x)=xex-2 increases and has oppo-
site signs at the end-points of the interval (0, l).
3.13.18. Hint. Show that the derivative

f' (x) = -21 + 2x sin _.!_ - cos _!__ (x i= 0)


x x

is equal to ; at the points x=( 2 n~l):rc (n=O, 1. 2, ... ),and to-~ at

the points x = -2 1 , i.e. the derivative changes sign in any vicinity of the
n:rc
origin.
3.13.19. Hint. Ascertain that the auxiliary function 1p (x) = f (x)-<p (x) iPJ-
creases.
3.13.20. Hint. Make sure that at all points of the domain of definition of the
function the derivative retains its sign if ad-be i= 0. But if ad-bc=O, i.e.
; = ~, then the function is constant. 3.13.21. P=-6, q= 14.
3.13.22. A mmimum f (x 0 ) = 0 if <p (x 0 ) > 0 and n is even; a maximum
f (x0 ) = 0 if <p (x0) < 0 and n is even; the point x0 is not an ext rem um if n
is odd. Hint. Al an even n, in a certain neighbourhood of the point x0 the func-
tion retains its 5ign and is either rigorously greater than zero or rigorously less
tlian zero, depending on the sign of <p (x0 ). At an odd n the function changes
sign in a certain neighbourhood of the point x0
Answers and Hints to Ch. IV 439

3.13.23. Hint. For x :j: 0 f (x) > 0, hence f (0) is a minimum. For x >0
tiie denva . I + -cos-
. t.1ve f' (x ) = 2 -sm- I . positive at the points x =1-
I 1s
x x x 2nn
and negative at the points X= ( 2 n~ I) :rt. The case x <0 is investigated ana-
logously. 3.13.24. (a) I and O; (b) I and -2.
3.13.25. (a) The least value is non-existent, the greatest value equals I; (b)
the function has neither the greatest, nor the least value.
3.13.30. Yes. Hint. Since f" (x) changes sign when passing through the point x0 ,
the latter is a point of extremum for the function f' (x).
3.13.31. The graph passes through the point M (-!, 2) and has a tangent
line y-2=- (x+ I); M is a point of inflection, the curve being concave down-
ward to the left of the point M, and upward to the right of it. Hint. The func-
tion f" (x) increases and changes sign when passing through x = -1.
I
3.13.32. h= .r-.
ar2
3.13.33. Hint. According to the Rolle theorem, between the roots of the first
derivative there is at least one root of the second derivative. When passing
through one of these roots the second derivative must change sign.
3.13.35. Hint. The polynomial has the form a0 x2 n+a 1 x2n- 2+ ... +an_ 1 x 2 +an.
Polynomials of this form with positive coefficients have no real roots.
3.13.36. Hint. Take advantage of the fact th1t a polynomial of an odd degree
(and, hence, also its second derivative) has at least one real root and changes
sign at least once.
3.13.37. Hint. Find lim ( 2x4+x3+ ') .
x- 00 x 3 - 0. . x-- 1

Ch apter IV
4.1.2. /=x 3 +x 2 +0.51n\2x-1 l+C.
3 3
4.1.7. I=~ (x+l)2 + ~ x2 +c. Hint. Eliminate the irrationality from the
denominator.
4. 1. 14 . I = I arc t an 2x + C .
10 5
2 2x+ I
4.1.15. I= .r- arctan .r- +c.
r 3 r 3
4.1.18. l=lnlx+3+V'--x2_...J.._,-6.-r1_'_1 l+c.
= - ' - I n I v-Tiix-
4 1. 20 / 2 V10 Viox+ V1
1 vr + c.
4.1.21. (a) 2I arctan- x-3
2 -+c,
. (b) 3 (x-4)
4
,VJ -
x+C.. (c) 3 tan x +
2
+2 cot x+C; (d) - x--!-arc tan x+ C.
4.1.22. (a) In (x+ YI -f-x 2 )-t-arc sin x-f-C; (b) sinx-cos x+C;
(c) -Ii~ 5 5-x+ ~ 2-x+c; (d) -0.2 cos.Sx-x sin Sa+C.

4.2.3. I=~ Jr(2x-5)'1+~ V2x-5- .37 +c.


12 2 4 Y2x-s
440 Answers and Hints

-- I ~
4.2.8. I =-2 Vcos x+c. 4.2.10. I =T (x3+3x+ I) a +c.
I 2
4.2.13. (a)0.75 V(l+lnx)'+C; (b) lnllnxl+C; (c) 2 arcsin;3 +c;
I xn
(d) - arc tan -+C;
.r-
(e) -2 cos r x +c;
I
(f) -2 ln 2 x+ Jn I In x I+c.
na a
3 ~
4.2.14. (a) - 140 (35-40x+14x2 ) (1-x) a +c;

(b) ; (lnx-5) V l+lnx+C;


(c) (~-~
3 7
sin 2 x+~ sin' x) JI sin 3 x+C
11 '
(d) - /5 (8+4x 2 +3x') JI I x2+C.
4.3.2. x arc sin x+ YI x2 +c.
4.3.14. - cosx In tan x+ln Itan ( ; ) l+c.
4.3.17. x In (x+ VI+ x2 ) - VI +x2 +c.
4.3.18. !x Vx [(In x) 2 - ~ In x+ +c.
4.3.19. 2 VI +x arc sin x+4 VT=X+c.
n
4.3.20. -0.5 (++cot x) +c.
Sin X

4321 3X(sinx+cosxln3) C
. . . l+(ln3)2 + .
4.3.22. (; x3 -x2 +; x+ 1:) eax+c.
4.3.23. (x 4 -10x 2+21) sin x+x (4x2-20) cos x+C.
4324
. . .
9x 2 +1Bx-ll cos 3x+- 2x+2. 3
s1n x+ .
C
27 9-
x3 )
4.3.25. ( --x2 +3x In x-.::._+~-3x+C.
1 2
3 9 2
x4 - I x3 x
4.3.26. - 4- arc tan x- 12 4 +c. +
x3 2+x2 . r - -
4.3.27. 3 arccosx--9- r 1-x2+c.

4.3.28. (a) ;-t


IBx 2 + 75 3 sin(6x+2)- 5x
7 tl cos(6x+2)+ ~ x3 +
3 2 9 ~
++x2 -x+C; (b) 4 (x2-7x+I) (2x+I)3 - 40 (2x-7)(2x+l) 3 +
'i.7 ~
+ 320 (2x+ I) a +c.
4.4.2. (d) Hint. Apply the generalized formula for integration by parts and
express In from the relation thus obtained
ex n (n-1) 11 2
ln--a. 2 sinn- 1 x(a.sinx-ncosx)+ a. 2 ln-2-a~ In.
Answers and flints to Ch. V 441

4.4.3. I = - cos x +n-2 I 2)


" (n-J)sin"- 1 x n-1 n- 2 (n~ ;
..x, +-
/3=- -2cos cos x
I 1 = - -.-.,-+-
smx 2 1
I x
2 smx 2 In tan -2 +c.
I I

Chapter V
2
5.1.2. x2 -2x+ 6I 1n J (x+
x-1!)
3
J
+ 316 In I x+21+c.
x
5.1.5. 21nlx-1 l-1nlxl-{X-l) 2 +c.

5.1.8. ~- arctan 2 ~: 1 -_!_arctan(x+2)+C.


3 f 7 f 7 3
5.1.10. 5x+ln x 2 (x+2) 4 I x-21 3 +C.
9x2+sox+68
5.1.11. 4 (x+2) (x+3J2 +a In
I J (x+ I) (x+2) 16
(x+3)11
I+c.
I
5.1.12. - x- 2 -arc tan (x-2)+C.
I (l+x) 2
5.1.13. - 6 (I +x) +6I In l-x +x2 +2I arc tan x-
i 2x-1
- 3 _3 arc tan VJ +c.
x+2
5.1.14. 2 (x 2 +I) +2 arc tan x +In Vvx:tT
x + 1 +c.
2

5.2.2. 4 Vx+6 Vx+24 1~-x+241n I ~x-1 l+c.

5.2.4. - I arc tan 21+


.r- .r-I +In IVt-lYt
3
Vv+2fi I+
f

.. /x+I
3 f

+c.
3
where t =

3 v1+x
vx- x
2

I.
+t+1

5.2.7. V l-x +c. 5.2.8. 2 l-x + C.


5.2.9. ( 1-f x) VT"=X1-f arc sin x+C.
5.3.3. -2 arc tan ( V 1+x;xLf- 1 +I) +c.

5.3.5. 2 In I V xi+ 2x + 4 -- x I- . 3 -
2 ( V x 2 +2x+4-x- I)

- -x:i In 11--- I
~ x~+2x-i-4 -x-1 + c.
442 Amwers and Hints

536 l+Jf'f"=X"2+2arctan -./l+x+c .


. . " x V 1- x
5.3.7. x- I +c. 5.3.8. (x+ Vf+XI)u +c.
Y2x-x 2 15
5.4.2. 5 Yx~+2x +-5-ln (x+ I+ Vx_2_+_2x_+_5)+C.
5.4.5. 3x2 + 3x-I Y3x 2 -2x+1+c.

5.4.6. 2x:I J.l"x 2 +x+I +} lnl2x+1+2 JfxLt-x+t l+c.

5.4.8. + (x 2 -14x + 111) Vx 2 +4x+ 3- 66 In I x+2+ Jf x" --i-4x+3 l+c.

5.4.9 . ..!..(32x2 -20x-373) v~x'+5x+7+ 32;; ln/4x+5+


64 128 r 2
+ 2 Jf 4x 2 + IOx + 14 I+ C.
3x+5 ,/-2- 2- 3 . I C
5.4.10. tl (x + l) 2 r x + x- 8 arc sm (x+ I)+ .
Yx 2 -4x+3 . I
5.4.11. - 1 -2arc sm------.;+C.
X- X-.&.

-~ .., /x+28x 2 +12x+7 C


5.4 12 15 V x+ I (x+ 1) 2 +
5.4.13. 1nlx 2 + 1 +V; 4 + 3x2 + 1 l+c. Hint. First make the substitution
x2 =t.
5.5.2. 3 arc tan V-x+C. 5.5.4.
2
3
( 2+x--a2)Y4 - 12
5
( 2)5
2+x3 4 +c.
11 8 !j

5.5.5. 22 a - 3 (I +x
3 ( I +x)
8 2>3+ TO3 (I +x) a + c. 9

5.5.7. 172 V(t+Vx)7-3 V(i+v xt+c.


5.5.8. 3 In
v--xa/ +
3
av-+C.
l+v x I+ x
a
(I +x 2 ) 2 (3x 2 -2)
5.5.9. 15 +c.

5 51.o Y~ (2x 2 - I ) +c
3x3

5.5.11. ~~ V(1+vx~) 8 +c.


4 D

5.5.12. ~ ( 1++ )5 - ~ (I+~ ) 5-1-C.

5.6.2. ~
3 Stn --
X 5 Stn.1 6 X +c.5.6.6. tan x+ 31 tnn'l x+C.
I I
5.6.IO. (a) -cotx+ 3 cot3x- 5 cot 6 x-x-f-C;

(b) ~ tan 2 x-~ ln(l+tan 2 x)+C~~ ~ tan 2 x-j-lnlcosx\+C.


Answers and Hints to Ch. VI 443

5.6.12. -sinx-...!_ sin 3 x+_!_ In 1 1 + sinxl+c.


3 2 I- sin x

2 (1+2tani)
5.6.14. VIS arc tan VIS ;+c.

,
5.6.22. (a) -
x sinh 4x
+32 +c;
2
(b) V 3 arc tan
( 2 tanh ;
V3
+ 1) +c.
8
5.7.3. -! ln(x+ Vx2 -1)+ ! x(2x 2 -I) Vx 2 -l+C.

5.7.4. In (x+ Vx 2 +1)- VX2+T


x
+c.

5.7.7. i=arcsinxtl+c.
x-1
5.7.8. I= V +c.
x2 -2x +5
4
5.8.2. I =4 VI -x+ 2 In (2-x-2 Vl=X)-2 (I+ Y1 x) In x+C.
ta cos t +sin t
5.8.5. I ~"e a. 2 + 1 + C, where t =arc tan x.

Ch apter VI

6.1.9. 1=4 3 t 19 =44 as the area of a trapezoid whose height is5-1=4


and bases 4Xl-1=3 and 4X5-1=19.
I 175 125 1 175 125
6.1.12. Sn=16-;r-2ii'"+ 4n 2 ; Sn=l64+2ii'"+ 4n 2
3 :rt 7 I 3
6.2.2. (a) I; (b) 2 ; (c) 6 . 6.2.10. (a) 72 ; (b) 2 In 2 ; (c) n;
:rt :rt n n ') T5;
14
(d) 4 - arc tan T; (e) In 2; (f) I; (g) arc tan e- 4 ; (h) 16 : (1
4
(J) 3 : (k)
v3-2 . 6.3.1. (c)3</ <5. Hint.
. M=/(0)= 2
5
, m=
2
3 , sin2x ,r-- . n dy 2
=/(2)= 2 . 6.3.11. (a)-x-: (b) - r i+x4 6.3.14. (b) 4 .63.15.(b)dx=
. 3 . , t , tan t
= - e-IJ smx. 6.3.23. (a) In x; (b) x. 6.3.24. (a) Y, =mt: (b) Yx= T
6.3.25. (a) The maximum is at x= I, the minimum at X=- I; (b) the
minima are at x = - 2; O; 2, the maxima at X= I.
6.4.3. (a) ~~4 (substitution X= a sin t); (b) Jf-3-; Jf2 (substitution X= tan/) .
6.4.6. (a) rr-2-,r-+ln
2 2+ v_;
3 '"'3-1); (c) 8+-2-n.
(b) 2(r 3 J/3
r 3 l+ 2
. n
sm 24
6.4.15. (a) 2-2 ln2; (b) 0.2 In 112; (c) - - - - (d) y'3-0.5 In (2+
. :rt . :rt
sm 8 sm 12
444 Answers and Hints

+ Jf 3); (e) 0.25 In 3 (substitution sin x- cos x= t); (f) a 3 ( ~ - ; )

(substitution X= a cost); (g) :n; (substitution X= 2a sin 2 t); 2


(h) ~ +f.
6.4.16. (a) ~ ; (b) : ; (c) {-In~~ (substitution x4 = t); (d) ~ (substi-
tution x2 = a 2 cos 2 t + b2 sin 2 t).
6.4.17. The substitution x
nuous at t = 0.
=+ will not do, since this function is disconti-

6.4.18. The substitution t =tan ; will not do, since this function is discon-
tinuous at x = :n.
6.4.19. Hint. The inverse function x= Jf/5 is double-valued. To obtain
the correct result it is necessary to divide the initial interval of integration into
two parts:
2 0 2

~ Vx 2 dx= ~ Vx 2 dx+ ~ Vx 2 dx
-2 -2 0
and apply the substitutions x=- in -2 < x < 0 and x=+ Jf75 in 0 < Vto
< x < 2.
6.4.20. It is impossible, since sec t ~I and the interval of integration is
(0, I].
6.4.21. It is possible; see Problem 6.4.12.
a 0 a
G.4.22. Hint. On writing ~ f (x) dx = ~ f (x) dx + ~ f (x) dx, make the sub-
-a -u O
stitution x= - t in the first integral.
I -1 0
6.4.23. ~ f(arcsint)dt+ ~f (n-arcsint)dt+ ~ f(2:n+arcsint)dt.
0 I -1
Hint. Represent the given integral as the sum of three integrals for the in
tervals: ( 0, 2:It ) , ( 2,
:It T3:rt ) , ( T,
3:rt 2:n ) and substitute
. the variable: x =
=arcsint, X=:rt-arcsint, x=2n+arcsint respectively.
6.5.3. (I) If f (x) is an even function, then
:t n :re
~ f(x)cosnxdx=2 ~ f (x)cosnxdx, and ~ f(x)sinxdx=~O.
-rr 0 -IT
rr rr
(2) If f (x) is an odd function, then ~ f (x) cos nx dx == 0, and ~ f (x) sin nx dx=
-IT -rr
IT

= 2 ~ f (x) sin n dx.


0
6.5.4. 0. 6.6.3. 6-2e. 6.6.5. rr y2-4. 6.6.6. rr-2. 6.6.13.(a) ~--I;

(c) .::._ Jf-3 rr+..!..1n2- (u) _::._ _ _!__. (c) ln2-_!_. (f)ln2
(b) --e 4 9 2 :.:! ' 4 2 2 ' 8
:n 16:11 .r-
(g) 2-I; (h) 3-2 r 3.
Answers and Hints to Ch. VI 445

6.6.14. /lint. Integrate by parts twice, putting u = (arc cos x)n the first
time and u =(arc cos x)n- 1 the second time.
6.6.15. Hint. Integrate by parts, putting u=x.
6.7.4. (a) 0.601. Hint. Estimate I flV (x) I on the interval [ ~, ~ J and put
2n=6; (b) 0.7462. 6.7.5. 0.96
6.8.1.
( x-x2
for O~x~I.
I
F (x)= i 2

(x-2) 3
I
2
I
for I< x~2.

1-+-
3 2
for 2 < x~3.

Continuity is checked directly. The assertion concerning the derivative re


quires checking only at the points x= l, x=2.
6.8.2. Hint. Make sure that the function f (x) is continuous both m~1de the
interval (0, I) and at the end-points ( lim f (x) = f (O) and Jim f (x) = f (I)).
X->+0 x-J-0
6.8.3. No. Hint. Consider the function
I if x is rational,
{
<p (x) = - I if x is irrational on the interval [0, I).
b
6.8.4. l-Jf3. Hint. ~f"(x)dx=f'(b)-f'(a).
a
6.8.5. Hint. Putting for definiteness x > 0 and
E (x)=n~x < n+ I,
take advantage of the additivity of the integral
x I 2 n x
~ E (x) dx= ~ E (x) dx+ ~ E (x) dx+ ... + ~ E (x) dx+ ~ E (x) dx.
0 0 I n-1 n
6.8.6. The antiderivative f 1 (x) will lead to the correct result and f 2 (x) to
the wrong one, since this function is discontinuous in the interval [O, :re].
x
6.8.7. F (x) =Yo+~ f (t) dt. Hint. Any an ti derivative F (x) can be represen-
Xo
x
ted in the form F(x)=~f(t)dt+C. Puttingx=x0 , find C=y 0
Xo
I e2b_e2a
6.8.8. ~ =2 ln 2b-2a .
6.8.9. The function is defined on the interval [-I, I], it is ncld, and in-
creasing; convex on the interval [-I, O] and concave on the interval [O, I];
the point [O, OJ is a point of inflection.
6.8.10. Hint. The function
_ { xx at 0 < x ~ I
I (x)=
I at x~-"o
I
is continuous on the interval, it reaches the least value m = e e ~ 0.692 at
I
x=- and the greatest value M ~ l at x=O and at x= l.
e
446 Answers and Hints

6.8.11. H.int. I negrae


t t th e mequa1y
2 sin x
1't n~-x-~. I
6.8.12. Hint. Integrate the inequality

V xsinx > yx2 -i=x y i - ~~at O~x~ ~


and write Schwarz-Bunyakovsky inequality
~
2
;l't E.. 2 2

5Vxsinxdx~ l/ 5 5
0 " 0
xdx
22 0
sinxdx= ~'=~
6.8.14. Hint. Apply the Schwarz-Bunyakovsky inequality in the form

r!v b t (x). f ~x) dx]


2
~ ! !
f (x) dx f ;x) dx.

6.8.15. Hint. Make the substitution arc tan x=;.


x
6.8.16. Hint. If f (x) is an even !unction, then F (x) = ~ f (l) dt is an odd

!unction, since
-x x
F(-x)= ~ f(t)dt=-~f(-z)dz=-F(x) (l=-z).

,
And ii f (x) is an odd function, then F (x) = ~ f (t) dt is an even !unction,

since
-x ,
F(-x)= ~ f (t)dt=-~ f(-z)dz=F(x) (t=-z);

all the remaining antiderivatives have the form F (x) +c and, therefore, are
also even functions.
6.8.17. Hint. The derivative of the integral I with respect to a equals zero:
di
da =f (a+T)-f (a)=O.

Ch apter VII
3 I
7.1.4. (a) In 2; (b) ~ (2 2-1); (c) T; (d) I; (e) 2.
I I I 2 .n 2i
7.2.2. (a) -2 ; (b) -+-In--~ 0.283. 7.2.5. -4 . 7.2.10. .r
2 2 e-:- I hr d2+h2
5 8 21! 2/ 0
7.2.13.(a) =a; (b) =ln2;(c) = 1n 3 +2. 1.2.15. 3 . 1.2.16. 11 .
35 2 5 . 3 8 I
7.3.4. 6 . 7.3.6. 3 + 2 arcsm 5 . 7.3.11. 15 . 7.3.13. 9. 7.3.16. --1
m+.
Answers and Hints to Ch. VII 447

64 8
7.3.19. 3 7.3.20. 3 7.3.21. 2:rt-(2 y3) In (2+ y3). 7.3.22. 0.75:rt.
128 I 4 8 1 91
7.3.23.15. 7.3.24. 3 7.3.25. 3. 7.3.26. 15. 7.3.27. 12. 7.3.28. 30.
8
7.4.6. 5 . 7.4.8. 0.75:n:ab. Hint. The curve is symmetrical about the coordinate
axes and intersects them at the points X= a, y= b.
7.4.9. (a) ~5 . Hint. The curve is symmetrical about the x-axis, intersecting
it twice at the origin at I= I. The loop is situated in the second and third
quadrants; (b) ~5 . Hint. The points of self-intersection ot the curve are found
in the following way: y=tx(t), therefore y(t 1)=t 1x(ti)=t 2 x(t 2 ) at 11 f= t 2 and
x(t1)=x(t2), only if x(t 1 )=x(t 2 )=0, i.e. t 1 =0; t 2 =2; (c) 8 ~3 .
7.4.10. 0.25:n:ab. Hint. The curve is symmetrical with respect to both axes of
coordinates and passes twice through the origin forming two loops. Therefore, it
is sufficient to compute a quarter of the desired area corresponding to the variation
of t from 0 to ; and multiply the obtained result by 4.

7.4.11. ~;; . Hint. The curve resembles an astroid extended in the vertical
direction.
7.5.2. (a) 3; ; (b) :n::2 Hint. The curve is a circle of radius ~ passing

through the pole and symmetrical about the polar axis, - ~ ..:;;;;; cp..:;;;;; ;

7.5.6. 2a 2 (
5:rt - I)
8 7.5.8. (a) :n:; 2 ; (b) :n:t. 7.5.9. a2 ( ~;- r3).
7.5.10. ~~ 2
Hint. The curve passes through the pole forming two loops located
symmetrically about the y-axis in the first and fourth quadrants. It is suffirient
to calculate the area enclosed by one loop corresponding to variation of qi from O
to ~ and double the result thus obtainer!.

7.5.11. ~ na 2 Hint. The curve passes through the pole. it ts symmetrical


about the polar axis and situated in the first and fourth quadrants. It is suffiricnt
to calculate the area of the upper portion of the figure which corresponds to
variation of qi from 0 to ; and double the result thus obtained.

7.5.12. a( 1+ 6:rt - -2-3) .

7.5.13. n; 2 Hint. The curve is symmetrical about the coordinate axes and
intersects them only at the origin, forming four loops-one in each quadrant
(a four-leaved rose). Therefore, it is sufficient to find the area of one loop corres-
ponding to the variation of qi from 0 to ; and multiply the result by 4.
7.5.14. 2 :n:a 2 Hint. The curve is symmetrical about the axes of coordi-
nates and the bisectors of the coordinate angles; it cuts olT equal intercepts on
the axes. The origin is an isolated point. It is sufficient to compute the area of
448 Answers and Hints

one-eighth of the figure corresponding to variation of rp from 0 to -i- anrl mul-


tiply the result by 8.
7.6.2. 9; :rt. Hint. A plane perpendicular to the x-axis at the point x will
cut the sphere along a circle of radius r= V16-x2, therefore the cross-sectio-
nal area S (x)=:rt (16-x2).
7.6.5. 0.5:rta 2h. Hint. The area of a triangle situated at a distance x from
the centre of the circle is equal to h Va 2 -x2
7.6.10. 2:rt 2a 2 b. 7.6.11. ~ (see Problem 7.3.9). 7.6.14. 5:rt 2a 3.

7.6.16. (a) 2nab (I + 3~ 2 ) ; (b) 136 a; (c) ~ abk 2:rt. 7.6.17. ; a 3 tan Cl.

7.6.18. (a) I 2:rt,. (b) 16 :rt,.


15 (c) 564 :rt,. d 2.
():rt, 64 :rt,.
(e) 3 (f ) 3na.
4 3

7.6.19. ~~3 . 7.6.20. 7~. 7.6.21. {-:rta3 (e :" -e - ~c) +na2c= n;3 sinh ~ +
+ :rra 2c.7.6.22. ; 0 (6:rt+5 Jl3). Hint. The abscissas of the points of intersec-
:rt n 19 127 16:ric6
tion are: x1 = - 3 ; x2 = 3 . 7.6.23. 48 :rt. 7.6.24.7:rt. 7.6.2s. 105 ab 2

Hint. Represent the evolute of the ellipse parametrically as follows: x= c2 cosa t;


a
2 4 . n 2a3
y=--7J-sin 3 t, where c=Va2 b2. 7.6.26. 3 :rra 3 7.6.27. 4 V2";

:rt;3 l V2 In (1 + V2)- ~ J. Hint. Pass over to polar coordinates.


4 112 eb-e-b .r .r
3
7.6.28. 21 na. 7.7.2. 2f. 7.7.4. In ea-e~. 7.7.8. (a) r 6+In ( r 2+ Jl3);
n . :rt . 2 Jl3 a (a+ 2)
(b) 2ln(2-V3). Hint. x1 = - 2 . x 2 =3 (c) - 3 -. 7.7.9. -- 2- .
7.7.10. 10 G;+ VS)- 7.8.2. Ba. 7.8.5. 1: . Hint. The curve intersects the
4/-
axes at t 1 =0 and t 2 =1/ 8. 7.8.7. 4 V3. 7.8.8. 16a. 7.8.9. B:rta. Hint.
4 (a3-b3) :rta 2n
Sre Fig. 79. 7.8.10. ab . 7.8.11. 3 . 7.8.12. At t= 3 the point

Mr a ( 2; - ~ 3 )' 3;] . 7.9.5. l.5:rta. 1


7.9.9. 52+ In ; . 7.9.10. 2V2 :rta.

Hint. The curve p = 2 JI~ a cos ( rp- ~ ) is a circle.

7.9.11. P[V2+In(l+V2)]. 7.10.3. (a) I 4:r (b) 623Jt .


3 ;
7.10.5. 2:rr (I+ 3 ~-:3). 7.10.8.
2
7.10.14. (34 17-2) ; .

7.10.15. 2:rt [ V2+ In(!+ V 2)]. 7.10.16. 536 na 2


2 V2
7.10.17. -g- rr (e"-2).

7.10.18. 29.6n. 7.10.19. 4:t 2 a 2 7.10.20. 1 ~8 na2. 7.11.i. 16a 2 where a 1s


449

8
the radius of the cylinders base. 7.11.8. l.5:n:. 7.11.10. (a) 15 ;
7 I I :rra 2 p2 ( .r-
(b) 50 - 4 arc tan 2 . 7.11.11. (a) T; (b) 6 3+4 r 2);

(c) 8I (5:rt+6 ,r1 -3). 7.11.13. 2a (2ln3-I). 7.11.14. - 3 -


2 (5 .r -
r 5-2 ,r1 -)
2.

7.11.17. 2:rt ~/. 7.11.18. :rra2 fpq. 7.11.19. :rtab ( 3~2 -1+ 2; ) .
7.11.20. nabh .
3 7.11.21. I 2rr. 7. 1 1.22. ( 4 y39 - 6 ) :rtb2 a. 7. I 1.23. 2f
4 :rraa.

7.11.24. (a) :rtr( .r-


r 5-".r-)
2 + ( .r- 5-1
r 2+1)- 2-
J; (b) 4:rra
243
2 (
21 .r-
r 13+
3+ yT3) 2 :rtR MR-
2 w2
+2 ln 2 2nrh.
; 3 yR.
(c) 7.12.2. 3
7.12.4. T.
4
7.12.9. - .4

7.12.11. :rrabhd. 7.12.12. :rtrdh 2 7.12.13. / 2 :rrR 2 H. 7.13.3. 0.25nRa.

7.13.7. Mx=f(5YS-1); My=; rs+/51n(2+Y5). 7.13.8. Mx=

= ~ ya2+b 2 ; My=~ ya2+b2. 7.13.9. 2+1n(l+ y2). 7.13.10. 0.15.


_aba. 1y_aab (a+3b)h 3
7.13.11. I x- 12 , - 12 . 71312
. . . 12 . 7.13.16. Xe=Ye=0.4a.
a sin rx 5a
7.13.19. Xe=Ye=s. 7.13.26. Xe= R --a-; Ye=O. 7.13.28. Xc=s; Yc=O.
0.2 (2e 2"-en) 0.2a (e 2" - 2e")
7.13.29. Xe=- n Ye n 7.13.30. 4.5:n:a~.

en-e2 en-e2
7.13.31. Xc=O; Ye=!~ 7.14.t.1:::+~1 1; 41:+~1if bothmand nare

even; 2 m-nim+n J if both m and n are odd; I I


m-n
m+n
if m and n are of different
evenness. Hint. The curves ym=xn and yn=xm have two common points (0, O)
and (I, I) in the first quadrant. The area of the figure situated in the first

quadrant is equal to Ij (x; -x :


1
) dx I Depending on evenness and oddness
of m and n this figure is mapped symmetrically either about the coordinate
axes (m, n even) or about the origin (m, n odd). If m and n are of different
evenness, then the curves enclose only the area lying in the first quadrant.
7.14.3. Hint. Take advantage of the formula for computing the area in
polar coordinates.
7.14.4. Hint. Since the figures are of equal area, the function S (x) appear-
b
ing in the formula for the volume V = ~ S (x) dx is the same and, consequently,
a
the values of the integrals are also equal.
7.14.5. Hint. The formula follows rlirectly from Simpson's formula
h

SJ(x)dx=~ [t(O)-Ht(%)+f(h)].
0
450 An~wers and Hints

nr2x 2
for a sphere S (x) = n (r 2-x2); for a cone S (x) = ---p- ; for a paraboloid of
revolution S (x) = 2npx and so on.
7.14.6. Hint. Divide the curvilinear trapezoid into strips 11x wide and write
an expression for the element of volume 11V = 2n xy 11x.
7.14.8. Hint. Use the formula for calculating the length of a curve represen-
ted parametrically.
7.14.9. In ~. Hint. The point (t= I) nearest to the origin with a vertical
:rt
tangent corresponds to t =2'.

7.14.13.
V3
2n l5. 7.14.14. V2z. 7.14.16. (a) 0.5 In (x+y);

(b) ~-0.5arcsinx.

Chapter VIII
I I
8.1.2. (b) 2 In 2; (c) I; (d) I - In 2; (e) n; (f) 2 .
8.1.6. (a) It diverges. Hint. In (x 2 + I)>_!_x
x
for x > Y e-1; (b) converges;

.
(c) d 1verges. H"t2+cosx
in . x > V Vx,
l. (d) converges; ()d'
e 1verges.
8.1.17. (a) 0. Hint. Represent the integral as the sum of two items:
oo I oc

S 1 1~:2 dx=S 1 ~: 2 dx+ S 1 ~: 2 dx. Make the substitution x=+ in these-


o 0 I

(' In x
cond summand and show that .) 1 +x2 dx=
"'
- s
I
In x
1 +x2 dx; (b) ~!.
I 0
2
8.2.2. (a) 9a 3 ; (b) it diverges; (c) diverges; (e) 3'
(f) converges for p < l and diverges for p >- I.
8.2.7. (a) It converges; (b) diverges; (c) converges; (d) converges;
(e) diverges; (f) converges. 8.2.11. (a) It diverges; (b) 2
.r r -
In 2;
51
(c) 7 .
8.2.14. (a) It converges;
(e) converges. 8.3.7. (a) ; ;
(b)
(a) 2n.
diverges;
8.3.8. 3:rra 2 .
8.3.14. mgR. Hint. The law of attraction of a body by the Earth is deter-
(c) diverges;
8.3.9. +. (d) converges;
8.3.10. ~n.
ml{ R2
mined by the formula f = -r-2 - , where m is the mass of the body, r is the
diStance between the body and the centre of the Earth, R is the radius of tl~e
Earth.
8.3.15. e1 Hint. Electric charges interact with a force e,~ 2 , where e1 and e2
r
are the magnitudes of the charges and r is the distance between them.
8.4.1. Hint. Represent the integral in the form of the sum
+oo a +oo

S xP~:qx=S xP~:qx+ S xP~~qx (a> I)


1 I a
Answers and Hints to Ch. VIII 451

and apply special tests for convergence, taking into consideration that in the
first integral lnx=ln[I+(x-1)]-x-l as X-+ I, and in the second integral
the logarithmic function increases slower for q < 0 than any power function.
8.4.2. Hint. Making the substitution xq=t, reduce the given integral to the

form +s"' t:~


0
11
sin t dt. Represent the integral S'" t~~: sin t dt
0
as the sum

I +oo
Ss~~ / dt + S s~~ t dt, where a= 1-p~ 1 , and show that the integral conver-
o I
ges absolutely for I <a < 2 and conditionally for 0 < a,,;;;;; I. Note that at
P+q 1 =0 the integral is reduced to the conditionally converging integral
+~

S sint dt, and a t -q-


-t- . .in t egra 1
P+I = - l t o t he d'1verging s
+~

ysintdt.
0 0
1/2
8.4.3. Hint. Represent the given integral as the sum ~ xP- 1 (1-x)q-idx+
0
I

+ ~ xP- 1 (1-x)q-i dx and apply the special comparison test.


1/2
T
8.4.4. Hint. If /a I ::j: I~/, then ~ sin ax sin ~x dx is bounded.
0
8.4.5. Hint. By substituting t =x 2 the integral is reduced to the Euler
gamma-function.
"' "' "' afl
8.4.6. Hint. Sf (ax)-:-f (~x) dx= f ~) dx- f ~x) dx= S f ~x) dx= S 5
a aa afl aa
fla
=A lnl+S
a
f(x)-Adx. Applying the generalized mean value theorem, show
x
aa
that the last integral tends to zero as a-+ 0.
8.4.7. Hint. Take the function f (x)=e-x for the first integral, the function
f (x) =cos x for the second and take advantage of the results of Problem 8.4.6.
8.4.8. It converges for m < 3 and diverges for m:;,, 3. Hint. Take advantage
x2
of the equivalence of I -cos x and 2 as X-+ 0.
1t
1t 2
8.4.9. Hint. Represent 5~as
0 (sin x)
the sum of two integrals S (sin.dxx)
0
k +
1t

+ S~;
( sin x)k
reduce the second integral to the first one by making the substi-
1t
2
tu ti on x= r t - t and take advantage of the equivalence of sin x and x as x-+ 0.
452 Answers and Hints

l't
00 2
8.4.10. Hint. Ssinx(l-cosx)
xs dx=
ssinx(l-cosx)
dx +
0 0 ~
00

-f Ssin x (I_ xs-cos x) dx. The integrand of the first summand on the right side is

2
an infinitely large quantity of order s-3 as x-+ 0. By the special comparison
test the first integral converges absolutely for s-3 < I, i.e. s < 4, and diverges
for s;;;;. 4. The second integral in the right side converges absolutely for s > I,
since the function sin x (I -cos x) is bounded. But if 0 < s.;;;;; I, the second in
tegral converges conditionally as the difference of two conditionally converging

integra Is S"' sinxs x dx and S"" sin X


xs
cos x dx (see Problem 8.1.13).
Tl l't
2 2
8.4.11. Hint. Integral (2) can diverge. For example, let

q:>(x)= {
+ I) :rt,
I, 2n:rt .;;;;; x .;;;;; (2n
- I, (2n + I) :rt < x < (2n + 2) :rt.

The .integra I s
00

-x-x dx converges (see p ro bl em 8.I. 13 ).


sin But s
"'

.,, (x) dx =
sixn x m
0 0
00 00

=SI si~ x I dx diverges (see the same problem). But if the integral Sf (x) dx con
0 u
00

verges absolutely, then the integral Sf (x) q:> (x) dx also converges absolutely: if
a
I q:> (x) I < C, then If (x) q:> (x) I < CI f (x) I. and it remains to use the comparison
theorem.
l't
2-.1:
8.4.12 Hint. Transform the integral f (x) into f (x)= ~ In sin z dz by the
l't
2
substitution y=; -z. Taking into account that sin z= 2 sin ~ cos ~ , reduce
the above to the sum ol three integrals.
8.4.13. Hint. Putting u= In cos x, cos 2nxdx=dv, integrate by part~ and get

the equality I ,.=-I


2n
s.
2
sin-
sm 2nx - x dx,
cos ll'.
n : Cl . s1r.ce
p

sin '2.nx =sin (2n-2) XCOS 2.!.+ sin 2xcos (2n-2) x,


Answers and Hints to Ch. Vl/1 453

r n

ln=-
2
I
n
- s.
2
sin
sin x
(2n-2)x--dx+
cos x
L o

+ l
0
In (2n - 2) r-oln 2xdx +,]
0
l
,1nxm (2n-2) xdx
~

Check by direct calculation that for n ~ 2 the second and the third summands
equal zero. Therefore, for n ~ 2

2"
I =- -
" 2n
1 s. sin x n-l
sm (2n-2) x--dx=- - - I - i
cos x n "
0

and by mductwn, l,,=(-l)n- 1 Bi

Potrebbero piacerti anche